ΠΡΟΤΕΙΝΟΜΕΝΕΣ ΑΣΚΗΣΕΙΣ ΓΙΑ ΜΑΘΗΤΙΚΟΥΣ ΔΙΑΓΩΝΙΣΜΟΥΣ_ALL

445
http://www.mathematica.gr/forum/viewtopic.php?f=109&t=15584 Επιμέλεια: xr.tsif Σελίδα 1

Transcript of ΠΡΟΤΕΙΝΟΜΕΝΕΣ ΑΣΚΗΣΕΙΣ ΓΙΑ ΜΑΘΗΤΙΚΟΥΣ ΔΙΑΓΩΝΙΣΜΟΥΣ_ALL

Page 1: ΠΡΟΤΕΙΝΟΜΕΝΕΣ ΑΣΚΗΣΕΙΣ ΓΙΑ ΜΑΘΗΤΙΚΟΥΣ ΔΙΑΓΩΝΙΣΜΟΥΣ_ALL

http://www.mathematica.gr/forum/viewtopic.php?f=109&t=15584

Επιμέλεια: xr.tsif Σελίδα 1

Page 2: ΠΡΟΤΕΙΝΟΜΕΝΕΣ ΑΣΚΗΣΕΙΣ ΓΙΑ ΜΑΘΗΤΙΚΟΥΣ ΔΙΑΓΩΝΙΣΜΟΥΣ_ALL

http://www.mathematica.gr/forum/viewtopic.php?f=109&t=15584

Επιμέλεια: xr.tsif Σελίδα 2

« Επειδή πολλοί μαθητές του Γυμνασίου ενδιαφέρονται για τους διαγωνισμούς

που γίνονται κάθε χρόνο, αλλά δεν έχουν αποκτήσει την εμπειρία σχετικά με τα

θέματα που μπαίνουν, προτείνω να ξεκινήσουμε να βάζουμε θέματα που είτε

έχουν τεθεί παλιά ή είναι παρόμοιου επιπέδου αρχίζοντας με εύκολα, ώστε να

μπουν σιγά – σιγά στο νόημα οι αρχάριοι αλλά ταλαντούχοι μαθητές ».

Με αυτό το μήνυμα ξεκίνησε ο Δημήτρης ΙΩΑΝΝΟΥ την πρωτοβουλία

συλλογής θεμάτων για τους διαγωνισμούς της ΕΛΛΗΝΙΚΗΣ

ΜΑΘΗΜΑΤΙΚΗΣ ΕΤΑΙΡΕΙΑΣ.

Σε αυτή την προσπάθεια έδωσαν το παρόν αρκετοί συνάδελφοι, εξαίρετοι

ΜΑΘΗΜΑΤΙΚΟΙ όπως ο Θανάσης Κοντογιώργης και πολλοί ταλαντούχοι

μαθητές προτείνοντας και λύνοντας θέματα. Όλοι έχουν το ίδιο μικρόβιο, την

αγάπη για τα μαθηματικά .

Και φυσικά όταν έχεις το μικρόβιο, είσαι καταδικασμένος να δημιουργήσεις

κάποια στιγμή όμορφα πράγματα.

Ξεκίνησα την αποδελτίωση των θεμάτων πριν από ένα χρόνο με την

συμπαράσταση του Μιχάλη Νάννου, ο οποίος έφτιαξε και το εξώφυλλο.

Μέχρι στιγμής είναι έτοιμα τρία τεύχη από 100 ασκήσεις το καθένα

(1 – 100 , 101 – 200 , 201 – 300) και ελπίζω μέχρι το καλοκαίρι να είναι έτοιμο

το τέταρτο. Επειδή όλο και κάποιο λάθος ενδέχεται να έχει ξεφύγει σας

παρακαλώ να στέλνετε τις παρατηρήσεις σας στο [email protected].

Αφιερωμένο σε κάθε μαθητή που ασχολείται ή πρόκειται να ασχοληθεί με

Μαθηματικούς διαγωνισμούς

Τσιφάκης Χρήστος : xr.tsif

Page 3: ΠΡΟΤΕΙΝΟΜΕΝΕΣ ΑΣΚΗΣΕΙΣ ΓΙΑ ΜΑΘΗΤΙΚΟΥΣ ΔΙΑΓΩΝΙΣΜΟΥΣ_ALL

http://www.mathematica.gr/forum/viewtopic.php?f=109&t=15584

Επιμέλεια: xr.tsif Σελίδα 3

ΠΡΟΤΕΙΝΟΜΕΝΕΣ

ΑΣΚΗΣΕΙΣ ΓΙΑ ΜΑΘΗΤΙΚΟΥΣ

ΔΙΑΓΩΝΙΣΜΟΥΣ

ΤΕΥΧΟΣ 1ο

ΑΣΚΗΣΕΙΣ 1 - 100

Αφιερωμένο σε κάθε μαθητή που ασχολείται ή πρόκειται να ασχοληθεί με

Μαθηματικούς διαγωνισμούς

Τσιφάκης Χρήστος : xr.tsif

Page 4: ΠΡΟΤΕΙΝΟΜΕΝΕΣ ΑΣΚΗΣΕΙΣ ΓΙΑ ΜΑΘΗΤΙΚΟΥΣ ΔΙΑΓΩΝΙΣΜΟΥΣ_ALL

http://www.mathematica.gr/forum/viewtopic.php?f=109&t=15584

Επιμέλεια: xr.tsif Σελίδα 4

ΘΕΜΑ 1 (ΔΗΜΗΤΡΗΣ ΙΩΑΝΝΟΥ)

Να υπολογίσετε τις παραστάσεις: 2

10 6 12 9 3 2A 2 : 2 3 : (3 ·3 5 3) 2 και

3 3 2B 5 2 1 8 3 20 8 5 15 .

Λύση:

2

10 6 12 9 3 2A 2 : 2 3 : (3 ·3 5 2 3)

2

4 12 10 8 22 3 : 3 5 8 9 2 3 5 ·17 256 9 8 332 5

3 3 2B 5 2 1 8 3 20 8 5 15 5 8 1 8 27 20 8 25 15

5 ·7 8·7 8·10 35 56 80 91 80 11 .

ΘΕΜΑ 2 (ΔΗΜΗΤΡΗΣ ΙΩΑΝΝΟΥ)

Να βρείτε την τιμή της παράστασης: 2n 2011 2n 2012 100 101

1 ( 1) 0,2 5A .

Λύση:

Το 2n είναι σίγουρα άρτιος αριθμός οπότε την πρώτη φορά προσθέτοντας του

το 2011 που είναι περιττός το άθροισμα τους γίνεται και αυτό περιττός

αριθμός, άρα το 2n 2011( 1)

θα γίνει 1 .

Εφόσον το 2n 2011 είναι περιττός το 2n 2012 είναι άρτιος άρα το 2n 2012( 1)

γίνεται 1 . Άρα έχω:

100 100 100 100

101

100 100 100

2 2 5 10A ( 1)(1) 5 1 5 1 5 1 5 4

10 10 10

.

Page 5: ΠΡΟΤΕΙΝΟΜΕΝΕΣ ΑΣΚΗΣΕΙΣ ΓΙΑ ΜΑΘΗΤΙΚΟΥΣ ΔΙΑΓΩΝΙΣΜΟΥΣ_ALL

http://www.mathematica.gr/forum/viewtopic.php?f=109&t=15584

Επιμέλεια: xr.tsif Σελίδα 5

ΘΕΜΑ 3 (ΔΗΜΗΤΡΗΣ ΙΩΑΝΝΟΥ)

Αν x y 2003 , να βρείτε την τιμή της παράστασης:

Α = 6 10x 2 (4x y 3) 1

2003 2(x ) 2y3(x z) 3(y z) 3

.

Λύση:

Θα ξεκινήσω αναλύοντας μόνο του το κλάσμα.

6 10x 8x 2y 6 2(x y) 2

3[(x z) (y z)] 3(x y) 3

. Άρα τώρα έχω:

2 2A 2003 2x 2y 2003 2(x y) 2003 2·2003 2003

3 3 .

ΘΕΜΑ 4 (ΔΗΜΗΤΡΗΣ ΙΩΑΝΝΟΥ)

Δίνονται οι αριθμοί:

998 499

1000

5001 2

A ( 2)2 3

3

2

και n n 1

B 2 3

όπου n άρτιος φυσικός αριθμός. Να συγκριθούν οι αριθμοί n

3 A , B .

Λύση:

501 499 499 499 499 499 501 4993 1 1 2 3 12 3A 2 ·2 ·( ) ·( ) ·( ) ·( ) · 2 ( ) ·

2 2 2 3 2 24 2

501

501 499 2

499

1 3 2 3 32 ( ) · ( )· 2 · 6

2 2 2 2 2 .

n n nB 2 ·3 ·3 6 ·3

Άρα ο αριθμός n n n3A 3·( 6) 3·6 αφού n είναι άρτιος. Άρα A B .

Page 6: ΠΡΟΤΕΙΝΟΜΕΝΕΣ ΑΣΚΗΣΕΙΣ ΓΙΑ ΜΑΘΗΤΙΚΟΥΣ ΔΙΑΓΩΝΙΣΜΟΥΣ_ALL

http://www.mathematica.gr/forum/viewtopic.php?f=109&t=15584

Επιμέλεια: xr.tsif Σελίδα 6

ΘΕΜΑ 5 (ΔΗΜΗΤΡΗΣ ΙΩΑΝΝΟΥ)

Αν n

2

( 2)A

2n

και

n

2

( 2)B

2n 3

όπου n θετικός ακέραιος, να βρεθεί ποιος

από τους αριθμούς A και B είναι μεγαλύτερος.

Λύση:

Αφού τα δυο κλάσματα έχουν τον ίδιο αριθμητή το μεγαλύτερο είναι αυτό

με το μικρότερο παρονομαστή.

Άρα πρέπει να συγκρίνω το 22n και 2

2n 3 .

Το 22n είναι σαφώς μικρότερο άρα A B .

Αυτό ισχύει μόνο όταν το n είναι άρτιος.

Όταν το n είναι περιττός το πρόσημο είναι μείον οπότε μεγαλύτερος είναι ο B .

Το A B ισχύει μόνο για τις απόλυτες τιμές.

ΠΑΡΑΤΗΡΗΣΕΙΣ

(α) Αν ένας αριθμός λήγει σε 0 ή 1 ή 5 ή 6 , τότε κάθε δύναμη που έχει βάση

τον αριθμό αυτό θα λήγει επίσης σε 0 ή 1 ή 5 ή 6αντίστοιχα.

(β) Ένας φυσικός αριθμός που λήγει σε 2 ή 3 ή 7 ή 8 , δεν μπορεί να είναι

τετράγωνος (δηλ. δεν μπορεί να πάρει την μορφή τετραγώνου φυσικού

αριθμού).

ΘΕΜΑ 6 (ΔΗΜΗΤΡΗΣ ΙΩΑΝΝΟΥ)

Αν 7 6 5 4 3 2 1000(8 9·8 9·8 9·8 9·8 9·8 1)α 9·8 και

200 10001024 625β , να συγκρίνετε τους αριθμούς

2α και β .

Page 7: ΠΡΟΤΕΙΝΟΜΕΝΕΣ ΑΣΚΗΣΕΙΣ ΓΙΑ ΜΑΘΗΤΙΚΟΥΣ ΔΙΑΓΩΝΙΣΜΟΥΣ_ALL

http://www.mathematica.gr/forum/viewtopic.php?f=109&t=15584

Επιμέλεια: xr.tsif Σελίδα 7

Λύση:

7 6 5 4 3 2 1000(8 9·8 9·8 9·8 9·8 9·8 9·8 1)α

7 6 5 4 3 2 1000[8 (8 1)·8 (8 1)·8 (8 1)·8 (8 1)·8 (8 1)·8 (8 1)·8 1]

7 7 6 6 5 5 4 4 3 3 2 2 1000(8 8 8 8 8 8 8 8 8 8 8 8 8 1)

1000 1000( 7) 7 .

Άρα 2 2000 2 1000 10007 (7 ) 49α .

200 1000 2000 2000 20001024 625 2 25 50β .

Επομένως 2α b .

ΘΕΜΑ 7 (ΔΗΜΗΤΡΗΣ ΙΩΑΝΝΟΥ)

Να βρείτε το ψηφίο των μονάδων του αριθμού 500α 129 .

Λύση:

Τον αριθμό 500129 μπορούμε να τον γράψουμε ως 2 125

(129 ) . Ο 2129 λήγει σε 1

άρα οποιαδήποτε δύναμη του θα λήγει σε 1 . Άρα ο αριθμός 500

129 λήγει σε 1 .

Β τρόπος

επειδή το 129 τελειώνει σε 9 όταν πολλαπλασιάζουμε τα εννιάρια παίρνουμε 1

αν τα εννιάρια είναι άρτιος αριθμός και 9 αν τα εννιάρια είναι περιττός αριθμός

π.χ.: 9 9 81 , 9 81 729 , 9 729 6561

εδώ το 129 το έχουμε 500ες φορές, άρα ο αριθμός 500

129 τελειώνει σε 1 .

Page 8: ΠΡΟΤΕΙΝΟΜΕΝΕΣ ΑΣΚΗΣΕΙΣ ΓΙΑ ΜΑΘΗΤΙΚΟΥΣ ΔΙΑΓΩΝΙΣΜΟΥΣ_ALL

http://www.mathematica.gr/forum/viewtopic.php?f=109&t=15584

Επιμέλεια: xr.tsif Σελίδα 8

ΘΕΜΑ 8 (ΔΗΜΗΤΡΗΣ ΙΩΑΝΝΟΥ)

Να αποδείξετε ότι ο αριθμός 1 2 3 101 2 3 .A ..10 δεν είναι τέλειο

τετράγωνο (ακεραίου).

Λύση:

Τον αριθμό A μπορώ να τον γράψω και με τη μορφή:

1 2 3 2 2 2 2 2 2 2 2 2 2 2 2 2 2A 1 2 3 4 ·4 5 ·5 ·5 6 ·6 ·6 7 ·7 ·7 ·7 8 ·8 ·8 ·8

2 2 2 2 109 ·9 ·9 ·9 ·9 10 1 4 27 16·16 25·25·5 36·36·36

1049·49·49·7 64·64·64·64 81·81·81·81·9 10

Άρα το τελευταίο ψηφίο του A είναι

τ(A) τ(1 4 7 6 5 6 3 6 9 0) τ(47) 7 .

που σημαίνει ότι σίγουρα δεν είναι τέλειο τετράγωνο κανενός ακέραιου

αριθμού.

ΘΕΜΑ 9 (ΔΗΜΗΤΡΗΣ ΙΩΑΝΝΟΥ)

Να βρεθεί το ψηφίο των μονάδων του αριθμού 101α 597 .

Λύση:

Το ψηφίο των μονάδων του 101

597 είναι ίσο με:

101 101 100 1 2 50 50 50τ(579 ) τ(9 ) τ(9 ·9 ) τ((9 ) ·9) τ(81 ·9) τ(1 ·9) 9 .

ΘΕΜΑ 10 (ΔΗΜΗΤΡΗΣ ΙΩΑΝΝΟΥ)

Να εξετάσετε αν ο αριθμός 100 100

A 7 658 διαιρείται :

(α) με το 2 .

(β) με το 5 .

Page 9: ΠΡΟΤΕΙΝΟΜΕΝΕΣ ΑΣΚΗΣΕΙΣ ΓΙΑ ΜΑΘΗΤΙΚΟΥΣ ΔΙΑΓΩΝΙΣΜΟΥΣ_ALL

http://www.mathematica.gr/forum/viewtopic.php?f=109&t=15584

Επιμέλεια: xr.tsif Σελίδα 9

Λύση:

Ο αριθμός 100 1007 658 λήγει σε:

Ο 1007 μπορεί να γραφτεί ως: 4 25

(7 ) . Το 47 λήγει σε 1 άρα όλος ο αριθμός θα

λήγει σε 1 . Ο αριθμός 100658 μπορεί να γραφτεί ως: 4 25

(658 ) .

το 4658 λήγει σε 6 , άρα όλος ο αριθμός θα λήγει σε 6 .

Τέλος: 6 1 7 άρα όλος αυτός ο αριθμός θα λήγει σε 7 .

Το 7 όμως δε διαιρεί ούτε το 2 ούτε το 5 .

ΘΕΜΑ 11 (ΔΗΜΗΤΡΗΣ ΙΩΑΝΝΟΥ)

Οι ακέραιοι x και y είναι ανάλογοι προς τον αριθμητή και τον παρονομαστή

αντίστοιχα του κλάσματος που προκύπτει από την μετατροπή σε κλασματική

μορφή του δεκαδικού περιοδικού αριθμού α 4,333... .

Να υπολογίσετε την τιμή της παράστασης: 6x 5y 21

Β6x 5y 31

.

Λύση:

134,333...

3 άρα

13 x3x 13y

3 y (1).

6x 5y 21 2·3x 5y 21Β

6x 5y 31 2·3x 5y 31

.

από την (1) συνεπάγεται ότι:

2·13y 5y 21 26y 5y 21 21y 21 21 21Β 0

2·13y 5y 31 26y 5y 31 31y 31 31 31

.

Β τρόπος

ο αριθμός α 4,333... είναι το κλάσμα 13

3 γιατί

Page 10: ΠΡΟΤΕΙΝΟΜΕΝΕΣ ΑΣΚΗΣΕΙΣ ΓΙΑ ΜΑΘΗΤΙΚΟΥΣ ΔΙΑΓΩΝΙΣΜΟΥΣ_ALL

http://www.mathematica.gr/forum/viewtopic.php?f=109&t=15584

Επιμέλεια: xr.tsif Σελίδα 10

1 13α 4,333... 4 0,333... 4

3 3 , άρα αν βάλουμε x 13 και y 3

6x 5y 21 6 13 5 3 21 78 15 21 63 21 3 21 21B

6x 5y 31 6 13 5 3 31 78 15 31 93 31 3 31 31

21 211 0

31 31 .

ΘΕΜΑ 12 (ΣΩΤΗΡΗΣ ΛΟΥΡΙΔΑΣ)

Να υπολογιστεί το ελάχιστο της παράστασης 2 2x 8xy 19y 6y 3 ,

προσδιορίζοντας ταυτόχρονα και τις τιμές των x,y για τις οποίες το έχουμε.

Λύση:

Λοιπόν ας ονομάσουμε A την παράσταση 2 2x 8xy 19y 6y 3 .Έτσι

2 2 2 2 2A x 8xy 19y 6y 3 (x 8xy 16y ) 3y 6y 3

2 2(x 4y) 3(y 2y 1) 2 2

(x 4y) 3(y 1) 0 .

Συνεπώς η ελάχιστη τιμή της παράστασης είναι το 0 και λαμβάνεται όταν

x 4y και όταν y 1 ,δηλαδή όταν x 4,y 1 .

ΘΕΜΑ 13 (ΣΩΤΗΡΗΣ ΛΟΥΡΙΔΑΣ)

Έστω M,N τα μέσα των πλευρών DC,AB ενός τετράπλευρου ABCD.

Υπολογίστε την τιμή της παράστασης:

(AND) (BNC) (ABM)X

(ABM) (AND) (BNC)

(*) όταν έχουμε ευθύγραμμο σχήμα μέσα σε παρένθεση εννοούμε το εμβαδόν

του.

Λύση:

Παίρνουμε τα σημεία G,H,E επί της ευθείας AB που είναι τέτοια ώστε:

DG AB, MH AB, CE AB .

Page 11: ΠΡΟΤΕΙΝΟΜΕΝΕΣ ΑΣΚΗΣΕΙΣ ΓΙΑ ΜΑΘΗΤΙΚΟΥΣ ΔΙΑΓΩΝΙΣΜΟΥΣ_ALL

http://www.mathematica.gr/forum/viewtopic.php?f=109&t=15584

Επιμέλεια: xr.tsif Σελίδα 11

Προφανώς τα τμήματα DG,MH,CE είναι παράλληλα και το τετράπλευρο

GECD είναι τραπέζιο ενώ αφού το M είναι το μέσο της CD ,τότε το H είναι

το μέσο της GE και άρα η HM είναι διάμεσος του τραπεζίου.

Άρα ισχύει: DG

HM2

CE

(1)

και έχουμε:

AN·DG BN·CEAND BNC 2 2

AB·MHABM

2

( AB 2AN,AN NB)AN·DG BN·CE

AB·MH

(1)AN·DG AN·CE DG CE 2MH1

2AN·MH 2MH 2MH

.

Δηλαδή: 1

X 1 1 0 .

ΘΕΜΑ 14 (ΣΩΤΗΡΗΣ ΛΟΥΡΙΔΑΣ)

Έστω a,b,c,d ώστε 2 2 2 24a 13b 13c 9d 12(ab bc cd)

Να αποδειχθεί ότι : a πολ27 .

Λύση:

Όπως και με την άσκηση 12 σχηματίζουμε τέλεια τετράγωνα.

Δηλαδή 2 2 2 24a 13b 13c 9d 12(ab bc cd)

Page 12: ΠΡΟΤΕΙΝΟΜΕΝΕΣ ΑΣΚΗΣΕΙΣ ΓΙΑ ΜΑΘΗΤΙΚΟΥΣ ΔΙΑΓΩΝΙΣΜΟΥΣ_ALL

http://www.mathematica.gr/forum/viewtopic.php?f=109&t=15584

Επιμέλεια: xr.tsif Σελίδα 12

2 2 2(2a 3b) (2b 3c) (2c 3d) 0 2a 3b , 2b 3c , 2c 3d .

Πολλαπλασιάζοντας κατά μέλη τις 3 σχέσεις παίρνουμε 8abc 27bcd και

εφόσον a,b,c,d έχουμε 8a 27d .Όμως (27.8) 1 , έτσι 8 / d και 27 / a

που είναι και το ζητούμενο.

ΘΕΜΑ 15 (ΔΗΜΗΤΡΗΣ ΙΩΑΝΝΟΥ)

Αν x,y,a,b 0 , x 2y , y 2x , a 3b , a 3b και αν ισχύει ότι

2x y 2y xk

a 3b a 3b

, να αποδείξετε ότι: x y 2k a , x y 2k b .

Λύση:

Έχουμε: 2x y 2y x

ka 3b a 3b

Άρα:

2x y k

a 3b 1

(1) και

2y x k

a 3b 1

(2).

Με χιαστί από την (1) παίρνουμε: 2x y ka 3bk .

Με χιαστί από την (2) παίρνουμε: 2y x ka 3bk .

Με πρόσθεση κατά μέλη του (1) και (2) έχουμε: x y 2k a (το ζητούμενο)

Με αφαίρεση κατά μέλη παίρνουμε: 3x 3y 6b k , διαιρούμε και τα δύο

μέλη της ισότητας με το 3 και παίρνουμε: x y 2k b (το ζητούμενο).

Β τρόπος

Θα χρησιμοποιήσουμε την ιδιότητα x z x z x z

y w y w y w

(όπου όλοι οι

παρονομαστές δεν είναι μηδέν)

οπότε έχουμε: 2x y 2y x x y

a 3b a 3b 2a

, άρα

x yk

2a

,

στην τελευταία σχέση κάνουμε μια χιαστί και τελειώσαμε.

Page 13: ΠΡΟΤΕΙΝΟΜΕΝΕΣ ΑΣΚΗΣΕΙΣ ΓΙΑ ΜΑΘΗΤΙΚΟΥΣ ΔΙΑΓΩΝΙΣΜΟΥΣ_ALL

http://www.mathematica.gr/forum/viewtopic.php?f=109&t=15584

Επιμέλεια: xr.tsif Σελίδα 13

Τώρα για να δείξουμε την δεύτερη ισότητα κάνουμε τα ίδια μόνο που το

κλάσμα 2y x

a 3b

το γράφουμε ως

x 2y

3b a

(για να εμφανίσουμε το x y ).

ΠΑΡΑΤΗΡΗΣΕΙΣ

Είναι γνωστό, ότι

για να βρούμε το άθροισμα 1 2 3 ... 100 , παρατηρούμε ότι :

1 100 101 , 2 99 101 , 3 98 101 , ...

συνεπώς, παίρνοντας ανά ζεύγη τους πιο πάνω αριθμούς, (όπου τα ζεύγη είναι

50 στο πλήθος) βρίσκουμε το ζητούμενο άθροισμα ίσο με 50 101 , δηλαδή

5050 .

Γενικά, ας γνωρίζουμε ότι n (n 1)

1 2 3 ... n2

.

Στη συνέχεια, θα δούμε με ποιον τρόπο μπορούμε να βρούμε ένα πλήθος

αριθμών, όταν κάθε ένας από αυτούς (από τον δεύτερο και μετά) είναι

ίσος με τον προηγούμενό του συν ένα σταθερό αριθμό.

ΠΑΡΑΔΕΙΓΜΑ: Να βρεθεί το πλήθος των αριθμών: 4,9,14,...,1499 .

Παρατηρούμε ότι

1ος αριθμός: 4 4 0 5

2ος αριθμός: 9 4 1 5

3ος αριθμός: 14 4 2 5

.....

.....

.....

νιοστός αριθμός: 1499 4 (v 1) 5

Οπότε για να βρούμε το πλήθος v των αριθμών, αρκεί να λύσουμε την εξίσωση

Page 14: ΠΡΟΤΕΙΝΟΜΕΝΕΣ ΑΣΚΗΣΕΙΣ ΓΙΑ ΜΑΘΗΤΙΚΟΥΣ ΔΙΑΓΩΝΙΣΜΟΥΣ_ALL

http://www.mathematica.gr/forum/viewtopic.php?f=109&t=15584

Επιμέλεια: xr.tsif Σελίδα 14

1499 4 (v 1) 5 από όπου βρίσκουμε v 300 .

Ας δούμε και ένα ακόμα παράδειγμα, που θα χρησιμοποιήσουμε την πιο πάνω

γνώση:

Να βρεθεί το άθροισμα: 3 7 11 ... 399 .

Έχουμε:

3 3 0 4

7 3 1 4

11 3 2 4

.....

.....

399 3 (ν 1) 4

Από την τελευταία εξίσωση βρίσκουμε ν 100 .

Με πρόσθεση τώρα κατά μέλη των παραπάνω ισοτήτων βρίσκουμε:

3 7 11 ... 399 100 3 1 4 2 4 ... 99 4

99(99 1)300 4 (1 2 3 ... 99) 300 4

2

300 4 4950 20100 .

ΘΕΜΑ 16 (ΔΗΜΗΤΡΗΣ ΙΩΑΝΝΟΥ)

Ο αριθμός Α προκύπτει από το γινόμενο δύο διαδοχικών θετικών ακεραίων και

είναι μικρότερος του 20 ενώ ο αριθμός Β προκύπτει από το γινόμενο τριών

θετικών διαδοχικών ακεραίων και είναι μικρότερος του 30 . Αν το πηλίκο Α

Β

έχει την ιδιότητα να είναι τέλειο τετράγωνο ακεραίου, να βρεθεί η τιμή της

παράστασης:

1000

2 2AΚ 1000 2004 A 2004 B

B

.

Page 15: ΠΡΟΤΕΙΝΟΜΕΝΕΣ ΑΣΚΗΣΕΙΣ ΓΙΑ ΜΑΘΗΤΙΚΟΥΣ ΔΙΑΓΩΝΙΣΜΟΥΣ_ALL

http://www.mathematica.gr/forum/viewtopic.php?f=109&t=15584

Επιμέλεια: xr.tsif Σελίδα 15

Λύση:

πρέπει A α(α 1) 20 ,B b(b 1)(b 2) 30 .

Το Β είναι προφανώς πολλαπλάσιο του 3 . Αφού Α

ΖΒ , πρέπει Β / Α . Αφού

3 / Β , πρέπει και 3 / Α . Άρα ή 3 / α ή 3 / α 1 .

Επειδή 4 5 20 , α , (α 1) 1,2,3,4

άρα απαραίτητα α 3 , α 1 4 ή α 2 , α 1 3 , οπότε Α 12 ή Α 6 .

Οι διαιρέτες του 12 είναι 1,2,3,4,6,12 .

Οπότε Β 1 2 3 6 ή Β 2 3 4 24 .

Σε καμία από τις δυο περιπτώσεις το Α

Β δεν είναι τέλειο τετράγωνο.

Οπότε Α 6 .

Οι διαιρέτες του 6 είναι 1,2,3,6 .

Οπότε πρέπει Β 1 2 3 6 και άρα Α

1Β .

Και άρα K 1000 2004 2004 1000 .

ΘΕΜΑ 17 (ΔΗΜΗΤΡΗΣ ΙΩΑΝΝΟΥ)

Να προσδιορίσετε το άθροισμα: A 200 198 196 194 ... 4 2 .

Λύση:

A 200 198 196 194 ... 4 2 (200 196 ... 4)

50 50(198 194 ... 2) ( )(200 4) ( )(198 2)

2 2

25 204 25 200 25 4 100 . (αριθμητικές σειρές)

Page 16: ΠΡΟΤΕΙΝΟΜΕΝΕΣ ΑΣΚΗΣΕΙΣ ΓΙΑ ΜΑΘΗΤΙΚΟΥΣ ΔΙΑΓΩΝΙΣΜΟΥΣ_ALL

http://www.mathematica.gr/forum/viewtopic.php?f=109&t=15584

Επιμέλεια: xr.tsif Σελίδα 16

2ος Τρόπος

200A 200 198 196 194 ... 4 2 2 2 2 2 2 .... 2 2 ( ) 100

4

ΘΕΜΑ 18 (ΔΗΜΗΤΡΗΣ ΙΩΑΝΝΟΥ)

Δίνονται οι παραστάσεις: 3 4 5 2001

A 2 ...2 3 4 2000

και

1 1 1 1B 1 ...

2 3 4 2000 . Να βρείτε τον αριθμό: Α Β .

Λύση:

2 1 3 1 4 1 2001 1A B ... 1 1 1 1.... 1 2000

1 2 3 2000

.

ΠΑΡΑΤΗΡΗΣΕΙΣ

Το γινόμενο δύο διαδοχικών φυσικών αριθμών, διαιρείται πάντα με το 2

Η απόδειξη της πρότασης αυτής είναι εύκολη και αφήνεται ως άσκηση (δείτε εξ

άλλου και το επόμενο)

Το γινόμενο τριών διαδοχικών φυσικών αριθμών, διαιρείται πάντα με το

3

Απόδειξη:

Έστωα ν(ν 1)(ν 2) .

Από την ευκλείδεια διαίρεση του ν με τον 3 , έχουμε ν 3κ υ , όπου

υ 0,1,2 . Άρα ν 3κ ή ν 3κ 1 ή ν 3κ 2 .

1η Περίπτωση: ν 3κ

Page 17: ΠΡΟΤΕΙΝΟΜΕΝΕΣ ΑΣΚΗΣΕΙΣ ΓΙΑ ΜΑΘΗΤΙΚΟΥΣ ΔΙΑΓΩΝΙΣΜΟΥΣ_ALL

http://www.mathematica.gr/forum/viewtopic.php?f=109&t=15584

Επιμέλεια: xr.tsif Σελίδα 17

Τότε α 3κ(3κ 1)(3κ 2) και άρα ο α διαιρείται με το 3 .

2η Περίπτωση: ν 3κ 1

Τότε α (3κ 1)(3κ 2)(3κ 3) 3(κ 1)(3κ 1)(3κ 2) και άρα ο

α διαιρείται με το 3 .

3η Περίπτωση: ν 3κ 2

Τότε α (3κ 2)(3κ 3)(3κ 4) 3(κ 1)(3κ 2)(3κ 4) άρα και πάλι ο α

είναι πολλαπλάσιο του 3 .

Γενικά, το γινόμενο ν διαδοχικών φυσικών αριθμών, διαιρείται με το

ν

ΘΕΜΑ 19 (ΔΗΜΗΤΡΗΣ ΙΩΑΝΝΟΥ)

Αν 1 1 1 1

α 1 ...2 3 4 1999

και 2 4 6 3996

b 1 ...4 6 8 3998

, να βρείτε

τον αριθμό α b

2

.

Λύση:

1 1 1 1α 1 ...

2 3 4 1999 ,

2 4 6 3996 1 2 3 1998b 1 ... 1 ...

4 6 8 3998 2 3 4 1999 .

1 1 1 1 2 3 1998α b (1 ... ) (1 ... )

2 3 1999 2 3 4 1999

1 1 1 2 1 19981 (1 ... ) 1 (1 1 1 1 ... 1) 1 1999 2000

2 3 1999

a b1000

2

.

Page 18: ΠΡΟΤΕΙΝΟΜΕΝΕΣ ΑΣΚΗΣΕΙΣ ΓΙΑ ΜΑΘΗΤΙΚΟΥΣ ΔΙΑΓΩΝΙΣΜΟΥΣ_ALL

http://www.mathematica.gr/forum/viewtopic.php?f=109&t=15584

Επιμέλεια: xr.tsif Σελίδα 18

ΘΕΜΑ 20 (ΔΗΜΗΤΡΗΣ ΙΩΑΝΝΟΥ)

Αν για κάθε θετικό ακέραιο n ισχύει η ισότητα: 1 1 1

n (n 1) n n 1

,

να υπολογίσετε το άθροισμα: 1 1 1 1 1

S ...1·2 2·3 3·4 4·5 2000·2001

.

Λύση:

Από την ισότητα 1 1 1

n (n 1) n n 1

μπορούμε να γράψουμε:

1 1 1

1·2 1 2

1 1 1

2·3 2 3

1 1 1

3·4 3 4

....

1 1 1

2000·2001 2000 2001

Από τα παραπάνω έχουμε:

1 1 1 1 1S ...

1·2 2·3 3·4 4·5 2000·2001

1 1 1 1 1 1 1 1 1 1...

1 2 2 3 3 4 4 5 2000 2001

1 1 1 1 1 1 1 1...

1 2 2 3 3 4 2000 2001

Παρατηρούμε πώς γίνονται διαδοχικές απλοποιήσεις και αυτό που μένει είναι:

Page 19: ΠΡΟΤΕΙΝΟΜΕΝΕΣ ΑΣΚΗΣΕΙΣ ΓΙΑ ΜΑΘΗΤΙΚΟΥΣ ΔΙΑΓΩΝΙΣΜΟΥΣ_ALL

http://www.mathematica.gr/forum/viewtopic.php?f=109&t=15584

Επιμέλεια: xr.tsif Σελίδα 19

1 1 2001 1 2000

1 2001 2001 2001 2001 .

ΘΕΜΑ 21 (ΔΗΜΗΤΡΗΣ ΙΩΑΝΝΟΥ)

α) Να αποδείξετε ότι: 2 1 1 1 1

n (n 1) (n 2) n n 1 n 1 n 2

.

β) Να υπολογίσετε το άθροισμα:

1 1 1 1S ...

1 2 3 2 3 4 3 4 5 1999 2000 2001

.

Λύση:

α) 2 A B C

n(n 1)(n 2) n n 1 n 2

(μερικά κλάσματα)

με απαλοιφή παρονομαστών:

2 A(n 2)(n 1) B(n 2)n C(n 1)n

Μηδενίζοντας τα n,(n 1),(n 2) αντίστοιχα έχουμε

n 0 2 2A 0B 0C 2A A 1 .

n 1 2 0A ( 1)B 0C B B 2 .

n 2 2 0A 0B 2C 2C C 1 .

Οπότε

2 1 2 1 1 1 1 1

n(n 1)(n 2) n n 1 n 2 n n 1 n 1 n 2

. Οεδ.

Β τρόπος

1 1 1 1 1 1 2

n n 1 n 1 n 2 n(n 1) (n 1)(n 2) n(n 1)(n 2)

Page 20: ΠΡΟΤΕΙΝΟΜΕΝΕΣ ΑΣΚΗΣΕΙΣ ΓΙΑ ΜΑΘΗΤΙΚΟΥΣ ΔΙΑΓΩΝΙΣΜΟΥΣ_ALL

http://www.mathematica.gr/forum/viewtopic.php?f=109&t=15584

Επιμέλεια: xr.tsif Σελίδα 20

(το τελευταίο προκύπτει κάνοντας τα κλάσματα ομώνυμα).

β) 1 1 1 1

S ...1 2 3 2 3 4 3 4 5 1999 2000 2001

2 2 2 2( ... )1 2 3 2 3 4 3 4 5 1999 2000 2001

2

1 1 1 1 1 1 1 1 1 1 1(1 ) ( ) ( ) ( ) ... ( ) ( )

2 2 3 2 3 3 4 1999 2000 2000 2001

2

μετά από τις διαδοχικές απλοποιήσεις:

1 1 1 1 2001 2000(1 ) ( )

2 2000 2001 2 2000 2001S2 2

1 1

1 1 1000 2001 1 20009992 2000 2001

2 4 4000 2001 4000 2001 4000 2001

.

ΘΕΜΑ 22 (ΔΗΜΗΤΡΗΣ ΙΩΑΝΝΟΥ)

Να εξετάσετε αν ο παρακάτω αριθμός είναι θετικός ή αρνητικός:

A ( 13) ( 17) ( 21) ( 25) ... ( 4013) .

Λύση:

Οι αριθμοί από το 13 έως το 4013 γράφονται ως εξής:

13 13 0 4

17 13 1 4

21 13 2 4

.

Page 21: ΠΡΟΤΕΙΝΟΜΕΝΕΣ ΑΣΚΗΣΕΙΣ ΓΙΑ ΜΑΘΗΤΙΚΟΥΣ ΔΙΑΓΩΝΙΣΜΟΥΣ_ALL

http://www.mathematica.gr/forum/viewtopic.php?f=109&t=15584

Επιμέλεια: xr.tsif Σελίδα 21

.

.

4013 13 x 4

Λύνοντας την εξίσωση βρίσκουμε ότι το x 1000 . Άρα ο 4013 είναι ο

χιλιοστός πρώτος αριθμός (και το 13 στην αρχή το προσθέτουμε).Το 1001 είναι

περιττός άρα το αποτέλεσμα της παράστασης θα είναι αρνητικός αριθμός.

ΘΕΜΑ 23 (ΔΗΜΗΤΡΗΣ ΙΩΑΝΝΟΥ)

α) Να αποδείξετε ότι αν το τετράγωνο ενός θετικού ακεραίου αριθμού είναι

άρτιος, τότε και ο αριθμός αυτός είναι άρτιος.

β) Ο ακέραιος a δεν διαιρείται με το 5 και ο αριθμός 2a 2a 3 είναι άρτιος.

Να βρείτε το ψηφίο των μονάδων του a .

Λύση:

α) n(2a 1) 2m 1 . Οπότε κάθε δύναμη περιττού είναι περιττός αριθμός.

Οπότε, αν nx άρτιος, από τον νόμο της αντιθετοαντιστροφής έχουμε ότι x είναι

άρτιος.

β) 2 2 2a 2a 3 a 2a 1 2 (a 1) a 1 0(mod2) .

Άρα πρέπει a 1(mod2) . Οπότε το a είναι περιττός, το ίδιο και το τελευταίο

του ψηφίο. Άμεση συνέπεια της εκφώνησης είναι ότι το τελευταίο ψηφίο είναι

διάφορο του 5 , και άρα τ(a) 1,3,7,9 .

Β τρόπος

α) Έχουμε λοιπόν ότι το τετράγωνο του a είναι άρτιος και θέλουμε να δείξουμε

ότι ο a είναι και αυτός άρτιος.

Page 22: ΠΡΟΤΕΙΝΟΜΕΝΕΣ ΑΣΚΗΣΕΙΣ ΓΙΑ ΜΑΘΗΤΙΚΟΥΣ ΔΙΑΓΩΝΙΣΜΟΥΣ_ALL

http://www.mathematica.gr/forum/viewtopic.php?f=109&t=15584

Επιμέλεια: xr.tsif Σελίδα 22

Θα εφαρμόσουμε την μέθοδο της εις άτοπον απαγωγής . Δηλαδή θα

υποθέσουμε ότι ο a δεν είναι άρτιος και θα καταλήξουμε σε άτοπο (δηλαδή

σε κάτι που δεν είναι αλήθεια)

Αν λοιπόν ο a δεν είναι άρτιος, τότε θα είναι περιττός. Άρα a 2n 1 (με n

φυσικό αριθμό). Τότε

2 2 2 2(2n 1) 4n 4n 1 2(2na 2) 1

και αυτό σημαίνει ότι ο 2a είναι περιττός, πράγμα που είναι άτοπο. Άρα ο a

είναι υποχρεωμένος να είναι άρτιος.

β) Έχουμε ότι ο αριθμός 2a 2a 3 είναι άρτιος. Στο μεταξύ ο a θα είναι

άρτιος ή περιττός. Αν υποθέσουμε ότι είναι άρτιος τότε ο 2a 2a θα είναι

άρτιος (ως άθροισμα άρτιων) και ο 2a 2a 3 θα είναι περιττός (ως άθροισμα

άρτιου και περιττού). Τούτο όμως είναι άτοπο. Άρα ο a είναι περιττός. Και

αφού δεν διαιρείται με το 5 θα λήγει σε 1 ή 3 ή 7 ή 9 .

ΘΕΜΑ 24 (ΔΗΜΗΤΡΗΣ ΙΩΑΝΝΟΥ)

α) Αν 1 1 1

x y 3 με x,y 0 να αποδείξετε ότι y 3 και

9x 3

y 3

.

β) Να προσδιορίσετε τους θετικούς ακέραιους x,y που ικανοποιούν τη σχέση

1 1 1

x y 3 .

Λύση:

α) Αν y 3 πρέπει 1

0x , άτοπο.

Κάνοντας τα κλάσματα ομώνυμα προκύπτει ότι 3y 3x xy .

Γράφεται 3y xy 3x x(y 3)

Page 23: ΠΡΟΤΕΙΝΟΜΕΝΕΣ ΑΣΚΗΣΕΙΣ ΓΙΑ ΜΑΘΗΤΙΚΟΥΣ ΔΙΑΓΩΝΙΣΜΟΥΣ_ALL

http://www.mathematica.gr/forum/viewtopic.php?f=109&t=15584

Επιμέλεια: xr.tsif Σελίδα 23

3y 3y 9 9 3y 9 9 9x 3

y 3 y 3 y 3 y 3 y 3

.

β) Αφού ικανοποιούν την αρχική σχέση, πρέπει να ικανοποιούν και την

9x 3

y 3

.

Για να είναι το αριστερό μέλος ακέραιο, πρέπει να είναι και το δεξί. Για να είναι

το δεξί πρέπει να είναι και το 9

y 3. Άρα πρέπει y 3 / 9

Οπότε y 3 1 y 4 ή y 3 3 y 6 ή y 3 9 y 12 .

Έχουμε τα εξής ζεύγη λύσεων: (12,4) , (6,6) , (4,12) .

ΘΕΜΑ 25 (ΔΗΜΗΤΡΗΣ ΙΩΑΝΝΟΥ)

Δίνεται η παράσταση: 2 2 2P(x) (α b) x 4(α b) x c 4 , όπου a,b,c

είναι ακέραιοι με a,b 0 και c 0 . Αν η παράσταση αυτή παίρνει την τιμή 0

για x 1 , να βρεθούν οι αριθμοί a,b,c .

Λύση:

έχουμε ότι 2 2 2 20 (a b) 4(a b)x c 4 (a b 2) c

είναι γνωστό ότι τα τετράγωνα είναι είτε μεγαλύτερα είτε ίσα του 0 . Οπότε το

άθροισμα τετραγώνων είναι 0 , αν και μόνο αν κάθε τετράγωνο είναι ίσο με

μηδέν.

Oπότε a b 2 καιc 0 ,

και αφού a,b θετικοί ακέραιοι,a b 1 .

Page 24: ΠΡΟΤΕΙΝΟΜΕΝΕΣ ΑΣΚΗΣΕΙΣ ΓΙΑ ΜΑΘΗΤΙΚΟΥΣ ΔΙΑΓΩΝΙΣΜΟΥΣ_ALL

http://www.mathematica.gr/forum/viewtopic.php?f=109&t=15584

Επιμέλεια: xr.tsif Σελίδα 24

ΘΕΜΑ 26 (ΔΗΜΗΤΡΗΣ ΙΩΑΝΝΟΥ)

Πόσοι από τους αριθμούς 1,2,3,...,1999 δεν διαιρούνται με το 5 ούτε με το 7 ;

Λύση:

Ο αριθμός των πολλ. του 5 που είναι μικρότερα του 1999 είναι το ακέραιο

μέρος της 1999

5, που είναι: 399 .

Αντίστοιχα, τα πολλ. του 7 που είναι μικρότερα του 1999 είναι: 285 .

Αφού [5,7] 35 , οπότε οι αριθμοί που συμπεριλαμβάνονται και στις δυο πιο

πάνω περιπτώσεις είναι με όμοιο τρόπο: 57 .

Οπότε, συνολικά οι αριθμοί που διαιρούνται είτε με το 5 είτε με το 7 είναι :

684 57 627 .

Και αυτοί που δεν διαιρούνται με κανέναν από τους δυο είναι :

1999 627 1372 .

ΑΝΑΛΥΤΙΚΗ ΕΞΗΓΗΣΗ

Τα πολλαπλάσια του 5 είναι 1 5,2 5,3 5,.... . Το ζήτημα είναι να βρούμε πιο

πολλαπλάσιο του 5 είναι αμέσως πριν το 1999 . Κάνουμε λοιπόν την διαίρεση

1999 δια το 5 και βρίσκουμε πηλίκο 399 και υπόλοιπο 4 . Αυτό μας δείχνει ότι

το τελευταίο πολλαπλάσιο του 5 αμέσως πριν το 1999 είναι το 399 5 .

Άρα όλα τα πολλαπλάσια του 5 που βρίσκονται ανάμεσα στο 51 και στο 1999

είναι 399 στο πλήθος.

Όμοια βρίσκουμε ότι όλα τα πολλαπλάσια του 7 που βρίσκονται ανάμεσα στο 1

και στο 1999 είναι 285 .

Page 25: ΠΡΟΤΕΙΝΟΜΕΝΕΣ ΑΣΚΗΣΕΙΣ ΓΙΑ ΜΑΘΗΤΙΚΟΥΣ ΔΙΑΓΩΝΙΣΜΟΥΣ_ALL

http://www.mathematica.gr/forum/viewtopic.php?f=109&t=15584

Επιμέλεια: xr.tsif Σελίδα 25

Φαίνεται λοιπόν ότι όλα τα πολλαπλάσια του 5 και του 7 που είναι ανάμεσα

στους αριθμούς 1 και 1999 είναι 399 285 δηλαδή 684 . Όμως είναι λιγότερα.

Γιατί μέσα στα πολλαπλάσια του 5 βρίσκονται και μερικά πολλαπλάσια του 7 .

Συγκεκριμένα, τα πολλαπλάσια του 7 που είναι μέσα στα πολλαπλάσια του 5

είναι: 1 35,2 35,3 35,.... και όπως πριν , βρίσκουμε ότι το τελευταίο

πολλαπλάσιο του 35 , αμέσως πριν το 1999 είναι το 57 35 .

Άρα τα πολλαπλάσια του 7 που βρίσκονται μέσα στα πολλαπλάσια του 5 είναι

57 στο πλήθος. Αυτά, πρέπει να τα αφαιρέσουμε από το 684 (γιατί τα έχουμε

πάρει δύο φορές). Άρα τελικά τα πολλαπλάσια του 5 και του 7 που είναι

ανάμεσα στους αριθμούς 1 και 1999 είναι 684 57 δηλαδή 627 στο πλήθος.

Οπότε οι αριθμοί που ζητάμε είναι 1999 627 1372 (τόσοι δηλ. είναι οι

αριθμοί που δεν είναι πολλαπλάσια του 5 ούτε του 7 ).

ΘΕΜΑ 27 (ΔΗΜΗΤΡΗΣ ΙΩΑΝΝΟΥ)

Ο θετικός ακέραιος x είναι άρτιος και όταν διαιρείται με το 7 δίνει υπόλοιπο

2 . Να βρεθεί ο x αν είναι μεταξύ των αριθμών 512 και 521 .

Λύση:

Οι άρτιοι αριθμοί ανάμεσα από 512 και 521 είναι ο 514,516,518 και 520 . Ο

μόνος αριθμός που διαιρείται με το 7 είναι το 518 από τους ανάμεσα από τους

αριθμούς 512 και τον 521 και το 2 που είναι το υπόλοιπο μας κάνει 520 , αν

διαιρέσουμε το 514 με το 7 θα έχουμε 73 και 3 υπόλοιπο άρα μας λείπουν

ακόμη 4 για να διαιρείται με το 7 και έτσι φθάνουμε στον αριθμό 518 και 2 το

υπόλοιπο 520 .

ΘΕΜΑ 28 (ΔΗΜΗΤΡΗΣ ΙΩΑΝΝΟΥ)

Οι δύο διαστάσεις ενός ορθογωνίου είναι οι θετικοί ακέραιοι x και y . Αν

αυξήσουμε την μία διάσταση κατά 1 και την άλλη κατά 2 τότε το ορθογώνιο

που προκύπτει έχει εμβαδόν διπλάσιο του αρχικού. Να βρεθούν οι διαστάσεις x

και y .

Page 26: ΠΡΟΤΕΙΝΟΜΕΝΕΣ ΑΣΚΗΣΕΙΣ ΓΙΑ ΜΑΘΗΤΙΚΟΥΣ ΔΙΑΓΩΝΙΣΜΟΥΣ_ALL

http://www.mathematica.gr/forum/viewtopic.php?f=109&t=15584

Επιμέλεια: xr.tsif Σελίδα 26

Λύση:

Πρέπει

(x 1)(y 2) 2xy xy 2x y 2 2xy y 2 x(y 2) .

Πρέπει y 2 / y 2 . Επειδή (y 2) (y 2) 4 , πρέπει y 2 / 4 ,

οπότε y 2 1 y 3 ή y 2 2 y 4 ή y 2 4 y 6 .

Με αντικατάσταση, προκύπτουν τα ζεύγη (5,3) , (3,4) , (2,6) .

ΘΕΜΑ 29 (ΔΗΜΗΤΡΗΣ ΙΩΑΝΝΟΥ)

Να αποδειχθεί ότι ο αριθμός α = 2

222223 444441 222220 222216a

222222

είναι

ακέραιος και να βρεθεί.

Λύση:

Έχουμε ότι : 2

222223 444441 222220 222216a

222222

επειδή παντού υπάρχει το 222222 , έμμεσα ή άμεσα, θα ονομάσουμε αυτόν τον

αριθμό x . Άρα x 222222 .

Οι αριθμοί του αριθμητή θα γίνουν:

222223 x 1

444441 2x 3

222220 x 2

222216 x 6

Page 27: ΠΡΟΤΕΙΝΟΜΕΝΕΣ ΑΣΚΗΣΕΙΣ ΓΙΑ ΜΑΘΗΤΙΚΟΥΣ ΔΙΑΓΩΝΙΣΜΟΥΣ_ALL

http://www.mathematica.gr/forum/viewtopic.php?f=109&t=15584

Επιμέλεια: xr.tsif Σελίδα 27

Το αποτέλεσμα είναι: 2

(x 1)·(2x 3)·(x 2) (x 6)

x

2

2

(2x 3x 2x 3)(x 2) x 6

x

3 2 2 2

2

2x 4x 3x 6x 2x 4x 3x 6 x 6

x

3 4 2 2

2 2

2x 4x x x (2x 5)2x 5 (2·222222) 5 444439

x x

.

ΘΕΜΑ 30 (ΔΗΜΗΤΡΗΣ ΙΩΑΝΝΟΥ)

Να αποδειχθεί ότι ο αριθμός 2

333334 666663 333331 333327b

333333

είναι

ακέραιος και να βρεθεί.

Λύση:

Θέτουμε όπου n 333333 . Έτσι έχουμε:

2

333334·666663·333331 333327

333333

2

(n 1)(2n 3)(n 2) (n 6)

n

2

2

(2n 3n 2n 3)(n 2) (n 6)

n

3 2 2 2

2

2n 4n 3n 6n 2n 4n 3n 6 n 6

n

3 2 2

2

2n 7n 2n 2n 3n n

n

3 2 2

2 2

2n 5n n (2n 5)2n 5 (2·333333) 5 666661

n n

.

Page 28: ΠΡΟΤΕΙΝΟΜΕΝΕΣ ΑΣΚΗΣΕΙΣ ΓΙΑ ΜΑΘΗΤΙΚΟΥΣ ΔΙΑΓΩΝΙΣΜΟΥΣ_ALL

http://www.mathematica.gr/forum/viewtopic.php?f=109&t=15584

Επιμέλεια: xr.tsif Σελίδα 28

ΘΕΜΑ 31 (ΔΗΜΗΤΡΗΣ ΙΩΑΝΝΟΥ)

Τριψήφιος αριθμός είναι μεγαλύτερος του 610 και μικρότερος του 650 και

διαιρούμενος με το 7 δίνει υπόλοιπο 3 . Να βρεθεί ο αριθμός αυτός, αν είναι

γνωστό ότι είναι πολλαπλάσιο του 5 .

Λύση:

Τα πολλαπλάσια του 7 μεταξύ των 610 και 650 είναι τα :

616,623,630,637,644 . Άρα ο τριψήφιος αριθμός που ψάχνουμε είναι κάποιος

από τους παραπάνω προσθέτοντας το 3 .

Αλλά μόνο ο αριθμός 637 3 640 διαιρείται με το 5 . Άρα ο ζητούμενος

αριθμός είναι ο 640 .

Β τρόπος

Αφού ο αριθμός διαιρείται με το 7 και δίνει υπόλοιπο 3 , θα γράφεται στην

μορφή : a 7k 3 , k Z . Άρα θα έχουμε

607 647610 a 650 610 7k 3 650 607 7k 647 k

7 7

86,71 k 92,42k 87,88,89,90,91,92

k Z

.

άρα ο a 612,619,626,633,640,647 και αφού είναι πολλαπλάσιο του 5 , τελικά

a 640 .

ΘΕΜΑ 32 (ΔΗΜΗΤΡΗΣ ΙΩΑΝΝΟΥ)

Να αποδειχθεί ότι ο αριθμός 2 2 2 2 2 2

A 1998 1997 1996 1995 ... 2 1

είναι πολλαπλάσιο του 1999 .

[ Μια σπουδαία ισότητα (ταυτότητα) είναι η ακόλουθη (που λέγεται διαφορά

τετραγώνων) 2 2

x y (x y)(x y) ].

Page 29: ΠΡΟΤΕΙΝΟΜΕΝΕΣ ΑΣΚΗΣΕΙΣ ΓΙΑ ΜΑΘΗΤΙΚΟΥΣ ΔΙΑΓΩΝΙΣΜΟΥΣ_ALL

http://www.mathematica.gr/forum/viewtopic.php?f=109&t=15584

Επιμέλεια: xr.tsif Σελίδα 29

Λύση:

Με τη βοήθεια της ταυτότητας έχουμε:

2 2•1998 1997 (1998 1997)(1998 1997) 1998 1997

2 2•1996 1995 (1996 1995)(1996 1995) 1996 1995

2 2• 2 1 (2 1)(2 1) 2 1

Άρα:

2 2 2 2 2 2A 1998 1997 1996 1995 ... 2 1

1998 1997 1996 1995 ... 2 1

(1998 1) (1997 2) (1996 3) ... (1000 999)

1999·999 πoλ.1999 .

ΘΕΜΑ 33 (ΔΗΜΗΤΡΗΣ ΙΩΑΝΝΟΥ)

Αν ο αριθμός n είναι θετικός ακέραιος, να δείξετε ότι ο αριθμός

1

A 11

11

1n

δεν είναι ποτέ ακέραιος.

Page 30: ΠΡΟΤΕΙΝΟΜΕΝΕΣ ΑΣΚΗΣΕΙΣ ΓΙΑ ΜΑΘΗΤΙΚΟΥΣ ΔΙΑΓΩΝΙΣΜΟΥΣ_ALL

http://www.mathematica.gr/forum/viewtopic.php?f=109&t=15584

Επιμέλεια: xr.tsif Σελίδα 30

Λύση:

Έστω ότι ο A είναι ακέραιος: Πρέπει και ο 1

11

11

n

να είναι ακέραιος. Οπότε

απαραίτητα, η απόλυτη τιμή του 1

1 11

1n

. Αλλά 1

01

1n

, επειδή n

θετικός, και άρα 1

1 11

1n

. Άτοπο. Οπότε ο A δεν μπορεί να είναι ακέραιος.

ΘΕΜΑ 34 (ΣΩΚΡΑΤΗΣ ΛΥΡΑΣ)

Να προσδιορίσετε τις τιμές του θετικού ακέραιου n για τις οποίες ο αριθμός 3 2

A n n n 1 είναι πρώτος. ( Αρχιμήδης juniors)

Λύση:

3 2 2 2A n n n 1 n (n 1) (n 1) (n 1)(n 1)

Οπότε n 2 και A 5 .

Θεωρία:

Ο πρώτος αριθμός διαιρείται μόνο με τον αριθμό 1 και με τον εαυτό του.

Σημείωση: Υπενθυμίζουμε τις ταυτότητες: 2 2 2

(x y) x 2xy y και 2 2 2

(x y) x 2xy y 3 3 2 2 3 3 3

(x y) x 3x y 3xy y x y 3xy(x y) και 3 3 2 2 3 3 3

(x y) x 3x y 3xy y x y 3xy(x y)

Δύο επίσης χρήσιμες ταυτότητες είναι και οι εξής: 3 3 2 2

x y (x y)(x xy y ) (άθροισμα κύβων) και η 3 3 2 2

x y (x y)(x xy y ) (διαφορά κύβων)

Page 31: ΠΡΟΤΕΙΝΟΜΕΝΕΣ ΑΣΚΗΣΕΙΣ ΓΙΑ ΜΑΘΗΤΙΚΟΥΣ ΔΙΑΓΩΝΙΣΜΟΥΣ_ALL

http://www.mathematica.gr/forum/viewtopic.php?f=109&t=15584

Επιμέλεια: xr.tsif Σελίδα 31

Επίσης η 2 2 2x y (x y) 2xy και 2 2 2 2

(x y z) x y z 2xy 2yz 2xz .

ΘΕΜΑ 35 (ΔΗΜΗΤΡΗΣ ΙΩΑΝΝΟΥ)

α) Να παραγοντοποιηθεί η παράσταση: 4 4x 4y .

β) Αν x,y θετικοί ακέραιοι και y 2 να δείξετε ότι ο αριθμός 4 4x 4y είναι

σύνθετος (δηλαδή δεν είναι πρώτος).

Λύση:

Για το πρώτο ερώτημα:

4 4 2 2 2 2x 4y (x 2xy 2y )(x 2xy 2y )

Για το δεύτερο ερώτημα:

Είναι άμεση απόρροια του πρώτου ερωτήματος, αφού η παράσταση γράφεται

ως γινόμενο, άρα ο αριθμός είναι σύνθετος.

Β τρόπος

α) Έχουμε:

4 4 2 2 2 2 2 2 2 2 2 2 2 2x 4y (x ) (2y ) (x ) (2y ) 2 x (2y ) 2 x (2y )

2 2 2 2 2 2 2 2(x 2y ) (2xy) (x 2y 2xy)(x y 2xy)

β) Πρέπει να δείξουμε ότι 2 2 2 2x 2y 2xy 1 , x 2y 2xy 1

Πράγματι, έχουμε: 2 2 2 2 2 2 2x 2y 2xy x y y 2xy (x y) y ,

Όμως αφού είναι 2 2 2 2y 2 y 4 (x y) y (x y) 4 4 1 .

Άρα 2 2(x y) y 1 .

Όμοια δείχνουμε ότι και ο άλλος παράγοντας είναι διάφορος της μονάδας,

οπότε έχουμε ότι ο αριθμός 4 4

x 4y είναι σύνθετος.

Page 32: ΠΡΟΤΕΙΝΟΜΕΝΕΣ ΑΣΚΗΣΕΙΣ ΓΙΑ ΜΑΘΗΤΙΚΟΥΣ ΔΙΑΓΩΝΙΣΜΟΥΣ_ALL

http://www.mathematica.gr/forum/viewtopic.php?f=109&t=15584

Επιμέλεια: xr.tsif Σελίδα 32

ΘΕΜΑ 36 (ΔΗΜΗΤΡΗΣ ΙΩΑΝΝΟΥ)

Αν a και x είναι πραγματικοί αριθμοί και a 1 να δείξετε ότι 2

2

x a2

x a 1

.

Πότε ισχύει η ισότητα;

Λύση:

Όπως είδαμε παραπάνω, αν θέσουμε 2y x a , αρκεί να δείξουμε ότι

y y2 2 0

y 1 y 1

.

Έχουμε 2

y 2 y 2 ( y ) 2 y 1 1 1y2

y 1 y 1 y 1

2( y 1) 1

y 1

.

Αφού τώρα 2 2a 1 x a x 1 1 για κάθε x διάφορο του μηδενός.

Άρα y 1 δηλαδή y 1 0 . Και αφού 2( y 1) 1 0 θα πρέπει:

2( y 1) 1

0y 1

οπότε

y2 0

y 1

και άρα

y2

y 1

.

Έχουμε μια εκκρεμότητα, για το τι συμβαίνει αν είναι x 0 . Τότε έχουμε y a

οπότε αρκεί να δείξουμε ότι α

α2

1

. Πρέπει να είναι το a 1 (για να μην

μηδενίζεται ο παρονομαστής). Δηλαδή πρέπει να είναι μόνοa 1 και άρα

a 1 a 1 0 .

Οπότε έχουμε : α

α2

1

(δικαιολογήστε το γιατί)

Άρα και στην περίπτωση αυτή, έχουμε το ζητούμενο.

ΠΑΡΑΤΗΡΗΣΗ: Η ισότητα, όπως είδαμε πιο πάνω, δεν μπορεί να ισχύει ποτέ.

Page 33: ΠΡΟΤΕΙΝΟΜΕΝΕΣ ΑΣΚΗΣΕΙΣ ΓΙΑ ΜΑΘΗΤΙΚΟΥΣ ΔΙΑΓΩΝΙΣΜΟΥΣ_ALL

http://www.mathematica.gr/forum/viewtopic.php?f=109&t=15584

Επιμέλεια: xr.tsif Σελίδα 33

ΘΕΜΑ 37 (ΔΗΜΗΤΡΗΣ ΙΩΑΝΝΟΥ)

Αν για τους πραγματικούς αριθμούς x,a,b,y ισχύει ότι xy ab 1 , να

αποδειχθεί ότι: 2 2 2 2a b x y ax by 1 .

Λύση:

Η προς απόδειξη ανισότητα γράφεται με αντικατάσταση όπου xy ab 1

2 2 2 2a b x y ax by xy ab 0

2 2 2 22a 2ab 2b 2x 2xy 2y 2ax 2by 0 .

Tώρα σχηματίζουμε ταυτότητες της μορφής: 2 2 2(α β) α 2αβ β

2 2 2 2 2 2 2 2... a 2ab b x 2xy y a 2ax x b 2by y

2 2 2 2(a b) (x y) (a x) (b y) 0 .

Όμως ως αθροίσματα τετραγώνων ισχύει : 2 2 2 2(a b) (x y) (a x) (b y) 0 . Η ισότητα ισχύει αν και μόνον αν

a b,x y ,a x,b y

Τώρα λύνοντας το σύστημα

a b 0

x y

a x 0

b y 0

προκύπτει a b x y 0

Όμως κάτι τέτοιο δεν μπορεί να συμβαίνει καθώς δεν θα ικανοποιείται η

δοσμένη σχέση: xy ab 1 .

Συνεπώς η ισότητα δεν μπορεί να ισχύει άρα 2 2 2 2(a b) (x y) (a x) (b y) 0 και το ζητούμενο δείχθηκε.

Page 34: ΠΡΟΤΕΙΝΟΜΕΝΕΣ ΑΣΚΗΣΕΙΣ ΓΙΑ ΜΑΘΗΤΙΚΟΥΣ ΔΙΑΓΩΝΙΣΜΟΥΣ_ALL

http://www.mathematica.gr/forum/viewtopic.php?f=109&t=15584

Επιμέλεια: xr.tsif Σελίδα 34

ΘΕΜΑ 38 (ΔΗΜΗΤΡΗΣ ΙΩΑΝΝΟΥ)

Για όλους τους θετικούς πραγματικούς αριθμούς x,y να αποδείξετε ότι: 3 3

2 2

x yx y

x xy y

.

Λύση:

Έχουμε ότι: 3 3 2 2

2 2 2 2

x y (x y)(x xy y )

x xy y x xy y

και το

2 2

2 2

(x y)(x xy y )x y

x xy y

Οπότε η ζητούμενη ανισότητα γράφεται

2 2 2 2

2 2 2 2

(x y)(x xy y ) (x y)(x xy y )

x xy y x xy y

ίσοι αριθμητές, άρα αρκεί να δείξουμε ότι για κάθε x,y θετικούς

2 2 2 2x xy y x xy y , ή ακόμα ότι xy xy . Πράγμα που προφανώς

ισχύει, με την ισότητα στο x 0 ή y 0 (η πιθανότητα να ισχύουν και τα δυο

απορρίπτεται.)

ΠΑΡΑΤΗΡΗΣΗ

Να τονίσουμε επί πλέον ότι οι αριθμοί:

2 2 2 2,x xy y x xy y είναι μη αρνητικοί για κάθε x,y πραγματικούς

αριθμούς (ενώ είναι μηδέν μόνο για x y 0 ).

Παρακάτω θα δούμε την απόδειξη της ανισότητας: 2 2x xy y 0 για κάθε

x,y .

Έχουμε:2 2 2 2 2 2

2 2 2x 2xy 2y x x 2xy y yx xy y 0

2 2

Page 35: ΠΡΟΤΕΙΝΟΜΕΝΕΣ ΑΣΚΗΣΕΙΣ ΓΙΑ ΜΑΘΗΤΙΚΟΥΣ ΔΙΑΓΩΝΙΣΜΟΥΣ_ALL

http://www.mathematica.gr/forum/viewtopic.php?f=109&t=15584

Επιμέλεια: xr.tsif Σελίδα 35

(δώστε μόνοι την εξήγηση) για κάθε x,y (όπου για να ισχύει η ισότητα, θα

πρέπει να είναι x 0 και x y 0 και y 0 , δηλαδή x y 0 ).

Επειδή λοιπόν στην εκφώνηση της άσκησης μας έδιναν ότι τα x,y είναι θετικοί

πραγματικοί αριθμοί, είναι φανερό ότι οι αριθμοί 2 2 2 2,x xy y x xy y

είναι θετικοί.

ΘΕΜΑ 39 (ΔΗΜΗΤΡΗΣ ΙΩΑΝΝΟΥ)

Να παραγοντοποιηθεί η παράσταση: 2 2 3A (2 x x ) x .

Λύση:

2 2 3 2 2 3(2 x x ) x [1 (1 x x )] x 2 2 2 3

1 2(1 x x ) (1 x x ) x

2 2 3 2(1 x x )(3 x x ) (1 x ) (1 x x )

2 2(3 x x ) (1 x)(1 x x ) 2 2

(1 x x )(x 4) .

ΘΕΜΑ 40 (ΔΗΜΗΤΡΗΣ ΙΩΑΝΝΟΥ)

(α) Να αποδείξετε ότι για κάθε θετικό ακέραιο n ισχύει ότι:

22n n

2n 1 n 1

.

(β) Να αποδείξετε ότι:

22 4 6 2000 1

...3 5 7 2001 2001

.

Λύση:

α) πρέπει 2

2

4n n

4n 4n 1 n 1

Απλοποιούμε τα n των αριθμητών. Επειδή n θετικός, πρέπει να ισχύει

2 2 24n(n 1) 4n 4n 1 4n 4n 4n 4n 1 0 1 , που ισχύει.

Page 36: ΠΡΟΤΕΙΝΟΜΕΝΕΣ ΑΣΚΗΣΕΙΣ ΓΙΑ ΜΑΘΗΤΙΚΟΥΣ ΔΙΑΓΩΝΙΣΜΟΥΣ_ALL

http://www.mathematica.gr/forum/viewtopic.php?f=109&t=15584

Επιμέλεια: xr.tsif Σελίδα 36

Άρα ισχύει και η αρχική ανισότητα.

β) 22 1( )3 2

24 2( )5 3

26 3( )7 4

...

22000 1000( )2001 1001

Με πολλαπλασιασμό προκύπτει ότι

22 4 6 2000 1 2 3 1000 1( ... ) ...3 5 7 2001 2 3 4 1001 1001 (με διαδοχικές απλοποιήσεις).

ΘΕΜΑ 41 (komi)

Εάν a,b πραγματικοί μη μηδενικοί και 2 2 3 3 3 2(a b ) (a b ) να βρεθεί η τιμή

της παράστασης a b

b a .

Λύση:

Η σχέση 2 2 3 3 3 2(a b ) (a b ) γράφεται

6 4 2 2 4 6 6 3 3 6 4 2 2 4 3 3a 3a b 3a b b a 2a b b 3a b 3a b 2a b

2 2 2 2

2 2 2 2 3 3

3 3

a b (a b ) 23a b (a b ) 2a b

a b 3

2 2a b 2 b a 2

ab 3 a b 3

.

Page 37: ΠΡΟΤΕΙΝΟΜΕΝΕΣ ΑΣΚΗΣΕΙΣ ΓΙΑ ΜΑΘΗΤΙΚΟΥΣ ΔΙΑΓΩΝΙΣΜΟΥΣ_ALL

http://www.mathematica.gr/forum/viewtopic.php?f=109&t=15584

Επιμέλεια: xr.tsif Σελίδα 37

ΘΕΜΑ 42 (komi)

Εάν a,b,c ακέραιοι πραγματικοί , να δείξετε ότι η παράσταση 2

A 4abc(b c a) [(b a)(c a)] είναι τέλειο τετράγωνο.

Λύση:

2 2 2 2 2 2A 4abc(b c a) [(b a)(c a)] 4ab c 4abc 4a bc [bc a(b c) a ]

2 2 2 2 2 2 2 3 3 2

4ab c 4abc 4a bc b c 2ab c 2abc 2a b 2a c 2a bc 2 2 2 2 2 2 2 2 2 2 4 2 2 3

4ab c 4abc 4a bc b c a b 2a bc c a a 2ab 2ac 2a b 3 2

2a c 2a bc 2 2 2[c(b a)] 2ac(b a)(b a) [a(b a)] [(c a)(b a)] .

ΘΕΜΑ 43 (ΔΗΜΗΤΡΗΣ ΙΩΑΝΝΟΥ)

Το άθροισμα δύο ακεραίων αριθμών είναι 26 ενώ αν διαιρέσουμε τον

μεγαλύτερο με τον μικρότερο βρίσκουμε πηλίκο 4 και υπόλοιπο 1 . Να βρεθούν

οι αριθμοί.

Λύση:

Ισχύει ότι a b 26 (1)

Χωρίς βλάβη της γενικότητας υποθέτουμε ότι b a .

b 4a 1 (2)

από (1) και (2) με σύστημα a 5 , b 21 .

ΘΕΜΑ 44 (ΔΗΜΗΤΡΗΣ ΙΩΑΝΝΟΥ)

Αν a είναι περιττός ακέραιος, να δείξετε ότι ο αριθμός 4 2

a 6a 7 είναι

πολλαπλάσιο του 128 .

Page 38: ΠΡΟΤΕΙΝΟΜΕΝΕΣ ΑΣΚΗΣΕΙΣ ΓΙΑ ΜΑΘΗΤΙΚΟΥΣ ΔΙΑΓΩΝΙΣΜΟΥΣ_ALL

http://www.mathematica.gr/forum/viewtopic.php?f=109&t=15584

Επιμέλεια: xr.tsif Σελίδα 38

Λύση:

4 2 2 2a 6a 7 (a 7)(a 1) .

περιττά τετραγωνικά υπόλοιπα modulo 16 είναι μόνο τα 1,9 .

Αν το υπόλοιπο του 2a (mod16) είναι1 , τότε

ο πρώτος παράγοντας γίνεται της μορφής

16k 1 7 16k 8 8(2k 1) mult.8 , και ο δεύτερος της μορφής

16k 1 1 16k mult.16 . Οπότε

4 2 2 2a 6a 7 (a 7)(a 1) (mult.8)(mult.16) mult.128

Αν το υπόλοιπο του 2a (mod16) είναι 9 , τότε

ο πρώτος παράγοντας γίνεται της μορφής

16k 9 7 16k 16 16(k) mult.16 , και ο δεύτερος της μορφής

16k 9 1 16k 8 8(2k 1) mult.8 . Οπότε

4 2 2 2a 6a 7 (a 7)(a 1) (mult.16)(mult.8) mult.128

Σημείωση : mult.16 σημαίνει πολ16 .

Β τρόπος

Έχουμε:

4 2 4 2 4 2 2 2 2a 6a 7 a 6a 6 1 (a 1) 6(a 1) (a 1)(a 1) 6(a 1)

2 2 2(a 1)(a 1 6) (a 1)(a 1)(a 7), (*)

Αφού όμως ο a είναι περιττός φυσικός αριθμός, θα έχουμε a 2k 1 , με k

φυσικό αριθμό. Οπότε η σχέση (*) γράφεται:

2 2(*) (2k 1 1)(2k 1 1)[(2k 1) 7] 2k(2k 2)(4k 4k 1 7)

2 22k 2(k 1)(4k 4k 8) 4k(k 1) 4(k k 2)

16k(k 1)[k(k 1) 2], (**)

Page 39: ΠΡΟΤΕΙΝΟΜΕΝΕΣ ΑΣΚΗΣΕΙΣ ΓΙΑ ΜΑΘΗΤΙΚΟΥΣ ΔΙΑΓΩΝΙΣΜΟΥΣ_ALL

http://www.mathematica.gr/forum/viewtopic.php?f=109&t=15584

Επιμέλεια: xr.tsif Σελίδα 39

Γνωρίζουμε όμως (βλέπε κάποια από τις προηγούμενες ασκήσεις)

ότι το γινόμενο δύο διαδοχικών φυσικών αριθμών διαιρείται πάντα με το 2

(όπως και το γινόμενο τριών διαδοχικών φυσικών αριθμών διαιρείται πάντα με

το 3 , κ.ο.κ)

Άρα k(k 1) 2m όπου m είναι φυσικός αριθμός. Άρα η σχέση (**)

γράφεται: (**) 16 2m (2m 2) 64m(m 1), (***)

Και πάλι θα είναι m(m 1) 2n , όπου n είναι φυσικός αριθμός. Άρα η σχέση

(***) γράφεται: (***) 64 2n 128 n ,

από όπου προκύπτει το ζητούμενο.

Γ τρόπος 4 2 2 2

α 6α 7 α 6 7(α )

το τετράγωνο περιττού ακεραίου αριθμού είναι της μορφής 8λ 1

2 2(8λ 1)(8λ 1 6) 7 64λ 56λ 8λ 7 7 64λ 64λ 64λ λ 1

Άρα είναι πολ64 και εφόσον το λ παίρνει τιμές 1,2,3,... είναι και πολ128 .

ΘΕΜΑ 45 (ΛΩΛΑΣ ΠΑΝΑΓΙΩΤΗΣ)

Αν a,b,c πλευρές τριγώνου και x,y,z πραγματικοί, νa δείξετε ότι: 2 2 2

a (x y)(x z) b (y z)(y x) c (z x)(z y) 0 .

Πότε ισχύει η ισότητα;

Λύση:

Προτού δώσουμε μια λύση, είναι χρήσιμο να αποδείξουμε μια ανισότητα (που

καλό είναι να απομνημονευθεί).

Αν κ,μ πραγματικοί αριθμοί και λ, ν θετικοί, τότε ισχύει:

Page 40: ΠΡΟΤΕΙΝΟΜΕΝΕΣ ΑΣΚΗΣΕΙΣ ΓΙΑ ΜΑΘΗΤΙΚΟΥΣ ΔΙΑΓΩΝΙΣΜΟΥΣ_ALL

http://www.mathematica.gr/forum/viewtopic.php?f=109&t=15584

Επιμέλεια: xr.tsif Σελίδα 40

2 2 2κ μ (κ μ)

λ ν λ ν

.

ΑΠΟΔΕΙΞΗ:

Επειδή τα λ, ν είναι θετικοί αριθμοί, άρα το ΕΚΠ των παρονομαστών που είναι

λ ν (λ ν) θα είναι θετικός αριθμός. Οπότε μπορούμε να πολλαπλασιάσουμε

τα μέλη της ανισότητας που ζητάμε να αποδείξουμε με το ΕΚΠ και έτσι

ισοδύναμα, αρκεί να αποδείξουμε ότι:

2 2 2ν(λ ν)κ λ(λ ν)μ λν(κ μ)

2 2 2 2 2 2 2 2νλκ ν κ λ μ λνμ λνκ 2λνκμ λνμ

2 2 2 2 2ν κ λ μ 2λνκμ 0 (νκ λμ) 0 ,

πράγμα που είναι αληθές. Άρα αληθές είναι και το ζητούμενο.

Τώρα , θα οδηγήσουμε την άσκηση 45, στην προηγούμενη ανισότητα (που ήδη

αποδείξαμε)

Αρχικά, παρατηρούμε ότι αν ήταν x y , τότε αρκεί να δείξουμε ότι

2 2c (z x) 0, , που προφανώς αληθεύει (και μάλιστα η ισότητα ισχύει όταν

x z , δηλαδή τελικά όταν x y z ).

Με τον ίδιο τρόπο καταλήγουμε στο ζητούμενο και όταν είναι x z ή y z .

Ας υποθέσουμε τώρα ότι τα x,y,z δεν είναι ίσα ανά δύο, αλλά άνισα. και έστω

ότι x y z .

Τότε είναι (x y)(y z)(x z) 0 .

Διαιρώντας τώρα τα μέλη της ανισότητας που ζητάμε να αποδείξουμε με το

(x y)(y z)(x z) 0 , έχουμε ισοδύναμα:

2 2 2 2 2 2a b c a c b

0 (1)y z x z x y y z x y x z

Page 41: ΠΡΟΤΕΙΝΟΜΕΝΕΣ ΑΣΚΗΣΕΙΣ ΓΙΑ ΜΑΘΗΤΙΚΟΥΣ ΔΙΑΓΩΝΙΣΜΟΥΣ_ALL

http://www.mathematica.gr/forum/viewtopic.php?f=109&t=15584

Επιμέλεια: xr.tsif Σελίδα 41

Όμως , από την (βοηθητική) ανισότητα που αποδείξαμε στην αρχή, έχουμε ότι:

2 2 2 2a c (a c) b

y z x y x z x z

(χρησιμοποιήσαμε την τριγωνική ανισότητα)

Άρα η σχέση (1) αποδείχθηκε, οπότε αποδείχθηκε και η αρχικά ζητούμενη.

Ακριβώς με τον ίδιο τρόπο, μπορούμε να αποδείξουμε το ζητούμενο και στις

περιπτώσεις που είναι x z y , y x z , κλπ...

Β τρόπος

Θέτω: x y k  ,y z και η προς απόδειξη ανισότητα γράφεται:

2 2 2 2 2 2 2 2 2 2a k(k ) b ( k) c ( k )( ) a k a k b k c k c

2 2 2 2 2 2 2a k (a b c )k c 0

το οποίο είναι τριώνυμο ως προς k . Αρκεί λοιπόν να είναι D 0

Έχουμε: 2 2 2 2 2 2 2 2 2 2 2 2

D (a b c ) (2ac ) (a 2ac c b )(a 2ac c b ) όμως

από τριγωνική ανισότητα έχουμε:

2 2 2a c b a 2ac c b 0 και:

2 2 2 2 2 2b c a b a 2ac c a 2ac c b 0 και η απόδειξη

ολοκληρώθηκε.

ΠΑΡΑΤΗΡΗΣΕΙΣ

(α)Τριγωνική ανισότητα: Κάθε πλευρά τριγώνου είναι μικρότερη από το

άθροισμα των δύο άλλων πλευρών (και μεγαλύτερη από την απόλυτη τιμή της

διαφοράς των άλλων πλευρών).

(β) Αν κ,μ πραγματικοί αριθμοί και λ, ν θετικοί, τότε ισχύει η ανισότητα:

Page 42: ΠΡΟΤΕΙΝΟΜΕΝΕΣ ΑΣΚΗΣΕΙΣ ΓΙΑ ΜΑΘΗΤΙΚΟΥΣ ΔΙΑΓΩΝΙΣΜΟΥΣ_ALL

http://www.mathematica.gr/forum/viewtopic.php?f=109&t=15584

Επιμέλεια: xr.tsif Σελίδα 42

2 2 2κ μ (κ μ)

λ ν λ ν

.

(την απόδειξη, την αφήνω ως άσκηση μιας και δεν έχει ιδιαίτερη δυσκολία)

(γ) Παρατηρείστε ότι αν x y ή y z ή z x τότε το ζητούμενο

αποδεικνύεται εύκολα. Θεωρείστε στη συνέχεια ότι x y z και προσπαθήστε

να οδηγήσετε την άσκηση στην παρατήρηση (β) η οποία θα αποδειχθεί με την

παρατήρηση (α).

δ) αν ένα πολυώνυμο της μορφής 2f (x) ax bx c , a,b,c πραγματικοί, έχει

a 0 και αρνητική Διακρίνουσα τότε f (x) 0 για κάθε πραγματικό x .

ΘΕΜΑ 46 (ΔΗΜΗΤΡΗΣ ΙΩΑΝΝΟΥ)

Να παραγοντοποιηθεί η παράσταση: 2 2 3A [1 x(1 x x )] x

Λύση:

(Νομίζω ότι πρέπει να δοθεί μια αρχή για την άσκηση αυτή, ώστε να μπορεί να

αντιμετωπιστεί και από μη έμπειρους μαθητές Γυμνασίου:

Ξεκινάμε λοιπόν: 2 2 3A [1 x(1 x x )] x

στη συνέχεια αναπτύξτε το τετράγωνο αθροίσματος και κάποια στιγμή

θυμηθείτε το άθροισμα κύβων...).

Δηλαδή 2 3 3 2 2 3 2

(1 x x x ) x [1 x(1 x x )] x 1 2x(1 x x ) 2 2 2 3 2 2 3 4 3

x (1 x x ) x (1 x x )(2x x x x ) (x 1) 2 2 3 4 2

(1 x x )(2x x x x ) (x 1)(x x 1)

2 2 3 4(1 x x )(1 3x x x x ) .

Page 43: ΠΡΟΤΕΙΝΟΜΕΝΕΣ ΑΣΚΗΣΕΙΣ ΓΙΑ ΜΑΘΗΤΙΚΟΥΣ ΔΙΑΓΩΝΙΣΜΟΥΣ_ALL

http://www.mathematica.gr/forum/viewtopic.php?f=109&t=15584

Επιμέλεια: xr.tsif Σελίδα 43

ΘΕΜΑ 47 (ΔΗΜΗΤΡΗΣ ΙΩΑΝΝΟΥ)

Δίνεται το πολυώνυμο 2 2 2 2

P(x,y,z) x yz 3x y 2x z 6x 11xyz 22xz 33xy 66x

α) Να γράψετε το πολυώνυμο αυτό ως γινόμενο πρωτοβάθμιων παραγόντων

β) Για ποιες τριάδες φυσικών αριθμών (x,y,z) ισχύει ότι P(x,y,z) 2002 ;

Λύση:

α) 2 2 2 2P(x,y,z) x yz 3x y 2x z 6x 11xyz 22xz 33xy 66x

2 2x (yz 3y 2z 6) 11x(yz 3y 2z 6) (x 11x)(yz 3y 2z 6)

x(x 11)[y(z 3) 2(z 3)] x(x 11)(y 2)(z 3)

β) To 2002 γράφεται: 2002 2 7 11 13

Αν x άρτιος x 11 περιττός. ή το αντίστροφο.

επειδή ο 2 είναι ο μικρότερος από τους παράγοντες, x 2 και x 11 13 .

ή y 2 7 y 5 και z 3 11 z 8

ή y 2 11 y 9 και z 3 7 z 4 .

Οπότε οι λύσεις είναι (2,5,8) και (2,9,4) .

ΘΕΜΑ 48 (ΣΩΚΡΑΤΗΣ ΛΥΡΑΣ)

Αν ο αριθμός p είναι πρώτος να βρεθούν οι φυσικοί αριθμοί για τους οποίους

ισχύει: 2n n p 1982 .

Λύση:

2n n p 1982 n(n 1) p 1982 p 1982 2m

Page 44: ΠΡΟΤΕΙΝΟΜΕΝΕΣ ΑΣΚΗΣΕΙΣ ΓΙΑ ΜΑΘΗΤΙΚΟΥΣ ΔΙΑΓΩΝΙΣΜΟΥΣ_ALL

http://www.mathematica.gr/forum/viewtopic.php?f=109&t=15584

Επιμέλεια: xr.tsif Σελίδα 44

p2(991 m) p 2

991 m

.

Η μόνη τιμή που μπορεί να πάρει το 991 m είναι το 1 . Επομένως p 2 . Στη

συνέχεια από την εξίσωση n(n 1) 1980 με δοκιμές βρίσκω ότι n 44 .

Β τρόπος

o 2n n n(n 1) είναι άρτιος, επειδή είναι γινόμενο διαδοχικών.

αν p περιττός πρώτος, 2n n p περιττός, άτοπο. Άρα p 2 .

Λύνουμε την 2n n 1980 0 , και βρίσκουμε ως μόνη θετική λύση την

n 44 .

ΘΕΜΑ 49 (Socrates)

Χωρίζουμε το σύνολο A {1,2,3,...,19} σε δύο μη κενά σύνολα M και N έτσι

ώστε M N , M N A και για κάθε x M είναι x 10 M ή

x 10 M . Αν m το άθροισμα των στοιχείων του M και n το άθροισμα των

στοιχείων του N , να βρείτε την ελάχιστη τιμή του m n .

Λύση:

Αν 1 M το 11 M και αν 11 M τότε το 1 M , όμοια ισχύει και για τα

υπόλοιπα πιθανά στοιχεία. Άρα μπορούμε να ομαδοποιήσουμε τα πιθανά

στοιχεία του M στο σύνολο X 10,12,14,16,18,20,22,24,26,28 , όπου κάθε

στοιχείο αντιστοιχεί στο άθροισμα δυο αντιστοίχων του A (εκτός του 10 που

είναι 10 2 0 ) δηλαδή το m ισούται με το άθροισμα των αντίστοιχων

στοιχείων του M στο X . Άμεση συνέπεια είναι ότι ισχύει το ίδιο και για το n ,

το οποίο είναι στην ουσία το άθροισμα όσων στοιχείων του X περισσέψουν.

Είναι προφανές ότι τα n και m είναι άρτιοι. Το άθροισμα των στοιχείων του X

είναι 190 , επειδή 190

852

, m n 0 . Οπότε η αμέσως μικρότερη τιμή που

ψάχνουμε είναι το 2 . Παρατηρούμε ότι οι όροι στο X είναι διαδοχικοί

αριθμητικής προόδου. Οπότε

Page 45: ΠΡΟΤΕΙΝΟΜΕΝΕΣ ΑΣΚΗΣΕΙΣ ΓΙΑ ΜΑΘΗΤΙΚΟΥΣ ΔΙΑΓΩΝΙΣΜΟΥΣ_ALL

http://www.mathematica.gr/forum/viewtopic.php?f=109&t=15584

Επιμέλεια: xr.tsif Σελίδα 45

10 28 12 26 14 24 16 22 18 20 38 . Βάζοντας τα πρώτα δυο

ζεύγη στο M , τα δυο αμέσως επόμενα στο N και τα δυο τελευταία ένα στο

καθένα, έχουμε m n 2 που είναι και η απάντηση.

ΘΕΜΑ 50 (Socrates)

α) Να δείξετε ότι μπορούμε να τοποθετήσουμε 20 μη μηδενικούς ακεραίους, όχι

απαραίτητα διαφορετικούς, στη σειρά έτσι ώστε το άθροισμά τους να είναι

θετικό, ενώ το άθροισμα οποιονδήποτε τριών διαδοχικών να είναι αρνητικό.

β) Δείξτε ότι δε μπορούμε να κάνουμε το ίδιο σε ένα κύκλο.

Λύση:

Βάζω μια απόδειξη μιας και έχει μείνει άλυτη αρκετό καιρό. Θα προσπαθήσω

να γράψω κάπως αναλυτικά το σκεπτικό.

Για το (α) θέλουμε να βρούμε το παράδειγμα χωρίς να μπλέξουμε σε πράξεις.

Θα προσπαθήσουμε λοιπόν όσο το δυνατόν περισσότεροι αριθμοί να είναι ίσοι.

Μια ιδέα είναι να δοκιμάσουμε τους ακεραίους

m,m,n,m,m,n, ,m,m,n,m,m . Δηλαδή όλοι οι ακέραιοι που είναι σε θέσεις

πολλαπλάσια του τρία να ισούνται με n και όλοι οι υπόλοιποι να ισούνται με

m . Με αυτήν την επιλογή, κάθε τριάδα διαδοχικών ακεραίων έχει άθροισμα

2m n . Το συνολικό άθροισμα ισούται με 6n (20 6)m . Για να δουλέψει

λοιπόν αυτή η μέθοδος θέλουμε να βρούμε n,m ώστε 2m n 0 αλλά

14m 6n 0 . Για την πρώτη ανισότητα συμφέρει να επιλέξουμε

n (2m 1) . Τότε για να ικανοποιείται η δεύτερη ανισότητα πρέπει να

βρούμε m ώστε 14m 6(2m 1) 0 . Παρατηρούμε τώρα ότι το m 4 (και

άρα n 9 ) δουλεύει.

Μπορούμε τώρα να γράψουμε την λύση και πιο σύντομα. Παίρνουμε τους

ακεραίους 4,4, 9,4,4, 9, ,4,4, 9,4,4 . Το άθροισμα οποιονδήποτε τριών

διαδοχικών ισούται με 4 4 9 1 0 ενώ το ολικό άθροισμα ισούται με

14·4 6·9 2 0 .

Page 46: ΠΡΟΤΕΙΝΟΜΕΝΕΣ ΑΣΚΗΣΕΙΣ ΓΙΑ ΜΑΘΗΤΙΚΟΥΣ ΔΙΑΓΩΝΙΣΜΟΥΣ_ALL

http://www.mathematica.gr/forum/viewtopic.php?f=109&t=15584

Επιμέλεια: xr.tsif Σελίδα 46

ΘΕΜΑ 51 (Socrates)

Α) Να προσδιορίσετε τους φυσικούς αριθμούς n,m αν οι αριθμοί 42m 3 και

42n 5 είναι δίδυμοι πρώτοι, δηλαδή είναι πρώτοι αριθμοί που διαφέρουν

κατά 2 .

Β) Ποιο μπορεί να είναι το τελευταίο ψηφίο του 2a ; Ποιο του 4

a ;

Λύση:

Α) είναι 4m 1(mod5) ή 4

m 0(mod5)

Στην πρώτη περίπτωση: αν 4m 1(mod5) οπότε 4

2m 0(m 5)3 od

Οπότε πρέπει 432m 5 , m 1

άρα 42n 5 7 ή 4

2n 5 3 .

Η δεύτερη απορρίπτεται, και στην πρώτη έχουμε n 1 .

Στην δεύτερη περίπτωση: 42m 3(m 5)3 od

Αλλά επειδή ο δεύτερος ακέραιος είναι κατά 2 μεγαλύτερος ή μικρότερος από

τον πρώτο, έχουμε 42n 0(m 5)5 od ή 4

2n 1(m 5)5 od

Στην πρώτη υποπερίπτωση, πρέπει 4n n 052 5 , απορρίπτεται.

Στην δεύτερη υποπερίπτωση, 42n 1(m 5)5 od , αλλά επειδή

4n 1 ή 0(mod5) πρέπει 4

2n 2 ή 0(mod5) , άτοπο.

Μοναδική λύση η m n 1 .

Β) Το τετράγωνο ενός ακεραίου τελειώνει πάντα σε 0,1,4,5,6,9 οπότε

υψωμένο πάλι στο τετράγωνο 2 2 4(α ) α έχει ως πιθανά τελευταία ψηφία το

0,1,5,6 . Άρα:

4τ(n ) (0,1,5,6)

4τ(2n ) (0,2)

Page 47: ΠΡΟΤΕΙΝΟΜΕΝΕΣ ΑΣΚΗΣΕΙΣ ΓΙΑ ΜΑΘΗΤΙΚΟΥΣ ΔΙΑΓΩΝΙΣΜΟΥΣ_ALL

http://www.mathematica.gr/forum/viewtopic.php?f=109&t=15584

Επιμέλεια: xr.tsif Σελίδα 47

4τ(2n 5) (5,7)

4τ(m ) (0,1,5,6)

4τ(2m ) (0,2)

4τ(2m 3) (3,5)

Για να σχηματίσω την διαφορά των δυο παραστάσεων που είναι 2 (δηλ. το

τελευταίο ψηφίο της είναι 2 ) εξετάζω όλα τις πιθανές περιπτώσεις των

παραπάνω τελευταίων ψηφίων και καταλήγω ότι οι πιθανές περιπτώσεις είναι

δυο.

1η περίπτωση

τ(5) τ(3) 2 άρα 4τ(2n 5) 5 πρέπει αναγκαστικά το 4

2n 5 5 καθώς ο

μόνος πρώτος αριθμός που τελειώνει σε 5 είναι το 5 . Άρα 42n 5 5 άρα

n 0 που είναι άτοπο καθώς ο n είναι φυσικός

2η περίπτωση

τ(7) τ(5) 2 άρα 4τ(2m 3) 5 όπως και παραπάνω πρέπει αναγκαστικά το

4m m 132 5

Αφού το 42m 53 το

4n n 152 7 . Άρα m n 1 .

Οπότε ως συμπέρασμα έχουμε ότι ο μόνος πρώτος αριθμός που τελειώνει σε 5

είναι το 5 .

Να γράψω μόνο για τον συμβολισμό π.χ 4τ(n ) (0,1,5,6) διαβάζεται:

"Το τελευταίο ψηφίο του 4

n είναι 0 ή 1 ή 5 ή 6".

ΠΑΡΑΤΗΡΗΣΕΙΣ

Θα πρέπει να εξηγήσουμε το γιατί κάθε φυσικός αριθμός όταν υψωθεί εις την

τετάρτη και ύστερα διαιρεθεί με το 5, αφήνει πάντα υπόλοιπο ή μηδέν ή 1 (μου

ζητήθηκε να το εξηγήσουμε από κάποιους μαθητές γυμνασίου).

Page 48: ΠΡΟΤΕΙΝΟΜΕΝΕΣ ΑΣΚΗΣΕΙΣ ΓΙΑ ΜΑΘΗΤΙΚΟΥΣ ΔΙΑΓΩΝΙΣΜΟΥΣ_ALL

http://www.mathematica.gr/forum/viewtopic.php?f=109&t=15584

Επιμέλεια: xr.tsif Σελίδα 48

Έστω λοιπόν ένας φυσικός αριθμός m. Αν κάνουμε την διαίρεση του m με το 5

θα βρούμε πηλίκο τον φυσικό αριθμό k και υπόλοιπο τον αριθμό u όπου το u

παίρνει τις τιμές 0 ή 1 ή 2 ή 3 ή 4.

Επειδή "Ο διαιρετέος ισούται με τον διαιρέτη επί το πηλίκο συν το υπόλοιπο",

θα έχουμε: m 5k u .

Διακρίνουμε τώρα τις περιπτώσεις:

1η περίπτωση: u 0

Τότε m 5k . Άρα 4 4m 625k πολ5 .

Αυτό σημαίνει ότι η διαίρεση του 4m με το 5 δίνει υπόλοιπο 0 .

2η περίπτωση: u 1

Τότε m 5k 1

Άρα 2 2 2m 25k 10k 1 5(5k 2k) 1 πολ5 1 .

Αυτό σημαίνει ότι η διαίρεση του 2m με το 5 δίνει υπόλοιπο 1 .

Άρα

2 4 2 4 2 2m 5a 1 m (5a 1) m 25a 10a 1 5(5a 2a) 1 πολ5 1

Αυτό σημαίνει ότι η διαίρεση του 4m με το 5 δίνει υπόλοιπο 1 .

3η περίπτωση: u 2

Τότε m 5k 2

Άρα 2 2 2m 25k 20k 4 5(5k 4k) 4

Αυτό σημαίνει ότι το υπόλοιπο της διαίρεσης του 2

m με το 5 δίνει υπόλοιπο 4 .

Άρα 2 4 2 2 2

m 5b 4 m 25b 40b 16 25b 40b 15 1 5(5b 8b 3) 1

Αυτό πάλι σημαίνει ότι το υπόλοιπο της διαίρεσης του 4

m με το 5 δίνει

υπόλοιπο 1 .

Page 49: ΠΡΟΤΕΙΝΟΜΕΝΕΣ ΑΣΚΗΣΕΙΣ ΓΙΑ ΜΑΘΗΤΙΚΟΥΣ ΔΙΑΓΩΝΙΣΜΟΥΣ_ALL

http://www.mathematica.gr/forum/viewtopic.php?f=109&t=15584

Επιμέλεια: xr.tsif Σελίδα 49

Όμοια και στις υπόλοιπες δύο περιπτώσεις , καταλήγουμε ότι το υπόλοιπο της

διαίρεσης του 4m με το 5 είναι το1 .

ΘΕΜΑ 52 (Socrates)

Αν 1 2 n

x ,x ,...,x ακέραιοι τέτοιοι ώστε

1 2 n 1 2 n( x x ... x ) x x ... x 2 , να δείξετε ότι ένας τουλάχιστον

από αυτούς ισούται με 1 ή 1 .

Λύση:

Αν όλοι οι ακέραιοι αριθμοί (που μας έχει δώσει η εκφώνηση) ήταν θετικοί,

τότε θα ήταν και 1 2 n

x x ... x 0 .

Άρα θα είχαμε ότι: 1 2 n 1 2 nx x ... x x x ... x 2 0 2 .

πράγμα άτοπο.

Αν πάλι όλοι ήταν αρνητικοί, τότε θα ήταν και 1 2 n

x x ... x 0 .

Άρα θα είχαμε ότι: 1 2 n 1 2 nx x ... x x x ... x 2 0 2 που

και πάλι είναι άτοπο.

Άρα κάποιοι από τους 1 2 n

x ,x ,...,x θα είναι θετικοί ή μηδέν και κάποιοι

αρνητικοί.

Έστω λοιπόν ότι οι ακέραιοι 1 2 n

x ,x ,...,x είναι θετικοί ή μηδέν και οι ακέραιοι

k 1 k 2 nx ,x ,...,x

είναι αρνητικοί (δεν βλάπτει την γενικότητα αυτό που

θεωρήσαμε)

Τότε η δομένη σχέση γράφεται:

1 2 k k 1 k 2 n 1 2 k k 1 k 2 nx x ... x x x ... x x x ... x x x ... x 2

1 2 k k 1 k 2 nx x ... x x x ... x

1 2 k k 1 k 2 nx x ... x x x ... x 2

.

Page 50: ΠΡΟΤΕΙΝΟΜΕΝΕΣ ΑΣΚΗΣΕΙΣ ΓΙΑ ΜΑΘΗΤΙΚΟΥΣ ΔΙΑΓΩΝΙΣΜΟΥΣ_ALL

http://www.mathematica.gr/forum/viewtopic.php?f=109&t=15584

Επιμέλεια: xr.tsif Σελίδα 50

Για ευκολία, ας θέσουμε 1 2 k

x x ... x F 0 , k 1 k 2 n

x x ... x R 0

Τότε η σχέση (*) γράφεται: F R F R 2 (1).

Διακρίνουμε τώρα 3 περιπτώσεις:

1η περίπτωση: F R 0 .

Τότε από την σχέση (1) έχουμε F R 2 και με πρόσθεση κατά μέλη αυτών

των σχέσεων βρίσκουμε 2F 2 F 1 , οπότε προκύπτει ότι R 1 .

Άρα k 1 k 2 n

x x ... x 1 και αφού οι αριθμοί αυτοί είναι ακέραιοι και

αρνητικοί , θα πρέπει τουλάχιστον ένας από αυτούς να ισούται με 1 (αλλιώς

το άθροισμά τους θα ήταν μικρότερο του 1 , πράγμα άτοπο)

2η περίπτωση: F R 0 .

Τότε από την σχέση (1) έχουμε: F R (F R) 2 2R 2 R 1 ,

οπότε πάλι όπως και στην 1η περίπτωση, έχουμε ότι ένας τουλάχιστον από τους

ακέραιους ισούται με το 1 .

3η περίπτωση: F R 0 .

Τότε από την σχέση (1) έχουμε: F R [ (F R)] 2 2F 2 F 1 .

Δηλαδή 1 2 k

x x ... x 1 και αφού οι ακέραιοι αυτοί είναι θετικοί ή μηδέν,

τότε αν όλοι ήταν διάφοροι της μονάδας, το άθροισμά τους θα ήταν ή μηδέν ή

μεγαλύτερο του 2 , πράγμα άτοπο. Άρα ένας τουλάχιστον από αυτούς θα

ισούται με την μονάδα.

Από τις παραπάνω περιπτώσεις, συμπεραίνουμε ζητούμενο.

Ας δούμε μία παραλλαγή της λύσης:

Μπορούμε να υποθέσουμε ότι 1 2 n

x x ... x 0 (αλλιώς αλλάζουμε το

πρόσημο κάθε αριθμού. Η παράσταση τότε δεν αλλάζει τιμή). Άρα

1 2 n 1 2 n2 x x ... x x x ... x

1 2 n 1 2 nx x ... x x x ... x

Page 51: ΠΡΟΤΕΙΝΟΜΕΝΕΣ ΑΣΚΗΣΕΙΣ ΓΙΑ ΜΑΘΗΤΙΚΟΥΣ ΔΙΑΓΩΝΙΣΜΟΥΣ_ALL

http://www.mathematica.gr/forum/viewtopic.php?f=109&t=15584

Επιμέλεια: xr.tsif Σελίδα 51

1 1 2 2 n n| x | x | x | x ... | x | x

παρατηρούμε ότι όλες οι παρενθέσεις είναι θετικοί αριθμοί ή μηδέν. Επίσης, αν

κάποιος k

x είναι 0 τότε k k

| x | x 0 . Αυτούς τους σβήνουμε. Δεν αλλάζει το

άθροισμα. Συνεπώς έχουμε μόνο αρνητικά x(m) και το άθροισμά μας είναι της

μορφής: 2 2x(m) ... 2x(p) δηλαδή x(m) ... x(p) 1 .

Επειδή όμως οι x(m),...,x(p) είναι (αρνητικοί) ακέραιοι σημαίνει ότι ένας από

αυτούς είναι 1 (και οι υπόλοιποι 0). Όπως θέλαμε.

(η περίπτωση του 1 προκύπτει αν αρχικά έχουμε 1 2 n

x x ... x 0 ).

ΘΕΜΑ 53 (ΣΩΚΡΑΤΗΣ ΛΥΡΑΣ)

Ένας σκύλος καταδιώκει μια αλεπού που απέχει εξήντα πηδήματα από αυτόν.

Όταν η αλεπού κάνει εννέα πηδήματα, ο σκύλος κάνει έξι πηδήματα. Αλλά τρία

πηδήματα του σκύλου ισούνται με επτά της αλεπούς. Μετά από πόσα πηδήματα

θα φτάσει ο σκύλος την αλεπού?

Λύση:

Έστω y τα πηδήματα τα οποία θα φτάσει ο σκύλος την αλεπού. Αλλά τότε η

αλεπού θα έχει κάνει 9

60 y6

πηδηματάκια.

Αφού τα 7 πηδήματα της αλεπούς είναι 3 του σκύλου, τότε 1 της αλεπούς είναι

3

7 του σκύλου.

Και άρα τα 9

60 y6

της αλεπούς είναι 3 9

(60 y)7 6 του σκύλου.

Συνεπώς 3 9

(60 y) y y 727 6 πηδηματάκια.

Page 52: ΠΡΟΤΕΙΝΟΜΕΝΕΣ ΑΣΚΗΣΕΙΣ ΓΙΑ ΜΑΘΗΤΙΚΟΥΣ ΔΙΑΓΩΝΙΣΜΟΥΣ_ALL

http://www.mathematica.gr/forum/viewtopic.php?f=109&t=15584

Επιμέλεια: xr.tsif Σελίδα 52

ΘΕΜΑ 54 (ΣΩΚΡΑΤΗΣ ΛΥΡΑΣ)

Έστω x πραγματικός αριθμός. Αν οι αριθμοί 3x x και 2

x 2x είναι ρητοί, να

αποδειχθεί ότι ο x είναι ρητός.

Λύση:

Έστω 3x x b και 2

x 2x a με a και bρητούς.

3 3 2 2 3 2 2b x x x x 2x 2x 4x 4x x x 2x 2x 4x 5x

2 2x x 2x 2 x 2x 5x ax 2a 5x

2a bb ax 2a 5x b 2a ax 5x x

a 5

αφού a 5 μίας και η

εξίσωση 2x 2x 5 είναι αδύνατη. Άρα ο αριθμός x είναι ρητός ως πηλίκο

ρητών.

ΠΑΡΑΤΗΡΗΣΕΙΣ

Για να την λύσουμε, θα χρησιμοποιήσουμε μόνο ότι :

Το άθροισμα, η διαφορά, το γινόμενο και το πηλίκο ρητών, είναι ρητός (αρκεί

στο πηλίκο, ο παρονομαστής να είναι διάφορος του μηδενός)

Έστω 3

x x k (1)

2

x 2x r (2) όπου k,r είναι ρητοί αριθμοί.

Λύστε τώρα την σχέση (2) ως προς 2

x και αντικαταστήστε στην (1) με τελικό

σκοπό να βρείτε το x ....

Β τρόπος

Έστω 3

x x k (1)

2

x 2x r (2) όπου k,r είναι ρητοί αριθμοί.

Η σχέση (2) γράφεται: 2

x r 2x

Page 53: ΠΡΟΤΕΙΝΟΜΕΝΕΣ ΑΣΚΗΣΕΙΣ ΓΙΑ ΜΑΘΗΤΙΚΟΥΣ ΔΙΑΓΩΝΙΣΜΟΥΣ_ALL

http://www.mathematica.gr/forum/viewtopic.php?f=109&t=15584

Επιμέλεια: xr.tsif Σελίδα 53

Τώρα η (1) γράφεται: 2 2

x x x k x(r 2x) x k xr 2x x k xr 2(r 2x) x k

xr 2r 4x x k (r 5)x 2r k (3).

Αν τώρα ήταν r 5 0 r 5 οπότε η (2) γράφεται 2x 2x 5 0 .

Η εξίσωση όμως αυτή είναι αδύνατη, αφού έχει Διακρίνουσα 16 0

Άρα η σχέση (3) γράφεται k 2r

xr 5

.

Άρα ο αριθμός x είναι ρητός, ως πηλίκο ρητών αριθμών.

ΘΕΜΑ 55 (ΔΗΜΗΤΡΗΣ ΙΩΑΝΝΟΥ)

Το τριπλάσιο ενός αριθμού αυξημένο κατά 18 ισούται με το τετράγωνο του

αριθμού. Να βρεθεί ο αριθμός αυτός.

Λύση:

Έχω : 2x 3x 18 0 x 6 ή x 3 .

ΘΕΜΑ 56 (ΔΗΜΗΤΡΗΣ ΙΩΑΝΝΟΥ)

Δείξτε ότι η εξίσωση 2x x 2n 1 , όπου n είναι φυσικός αριθμός, έχει

πραγματικές ρίζες. Στη συνέχεια να εξετάσετε αν είναι δυνατόν οι ρίζες αυτής

της εξίσωσης να είναι ακέραιες (για κάποιονn φυσικό αριθμό)

Λύση:

Τα φέρνω όλα στο 1ο μέλος και έχω μια δευτεροβάθμια εξίσωση ως προς x . H

Διακρίνουσα είναι θετική άρα έχει λύσεις στους πραγματικούς αλλά όχι στους

ακέραιους αφού ένας άρτιος ισούται με έναν περιττό.

Page 54: ΠΡΟΤΕΙΝΟΜΕΝΕΣ ΑΣΚΗΣΕΙΣ ΓΙΑ ΜΑΘΗΤΙΚΟΥΣ ΔΙΑΓΩΝΙΣΜΟΥΣ_ALL

http://www.mathematica.gr/forum/viewtopic.php?f=109&t=15584

Επιμέλεια: xr.tsif Σελίδα 54

Β τρόπος

Αν υποθέσουμε λοιπόν ότι η εξίσωση έχει ρίζα τον ακέραιο αριθμό k . Τότε θα

πρέπει: 2k k 2n 1 k(k 1) 2n 1

Όμως έχουμε δει σε προηγούμενες ασκήσεις ότι το γινόμενο δύο διαδοχικών

ακεραίων είναι πάντα άρτιος. Άρα το πρώτο μέλος της (1) είναι άρτιος ενώ το

δεύτερο περιττός. Τούτο όμως είναι άτοπο και άρα η εξίσωση δεν μπορεί να

έχει ρίζα ακέραιο αριθμό.

ΘΕΜΑ 57 (ΔΗΜΗΤΡΗΣ ΙΩΑΝΝΟΥ)

Να βρείτε την ελάχιστη τιμή της παράστασης: 2 2A a 10ab 27b 8b 8 .

Για ποιες τιμές των a,b λαμβάνεται η ελάχιστη τιμή της παράστασης A ;

Λύση:

2 2 2 2 2A a 10ab 27b 8b 8 a 10ab 25b 2b 8b 8

2 2 2 2 2a 10ab 25b 2(b 4b 4) (a 5b) 2(b 2)

έχουμε άθροισμα δύο μη αρνητικών όρων, οπότε η ελάχιστη τιμή της

παράστασης A είναι το 0 .

Αυτό γίνεται για b 2 και a 10 .

ΘΕΜΑ 58 (ΔΗΜΗΤΡΗΣ ΙΩΑΝΝΟΥ)

Να βρεθεί η μέγιστη τιμή του θετικού ακεραίου x για την οποία ο αριθμός x13

διαιρεί τον αριθμό 500!. (ΣΥΜΒΟΛΙΣΜΟΣ: n! 1 2 ... (n 1) n )

Λύση:

Στο γινόμενο 500! διακρίνουμε τους εξής όρους: 13 1,13 2,...,13 38 .

Αλλά ανάμεσά τους έχουμε τους: 13 13 και 2 13 13 άρα ο ζητούμενος

αριθμός είναι ο 40 .

Page 55: ΠΡΟΤΕΙΝΟΜΕΝΕΣ ΑΣΚΗΣΕΙΣ ΓΙΑ ΜΑΘΗΤΙΚΟΥΣ ΔΙΑΓΩΝΙΣΜΟΥΣ_ALL

http://www.mathematica.gr/forum/viewtopic.php?f=109&t=15584

Επιμέλεια: xr.tsif Σελίδα 55

με άλλα λόγια ,

Ονομάζουμε το ζητούμενο a .

επειδή 313 500 , 2

500 500a 40

13 13 .

ΘΕΜΑ 59 (ΔΗΜΗΤΡΗΣ ΙΩΑΝΝΟΥ)

Τρίγωνο ΑΒΓ έχει πλευρές ΑΒ x , ΑΓ x 2 και ΒΓ 10 . Αν ισχύει ότι 2 2

(x 2) x 28 , να αποδείξετε ότι το τρίγωνο ΑΒΓ είναι ορθογώνιο με ορθή

την γωνία Α

.

Λύση:

Από τη σχέση που μου δίνει το πρόβλημα έχω ότι :

2 2 2 2(x 2) x x 4 4x x 28 4x 24 x 6 .

Άρα x 6 και x 2 8 .

Οπότε το πυθαγόρειο θεώρημα επαληθεύεται ( 2 2 26 8 10 ).

ΘΕΜΑ 60 (ΔΗΜΗΤΡΗΣ ΙΩΑΝΝΟΥ)

Αν για τους πραγματικούς αριθμούς x,y,z ισχύει ότι xyz 1 , να αποδείξετε

ότι 1 1 1

2y z x

y 1 z 1 x 1x 1 y 1 z 1

.

Λύση:

1 1 1

y z xy 1 z 1 x 1

x 1 y 1 z 1

Page 56: ΠΡΟΤΕΙΝΟΜΕΝΕΣ ΑΣΚΗΣΕΙΣ ΓΙΑ ΜΑΘΗΤΙΚΟΥΣ ΔΙΑΓΩΝΙΣΜΟΥΣ_ALL

http://www.mathematica.gr/forum/viewtopic.php?f=109&t=15584

Επιμέλεια: xr.tsif Σελίδα 56

x 1 y 1 z 1

(y 1)(x 1) y (z 1)(y 1) z (x 1)(z 1) x

x 1 y 1 z 1A

xy x 1 yz y 1 zx z 1

Αλλά xyz 1 άρα 1

zxy

. Άρα

11

x 1 y 1 xyA

1 1 1xy x 1y y 1 x 1

xy xy xy

x 1 x(y 1) xy 1 2xy 2x 2 2(xy x 1)2

xy x 1 xy x 1 xy x 1 xy x 1 xy x 1

.

ΘΕΜΑ 61 (ΜΠΑΜΠΗΣ ΣΤΕΡΓΙΟΥ)

Δίνεται ισοσκελές τρίγωνο ΑΒΓ όπου ΑΒ ΑΓ με oB 30

και σημείο Μ

στο εσωτερικό του , τέτοιο ώστε oMBΓ 30

και 3

M AB BAΓ4

. Να

αποδειχθεί ότι η γωνία οΑΜ Γ 150

.

Λύση:

Φέρνουμε τη διάμεσο ΑΝ που τέμνει την ΜΒ στο Ρ . Φέρνουμε

και τη ΡΓ . Οι οξείες γωνίες γύρω από το Ρ είναι ο

60 μοίρες, οπότε η ΡΜ είναι

διχοτόμος της γωνίας Α Ρ Γ

. Αλλά και η ΑΜ είναι διχοτόμος της γωνίας Ρ Α Γ

,

οπότε το Μ είναι το έκκεντρο του τριγώνου ΡΑΓ . Από βασική πρόταση ή με

απλούς υπολογισμούς προκύπτει ότι η γωνία ΑΜ Γ

είναι ίση με μια ορθή συν

το μισό της γωνίας Α Ρ Γ

, δηλαδή ο

150 μοίρες.

Page 57: ΠΡΟΤΕΙΝΟΜΕΝΕΣ ΑΣΚΗΣΕΙΣ ΓΙΑ ΜΑΘΗΤΙΚΟΥΣ ΔΙΑΓΩΝΙΣΜΟΥΣ_ALL

http://www.mathematica.gr/forum/viewtopic.php?f=109&t=15584

Επιμέλεια: xr.tsif Σελίδα 57

Β τρόπος

Αρκεί να ονομάσουμε Τ το σημείο τομής της ΒΜ με το

ύψος ΑΔ και να αποδείξουμε οτι το Μ είναι έκκεντρο

του τριγώνου ΑΤΓ(σημείο τομής των διχοτόμων

των γωνιών του).

ΘΕΜΑ 62 (Socrates)

Αν x,y,z [ 1, ) ώστε 1 x 1 y 2 1 z να δείξετε ότι x y 2z .

Πότε ισχύει η ισότητα;

Λύση:

Θέλουμε να αποδείξουμε ότι

x y 2z x 1 y 1 2z 2 (x 1) (y 1) 2(z 1) .

Ας ονομάσουμε για ευκολία x 1 a ,y 1 b και z 1 c

Τότε αρκεί να αποδείξουμε ότι 2 2 2

a b 2c ( a) ( b) 2c ( a b) 2 a b 2c (1)

Από την υπόθεση, έχουμε: a b 2 c (*)

Άρα 2(1) (2 c) 2 ab 2c 4c 2 ab 2c ab c (2)

Γνωρίζουμε όμως ότι αν k,m 0 τότε k m

km2

(Η απόδειξη αυτή είναι

απλή και αφήνεται ως άσκηση. Αργότερα θα δώσουμε και την γενικότερη

μορφή της ανισότητας αυτής)

Αν λοιπόν δούμε το a στην θέση του k και το b στην θέση του m τότε

έχουμε:

Page 58: ΠΡΟΤΕΙΝΟΜΕΝΕΣ ΑΣΚΗΣΕΙΣ ΓΙΑ ΜΑΘΗΤΙΚΟΥΣ ΔΙΑΓΩΝΙΣΜΟΥΣ_ALL

http://www.mathematica.gr/forum/viewtopic.php?f=109&t=15584

Επιμέλεια: xr.tsif Σελίδα 58

2a b 2 ca b a b a b c ( a b ) c

2 2

ab c .

Άρα η σχέση (2) είναι αληθής και η απόδειξη ολοκληρώθηκε.

Βάζω μια συντομότερη λύση:

Από τη γνωστή ανισότητα 2 2 22(m n ) (m n) (η οποία είναι ισοδύναμη με

την 2(m n) 0 )

έχουμε ότι 22(1 x 1 y) ( 1 x 1 y) 4(1 z),

άρα προκύπτει το ζητούμενο

Προφανώς, το ζητούμενο ισχύει αν και μόνο αν x y z .

ΘΕΜΑ 63 (Socrates)

Να βρείτε τους πρώτους αριθμούς x,y αν ισχύει 3 3 2 2 2 2

x 2y 6y y 3 2xy x y 3x x .

Λύση:

3 3 2 2 2 2x 2y 6y y 3 2xy x y 3x x

3 2 2 3 2 2(x 3x x y) (2y 6y 2xy ) ( x y 3) 0

2 2x ( x y 3) 2y ( x y 3) x y 3 0

2 2( x y 3)( x 2y 1) 0 .

Συνεπώς είτε x y 3 0 x y 3 (1) είτε

2 2 2x 2y 1 0 x 2y 1 (2)

(1): x y 3 άρα (x,y) (5,2),( 2, 5)

Page 59: ΠΡΟΤΕΙΝΟΜΕΝΕΣ ΑΣΚΗΣΕΙΣ ΓΙΑ ΜΑΘΗΤΙΚΟΥΣ ΔΙΑΓΩΝΙΣΜΟΥΣ_ALL

http://www.mathematica.gr/forum/viewtopic.php?f=109&t=15584

Επιμέλεια: xr.tsif Σελίδα 59

(2): 2

x 2y 1 άρα (x,y) (3,2)

Συνεπώς καταλήγουμε ότι: (x,y) (5,2),( 2, 5),(3,2) .

Υ.Γ. Αξίζει να σημειώσουμε για όσους δεν το γνωρίζουν ότι το 1 (και φυσικά

το 0) δεν είναι πρώτοι αριθμοί.

ΘΕΜΑ 64 (Socrates)

Αν a,b 0 να δείξετε ότι a b a b 2(a b)

1 a b 1 a 1 b 2 a b

.

Πότε ισχύει η ισότητα;

Λύση:

Ξεκινάμε με την αριστερή.

Είναι a b a b a b

,1 a b 1 a b 1 a b 1 a 1 b

αφού a,b 0 .

Πάμε στη δεξιά.

Είναι a b 2(a b) 1 a 1 1 b 1 2 a b 2

21 a 1 b 2 a b 1 a 1 b 2 a b

1 1 4 1 1 41 1 2 0,

a 1 1 b 2 a b a 1 1 b 2 a b( )

όπου η τελευταία ανισότητα, ισχύει ως εφαρμογή της κλασικής

1 1 4

k m k m

για k,m 0 .

ΘΕΜΑ 65 (ΔΗΜΗΤΡΗΣ ΙΩΑΝΝΟΥ)

Στο τραπέζιο ΑΒΓΔ οι γωνίες Α και Δ είναι ορθές ΑΒ α , ΑΔ β και

ΓΔ γ . Οι αριθμοί α,β,γ είναι ακέραιοι και ανάλογοι προς τους αριθμούς

1,2,3 αντίστοιχα και έχουν άθροισμα 30 .

Page 60: ΠΡΟΤΕΙΝΟΜΕΝΕΣ ΑΣΚΗΣΕΙΣ ΓΙΑ ΜΑΘΗΤΙΚΟΥΣ ΔΙΑΓΩΝΙΣΜΟΥΣ_ALL

http://www.mathematica.gr/forum/viewtopic.php?f=109&t=15584

Επιμέλεια: xr.tsif Σελίδα 60

Να βρεθεί το εμβαδόν του τραπεζίου ΑΒΓΔ .

Λύση:

Οι πληροφορίες της εκφώνησης μας

δίνουν τα εξής:

α β γ,

1 2 3

επειδή όμως οι α,β,γ έχουν

άθροισμα 30 , έχουμε:

305,

1 2 3 6 6

Επομένως: 5 , 10 , 15 .

Ο τύπος του εμβαδόν τραπεζίου είναι: 2(B β)υ (5 15)10Ε 100cm

2 2.

ΘΕΜΑ 66 (ΔΗΜΗΤΡΗΣ ΙΩΑΝΝΟΥ)

Αν a,b,x,y 0 και a b 1 τότε: 1

ax bya β

x y

. Πότε ισχύει το ίσον;

Λύση:

Έχουμε: 2 2 2a b x y(ax by)( ) a ab( ) b (a b) 1

x y y x

αφού για κάθε k,m 0 ισχύει ότι: k

m2

m

k .

Page 61: ΠΡΟΤΕΙΝΟΜΕΝΕΣ ΑΣΚΗΣΕΙΣ ΓΙΑ ΜΑΘΗΤΙΚΟΥΣ ΔΙΑΓΩΝΙΣΜΟΥΣ_ALL

http://www.mathematica.gr/forum/viewtopic.php?f=109&t=15584

Επιμέλεια: xr.tsif Σελίδα 61

ΒΑΣΙΚΗ ΑΝΙΣΟΤΗΤΑ (1):

Για κάθε x,y R (δηλαδή για κάθε x,y που είναι πραγματικοί αριθμοί) ισχύει:

2 2x y 2xy . Η ισότητα ισχύει όταν x y .

ΕΦΑΡΜΟΓΗ: Αν x,y 0 να αποδείξετε ότι x y 2 xy

ΑΠΟΔΕΙΞΗ: Στην βασική ανισότητα (1) βάλτε στην θέση του x το x και

στη θέση του y το y .

ΒΑΣΙΚΗ ΑΝΙΣΟΤΗΤΑ (2):

Για κάθε x,y R ισχύει: 2 2x y xy .

Ας δούμε μια απόδειξη: 2 2 2 2 2 2 2 2

x y xy 2(x y ) 2xy x y x y 2xy 0

2 2 2x y (x y) 0 .

Τούτο όμως ισχύει, οπότε θα ισχύει και το ζητούμενο

ΣΗΜ: Η ισότητα ισχύει όταν x 0 , y 0 , x y 0 δηλ. όταν x y 0 .

ΒΑΣΙΚΗ ΑΝΙΣΟΤΗΤΑ (3):

Για κάθε x,y,z R ισχύει : 2 2 2

x y z xy xz zy .

ΑΠΟΔΕΙΞΗ

1ος Τρόπος: Από την ΒΑΣΙΚΗ ΑΝΙΣΟΤΗΤΑ (1) έχουμε: 2 2

x y 2xy

2 2z y 2zy

2 2x z 2xz

Με πρόσθεση κατά μέλη των ανισοτήτων αυτών, έχουμε: 2 2 2 2 2 2

2x 2y 2z 2xy 2xz 2zy x y z xy xz zy

2ος Τρόπος: Πολλαπλασιάζουμε τα μέλη της ζητούμενης ανισότητας με το 2 ,

Page 62: ΠΡΟΤΕΙΝΟΜΕΝΕΣ ΑΣΚΗΣΕΙΣ ΓΙΑ ΜΑΘΗΤΙΚΟΥΣ ΔΙΑΓΩΝΙΣΜΟΥΣ_ALL

http://www.mathematica.gr/forum/viewtopic.php?f=109&t=15584

Επιμέλεια: xr.tsif Σελίδα 62

και με διασπάσεις και μεταφορά στο πρώτο μέλος, καταλήγουμε σε άθροισμα

τριών τετραγώνων μεγαλύτερο ή ίσο με το μηδέν, πράγμα που αληθεύει.

ΒΑΣΙΚΗ ΑΝΙΣΟΤΗΤΑ (4):

Αν x,y 0 τότε x y

2y x .

(Η απόδειξη είναι εύκολη , αν πολλαπλασιάσουμε ισοδύναμα τα μέλη της

αποδεικτέας με το x y ).

ΘΕΜΑ 67 (ΔΗΜΗΤΡΗΣ ΙΩΑΝΝΟΥ)

Για κάθε x,y,z 0 να αποδειχθεί ότι: x y x z y z

6y x z x z y

Λύση:

Ισχύουν: x y

2y x ,

x

z2

z

x ,

y

z2

z

y με πρόσθεση των τριών σχέσεων

προκύπτει η αρχική.

Β τρόπος

Με απλή εφαρμογή της ΑΜ – ΓΜ για 6 όρους.

ΘΕΜΑ 68 (ΔΗΜΗΤΡΗΣ ΙΩΑΝΝΟΥ)

Για κάθε x,y,z 0 πραγματικούς αριθμούς ότι (x y)(y z)(z x) 8xyz .

Λύση:

Ισχύουν: x y 2 xy , y z 2 yz , x z 2 xz οι οποίες

με πολλαπλασιασμό δίνουν την πρώτη σχέση.

Page 63: ΠΡΟΤΕΙΝΟΜΕΝΕΣ ΑΣΚΗΣΕΙΣ ΓΙΑ ΜΑΘΗΤΙΚΟΥΣ ΔΙΑΓΩΝΙΣΜΟΥΣ_ALL

http://www.mathematica.gr/forum/viewtopic.php?f=109&t=15584

Επιμέλεια: xr.tsif Σελίδα 63

ΘΕΜΑ 69 (ΔΗΜΗΤΡΗΣ ΙΩΑΝΝΟΥ)

Έστω ότι για τους θετικούς πραγματικούς αριθμούς a,b,c ισχύει ότι:

a b b c a cab( c) bc( a) ca( b) 0

2 2 2

. Να αποδειχθεί ότι a b c .

Λύση:

Έχουμε a b b c a c

ab( c) bc( a) ca( b) 02 2 2

.

Έχουμε: 2 2

ab(a b) a b ab... 3abc 3abc

2 2

.

Όμως από ΑM – GM παίρνουμε:

2 2 2 2 2 2 66a b ab b c bc c a ca 6 (abc) 6abc

2 2 2 2 2 2a b ab b c bc c a ca

3abc2

.

Επειδή όμως στην τελευταία θέλουμε ισότητα αναγκαστικά είναι: a b c .

Β τρόπος

Έχουμε: a b 2c b c 2a c a 2b

ab bc ca 02 2 2

2 2 2 2 2 2a b ab 2abc b c bc 2abc c a ca 2abc 0

2 2 2 2 2 2)a(b 2bc c ) b(a 2ac c ) c(b 2ab a 0

2 2 2a(b c) b(a c) c(b a) 0 .

Όμως οι αριθμοί a,b,c είναι θετικοί. Οπότε για να είναι αληθής η πιο πάνω

σχέση, θα πρέπει να ισχύει:

a b 0 και a c 0 και b c 0 , δηλαδή a b c .

Page 64: ΠΡΟΤΕΙΝΟΜΕΝΕΣ ΑΣΚΗΣΕΙΣ ΓΙΑ ΜΑΘΗΤΙΚΟΥΣ ΔΙΑΓΩΝΙΣΜΟΥΣ_ALL

http://www.mathematica.gr/forum/viewtopic.php?f=109&t=15584

Επιμέλεια: xr.tsif Σελίδα 64

ΘΕΜΑ 70 (ΣΩΤΗΡΗΣ ΛΟΥΡΙΔΑΣ)

Να λυθεί στους πραγματικούς αριθμούς η εξίσωση:

24 2 127 x x x (1 x) 4(3x x)

3 3 3

.

Λύση:

Ας δώσω και εγώ μια υπόδειξη (άλλο τρόπο) για την άσκηση 70

Έχουμε ισοδύναμα:

23x 4 3x 2 3x 127 (1 x) 4(3x x)

3 3 3

2(3x 4)(1 x)(3x 2)(3x 1) 4(3x x)

2 2 2[(3x x) 4][3(3x x) 2] 4(3x x) .

Μια συνήθης πρακτική σε τέτοιες ασκήσεις:

Θέτουμε 23x x y και η συνέχεια είναι απλή.

Υπόδειξη

Για να μην έχουμε συντελεστές μπροστά από τα x στους παράγοντες που

μετέχουν μετασχηματίζουμε την εξίσωση στην ισοδύναμη της

4 2 1 1

9 x x x x 1 4 x 0 x ,3 3 3 3

παρατηρώντας ότι:

4 2 1 11 0

13 3 3 3 ,6 6

οπότε επιχειρούμε τον

μετασχηματισμό που ακολουθεί:

1,2 3,4 1 2 3,4

1 1 5 17 2 1 17x y y x ,...,y , y ...x ,x 1,x .

6 6 6 6 3 6

Page 65: ΠΡΟΤΕΙΝΟΜΕΝΕΣ ΑΣΚΗΣΕΙΣ ΓΙΑ ΜΑΘΗΤΙΚΟΥΣ ΔΙΑΓΩΝΙΣΜΟΥΣ_ALL

http://www.mathematica.gr/forum/viewtopic.php?f=109&t=15584

Επιμέλεια: xr.tsif Σελίδα 65

ΘΕΜΑ 71 (ΔΗΜΗΤΡΗΣ ΙΩΑΝΝΟΥ)

Ένα ορθογώνιο παραλληλόγραμμο διαιρείται σε 4 μικρότερα ορθογώνια με δύο

ευθείες παράλληλες προς τις πλευρές του. Τα τρία από αυτά τα τέσσερα

ορθογώνια έχουν εμβαδά 10,18 και 25 τετραγωνικές μονάδες. Να βρεθεί το

εμβαδόν του τέταρτου ορθογωνίου.

Λύση:

Έχω λοιπόν xz 10 και yz 18 . Διαιρώ κατά μέλη (όλοι θετικοί !) και

παίρνω : xz 10 5

x yyz 18 9

Ομοίως : xz 10 και

2xt 25 z t

5

Συνεπώς έχω :

5 2 2 9xy 10 y t 10 yt 10 yt 10· yt 45 (?)!

9 5 9 2

Με παρόμοιο τρόπο μπορούμε να δούμε και την άλλη περίπτωση , δηλαδή

όταν το 25 και το ? εναλλαγούν.

Τότε θα βρούμε 125

? xt9

.Υπάρχει άλλη περίπτωση ;

Β τρόπος

Βασιζόμενος στο παραπάνω σχήμα έχω:

tx 25   (1)

xz 10 (2)

yz 18 (3) και θέλουμε το ty .

Page 66: ΠΡΟΤΕΙΝΟΜΕΝΕΣ ΑΣΚΗΣΕΙΣ ΓΙΑ ΜΑΘΗΤΙΚΟΥΣ ΔΙΑΓΩΝΙΣΜΟΥΣ_ALL

http://www.mathematica.gr/forum/viewtopic.php?f=109&t=15584

Επιμέλεια: xr.tsif Σελίδα 66

Πολλαπλασιάζοντας τις (1) , (2) , (3) κατά μέλη έχουμε

2

2 2

25 10 18 25 10 18ty(xz) 25 10 18 ty 45

(xz) 10

.

ΘΕΜΑ 72 (ΔΗΜΗΤΡΗΣ ΙΩΑΝΝΟΥ)

Οι αριθμοί m,n είναι ακέραιοι.

(α) Να βρεθούν τα ζεύγη (m,n) που επαληθεύουν την εξίσωση : 3 2 3 2

m 4mn 8n 2m n .

(β) Από τα ζεύγη που θα βρείτε να προσδιορίσετε εκείνα που επαληθεύουν την

εξίσωση 2m n 3 .

Λύση:

Είναι:

(a) 3 2 3 2 3 3 2 2m 4mn 8n 2m n m 8n 2m n 4mn 0

2 2 2(m 2n)(m 2mn 4n ) 2mn(m 2n) 0 (m 2n)(m 2n) 0

Aρα αναγκαστικά είναι: m 2n ή m 2n .

(b) Αντικαθιστώντας στην 2η εξίσωση βρίσκουμε ότι

γιαm 2n (m,n) (2,1) ( 6,ή 3)

και για m 2n (m,n) (2 ή, 1) ( 6,3).

ΘΕΜΑ 73 (Ferma_96)

Να εξεταστεί κατά πόσο ένας ακέραιος αριθμός, μπορεί να έχει την ρίζα του

στους ρητούς αλλά όχι και στους ακέραιους αριθμούς.

(είναι διάσημο πρόβλημα, οπότε μάλλον πολλοί θα το ξέρετε).

Λύση:

Αυτό δεν μπορεί να συμβεί .

Page 67: ΠΡΟΤΕΙΝΟΜΕΝΕΣ ΑΣΚΗΣΕΙΣ ΓΙΑ ΜΑΘΗΤΙΚΟΥΣ ΔΙΑΓΩΝΙΣΜΟΥΣ_ALL

http://www.mathematica.gr/forum/viewtopic.php?f=109&t=15584

Επιμέλεια: xr.tsif Σελίδα 67

Θέλουμε το κλάσμα a

b με a,b φυσικούς ,να μην είναι φυσικός αριθμός ενώ το

2

2

a

b να είναι φυσικός.

Προφανώς a b 0 , μπορώ να υποθέσω ότι (a,b) 1 διαφορετικά απλοποιώ το

κλάσμα. Τότε προφανώς και 2 2(a ,b ) 1 αυτό σημαίνει ότι το 2

b δεν διαιρεί το

2a , άρα το κλάσμα

2

2

a

bδεν είναι φυσικός.

ΘΕΜΑ 74 (ΣΩΚΡΑΤΗΣ ΛΥΡΑΣ)

Να λυθεί το σύστημα:

4 2 2 4

2 2

x x y y 91

x xy y 7

.

Λύση:

Διαπιστώνουμε ότι: x y,x y

Οι εξισώσεις του συστήματος δίνουν αντίστοιχα

6 6 2 2

3 3

x y 91 x y

x y 7 x y

Αν διαιρέσουμε κατά μέλη οδηγούμαστε στην 2 2x y xy 13 που μαζί με την

2 2x y xy 7 μας οδηγούν στην λύση.

Β τρόπος

Οι 2 σχέσεις γράφονται:

2 2 2 2(x y ) (xy) 91   (1)

2 2(x y ) (xy) 7  (2)

Διαιρώντας τις (1) και (2) κατά μέλη καταλήγουμε όπως παραπάνω.

Page 68: ΠΡΟΤΕΙΝΟΜΕΝΕΣ ΑΣΚΗΣΕΙΣ ΓΙΑ ΜΑΘΗΤΙΚΟΥΣ ΔΙΑΓΩΝΙΣΜΟΥΣ_ALL

http://www.mathematica.gr/forum/viewtopic.php?f=109&t=15584

Επιμέλεια: xr.tsif Σελίδα 68

ΘΕΜΑ 75 (ΔΗΜΗΤΡΗΣ ΙΩΑΝΝΟΥ)

Να λυθεί η εξίσωση: 2

2

42x x

x x 1

.

Λύση:

2

2

42x x

x x 1

, ας θέσουμε το 2

x x t . Τότε

242t t t 42 0 t 7,6

t 1

για t 7

2 2x x 7 x x 7 0 (Δεν πραγματική τιμή του x ).

Για t 6

2t 6 x x 6 0 x 3,2 .

ΘΕΜΑ 76 (ΔΗΜΗΤΡΗΣ ΙΩΑΝΝΟΥ)

Στον διαγωνισμό "ΑΡΧΙΜΗΔΗΣ" της ΕΜΕ συμμετέχουν αγόρια και κορίτσια

που χωρίζονται σε δύο κατηγορίες, στους "μικρούς" (με ηλικία κάτω των

15 ετών) και στους "μεγάλους". Τα αγόρια που λαμβάνουν μέρος στον φετινό

"ΑΡΧΙΜΗΔΗ" αποτελούν το 55% αυτών που συμμετέχουν. Ο λόγος του

πλήθους των "μικρών" αγοριών προς το πλήθος των "μεγάλων" αγοριών ισούται

με τον λόγο του πλήθους των "μικρών" προς το πλήθος των "μεγάλων".

Να βρεθεί ο λόγος του πλήθους των "μικρών" αγοριών προς το πλήθος των

"μικρών" κοριτσιών.

Λύση:

Έστω: x το πλήθος των μεγάλων αγοριών

y των μεγάλων κοριτσιών

Page 69: ΠΡΟΤΕΙΝΟΜΕΝΕΣ ΑΣΚΗΣΕΙΣ ΓΙΑ ΜΑΘΗΤΙΚΟΥΣ ΔΙΑΓΩΝΙΣΜΟΥΣ_ALL

http://www.mathematica.gr/forum/viewtopic.php?f=109&t=15584

Επιμέλεια: xr.tsif Σελίδα 69

z των μικρών αγοριών

w των μικρών κοριτσιών, άρα ψάχνουμε το z/w

Οι 2 σχέσεις της υπόθεσης δίνουν :

55 45 55 x z 11

x z (x y z w) (x z) (y w) ...   100 100 100 y w 9

(1)

και:

z z w z x x z 11xz zy xz xw

x x y w y y w 9

από την (1).

ΘΕΜΑ 77 (ΣΩΤΗΡΗΣ ΛΟΥΡΙΔΑΣ)

Να συγκρίνετε τους αριθμούς 3 33 3 33 3 3 3 , 2 3 λαμβάνοντας

σοβαρά υπ’ όψη ότι το εμφανισιακά μεγάλο δεν είναι πάντα και το μεγαλύτερο.

Λύση:

Αρκεί να συγκρίνουμε τους αριθμούς 3 3 33 3 3 k και 33 33 3 3

βασιζόμενος στην ταυτότητα 2 23 33 3 3a b ( a b)( a ab b )

η οποία δίνει 3 3

2 23 33

a ba b

a ab b

έχουμε:

3

3 3 3

23 3 33 3

33 3 3    (1)

(3 3) 3(3 3) 9

και: 3

33 3

23 3 33 3

33 3 3    (2)

(3 3) 3(3 3) 9

Από τις (1) κ (2) είναι φανερό ότι k άρα 3 33 3 32 3 3 3 3 3 .

Page 70: ΠΡΟΤΕΙΝΟΜΕΝΕΣ ΑΣΚΗΣΕΙΣ ΓΙΑ ΜΑΘΗΤΙΚΟΥΣ ΔΙΑΓΩΝΙΣΜΟΥΣ_ALL

http://www.mathematica.gr/forum/viewtopic.php?f=109&t=15584

Επιμέλεια: xr.tsif Σελίδα 70

Β τρόπος

3 332 2 3 3

3 33 3

a ba 3 3 0

a ab b ab θεωρία 4 a b a b

b 3 3 0 a b ab a b

3

3a b 24 a b 2 3.

ΘΕΜΑ 78 (ΔΗΜΗΤΡΗΣ ΙΩΑΝΝΟΥ)

Στην προηγούμενη μαθηματική Ολυμπιάδα, για ένα από τα προβλήματα που

τέθηκαν, στο οποίο η μέγιστη βαθμολογία ήταν 5 , είχαμε τα παρακάτω

αποτελέσματα:

Ο μέσος όρος των βαθμών των αγοριών ήταν 4 ,

ο μέσος όρος των βαθμών των κοριτσιών ήταν 3,25 και

ο μέσος όρος των βαθμών του συνόλου των μαθητών ήταν 3,6 .

Να βρείτε πόσα αγόρια και πόσα κορίτσια πήραν μέρος, αν ο αριθμός του

συνόλου των μαθητών ήταν μεταξύ 30 και 50 .

Λύση:

Εάν a ,bείναι το πλήθος των αγοριών και των κοριτσιών αντίστοιχα, έχουμε:

30 a b 50 ή 30 a b 50 αν συμπεριλάβουμε τα άκρα.

iA

i

xx 4 x 4a.

a

i

K i

yy 3,25 y 3,25b.

b

i ix y 4a 3,25b

x 3,6 3,6 0,35b 0,4a 8a 7ba b a b

.

Άρα 7 / a και 8 / b .Δηλαδή το a 7,14,21,28,35,42....

και το b 8,16,24,32,...

και τελικά πρέπει (a,b) (21,24) ή (a,b) (14,16) .

Page 71: ΠΡΟΤΕΙΝΟΜΕΝΕΣ ΑΣΚΗΣΕΙΣ ΓΙΑ ΜΑΘΗΤΙΚΟΥΣ ΔΙΑΓΩΝΙΣΜΟΥΣ_ALL

http://www.mathematica.gr/forum/viewtopic.php?f=109&t=15584

Επιμέλεια: xr.tsif Σελίδα 71

ΘΕΜΑ 79 (ΔΗΜΗΤΡΗΣ ΙΩΑΝΝΟΥ)

Δίνονται οι αριθμοί 41A 2 , 13

B 8 , 21C 4 , 8

D 32 .

α) Να βρείτε ποιος είναι ο μεγαλύτερος.

β) Να εκφράσετε το άθροισμα A B C D ως γινόμενο πρώτων

παραγόντων.

Λύση:

α) 41A 2 , 13 3 13 39

B 8 (2 ) 2 , 21 2 21 42C 4 (2 ) 2 , 8 5 8 40

D 32 (2 ) 2 .

Άρα Β D Α C .

β) 39 40 41 42 39 39 39Α Β C D 2 2 2 2 2 (1 2 4 8) 2 15 2 3 5 .

ΘΕΜΑ 80 (ΔΗΜΗΤΡΗΣ ΙΩΑΝΝΟΥ)

Να βρεθούν όλοι οι ακέραιοι αριθμοί n για τους οποίους ο αριθμός 2n 1

διαιρεί τον αριθμό 2n n 2 .

Λύση:

Επειδή ο 2n 1 είναι περιττός έχουμε

2 2 2 2n n 2 4(n n 2) 4n 4n 8 (2n 1) 9

2n 1 2n 1 2n 1 2n 1

92n 1 2n 1| 9 n 5, 2, 1,0,1,4

2n 1

.

ΘΕΜΑ 81 (qwerty)

Δείξτε ότι δεν υπάρχει φυσικός αριθμός n με την ιδιότητα: 5 3

3n 3n 30000001 . (Θαλής 1996)

Λύση:

Έχουμε : 5 3 5 33n 3n 3(n n ) πολ3

Page 72: ΠΡΟΤΕΙΝΟΜΕΝΕΣ ΑΣΚΗΣΕΙΣ ΓΙΑ ΜΑΘΗΤΙΚΟΥΣ ΔΙΑΓΩΝΙΣΜΟΥΣ_ALL

http://www.mathematica.gr/forum/viewtopic.php?f=109&t=15584

Επιμέλεια: xr.tsif Σελίδα 72

από την άλλη πλευρά όμως το 30000001 δεν διαιρείται με το 3 (αφού το

άθροισμα των ψηφίων του είναι 4),

οπότε δεν υπάρχει φυσικός αριθμός n με την παραπάνω ιδιότητα.

ΘΕΜΑ 82 (Socrates)

Τοποθετούμε τους αριθμούς 1,2,3,...,49 στα κελιά μιας 7x7 σκακιέρας, έναν

σε κάθε κελί. Υπολογίζουμε το άθροισμα των αριθμών κάθε γραμμής και κάθε

στήλης. Κάποια από αυτά τα 14 αθροίσματα είναι άρτιοι αριθμοί και κάποια

περιττοί. Έστω A το άθροισμα των περιττών αθροισμάτων και B το άθροισμα

των άρτιων. Είναι δυνατόν να τοποθετηθούν οι αριθμοί στην σκακιέρα με τέτοιο

τρόπο ώστε να ισχύει A B ;

Λύση:

Μία προσπάθεια με κάθε επιφύλαξη:

Στην σκακιέρα 7x7 το άθροισμα των ψηφίων των γραμμών προκύπτει από τον

τύπο: n(n 1)

S2

για n 49 άρα S 1225 . Επιπλέον το άθροισμα των

ψηφίων των στηλών είναι πάλι S΄ 1225 .

Δηλαδή θα πρέπει να ισχύει A B 2450 2B 2450 B 1225 , το οποίο

είναι άτοπο αφού το Β αποτελεί άθροισμα άρτιων.

Καταλήγοντας είναι αδύνατο να ισχύει A B .

ΘΕΜΑ 83 (Socrates)

Αν a,b,c θετικοί ακέραιοι τέτοιοι ώστε b c

(a 1) (a 25) να δείξετε ότι ο

αριθμός b c είναι πολλαπλάσιο του 4 .

Λύση:

Υπόδειξη:

Αν n n

a / b τότε a / b .

Page 73: ΠΡΟΤΕΙΝΟΜΕΝΕΣ ΑΣΚΗΣΕΙΣ ΓΙΑ ΜΑΘΗΤΙΚΟΥΣ ΔΙΑΓΩΝΙΣΜΟΥΣ_ALL

http://www.mathematica.gr/forum/viewtopic.php?f=109&t=15584

Επιμέλεια: xr.tsif Σελίδα 73

Δείξτε, λοιπόν, ότι

• b c

• a 1 / a 25 οπότε

• a 1 / 24...

ΛΥΣΗ

Αν b c τότε το δεξί μέλος είναι μεγαλύτερο από το αριστερό.

Άρα b c οπότε μπορούμε να γράψουμε b c c c(a 1) (a 1) (a 25) .

Ο αριθμός b c(a 1)

είναι ακέραιος, οπότε c c(a 1) (/ a 25) . και άρα

a 1| a 25 a 1| (a 25) (a 1) 24.

Έτσι, a 1 1,2,3,4,6,8,12,24 και αφού a 0 είναι a 1,2,3,5,7,11,23.

Αντικαθιστώντας καθεμιά τιμή του a και ελέγχοντας τους πρώτους παράγοντες

κάθε μέλους, βλέπουμε ότι οι μόνες που μπορούν να δώσουν λύση είναι οι

a 2 και a 7 .

Για a 2 είναι b c3 27 οπότε b 3c και b c 4c (πολλαπλάσιο του 4 ),

Για a 7 είναι b c8 32 οπότε 3b 5c.

Από την τελευταία σχέση 5 / 3b οπότε 5 / b , δηλαδή b 5k και c 3k

και τελικά b c 8k (πολλαπλάσιο του 4).

ΘΕΜΑ 84 (Socrates)

Θεωρούμε την διαδικασία: ξεκινώντας από μια τριάδα (a,b,c) (0,0,0)

παίρνουμε (την αντικαθιστούμε) την τριάδα (a b,b c,c a) . Δείξτε ότι αν

με, επανειλημμένη, εφαρμογή της διαδικασίας προκύψει η αρχική τριάδα, τότε

αυτό θα συμβεί μετά από ακριβώς έξι βήματα (εφαρμογές της διαδικασίας).

Page 74: ΠΡΟΤΕΙΝΟΜΕΝΕΣ ΑΣΚΗΣΕΙΣ ΓΙΑ ΜΑΘΗΤΙΚΟΥΣ ΔΙΑΓΩΝΙΣΜΟΥΣ_ALL

http://www.mathematica.gr/forum/viewtopic.php?f=109&t=15584

Επιμέλεια: xr.tsif Σελίδα 74

Λύση:

Μετά την n – οστή εφαρμογή της διαδικασίας επαγωγικά έχουμε ότι αν

εμφανίζεται η 3 – άδα (x,y,z) τότε nx y z 2 (a b c) , οπότε πρέπει για

κάποιο n 0 0n , na b c 2 (a b c) , άρα a b c 0 .

Με βάση αυτό αν παρατηρήσουμε την διαδικασία (θέτοντας c (a b) ) αφού

οι a,b,c δεν είναι όλοι 0 , έχουμε ότι η τριάδα (a,b,c) θα ξαναεμφανιστεί μετά

από 6 βήματα..

ΘΕΜΑ 85 (ΛΩΛΑΣ ΠΑΝΑΓΙΩΤΗΣ)

Οι φυσικοί αριθμοί από το 1 ως το 10 τοποθετούνται με τυχαία σειρά στην

περιφέρεια ενός κύκλου. Να δείξετε ότι υπάρχουν σε διπλανές θέσεις τρεις

αριθμοί με άθροισμα τουλάχιστον 18 .

Λύση:

Ξεχνάμε τον 1 . Οι υπόλοιποι εννιά

έχουν συνολικό άθροισμα

2 3 ... 10 54 . Τους χωρίζουμε

σε τρεις διαδοχικές τριάδες, κυκλικά,

αρχίζοντας από τον διπλανό του 1 .

Κάποια από τις τρεις αυτές τριάδες θα

έχει άθροισμα 18 γιατί αλλιώς το

συνολικό τους άθροισμα θα ήταν

3 17 51 54 . Άτοπο.

Αξίζει να προσθέσουμε ότι δεν μπορεί

να βελτιωθεί το αποτέλεσμα σε 19 ,

όπως δείχνει η παρακάτω διευθέτηση,

όπου όλα τα αθροίσματα τριών

διαδοχικών είναι το πολύ 18 :

Page 75: ΠΡΟΤΕΙΝΟΜΕΝΕΣ ΑΣΚΗΣΕΙΣ ΓΙΑ ΜΑΘΗΤΙΚΟΥΣ ΔΙΑΓΩΝΙΣΜΟΥΣ_ALL

http://www.mathematica.gr/forum/viewtopic.php?f=109&t=15584

Επιμέλεια: xr.tsif Σελίδα 75

ΘΕΜΑ 86 (Grigoris K.)

Βρείτε όλα τα ζεύγη πραγματικών αριθμών (x,y) τα οποία ικανοποιούν την εξής

σχέση: x 2 5 4 y | y 2x |

y xy x xy

.

Λύση:

x 2 5 4 y | y 2x |x(x 2) 5 y(4 y) | y 2x |

y xy x xy

2 2(x 1) (y 2) | y 2x | 0 .

πρέπει x 1 y 2 y 2x 0 , οπότε x 1,y 2.

ΘΕΜΑ 87 (Grigoris K.)

Σε μια σκακιέρα 6x6 έχουν τοποθετηθεί φυσικοί αριθμοί. Κάθε κίνηση

συνίσταται στην επιλογή ενός τετραγώνου μεγαλύτερου από 1x1 ( το οποίο

αποτελείται από "κουτάκια" της σκακιέρας ) και στην αύξηση όλων των

φυσικών αριθμών που βρίσκονται στο επιλεγμένο τετράγωνο κατά 1 . Είναι

πάντα εφικτό να κάνουμε κάποιες κινήσεις ώστε να οδηγηθούμε σε μια

κατάσταση όπου όλοι οι φυσικοί αριθμοί είναι διαιρετοί από το 3 ;

Λύση:

Μια ισχυρή υπόδειξη για τους μικρούς μας φίλους. Μπορεί τέτοιες ασκήσεις να

φαντάζουν δύσκολες και να φοβούνται οι juniors να τις αντιμετωπίσουν αλλά

σύντομα θα κατανοήσουν την σημασία και θα γευτούν την ομορφιά την

Συνδυαστικής:

Θα αντικρούσουμε το τον ισχυρισμό χρησιμοποιώντας αντιπαράδειγμα, δηλαδή

θα αναφερθούμε σε μία κατάσταση όπου κάνοντας n κινήσεις είναι αδύνατον να

οδηγηθούμε στην ζητούμενη κατάσταση.

Σκεφτείτε την παρακάτω περίπτωση (μπορείτε και να την γενικεύσετε με

πολλαπλάσια του 3 και μη):

Page 76: ΠΡΟΤΕΙΝΟΜΕΝΕΣ ΑΣΚΗΣΕΙΣ ΓΙΑ ΜΑΘΗΤΙΚΟΥΣ ΔΙΑΓΩΝΙΣΜΟΥΣ_ALL

http://www.mathematica.gr/forum/viewtopic.php?f=109&t=15584

Επιμέλεια: xr.tsif Σελίδα 76

3 3 3 3 3 3

3 3 3 3 3 3

3 3 3 3 3 3

3 3 1 3 3 3

3 3 3 3 3 3

3 3 3 3 3 3

Τώρα λαμβάνοντας υπόψη τον ορισμό της κινήσεως αποδείξτε ότι το ζητούμενο

δεν μπορεί να ισχύει σε αυτή την περίπτωση και συνεπώς δεν ισχύει πάντα.

ΘΕΜΑ 88 (ΣΩΤΗΡΗΣ ΛΟΥΡΙΔΑΣ)

Αν a,b,c 0 , να αποδείξετε ότι: 3 3 3 2 2 2a b c a b b c c a .

Λύση:

Από ΑM – GM έχουμε:

3 3 3 2a a b 3a b   (1)

3 3 3 2b b c 3b c   (2)

3 3 3 2c c a 3c a   (3)

με πρόσθεση κατά μέλη των (1),(2) και (3) και διαίρεση διά τρία προκύπτει το

ζητούμενο.

Β τρόπος

Απλά και μόνο για να δούμε και κάποιες άγνωστες ταυτότητες:

Ισχύει η ταυτότητα:

2 3 3 3 2 2 2

2a b a b 3 a b c a b b c c a .

Είναι κατανοητό ότι με βάση τη ταυτότητα αυτή η άσκηση επιλύεται αμέσως.

Page 77: ΠΡΟΤΕΙΝΟΜΕΝΕΣ ΑΣΚΗΣΕΙΣ ΓΙΑ ΜΑΘΗΤΙΚΟΥΣ ΔΙΑΓΩΝΙΣΜΟΥΣ_ALL

http://www.mathematica.gr/forum/viewtopic.php?f=109&t=15584

Επιμέλεια: xr.tsif Σελίδα 77

Γ τρόπος

Αν ήταν a b c , τότε το ζητούμενο ισχύει ως ισότητα.

Αν πάλι ήταν π.χ b c τότε αρκεί να δείξουμε ότι

3 3 2 2 2 2 2 2a b a b b a (a b)(a ab b ) ab(a b) a 2ab b 0 ,

πράγμα που είναι αληθές. Όμοια και αν a b ή a c .

Αν τώρα οι αριθμοί a,b,c είναι ανά δύο άνισοι, τότε χωρίς βλάβη της

γενικότητας, ας υποθέσουμε ότι a b c .

Τότε έχουμε: 2 2 2 2 2 3 2 2a b 2ab a(a b ) 2a b a ab 2a b

Ομοίως έχουμε ότι: 3 2 2 3 2 2b bc 2b cc ca 2c a

Με πρόσθεση κατά μέλη των τριών παραπάνω σχέσεων παίρνουμε:

3 3 3 2 2 2 2 2 2a b c 2a b 2b c 2c a ab bc ca

Οπότε αρκεί πλέον να αποδείξουμε ότι

2 2 2 2 2 2 2 2 22a b 2b c 2c a ab bc ca a b b c c a

2 2 2 2 2 2a b b c c a ab bc ca 0 2 2

a (b c) bc(b c) c (a b) 0 2 2

(b c)(a bc) c (a b) 0 .

Τούτο όμως αληθεύει αφού a,b,c 0 και a b c .

ΘΕΜΑ 89 (CARANUS)

Να εξετάσετε αν υπάρχουν πραγματικοί αριθμοί a,b διάφοροι του μηδενός ,

τέτοιοι ώστε: 1 13 10

ab a b 32 3

. (ΘΑΛΗΣ 2006)

Λύση:

23a 10b3 9a 20b 18ab 0

2b 3a

Page 78: ΠΡΟΤΕΙΝΟΜΕΝΕΣ ΑΣΚΗΣΕΙΣ ΓΙΑ ΜΑΘΗΤΙΚΟΥΣ ΔΙΑΓΩΝΙΣΜΟΥΣ_ALL

http://www.mathematica.gr/forum/viewtopic.php?f=109&t=15584

Επιμέλεια: xr.tsif Σελίδα 78

2 2 2 2 29(a 2ab b ) 11b 0 9(a b) 11b 0 .

Άρα πρέπει a b 0 , οπότε το ζητούμενο δεν ισχύει.

ΘΕΜΑ 90 (CARANUS)

Αν a,b,c 0 , να αποδειχθεί ότι: b c a

(a )(b )(c ) 8ca ab bc

.

Να αποδειχθεί πότε ισχύει η ισότητα;

Λύση:

Εφαρμόζουμε την ΑΜ – ΓΜ σε κάθε παρένθεση και πολλαπλασιάζουμε τις 3

σχέσεις και προκύπτει το ζητούμενο.

Εφαρμόζουμε ΑΜ – ΓΜ (ΑΜ – GM) για δύο αριθμούς (θετικούς):

b ba 2 a

ca ca . Ομοίως,

c cb 2 b

ab ab

a ac 2 c

bc bc

Οπότε με πολλαπλασιασμό κατά μέλη έχουμε το ζητούμενο.

(ΣΗΜΕΙΩΣΗ: Ο πολλαπλασιασμός κατά μέλη επιτρέπεται εφ όσον τα μέλη

είναι θετικά).

ΘΕΜΑ 91 (ΣΩΤΗΡΗΣ ΛΟΥΡΙΔΑΣ)

Να λυθεί ως προς x η εξίσωση: 2 327a x 2(x a) , a R* .

Λύση:

Μια υπόδειξη:

Η εξίσωση ισοδύναμα γίνεται:

Page 79: ΠΡΟΤΕΙΝΟΜΕΝΕΣ ΑΣΚΗΣΕΙΣ ΓΙΑ ΜΑΘΗΤΙΚΟΥΣ ΔΙΑΓΩΝΙΣΜΟΥΣ_ALL

http://www.mathematica.gr/forum/viewtopic.php?f=109&t=15584

Επιμέλεια: xr.tsif Σελίδα 79

3

x a2x54 2 ,

a a

οπότε (για να απαλλαγούμε από τονa που μας ενοχλεί)

θεωρούμε την αντικατάσταση που ακολουθεί: 2 x a

y ,...a

Η εξίσωση ισοδύναμα γίνεται

3

2 x a2x54 ,

a a

για να οδηγηθούμε στην

αντικατάσταση

2 x a

y 2x a y 2 ,a

που μας «διώχνει» το a

διευκολύνοντας, αφού αρκεί να λύσουμε πλέον την εξίσωση

3 2 2y 36y 108 0 y 6 y 6y 18 y 6 0 y 6 y 6y 18 0,

που έχει σαν λύσεις τις 1 2 3

y 6, y 3 3, y 3 3,

από τις οποίες έχουμε:

1 2 3

a 5 3 3 a 5 3 3x 2a, x , x .

2 2

ΒΑΣΙΚΗ ΑΝΙΣΟΤΗΤΑ (Ανισότητα των μέσων: ΑΜ – GM – HM)

Επειδή πολύ συχνά οι μαθητές μας για να λύσουν κάποια ανισότητα

χρησιμοποιούν την ανισότητα με τον αριθμητικό, γεωμετρικό και αρμονικό

μέσο, ήρθε η ώρα να την κάνουμε γνωστή και στους υπόλοιπους μαθητές που

δεν την γνωρίζουν

Για τους μη αρνητικούς πραγματικούς αριθμούς a,b,c ισχύει ότι:

3a b c 1abc

1 1 13

a b c

.

Έτσι όταν βλέπουμε κάπου να γράφει "από ΑΜ – GM" σημαίνει ότι

χρησιμοποιεί την ανισότητα:

3a b cabc

3

ή την ισοδύναμή της 3a b c 3 abc και αν σε αυτήν

Page 80: ΠΡΟΤΕΙΝΟΜΕΝΕΣ ΑΣΚΗΣΕΙΣ ΓΙΑ ΜΑΘΗΤΙΚΟΥΣ ΔΙΑΓΩΝΙΣΜΟΥΣ_ALL

http://www.mathematica.gr/forum/viewtopic.php?f=109&t=15584

Επιμέλεια: xr.tsif Σελίδα 80

θέσουμε όπου a το 3a , όπου b το 3

b και όπου c το 3c έχουμε την ανισότητα

3 3 3a b c 3 abc .

Παρατήρηση:

α) Η ανισότητα αυτή γενικεύεται και για περισσότερους (n στο πλήθος)

θετικούς αριθμούς.

β) Η ισότητα ισχύει μόνο όταν οι αριθμοί αυτοί είναι ίσοι.

ΘΕΜΑ 92 (CARANUS)

Αν a,b,c 0 , να αποδειχθεί ότι 1 1 1

(a b c)( ) 9a b c

.

Λύση:

Από ΑΜ – ΓΜ παίρνω:

3a b c 3 abc και 31 1 1 1

3a b c abc

Με πολλαπλασιασμό των 2 σχέσεων προκύπτει η ζητούμενη.

(Μπορεί να λυθεί πιο απλά με CS).

Β τρόπος

από AM – HM έχουμε

a b c 3

1 1 13

a b c

1 1 1(a b c)( ) 9

a b c

να προσθέσω στην ανισότητα AM GM HM

και το 2 2 2

a b cRM

3S

για το οποίο ισχύει RMS AM GM HM .

Page 81: ΠΡΟΤΕΙΝΟΜΕΝΕΣ ΑΣΚΗΣΕΙΣ ΓΙΑ ΜΑΘΗΤΙΚΟΥΣ ΔΙΑΓΩΝΙΣΜΟΥΣ_ALL

http://www.mathematica.gr/forum/viewtopic.php?f=109&t=15584

Επιμέλεια: xr.tsif Σελίδα 81

Γ τρόπος

κάνουμε χρήση της C – S. 21 1 1 1 1 1(a b c) ( ) (a b c )

a b c a b c

2RHS 3 9 που είναι και το ζητούμενο

Η άσκηση 92 μπορεί να γενικευτεί ως εξής:

1

2

1 2

n

n

2

1 1 1(x x ... x )( ..

x x x. ) n

και αποδεικνύεται άμεσα από Cauchy – Schwartz αφού

2

n2

1 2 n jj 1

1 2 n j

1 1 1 1(x x

x x x... x )( ... ) x n

x

.

Ανισότητα των Buniakowski – Cauchy – Schwarz (B-C-S)

Έστω τα σύνολα των πραγματικών αριθμών 1 2 n

{x ,x ,...,x } , 1 2 n

{y ,y ,...,y }

τότε ισχύει ότι : 2 2 2 2 2 2 2

1 2 n 1 2 n 1 1 2 2 n n(x x x ) (y y y ) (x y x y x y )

με την ισότητα να ισχύει όταν,

1 1 2 2 n nR, x y και x y και ...x y .

ΘΕΜΑ 93 (CARANUS)

Έστω a,b,c και οι τρεις αριθμοί διάφοροι του μηδενός. Αν οι αριθμοί ab ,bc

και ac είναι ρητοί, τότε και ο αριθμός 2 2 2

k a b c είναι ρητός.

Λύση:

το γινόμενο το πηλίκο, το άθροισμα, και η διαφορά ρητών, είναι ρητός.

Page 82: ΠΡΟΤΕΙΝΟΜΕΝΕΣ ΑΣΚΗΣΕΙΣ ΓΙΑ ΜΑΘΗΤΙΚΟΥΣ ΔΙΑΓΩΝΙΣΜΟΥΣ_ALL

http://www.mathematica.gr/forum/viewtopic.php?f=109&t=15584

Επιμέλεια: xr.tsif Σελίδα 82

2ab a aQ, ac a Q

bc c c

ομοίως, 2 2b ,c Q , άρα 2 2 2

k a b c Q .

ΘΕΜΑ 94 (CARANUS)

Δίνονται οι αριθμοί a n(n 1) και 2b (n 1) , όπου n θετικός ακέραιος.

Να αποδειχθεί ότι οι αριθμοί a και b έχουν διαφορετικό άθροισμα ψηφίων.

Ευκλείδης 1997

Λύση:

1η περίπτωση: n 1 πολ3 .

προφανώς το 2(n 1) πολ3 , ακόμα, ούτε το n , ούτε τοn 1 είναι πολ3 ,

αφού σε κάθε τριάδα αριθμών μόνο ο ένας είναι πολ3 , οπότε πρέπει ο ένας

είναι της μορφής 3k 1 , και ο άλλος της μορφής 3k 2 . Πολλαπλασιάζοντας

βρίσκουμε ότι ο n(n 1) είναι επίσης της μορφής 3k 2 , δηλαδή όταν

διαιρεθεί με το 3 δίνει υπόλοιπο 2 .

2η περίπτωση: n 1 πολ3

το τετράγωνο ενός αριθμού που δεν είναι πολ3 , είναι πάντα της μορφής 3k 1

δηλαδή όταν διαιρεθεί με το 3 δίνει υπόλοιπο1 .

Επειδή ανάμεσα σε τρεις διαδοχικούς αριθμούς ο ένας είναι πάντα πολ3 ,

έχουμε ότι είτε ο n , είτε ο n 1 είναι πολ3 , και άρα το ίδιο ισχύει και για το

γινόμενο τους.

Ξέρουμε όμως ότι ένας αριθμός είναι πολ3 αν και μόνο αν το ίδιο ισχύει και

για το άθροισμα των ψηφίων του. Και στις δυο πιο πάνω περιπτώσεις, ο ένας

από τους δυο αριθμούς μας διαιρείται με το 3 , και άρα το ίδιο ισχύει για το

άθροισμα των ψηφίων του, και ο δεύτερος δεν διαιρείται με το 3 και άρα το

ίδιο ισχύει για το άθροισμα των ψηφίων του. αλλά αφού το άθροισμα των

ψηφίων του πρώτου διαιρείται με το 3 , και το άθροισμα των ψηφίων του

δεύτερου όχι, τότε τα δυο αθροίσματα δεν μπορούν να είναι ίσα.

Page 83: ΠΡΟΤΕΙΝΟΜΕΝΕΣ ΑΣΚΗΣΕΙΣ ΓΙΑ ΜΑΘΗΤΙΚΟΥΣ ΔΙΑΓΩΝΙΣΜΟΥΣ_ALL

http://www.mathematica.gr/forum/viewtopic.php?f=109&t=15584

Επιμέλεια: xr.tsif Σελίδα 83

λίγο διαφορετικά.

Αν 2 αριθμοί έχουν ίδιο άθροισμα ψηφίων τότε η διαφορά τους είναι διαιρετή

από το 3 .

Έστω λοιπόν ότι αυτοί οι αριθμοί έχουν ίδιο άθροισμα ψηφίων. Η διαφορά τους

είναι ίση με: 2 2b a n 2n 1 n n 3n 1 ,άτοπο.

Y.Γ Ένας τρόπος να καταλάβει κάποιος την παραπάνω πρόταση είναι η

απόδειξη του ίδιου του κριτηρίου διαιρετότητας του 3 (και του 9).

Γράφω την απόδειξη για έναν τετραψήφιο και ομοίως είναι για αυθαίρετο

πλήθος ψηφίων.

Έχουμε: 2abcd 1000a 100b 10c d 3 (111a 11b c) a b c d

9k a b c d .

Από την τελευταία έπεται ότι...

1."Αν οι αριθμοί a και b έχουν το ίδιο άθροισμα ψηφίων, τότε η διαφορά

τους διαιρείται με το 3 ."

2."Το υπόλοιπο που αφήνει ένας αριθμός a διαιρούμενος με τον 3 είναι το

ίδιο με αυτό που αφήνει ο αριθμός που είναι το άθροισμα των ψηφίων του

a διαιρούμενο με τον 3 "

ΘΕΜΑ 95 (ΣΩΤΗΡΗΣ ΛΟΥΡΙΔΑΣ)

Αν a,b,c,d 0 και a b c d 1 τότε να αποδείξετε ότι

1 1 1 18

a b b c c d d a

.

Page 84: ΠΡΟΤΕΙΝΟΜΕΝΕΣ ΑΣΚΗΣΕΙΣ ΓΙΑ ΜΑΘΗΤΙΚΟΥΣ ΔΙΑΓΩΝΙΣΜΟΥΣ_ALL

http://www.mathematica.gr/forum/viewtopic.php?f=109&t=15584

Επιμέλεια: xr.tsif Σελίδα 84

Λύση:

1 1 1 1

a b (b c) (c d) (d a) 16a b b c c d d a

και

αφού a b c d 1 προκύπτει η ζητούμενη σχέση.

Μεθοδολογικές αναφορές:

α) Όταν μάς δώσουν ότι ένας αριθμός, έστω ο x είναι ΑΡΝΗΤΙΚΟΣ ή μηδέν

τότε μπορούμε να εμφανίσουμε με θετικό μέρος δηλαδή να λάβουμε υπ’ όψη

ότι: x R x R και x x .

β) Όταν μας πουν ότι ένας αριθμός, έστω x είναι ΘΕΤΙΚΟΣ μας δίνουν το

ελεύθερο να γράψουμε 2

x x , με ότι αυτό συνεπάγεται.

Ας δούμε, κάτω από αυτό το πρίσμα, το γνωστό παράδειγμα που ακολουθεί:

1 2 n 1 2 n

1 2 n

1 1 1a ,a ,...,a 0 (a a ... a )( ... )

a a a

n

n

α α ... α ...

α α α

2

2

1 2 n

1 2 n

1 1 1a a ... a n

a a a

.

Ανισότητα των Buniakowski – Cauchy – Schwarz (B – C – S)

και μεθοδικές πινελιές.

● Επειδή ο στόχος της στήλης αυτής είναι διδακτικός προς την κατεύθυνση να

διδαχθούν οι juniors παντού σε όλη την Πατρίδα και όχι μόνο ελάχιστοι εντός

των πυλών Θεωρία και Μεθοδολογία του είδους, ας ήμαστε όσο μπορούμε

κατατοπιστικοί προς αυτή τη κατεύθυνση της παρουσίασης των λύσεων μας.

Page 85: ΠΡΟΤΕΙΝΟΜΕΝΕΣ ΑΣΚΗΣΕΙΣ ΓΙΑ ΜΑΘΗΤΙΚΟΥΣ ΔΙΑΓΩΝΙΣΜΟΥΣ_ALL

http://www.mathematica.gr/forum/viewtopic.php?f=109&t=15584

Επιμέλεια: xr.tsif Σελίδα 85

● Σκέψη:

Εδώ μας δίνουν την πληροφορία της θετικότητας των αριθμών. Άρα μπορούμε

να χρησιμοποιήσουμε την συνεπαγωγή: 2

x 0 x x , που οδηγεί στο

ΠΙΘΑΝΟ ενδεχόμενο εφαρμογής της βασικής ανισότητας B – C – S.

Όταν θέλουμε να αποδείξουμε μία ανισότητα ένας τρόπος είναι να

χρησιμοποιήσουμε την μέθοδο της ΑΝΑΛΥΣΗΣ, δηλαδή να θεωρήσουμε ότι

πράγματι ισχύει , έστω A B και να παράξουμε Αληθείς προτάσεις

συνδεόμενες με το ρήμα ΑΡΚΕΙ (αφού μας ενδιαφέρει η «λογική

νομιμοποίηση» της αντίστροφης πορείας προς την A B ), έως ότου φτάσουμε

σε ΑΛΗΘΗ ΠΡΟΤΑΣΗ.

● Λύση:

Με βάση την υπόθεση

a,b,c,d 0

και ,

a b c d 1

έστω ότι ισχύει

1 1 1 18,

a b b c c d d a

αρκεί

1 1 1 1 1

a b b c c d d a 8,2 a b b c c d d a

αρκεί

2 2 2 2

a b b c c d d a ,

2 2 2 2

1 1 1 116,

a b b c c d d a

B C S

2

αρκει 1 1 1 1 16,

που είναι πρόταση ΑΛΗΘΗΣ.

ΦΥΣΙΚΑ πρέπει να κάνουμε μετά και Σύνθεση δηλαδή ουσιαστικά να

"αντιγράψουμε" την αντίθετη πορεία.

Page 86: ΠΡΟΤΕΙΝΟΜΕΝΕΣ ΑΣΚΗΣΕΙΣ ΓΙΑ ΜΑΘΗΤΙΚΟΥΣ ΔΙΑΓΩΝΙΣΜΟΥΣ_ALL

http://www.mathematica.gr/forum/viewtopic.php?f=109&t=15584

Επιμέλεια: xr.tsif Σελίδα 86

Για παράδειγμα:

Αν

*

2

Aν x,y με x y 1,a,b *

τότε, ισχύει τι .

a ba b

x y

ό

Για να μην έχουμε λύση χωρίς να φαίνεται το πώς μας έκοψε , κάνουμε πρώτα

Ανάλυση

Έστω ότι με βάση τις (*) ισχύει η 2a b

a b .x y

Για να ισχύει αυτή Αρκεί να ισχύει:

2a b

x y a b ,x y

αρκεί να ισχύει

2 2

2 2 2a bx y a b ,

x y

αρκεί να ισχύει

2 2

a b a b , που είναι αληθής.

Σύνθεση.

Γράφουμε την τελευταία και φτάνουμε με τις προφανής συνεπαγωγές (αφού

λειτουργήσαμε Μαθηματικά Λογικά το ρήμα Αρκεί κατά την διαδικασία της

Ανάλυσης) στην σχέση που θέλουμε να αποδείξουμε.

Page 87: ΠΡΟΤΕΙΝΟΜΕΝΕΣ ΑΣΚΗΣΕΙΣ ΓΙΑ ΜΑΘΗΤΙΚΟΥΣ ΔΙΑΓΩΝΙΣΜΟΥΣ_ALL

http://www.mathematica.gr/forum/viewtopic.php?f=109&t=15584

Επιμέλεια: xr.tsif Σελίδα 87

ΘΕΜΑ 95Α (ΣΩΤΗΡΗΣ ΛΟΥΡΙΔΑΣ)

Αν x,y 0 και 5x 6y 7 να υπολογιστεί το maximum (= Μέγιστο) της

παράστασης 2 3A x y .

Λύση:

► Αν έχουμε σαν υπόθεση:

i

1 2 n

x 0, i 1,...,n

και

x x ... x c, c θετική σταθερά,

τότε το γινόμενο

1 2 nm m mA x x ... x , γίνεται Μέγιστο όταν

*1 2 n

i

1 2 n 1 2 n

x x x c... , m .

m m m m m ... m

Επισημαίνουμε εδώ ότι επί του πρακτέου ,πιθανόν, να χρειαστεί να

«συμπληρώσουμε» με σταθερούς παράγοντες και αυτό καθορίζεται από το

δοθέν σταθερό άθροισμα (γραμμικός συνδυασμός).

Μία τέτοια μεθοδολογική αντίληψη θα δούμε στην διαδικασία επίλυσης της

άσκησης 95 Α.

► Στη συγκεκριμένη, λοιπόν, άσκηση έχουμε:

Η δεδομένη σχέση 5x 6y 7 , μας οδηγεί (με βάση τα προηγούμενα) στην

αντίληψη ότι θα προσδιορίσουμε το ζητούμενο Μέγιστο μέσω του Μέγιστου

της παράστασης:

2 3 2 32 3

2 3

1 15x 6y , αφού x y 5x 6y .

5 6

Αυτό με βάση τα προηγούμενα επιτυγχάνεται όταν:

5

2 3

7max

5x 6y 5x 6y 5x 6y 7 14 7 7x ,y x y .

2 3 2 3 5 5 25 10 2·5

Page 88: ΠΡΟΤΕΙΝΟΜΕΝΕΣ ΑΣΚΗΣΕΙΣ ΓΙΑ ΜΑΘΗΤΙΚΟΥΣ ΔΙΑΓΩΝΙΣΜΟΥΣ_ALL

http://www.mathematica.gr/forum/viewtopic.php?f=109&t=15584

Επιμέλεια: xr.tsif Σελίδα 88

ΘΕΜΑ 96 (ΑΝΤΩΝΗΣ ΚΥΡΙΑΚΟΠΟΥΛΟΣ)

Οι αριθμοί: 1 2 7

a ,a ,...,a είναι θετικοί ακέραιοι και ισχύουν:

1 2 7a a ... a 333 Να αποδείξετε ότι το πηλίκο δύο τουλάχιστον εξ αυτών

ανήκει στο διάστημα:2 5

,5 2

.

Λύση:

Οι 7 διαφορετικοί ακέραιοι είναι από το 1 έως τον 332

(συμπεριλαμβανομένου).

Εξετάζουμε τα διαστήματα [1,2], [3,7] , [8,20] , [21,52],[53,132]. Όλα είναι

της μορφής [a,b] με b 5

a 2 . Αφού τα διαστήματα είναι έξι και οι ακέραιοι επτά,

υπάρχει διάστημα [a,b] από αυτά που περιέχει δύο εξ αυτών, έστω τους c,d με

c d .

Τότε 2 d b 5

15 c a 2 , όπως θέλαμε.

Β τρόπος

Καθένας από τους αριθμούς: 32 7

1 2 6

aa a, ,...,

a a a (1) είναι μεγαλύτερος ή ίσος το 1

και άρα μεγαλύτερος του 2

5. Αρκεί, λοιπόν, να αποδείξουμε ότι ένας

τουλάχιστον από τους αριθμούς αυτούς είναι μικρότερος ή ίσος του 5

2. Έστω

ότι αυτό δεν συμβαίνει. Τότε, καθένας από τους αριθμούς (1) θα είναι

μεγαλύτερος του 5

2. Έτσι θα έχουμε: 2

1

a

a

5

2 και επειδή a 1 , θα έχουμε:

Page 89: ΠΡΟΤΕΙΝΟΜΕΝΕΣ ΑΣΚΗΣΕΙΣ ΓΙΑ ΜΑΘΗΤΙΚΟΥΣ ΔΙΑΓΩΝΙΣΜΟΥΣ_ALL

http://www.mathematica.gr/forum/viewtopic.php?f=109&t=15584

Επιμέλεια: xr.tsif Σελίδα 89

2

5a

2 και άρα:

2a 3 . Έχουμε:

3

2 3 3

2

a 515

a 2 a a 82

a 3

. Όμοια

βρίσκουμε ότι: 4 5 6

a 21 , a 53 , a 133 και 7

a 333 , άτοπο.

ΘΕΜΑ 97 (ΑΝΤΩΝΗΣ ΚΥΡΙΑΚΟΠΟΥΛΟΣ)

Για τους πραγματικούς αριθμούς a,b,c ισχύουν: abc 1 και

ab bc ca a b c . Να αποδείξετε ότι ένας τουλάχιστον από τους αριθμούς

αυτούς είναι ίσος με το 1 .

Λύση:

Έχουμε

(a 1)(b 1)(c 1) abc (ab bc ca) (a b c) 1 abc 1 0 0 , άρα

ένας από τους a 1,b 1,c 1 είναι 0 .

Β τρόπος (παραλλαγή):

Από Vieta, οι αριθμοί a,b,c είναι οι ρίζες εξίσωσης της μορφής 3 2

x Ax Ax 1 0 (εδώ A ab bc ca a b c ).

Παρατηρούμε ότι η x 1 είναι (προφανής) ρίζα της ίδιας εξίσωσης. Άρα ένας

από τους αριθμούς είναι ο 1 .

Γ τρόπος

Λίγο απλούστερα, χωρίς Vieta:

Είναι (a 1)(b 1)(c 1) abc ab bc ca a b c 1 0, άρα ..

Δ τρόπος

abc 1 a 0 b 0 c 0

1 1 1abc 1 ab b bc

c ac a .

Page 90: ΠΡΟΤΕΙΝΟΜΕΝΕΣ ΑΣΚΗΣΕΙΣ ΓΙΑ ΜΑΘΗΤΙΚΟΥΣ ΔΙΑΓΩΝΙΣΜΟΥΣ_ALL

http://www.mathematica.gr/forum/viewtopic.php?f=109&t=15584

Επιμέλεια: xr.tsif Σελίδα 90

Τότε

2 2 2 21 1 1αb bc ca a b c ca a c a c c a a c 1 ac

c a ca

2 2 2 2 2(c a a c) (c 1) (a ac ) 0 a c(c 1) (c 1) a(c 1)(c 1) 0

2(c 1)(a c 1 ac a) 0 (c 1)(a 1)(ac 1) 0

1(c 1)(a 1)( 1) 0 c 1 ή a 1 ή b 1

b .

ΘΕΜΑ 98 (ΣΩΤΗΡΗΣ ΛΟΥΡΙΔΑΣ)

Να εξεταστεί αν ο συλλογισμός που ακολουθεί είναι ΣΩΣΤΟΣ ή ΛΑΘΟΣ

δίνοντας πλήρη εξήγηση της απάντησης σας.

« Υπάρχει τουλάχιστον μία τριάδα φυσικών αριθμών k, ,m με

(k )( m)(m k) 0 τέτοιοι ώστε: k m2 2 2 »

Λύση:

Έστω , χωρίς βλάβη της γενικότητας ότι k 1 . Τότε

m l k l2 2 (2 1)

και αφού k 12 1

περιττός ,πρέπει k l m2 1| 2

, άτοπο.

ΘΕΜΑ 99 (ΣΩΤΗΡΗΣ ΛΟΥΡΙΔΑΣ)

Δίνονται τρείς περιττοί φυσικοί αριθμοί. Υπάρχει ένας άλλος περιττός φυσικός

αριθμός, ώστε το άθροισμα των τετραγώνων των τεσσάρων αυτών αριθμών να

είναι επίσης τέλειο τετράγωνο;

Λύση:

Το άθροισμα S των τριών περιττών τετραγώνων είναι περιττός αριθμός.

Κάθε περιττός αριθμός αποτελεί διαφορά διαδοχικών τετραγώνων. Το S είναι

της μορφής 4k 3 , και άρα αποτελεί διαφορά περιττού τετραγώνου από άρτιο

τετράγωνο.

Page 91: ΠΡΟΤΕΙΝΟΜΕΝΕΣ ΑΣΚΗΣΕΙΣ ΓΙΑ ΜΑΘΗΤΙΚΟΥΣ ΔΙΑΓΩΝΙΣΜΟΥΣ_ALL

http://www.mathematica.gr/forum/viewtopic.php?f=109&t=15584

Επιμέλεια: xr.tsif Σελίδα 91

Άρα ισχύει το ζητούμενο.

ΘΕΜΑ 100 (ΑΝΤΩΝΗΣ ΚΥΡΙΑΚΟΠΟΥΛΟΣ)

Να βρείτε τους φυσικούς αριθμούς x,y,z και w , για τις οποίες ισχύουν:

2x 7y 35z 210w 839 , x 4 , y 5 και z 6 .

Λύση:

Έχουμε: 839 7(y 5z 30w) 2x (1).

Επειδή: 0 x 4 , έχουμε: 0 x 3 0 2x 6 7 0 2x 7 . Συνεπώς η

(1) είναι η ισότητα της διαίρεσης 839:7 .

Εκτελούμε τη διαίρεση αυτή και βρίσκουμε πηλίκο 119 και υπόλοιπο 6 .

Άρα: 2x 6 και y 5z 30w 119 (2). Συνεπώς: x 3 .

Από την ισότητα (2) έχουμε: 119 5(z 6w) y (3).

Επειδή: 0 y 5 η (3) είναι η ισότητα της διαίρεσης 119:5 , οπότε y 4 και

z 6w 23 .

Η ισότητα αυτή, επειδή 0 z 6 ,η (3) είναι η ισότητα της διαίρεσης 23:6 και

συνεπώς z 5 και w 3 .

Άρα: x 3 , y 4 , z 5 και w 3 .

Όπως βρίσκομαι εύκολα, οι αριθμοί αυτοί επαληθεύουν τις δοσμένες σχέσεις

και άρα είναι οι μοναδικοί ζητούμενοι.

Page 92: ΠΡΟΤΕΙΝΟΜΕΝΕΣ ΑΣΚΗΣΕΙΣ ΓΙΑ ΜΑΘΗΤΙΚΟΥΣ ΔΙΑΓΩΝΙΣΜΟΥΣ_ALL

http://www.mathematica.gr/forum/viewtopic.php?f=109&t=15584

Επιμέλεια : xr.tsif Σελίδα 1

Page 93: ΠΡΟΤΕΙΝΟΜΕΝΕΣ ΑΣΚΗΣΕΙΣ ΓΙΑ ΜΑΘΗΤΙΚΟΥΣ ΔΙΑΓΩΝΙΣΜΟΥΣ_ALL

http://www.mathematica.gr/forum/viewtopic.php?f=109&t=15584

Επιμέλεια : xr.tsif Σελίδα 2

ΠΡΟΤΕΙΝΟΜΕΝΕΣ

ΑΣΚΗΣΕΙΣ ΓΙΑ ΜΑΘΗΤΙΚΟΥΣ

ΔΙΑΓΩΝΙΣΜΟΥΣ

ΤΕΥΧΟΣ 2ο

ΑΣΚΗΣΕΙΣ 101 - 200

Αφιερωμένο σε κάθε μαθητή που ασχολείται ή πρόκειται να ασχοληθεί με

Μαθηματικούς διαγωνισμούς

Τσιφάκης Χρήστος : xr.tsif

Page 94: ΠΡΟΤΕΙΝΟΜΕΝΕΣ ΑΣΚΗΣΕΙΣ ΓΙΑ ΜΑΘΗΤΙΚΟΥΣ ΔΙΑΓΩΝΙΣΜΟΥΣ_ALL

http://www.mathematica.gr/forum/viewtopic.php?f=109&t=15584

Επιμέλεια : xr.tsif Σελίδα 3

ΘΕΜΑ 101 (ΣΩΤΗΡΗΣ ΛΟΥΡΙΔΑΣ)

Έχουμε 9 διαφορετικές ανά δύο τριάδες (x,y,z) με x,y,z Z . Να αποδειχθεί

ότι υπάρχει τουλάχιστον ένα ζεύγος από αυτές τις τριάδες, έστω 1 1 1

(x ,y ,z ) και

2 2 2(x ,y ,z ) στο οποίο αντιστοιχίζεται τριάδα 3 31 1 2 2

x yx y x y( , , )

2 2 2

με

3 31 1 2 2x yx y x y

, , Z2 2 2

.

Λύση:

Ένα ζεύγος (x,y,z) μπορεί να πάρει 2 2 2 8 διαφορετικές τιμές στο 3

2 (οι

διατεταγμένες τριάδες αριθμών mod2) άρα υπάρχουν δύο τριάδες ίσες στο

σύνολο αυτό. Αλλά a bmod2 δίνει a b

2

, οπότε το ζητούμενο έπεται.

ΘΕΜΑ 102 (Socrates)

Αν οι πραγματικοί αριθμοί x,y είναι τέτοιοι ώστε 2 2 x yx y

2

, να βρεθεί η

μέγιστη τιμή της διαφοράς x y .

Λύση:

Θέτω: 2 2 22(x y ) a  και (x y) b .

Τότε 2(x y) a b x y a b και η δοθείσα γράφεται:

2 2a a b 1 1 1a a b b a a [(a ) ]

2 2 2 4 4

.

Άρα 1 1 1

b b x y4 4 2

με την ισότητα για 1

a2

. Δηλαδή για την

ισότητα θέλουμε: 2 2 1 1 1

x y   ,x y   και x y4 2 2

.

Page 95: ΠΡΟΤΕΙΝΟΜΕΝΕΣ ΑΣΚΗΣΕΙΣ ΓΙΑ ΜΑΘΗΤΙΚΟΥΣ ΔΙΑΓΩΝΙΣΜΟΥΣ_ALL

http://www.mathematica.gr/forum/viewtopic.php?f=109&t=15584

Επιμέλεια : xr.tsif Σελίδα 4

Απο τις παραπάνω σχέσεις εύκολα βρίσκουμε ότι 1

(x,y) ( ,0)2

,

τιμές πού ικανοποιούν την αρχική σχέση.

ΘΕΜΑ 103 (Socrates)

Δίνεται τρίγωνο ABCκαι έστω M το μέσο της πλευράς BC και N σημείο της

AB τέτοιο ώστε NA 2AN . Αν η CAB CM N

, να βρεθεί ο λόγος των

πλευρών AC

BC.

Λύση:

Αν η MNτέμνει την προέκταση της AC στο T τότε το N είναι το βαρύκεντρο

του τριγώνου BCT . Αυτό μπορούμε να το δούμε με δύο τρόπους:

α) Από το θεώρημα του Μενελάου στο τρίγωνο ABC με διατέμνουσα την TNM

έχουμε :

TA MC NB· · 1

TC MB NA οπότε TA TC δηλαδή η AB είναι διάμεσος στο BCT

.

β) Αν προεκτείνουμε την AC προς το A κατά AC , τότε η AB θα είναι διάμεσος

στο BCT.

Οπότε το N είναι το βαρύκεντρο του BCT

. (μοναδικό σημείο που

χωρίζει την διάμεσο σε λόγο 2:1). Συνεπώς η TN περνά από το μέσο M της BC .

Επειδή CAB CM N

είναι και TAB TMB

οπότε το τετράπλευρο AMBT

είναι εγγράψιμο σε κύκλο. Από αυτό και την παραλληλία των AM,TB έχουμε

N AM TBA TM A

και CAM AMC

διαφορά ίσων γωνιών,

δηλαδή το τρίγωνο ACM είναι ισοσκελές με AC CM .

Ο ζητούμενος λόγος είναι, λοιπόν, AC CM CM 1

BC BC 2CM 2 .

Page 96: ΠΡΟΤΕΙΝΟΜΕΝΕΣ ΑΣΚΗΣΕΙΣ ΓΙΑ ΜΑΘΗΤΙΚΟΥΣ ΔΙΑΓΩΝΙΣΜΟΥΣ_ALL

http://www.mathematica.gr/forum/viewtopic.php?f=109&t=15584

Επιμέλεια : xr.tsif Σελίδα 5

ΘΕΜΑ 104 (Socrates)

Να βρεθεί σκακιέρα nxn ελάχιστου εμβαδού, η οποία μπορεί να καλυφθεί

(χωρίς επικαλύψεις) από ίσο αριθμό σχημάτων της μορφής:

Λύση:

Αν χρησιμοποιήσουμε a πλακίδια από κάθε είδος, τότε 24a 3a n , άρα 2

7a n ,

οπότε 7 / n . Για n 14 μπορούμε εύκολα να καλύψουμε το τετράγωνο με τα

πλακίδια (τις πρώτες δύο γραμμές τις καλύπτουμε βάζοντας αρχικά 4 ,2x2

τετραγωνάκια κι έπειτα φαίνεται πως θα καλύψουμε τα υπόλοιπα με 4 σχήματα

του άλλου τύπου. Το ίδιο θα κάνουμε και με τις υπόλοιπες γραμμές). Θα

αποδείξουμε ότι n 14 είναι η απάντηση.

Αρκεί να δείξουμε ότι δεν γίνεται ένα 7x7τετράγωνο να καλυφθεί εάν

χρησιμοποιήσουμε 7 σχήματα κάθε τύπου, δηλαδή 14 συνολικά σχήματα.

Θεωρούμε ένα τέτοιο τετράγωνο το οποίο μπορεί να καλυφθεί με αυτόν τον τρόπο.

Βάφουμε μαύρο το 1o,3o,5o,7o κελί των γραμμών 1,3,5,7 . Συνολικά έχουμε

βάψει 16 κελιά. Κάθε σχήμα καλύπτει 1το πολύ από τα βαμμένα κελιά. Άρα θα

χρειαστούν τουλάχιστον 16 σχήματα για να γίνει η κάλυψη, άτοπο.

ΘΕΜΑ 105 (KOMI)

Εάν x,y πραγματικοί μη μηδενικοί να δείξετε την 2 2 2 2

x y 2 2

x y xy x y

.

Λύση:

Από την ανισότητα 22 2

2 x y x y έχουμε ότι 2 2x y 2 x y άρα

αρκεί να δείξουμε πως

2 2

2

2 2 2 2

2 x y 2 20 x y

x y xy x y

που ισχύει.

Page 97: ΠΡΟΤΕΙΝΟΜΕΝΕΣ ΑΣΚΗΣΕΙΣ ΓΙΑ ΜΑΘΗΤΙΚΟΥΣ ΔΙΑΓΩΝΙΣΜΟΥΣ_ALL

http://www.mathematica.gr/forum/viewtopic.php?f=109&t=15584

Επιμέλεια : xr.tsif Σελίδα 6

ΑΝΑΛΥΤΙΚΗ ΛΥΣΗ

Γνωρίζουμε ότι 2(x y) 0 και άρα 2 2

x 2xy y 0 άρα 2 2x y 2xy . Αν

προσθέσω τώρα στην τελευταία ανισότητα το 2 2x y και στα δύο μέρη θα πάρω

2 2 2 2 22(x y ) x y 2xy (x y) . Επίσης, επειδή το 2 2

2(x y ) είναι μη

αρνητικό μπορούμε να πάρουμε τετραγωνικές ρίζες και να γράψουμε

2 2x y 2(x y ) .

Φυσικά όταν γίνουμε κάπως πιο έμπειροι δεν είναι ανάγκη να κάνουμε όλη αυτήν

την διαδικασία. Μπορούμε να γράφουμε απευθείας 2 2x y 2xy ή

2 2 22(x y ) (x y) διότι θεωρούνται απλές ανισότητες.

Θέλουμε τώρα να αποδείξουμε την 2 2 2 2

x y 2 2

x y xy x y

.

Επειδή 2 2

2 2 2 2

2(x y )x y

x y xy x y xy

(*)

αρκεί να δείξουμε ότι 2 2

2 2 2 2

2(x y ) 2 2

x y xy x y

(**)

Στην (*) χρειάζεται προσοχή. Για να ισχύσει δεν αρκεί μόνο ότι

2 2x y 2(x y ) . Πρέπει και το 2 2

x y xy να είναι θετικό. Αν xy 0 τότε

σίγουρα 2 2x y xy 0 . Αν xy 0 τότε από την προηγούμενη ανισότητα έχουμε

2 2x y xy 2xy xy xy 0 . (Η 2 2

x y xy 0 με ισότητα μόνο αν x y 0

είναι άλλη μια απλή ανισότητα που στην συνέχεια επιτρέπεται να χρησιμοποιούμε

απευθείας.)

Ας πάμε πίσω στο πρόβλημα. Μένει να αποδείξουμε την (**) ή ισοδύναμα την

2 2 2 2 2 22(x y ) x y 2 2(x y xy)

Ισοδύναμα, αρκεί (διαιρώντας με 2 και αφαιρώντας το αριστερό μέλος από το

δεξί) να δείξουμε ότι: 2 2 2 22(x y xy) (x y ) 0.

Page 98: ΠΡΟΤΕΙΝΟΜΕΝΕΣ ΑΣΚΗΣΕΙΣ ΓΙΑ ΜΑΘΗΤΙΚΟΥΣ ΔΙΑΓΩΝΙΣΜΟΥΣ_ALL

http://www.mathematica.gr/forum/viewtopic.php?f=109&t=15584

Επιμέλεια : xr.tsif Σελίδα 7

Το αριστερό όμως μέλος ισούται με 2 2 2x y 2xy (x y) το οποίο είναι

πράγματι μη αρνητικό.

ΘΕΜΑ 106 (ΣΠΥΡΟΣ ΚΑΠΕΛΛΙΔΗΣ)

Αν *a,b,c να αποδείξετε ότι:

Α) Αν 23α Q , τότε 3 α Q

B) Αν 3 5α b Q , τότε 3 α Q και 5 b Q

Γ) Αν 3 5 7α b c Q , τότε 3 α Q , 5 b Q , και 7 c Q .

Λύση:

Χωρίς βλάβη μπορούμε να υποθέσουμε (και θα το κάνουμε) ότι σε όλους τους

ρητούςa,b,c,... που εμφανίζονται , το κλάσμα που τους παριστάνει είναι απλό

(χωρίς κοινούς παράγοντες στον αριθμητή και στον παρονομαστή.)

Πρώτα δείχνουμε το

Λήμμα : Αν a απλό , αν m,n πρώτοι προς αλλήλους φυσικοί και αν

mn a , τότε υπάρχει ρητός dμε na d .

Αφήνω την απόδειξη γιατί είναι παραλλαγή της γνωστής για την περίπτωση

m 1 . (Η μοναδικότητα της ανάλυσης σε πρώτους παράγοντες καθιστά εύκολη

την απόδειξη).

Έτσι "αποδείχθηκε" το Α).

Β) Υψώνουμε στην τρίτη δύναμη, οπότε 35a b , και άρα 35 b . Από το

Λήμμα είναι 5 b . Από την υπόθεση 3 5α b Q έπεται τώρα ότι και

3 α Q .

Γ) Υψώνουμε στην 15η δύναμη, οπότε 5 3 157a b c , και άρα 157 c . Από το

Λήμμα είναι 7 c Q . Επίσης, η υπόθεση τώρα δίνει 3 5α b Q , που

εξετάστηκε στο B).

Page 99: ΠΡΟΤΕΙΝΟΜΕΝΕΣ ΑΣΚΗΣΕΙΣ ΓΙΑ ΜΑΘΗΤΙΚΟΥΣ ΔΙΑΓΩΝΙΣΜΟΥΣ_ALL

http://www.mathematica.gr/forum/viewtopic.php?f=109&t=15584

Επιμέλεια : xr.tsif Σελίδα 8

ΘΕΜΑ 107 (ΣΠΥΡΟΣ ΚΑΠΕΛΛΙΔΗΣ)

Αν ο n είναι ακέραιος, να αποδειχθεί ότι ο 2 2n 3n 2 n n 1 είναι

άρρητος.

Λύση:

2 2 20 (n 2) n 3n 2 (n 1)

2 2 20 n n n 1 (n 1) .

αφού τα υπόριζα δεν είναι τέλεια τετράγωνα, αλλά είναι θετικοί ακέραιοι, τότε

έχουν άρρητη ρίζα. πρόκειται για άρρητους με τετράγωνο ακέραιο, και άρα το

άθροισμα τους είναι επίσης άρρητος.(ή τουλάχιστον αυτό θυμάμαι να ισχύει σε

τέτοιου είδους άρρητους).

ΘΕΜΑ 108 (DEMETRES)

α) Δίνονται 501διαφορετικοί θετικοί ακέραιοι όλοι μικρότεροι ή ίσοι του 1000 .

Να δειχθεί ότι δύο από αυτούς είναι σχετικώς πρώτοι μεταξύ τους. (Δηλαδή ο

μέγιστος κοινός διαιρέτης τους ισούται με 1 .)

β) Δίνονται 501διαφορετικοί θετικοί ακέραιοι όλοι μικρότεροι ή ίσοι του 1000 .

Να δειχθεί ότι υπάρχουν δύο (διαφορετικοί) από αυτούς ώστε ο ένας να διαιρεί τον

άλλο.

Λύση:

Για την λύση θα χρησιμοποιήσουμε την αρχή του περιστερώνα που λέει ότι αν

έχουμε n 1 περιστέρια σε n φωλιές τότε σε τουλάχιστον μια φωλιά θα έχουμε

τουλάχιστον δύο περιστέρια.

Για το α) παίρνουμε τα {1,2},{3,4}, ,{999,1000} σαν τις φωλιές και τους 501

δοσμένους ακεραίους σαν τα περιστέρια. Έχουμε σύνολο 500 φωλιές και 501

περιστέρια οπότε μια φωλιά θα έχει τουλάχιστον δύο περιστέρια. Αυτό όμως

συνεπάγεται ότι τουλάχιστον δύο από αυτούς θα είναι διαδοχικοί και άρα σχετικώς

πρώτοι μεταξύ τους.

Page 100: ΠΡΟΤΕΙΝΟΜΕΝΕΣ ΑΣΚΗΣΕΙΣ ΓΙΑ ΜΑΘΗΤΙΚΟΥΣ ΔΙΑΓΩΝΙΣΜΟΥΣ_ALL

http://www.mathematica.gr/forum/viewtopic.php?f=109&t=15584

Επιμέλεια : xr.tsif Σελίδα 9

Για το β) παίρνουμε τα 1,3,5, ,999 σαν τις φωλιές. Για κάθε ένα από τους

δοσμένους θετικούς ακεραίους τον γράφουμε σαν a2 b όπου b περιττός και τον

τοποθετούμε στην φωλιά b . Για παράδειγμα τον 20 τον γράφουμε σαν 22 5 και

άρα τον τοποθετούμε στην φωλιά 5 , τον 87 τον γράφουμε σαν 02 87 και τον

τοποθετούμε στην φωλιά 87 κ.τ.λ. Από την αρχή του περιστερώνα μια φωλιά,

έστω η b θα έχει τουλάχιστον δυο περιστέρια. Αυτά θα προέρχονται από δυο

αριθμούς της μορφής 1a

1n 2 b και 2a

2n 2 b . Χωρίς βλάβη της γενικότητας

1 2a a . Αλλά τότε

1 2n / n και τελειώσαμε.

ΘΕΜΑ 109 (DEMETRES)

Ο Ανδρέας και ο Βασίλης παίζουν το εξής παιγνίδι. Έχουν στον πίνακα

γραμμένους τους αριθμούς από το 1 μέχρι το 100 . Παίζει πρώτος ο Ανδρέας.

Σβήνει δυο αριθμούς, όποιους θέλει, και γράφει στον πίνακα την διαφορά τους.

Μετά κάνει το ίδιο ο Βασίλης και συνεχίζουν εναλλάξ μέχρι να μείνει μόνο ένας

αριθμός γραμμένος στον πίνακα. Αν είναι περιττός κερδίζει ο Ανδρέας ενώ αν

είναι άρτιος κερδίζει ο Βασίλης. Ποιος από τους δύο έχει στρατηγική νίκης;

Λύση:

Θα πω πως ο Βασίλης έχει στρατηγική νίκης αν και κάνει 49 κινήσεις έναντι των

50 κινήσεων του Ανδρέα. Από το 1 μέχρι το 100 υπάρχουν 50 άρτιοι και 50

περιττοί.

Ο Βασίλης επιδιώκει να αφήσει στις τελευταίες κινήσεις πλήθος άρτιων αριθμών,

πράγμα και το οποίο μπορεί να επιτευχθεί πιο εύκολα (σε σχέση με τους

περιττούς) αφού άρτιος – άρτιος = άρτιος και περιττός – περιττός = άρτιος, ενώ

άρτιος – περιττός ή περιττός – άρτιος = περιττός.

Συγκεκριμένα στην 48η κίνησή του ο Βασίλης επιθυμεί 5 άρτιους είτε 3 άρτιους

και 2 περιττούς, αφού αυτοί οι συνδυασμοί θα τους εξασφαλίζουν την νίκη καθώς

στην 49η κίνηση του Ανδρέα θα μείνουν 4 άρτιοι είτε 2 άρτιοι και 2 περιττοί,

συνδυασμός πράγματι νικηφόρος για τον Βασίλη αφού στην 49η και τελευταία

κίνηση του θα έχει 3 άρτιους ή 2 περιττούς και 1 άρτιο.

Page 101: ΠΡΟΤΕΙΝΟΜΕΝΕΣ ΑΣΚΗΣΕΙΣ ΓΙΑ ΜΑΘΗΤΙΚΟΥΣ ΔΙΑΓΩΝΙΣΜΟΥΣ_ALL

http://www.mathematica.gr/forum/viewtopic.php?f=109&t=15584

Επιμέλεια : xr.tsif Σελίδα 10

Λοιπόν ονομάζουμε x τους 50 άρτιους και y τους 50 περιττούς. Οι 2 παίκτες

μπορούν να κάνουν τις εξής κινήσεις:

(1)  • : x 1 , y σταθερό

(2)  • : x 1 ,y 2

Όπως είπαμε προηγουμένως ο Βασίλης επιδιώκει να αφήσει άρτιους για το τέλος.

Συνεπώς ο Ανδρέας προσπαθεί να τον πολεμήσει εμφανίζοντας στον πίνακα

περιττούς ( δηλαδή κάνει την κίνηση (1)).

Στο σύνολο θα γίνουν 99 κινήσεις και ο Βασίλης επιθυμώντας να αφανίσει τους

περιττούς θα προβεί στην κίνηση (2), 25 φορές και έτσι θα γίνει y 0 , x 75 .

Παράλληλα, ο Ανδρέας θα κάνει την κίνηση (1), 50 φορές, η οποία θα γίνει 24

φορές και από τον Βασίλη αναγκαστικά. Συνεπώς στο τέλος θα ισχύει x 1 , y 0

και νικητής ανακηρύσσεται ο Βασίλης.

Β τρόπος

Κερδίζει ο Βασίλης ακόμη και αν παίξει στα κουτουρού! Δηλαδή δεν χρειάζεται

στρατηγική. Κερδίζει έτσι και αλλιώς.

Πράγματι, με κάθε κίνηση (του Ανδρέα ή του Βασίλη)

[i]το άθροισμα όλων των αριθμών στον πίνακα αλλάζει κατά x y (x y) 2y

αν επιλέξει τους x,y . Δηλαδή αλλάζει κατά άρτια ποσότητα.

Στην αρχή το άθροισμα όλων των αριθμών είναι άρτιος ( 5050 ). Άρα σε κάθε

στάδιο, ακόμη και στο τελευταίο, παραμένει άρτιος.

ΘΕΜΑ 110 (DEMETRES)

Έχουμε τρεις στοίβες με σπίρτα. Η πρώτη έχει 2010 σπίρτα, η δεύτερη 2011 και

η τρίτη 2012 . Σε κάθε κίνηση μπορούμε να επιλέξουμε δύο στοίβες που έχουν

σπίρτα, να πάρουμε ένα σπίρτο από την κάθε μία και να τοποθετήσουμε και τα δύο

στην τρίτη. Να εξεταστεί αν μπορούν όλα τα σπίρτα να μεταφερθούν σε κάποια

από τις στοίβες.

Page 102: ΠΡΟΤΕΙΝΟΜΕΝΕΣ ΑΣΚΗΣΕΙΣ ΓΙΑ ΜΑΘΗΤΙΚΟΥΣ ΔΙΑΓΩΝΙΣΜΟΥΣ_ALL

http://www.mathematica.gr/forum/viewtopic.php?f=109&t=15584

Επιμέλεια : xr.tsif Σελίδα 11

Λύση:

Βάζουμε αξία 0 ευρώ σε κάθε σπίρτο της πρώτης στοίβας, 1 ευρώ σε κάθε σπίρτο

της δεύτερης, και 2 στης τρίτης.

Αυτή την στιγμή η συνολική αξία των σπίρτων είναι

2010 0 2011 1 2012 2 0 1 4mod3 2mod3 .

Μπορούμε να διαπιστώσουμε ότι με κάθε κίνηση η συνολική αξία όλων των

σπίρτων παραμένει 2mod3 . Πράγματι, η μεταφορά σπίρτου από την πρώτη και

την δεύτερη στοίβα στην τρίτη αλλάζει την συνολική αξία κατά 2 2 (0 1) 3

(αυξήθηκε 2 2 αλλά χάσαμε 0 1 , σύνολο 3 ). Όμοια η μεταφορά από την

πρώτη και τρίτη στοίβα στην δεύτερη αλλάζει την συνολική αξία κατά

1 1 (0 2) 0 , και από την δεύτερη και τρίτη στην πρώτη αλλάζει την

συνολική αξία κατά 0 0 (1 2) 3 . Όπως και να είναι, οι αλλαγές είναι

πολλαπλάσια του 3 , που δείχνει το ζητούμενο.

Τώρα, δεν μπορούν να μεταφερθούν όλα τα 2010 2011 2012 6033 σπίρτα

στην πρώτη στοίβα γιατί η συνολική αξία θα ήταν 6033 0 0mod3 2mod3 .

Όμοια δεν μεταφέρονται στην δεύτερη γιατί 6033 1 0mod3 2mod3 , ούτε

στην τρίτη γιατί 6033 2 0mod3 2mod3 .

Άρα δεν μπορούν όλα τα σπίρτα να μεταφερθούν στην ίδια στοίβα.

ΘΕΜΑ 111 (DEMETRES)

Η Λερναία Ύδρα έχει επτά κεφάλια. Ο Ηρακλής προσπαθεί να την σκοτώσει

κόβοντας τα κεφάλια της. Κάθε φορά όμως που κόβει ένα κεφάλι η Λερναία Ύδρα

βγάζει τέσσερα καινούργια κεφάλια. Κάθε φορά που της κόβει δύο κεφάλια βγάζει

είκοσι καινούργια ενώ κάθε φορά που της κόβει τρία κεφάλια δεν βγάζει κανένα

καινούργιο κεφάλι. Για να την σκοτώσει πρέπει να της κόψει όλα τα κεφάλια και

αυτή να μην βγάλει κανένα καινούργιο κεφάλι. Μπορεί να τα καταφέρει;

Λύση:

Παρατηρούμε το εξής: όταν ο Ηρακλής κόψει 1 κεφάλι από το τέρας τότε το

πλήθος των κεφαλιών του θα αυξηθεί κατά 1 4 3 δηλαδή πολ3 , όταν κόψει

Page 103: ΠΡΟΤΕΙΝΟΜΕΝΕΣ ΑΣΚΗΣΕΙΣ ΓΙΑ ΜΑΘΗΤΙΚΟΥΣ ΔΙΑΓΩΝΙΣΜΟΥΣ_ALL

http://www.mathematica.gr/forum/viewtopic.php?f=109&t=15584

Επιμέλεια : xr.tsif Σελίδα 12

2 τότε θα αποκτήσει 2 20 18 πολ3 παραπάνω κεφάλια και όταν κόψει 3

χάνει πάλι πολ3 κεφάλια.

Ο αρχικός αριθμός όμως είναι 7 πολ3 άρα είναι αδύνατο να φθάσει στο 0 (αφού

του προσθαφαιρούνται πολλαπλάσια του 3 ).

ΘΕΜΑ 112 (DEMETRES)

Στον πίνακα έχουμε γραμμένους τους αριθμούς 2,3,10 . Σε κάθε βήμα

μπορούμε να πάρουμε δύο από αυτούς έστω τους x,y και να τους

αντικαταστήσουμε με τους 3x 4y

x΄5

και

4x 3yy΄

5

. Να εξεταστεί αν

μπορούμε μετά από κάποια βήματα να καταλήξουμε στην τριάδα 1,7,8 .

Λύση:

Σε κάθε βήμα το άθροισμα των τετραγώνων των τριών αριθμών δεν αλλάζει (= η

αναλλοίωτή μας).

Πράγματι,

2 2

2 2 2 23x 4y 4x 3y(x') (y') ... x y

5 5

Αφού στην αρχή ήταν 2 2 2 2 2 22 3 10 113 114 1 7 8 , δεν θα καταφέρουμε

να πάμε από την 2,3,10 στην 1,7,8 .

ΘΕΜΑ 113 (DEMETRES)

Σε κάθε τετραγωνάκι μιας σκακιέρας γράφουμε ένα μη αρνητικό ακέραιο. Σε

κάθε βήμα επιτρέπεται να διαλέξουμε δυο γειτονικά τετραγωνάκια και να

αφαιρέσουμε από αυτά τον ίδιο ακέραιο, με την προϋπόθεση ότι οι αριθμοί που θα

μείνουν θα εξακολουθούν να είναι μη αρνητικοί ακέραιοι. Να βρεθεί ικανή και

αναγκαία συνθήκη ώστε να μπορούμε με μια ακολουθία επιτρεπόμενων βημάτων

να κάνουμε όλους τους αριθμούς ίσους με 0 .

Λύση:

Ικανή και αναγκαία συνθήκη: Το άθροισμα όλων των αριθμών στα μαύρα

τετραγωνάκια ισούται με το άθροισμα όλων των αριθμών στα άσπρα. (*)

Page 104: ΠΡΟΤΕΙΝΟΜΕΝΕΣ ΑΣΚΗΣΕΙΣ ΓΙΑ ΜΑΘΗΤΙΚΟΥΣ ΔΙΑΓΩΝΙΣΜΟΥΣ_ALL

http://www.mathematica.gr/forum/viewtopic.php?f=109&t=15584

Επιμέλεια : xr.tsif Σελίδα 13

Πράγματι, η αλλαγή που κάνουμε (δεδομένου ότι δύο γειτονικά τετραγωνάκια

είναι πάντα το μεν ένα μαύρο το δε άλλο άσπρο) διατηρεί την παραπάνω ισότητα.

Τώρα, αφού στο τέλος ισχύει προφανώς η εν λόγω ισότητα, τότε πρέπει να ισχύει

και στην αρχή. Αυτό δείχνει ότι η συνθήκη που έγραψα είναι αναγκαία.

Μένει να δείξουμε ότι είναι ικανή.

Θα δούμε πρώτα ότι μπορούμε να κάνουμε όλους τους αριθμούς ίσους. Από κει,

να τους κάνουμε 0 είναι τετριμμένο.

Σχεδιάζουμε μία γραμμή στην σκακιέρα μας η οποία πάει οριζόντια και κάθετα,

από τετραγωνάκι σε γειτονικό τετραγωνάκι, αλλά βαδίζουμε "βουστροφηδόν".

Δηλαδή όπως όταν γράφουμε αλλά με την διαφορά ότι όταν φτάσουμε στο τέλος

της γραμμής, δεν πάμε στην αρχή της επόμενης αλλά στο τέλος της, και μετά

περπατάμε ανάποδα. Με άλλα λόγια, προχωράμε όπως ακριβώς όταν οργώνουμε

ένα χωράφι με βόδια και άροτρο.

Τώρα, χωρίς βλάβη το αρχικό τετράγωνο έχει μεγαλύτερο αριθμό από το γειτονικό

του (αλλιώς μπορούμε να μεριμνήσουμε να το κάνουμε μεγαλύτερο).

Χρησιμοποιούμε το τρίτο τετραγωνάκι της σειράς για να αυξήσουμε το δεύτερο

μέχρι να γίνει ίσο με το πρώτο. Ομοίως, επαγωγικά, χρησιμοποιούμε το n 2

τετραγωνάκι (της βουστροφηδόν πορείας μας) για να κάνουμε το n 1

τετραγωνάκι ίσο με το n . Μπορούμε να μεριμνήσουμε να είναι θετικοί όλοι οι

αριθμοί που βλέπουμε: εν ανάγκη προσθέτουμε θετικούς, σύμφωνα πάντα με τον

κανόνα του παιχνιδιού, σε όσους έχουμε διευθετήσει.

Έτσι θα καταφέρουμε να κάνουμε όλους τους αριθμούς ίσους, εκτός ίσως τον

τελευταίο. Αλλά αυτός είναι αυτόματα ίσος με τον προηγούμενό του λόγω της

συνθήκης (*) (απλό). Τελειώσαμε.

ΘΕΜΑ 114 (DEMETRES)

Έστω n 4 θετικός ακέραιος και αριθμοί 1 2 n

x ,x ,...,x ώστε κάθε ένας από

αυτούς να ισούται είτε με 1 είτε με 1 .

Αν 1 2 3 4 2 3 4 5 n 1 2 3

x x x x x x x x ... x x x x 0 να δειχθεί ότι ο n είναι

πολλαπλάσιο του 4 .

Page 105: ΠΡΟΤΕΙΝΟΜΕΝΕΣ ΑΣΚΗΣΕΙΣ ΓΙΑ ΜΑΘΗΤΙΚΟΥΣ ΔΙΑΓΩΝΙΣΜΟΥΣ_ALL

http://www.mathematica.gr/forum/viewtopic.php?f=109&t=15584

Επιμέλεια : xr.tsif Σελίδα 14

Λύση:

Θα ξεκινήσουμε με δύο παρατηρήσεις.

1. Έχουμε n όρους που ισούνται είτε με 1 είτε με 1 και αθροίζονται σε 0, άρα

σίγουρα ο n είναι άρτιος έστω n 2k, k N .

2. To γινόμενο των όρων του αθροίσματος είναι ίσο με 4

1 2 3 n(x x x ...x ) 1

Έχουμε όμως ότι το γινόμενο των n αυτών όρων είναι ίσο με k k1 ( 1) από όπου

προκύπτει ότι και ο k είναι άρτιος σύμφωνα με τη δεύτερη παρατήρηση.

Άρα έχουμε:n 2k 2(2 ) 4  , N και τελειώσαμε.

Β τρόπος

Μιας και ο σκοπός της άσκησης είναι να εξοικειωθούμε με την χρήση

αναλλοίωτων δίνω και μια διαφορετική απάντηση.

Ας πάρουμε κάποιο i

x που ισούται με 1 και ας το αλλάξουμε σε 1 . Υπάρχουν

ακριβώς τέσσερις όροι που περιλαμβάνουν το i

x και θα αλλάξουν όλοι πρόσημο.

Αν π.χ. ήταν και οι τέσσερις ίσοι με 1 θα γίνουν και οι τέσσερις ίσοι με 1 , αν

τρεις ήταν ίσοι με 1 και ένας με 1 τότε τρεις θα γίνουν ίσοι με 1 και ένας με 1

κ.τ.λ. Τι αλλαγή γίνεται στο άθροισμα 1 2 3 4 2 3 4 5 n 1 2 3

S x x x x x x x x x x x x ;

Το S δεν είναι αναλλοίωτο αλλά αν κοιτάξουμε μία προς μια τις πιθανές

περιπτώσεις, είτε θα αυξηθεί κατά 4 ή 8 , είτε θα μειωθεί κατά 4 ή 8 είτε θα

μείνει το ίδιο. Σε όλες λοιπόν τις περιπτώσεις αυτό που μένει αναλλοίωτο είναι το

υπόλοιπο της διαίρεσης του S με το 4 . Και εφ' όσων αρχικά είχαμε S 0 τότε

όσες αλλαγές και να κάνουμε θα έχουμε τελικά ότι το S είναι πολλαπλάσιο του 4 .

Αν όμως κάνουμε όλες τις αλλαγές από 1 σε 1 το S θα γίνει ίσο με n . Άρα

λοιπόν το n πρέπει απαραίτητα να είναι πολλαπλάσιο του 4 .

ΘΕΜΑ 115 (ΑΝΤΩΝΗΣ ΚΥΡΙΑΚΟΠΟΥΛΟΣ)

Θεωρούμε ένα τρίγωνο ΑΒΓ . Παίρνουμε, στην πλευρά του ΑΒ ένα σημείοΔ

με 5

ΑΔ ΑΒ12

, στην πλευρά του ΒΓ ένα σημείο Ε με 1

ΒΕ ΒΓ3

και στην

Page 106: ΠΡΟΤΕΙΝΟΜΕΝΕΣ ΑΣΚΗΣΕΙΣ ΓΙΑ ΜΑΘΗΤΙΚΟΥΣ ΔΙΑΓΩΝΙΣΜΟΥΣ_ALL

http://www.mathematica.gr/forum/viewtopic.php?f=109&t=15584

Επιμέλεια : xr.tsif Σελίδα 15

πλευρά του ΓΑ ένα σημείο Ζ με 1

ΓΖ ΓΑ4

. Τα τμήματα ΑΕ και ΒΖ τέμνονται

στο σημείο Κ , τα ΒΖ και ΓΔ στο σημείο Λ και τα ΓΔ και ΑΕ στο σημείο Μ .

Να αποδείξετε ότι: (ΚΛΜ) (ΑΜΔ) (ΒΚΕ) (ΓΛΖ) .

Λύση:

Ονομάζουμε α

υ το ύψος του τριγώνου ΑΒΓ από το Α . Έχουμε:

α

1(ΑΒΓ) ΒΓ υ

2 και

α α

1 1 1 1(ABE) BE·υ BΓ υ (ABΓ)

2 2 3 3 . Όμοια

βρίσκουμε ότι: 1

(BΓZ) (ABΓ)4

και5

(ΓAΔ) (ABΓ)12

. Έτσι, προσθέτοντας

κατά μέλη, βρίσκουμε:

(ABΓ) (ΑΒΕ) (ΒΓΖ) (ΓΑΔ) (ΑΜΔ) (ΔΒΚΜ) (ΒΚΕ) (ΕΓΛΚ)

(ΓΛΖ) (ΖΑΜΛ) (ΚΛΜ) (ΑΔΜ) (ΔΒΚΜ) (ΒΕΚ) (ΒΚΕ)

(ΕΓΛΚ) (ΓΖΛ) (ΓΛΖ) (ΖΑΜΛ) (ΑΜΔ)

(ΚΛΜ) (ΑΜΔ) ΒΚΕ) (ΓΛΖ) .

Page 107: ΠΡΟΤΕΙΝΟΜΕΝΕΣ ΑΣΚΗΣΕΙΣ ΓΙΑ ΜΑΘΗΤΙΚΟΥΣ ΔΙΑΓΩΝΙΣΜΟΥΣ_ALL

http://www.mathematica.gr/forum/viewtopic.php?f=109&t=15584

Επιμέλεια : xr.tsif Σελίδα 16

ΘΕΜΑ 116 (ΔΗΜΗΤΡΗΣ ΙΩΑΝΝΟΥ)

Στις Δημοτικές εκλογές της 1ης Κυριακής (13 Οκτωβρίου 2002) σε ένα Δήμο

συμμετείχαν οι συνδυασμοί Α,Β και Γ . Ονομάζουμε ν τον αριθμό των

εγγεγραμμένων στους εκλογικούς καταλόγους ψηφοφόρων. Συνολικά ψήφισε το

75% του αριθμού ν και όλα τα ψηφοδέλτια ήταν έγκυρα. Ο συνδυασμός A

ψηφίστηκε από το 39% του αριθμού ν ενώ ο συνδυασμός B από το 27%του ν .

Λευκά δεν βρέθηκαν.

α) Να εξετάσετε αν ο αρχηγός του συνδυασμού A εξελέγη Δήμαρχος από την 1η

Κυριακή (δηλαδή αν έλαβε ποσοστό μεγαλύτερο του 50% ως προς τον αριθμό

των έγκυρων ψηφοδελτίων).

β) Να βρείτε το ποσοστό των ψήφων του συνδυασμού Γ ως προς τον αριθμό των

έγκυρων ψηφοδελτίων.

Λύση:

α) Αν οι εγγεγραμμένοι στους εκλογικούς καταλόγους ήταν 100 , τότε θα ψήφιζαν

οι 75 και ο συνδυασμός A θα έπαιρνε 39 στους 75 που ψήφισαν. Δηλαδή ο

συνδυασμός A πήρε 39

75 του συνόλου των έγκυρων ψηφοδελτίων, δηλαδή

ποσοστό 52% . Άρα εξελέγη από την πρώτη Κυριακή.

β) Ο συνδυασμός Γ πήρε (75 39 27)% 9% του ν . Δηλαδή πήρε το 9

75

του συνόλου των έγκυρων ψηφοδελτίων και άρα ποσοστό 12% .

ΘΕΜΑ 117 (ΔΗΜΗΤΡΗΣ ΙΩΑΝΝΟΥ)

Να προσδιορίσετε όλους τους διψήφιους αριθμούς που είναι ίσοι με το γινόμενο

που προκύπτει αν πολλαπλασιάσουμε τα ψηφία τους αυξημένα κατά 1 .

Λύση:

Έστω ο αριθμός που ψάχνω είναι ο ab . Άρα πρέπει να ισχύει ότι:

Page 108: ΠΡΟΤΕΙΝΟΜΕΝΕΣ ΑΣΚΗΣΕΙΣ ΓΙΑ ΜΑΘΗΤΙΚΟΥΣ ΔΙΑΓΩΝΙΣΜΟΥΣ_ALL

http://www.mathematica.gr/forum/viewtopic.php?f=109&t=15584

Επιμέλεια : xr.tsif Σελίδα 17

ab (a 1)(b 1) 10a b ab a b 1 9a ab 1 a(9 b) 1

19 b

a , και επειδή ο b πρέπει να είναι μονοψήφιος φυσικός επειδή είναι

ψηφίο, επομένως και το 9 b πρέπει να είναι φυσικός, το a πρέπει να διαιρεί το

1 δηλαδή a 1 . Άρα : 9 b 1 b 8 . Άρα ab 18 .

ΘΕΜΑ 118 (Socrates)

Χρησιμοποιώντας τους μετασχηματισμούς 2 1f (x) x f ( 1)

x και

2 1f(x) (x 1) f ( )

x 1

είναι δυνατόν από το τριώνυμο 2

1f (x) x 4x 3 να

προκύψει το 2

2f (x) x 10x 9 ;

Λύση:

Έχουμε ότι 2

1f (x) x 4x 3 , η οποία με βάση τον πρώτο μετασχηματισμό

γίνεται

2

2 2 21 1x 1 4 1 3 (x 1) 4x(x 1) 3x

x x

.

Με βάση τον δεύτερο μετασχηματισμό αυτή γίνεται 2

2 2 2

1

1 1 1 1(x 1) ( 1) 4 ( 1) 3 x 4x 3 f (x)

x 1 x 1 x 1 x 1

.

Δηλαδή παραμένει η ίδια.

Β τρόπος

Η τυχαία f(x) μέσω του 1ου μετασχηματισμού γίνεται 2 1

x f ( 1)x , εν συνεχεία

μέσω του 2ου μετασχηματισμού έχουμε : 2

2

1 1(x 1) f 1

1(x 1)

x 1

που είναι

η ιδία η f(x) . Ομοίως αν κάνω με την ανάποδη σειρά τους μετασχηματισμούς .

Page 109: ΠΡΟΤΕΙΝΟΜΕΝΕΣ ΑΣΚΗΣΕΙΣ ΓΙΑ ΜΑΘΗΤΙΚΟΥΣ ΔΙΑΓΩΝΙΣΜΟΥΣ_ALL

http://www.mathematica.gr/forum/viewtopic.php?f=109&t=15584

Επιμέλεια : xr.tsif Σελίδα 18

Άρα οι μετασχηματισμοί αφού μόνοι τους δεν μετατρέπουν την 1f (x)σε

2f (x) και

αφού αν τους χρησιμοποιήσω σε ζεύγη είναι αντίστροφη διαδικασία και δεν

αλλοιώνουν την όποια 1f (x) ,συμπεραίνω ότι δεν ειναι δυνατόν.

ΘΕΜΑ 119 (ΔΗΜΗΤΡΗΣ ΙΩΑΝΝΟΥ)

Να βρείτε την τιμή της παράστασης:

3 2 1 2004 0b 1 bK a (1 a) 4( ) [( 2004) ]

a 2 a

αν είναι 3

a2

και b 3 .

Λύση:

3 2 1 2004 0b 1 bK a (1 a) 4( ) [( 2004) ]

a 2 a

3 1

2

3 1 1 27 2 27 8( ) 4( 2 ) 1 4 4( ) 1 3

32 2 8 3 8 3(1 )

2

81 64 72 217

24 24 24 24 .

ΘΕΜΑ 120 (ΔΗΜΗΤΡΗΣ ΙΩΑΝΝΟΥ)

Σε μια διοργάνωση σκακιού μέσω διαδικτύου, συμμετείχαν 1119 αγόρια και

κορίτσια. Το πρώτο κορίτσι έπαιξε με 20 αγόρια, το δεύτερο κορίτσι έπαιξε με 21

αγόρια, το τρίτο κορίτσι με 22 αγόρια κ.ο.κ μέχρι το τελευταίο κορίτσι που έπαιξε

με όλα τα αγόρια. Να βρείτε πόσα ήταν τα κορίτσια και πόσα τα αγόρια.

Λύση:

Το άθροισμα της αύξουσας σειράς των κοριτσιών με το πλήθος των αγοριών που

παίζει κάθε κορίτσι σε κάθε παιχνίδι είναι περιττό. Δηλαδή 21,23,25,... . Άρα από

το 21 μέχρι το 1119 υπάρχουν 550 περιττοί . Άρα τα κορίτσια είναι 550 οπότε με

αφαίρεση βρίσκω ότι τα αγόρια είναι 569 .

Page 110: ΠΡΟΤΕΙΝΟΜΕΝΕΣ ΑΣΚΗΣΕΙΣ ΓΙΑ ΜΑΘΗΤΙΚΟΥΣ ΔΙΑΓΩΝΙΣΜΟΥΣ_ALL

http://www.mathematica.gr/forum/viewtopic.php?f=109&t=15584

Επιμέλεια : xr.tsif Σελίδα 19

ΘΕΜΑ 121 (ΔΗΜΗΤΡΗΣ ΙΩΑΝΝΟΥ)

Θεωρούμε τετράγωνο πλευράς a , a 1 . Το τετράγωνο που έχει πλευρά κατά 1

μικρότερη του a , έχει περίμετρο ίση αριθμητικά προς το εμβαδόν του αρχικού

τετραγώνου. Να βρεθεί η πλευρά a .

Λύση:

Έστω A το τετράγωνο με πλευρά a και B το τετράγωνο με πλευρά a 1 .

Άρα έχω ότι: 2 2 2Π(B) E(A) 4(a 1) a 4a 4 a a 4a 4 0

Επομένως : 2Δ ( 4) 4 1 4 16 16 0 και

β Δ β 4a 2

2a 2 2

. Άρα a 2 .

Αλλιώς: 2 2

a 4a 4 0 (a 2) 0 a 2 0 a 2 .

Page 111: ΠΡΟΤΕΙΝΟΜΕΝΕΣ ΑΣΚΗΣΕΙΣ ΓΙΑ ΜΑΘΗΤΙΚΟΥΣ ΔΙΑΓΩΝΙΣΜΟΥΣ_ALL

http://www.mathematica.gr/forum/viewtopic.php?f=109&t=15584

Επιμέλεια : xr.tsif Σελίδα 20

ΘΕΜΑ 122 (ΔΗΜΗΤΡΗΣ ΙΩΑΝΝΟΥ)

Οι αριθμοί x,y,z,w έχουν την ιδιότητα: « Αν προσθέσουμε τρεις οποιουσδήποτε

από αυτούς και από το άθροισμα που θα προκύψει αφαιρέσουμε τον αριθμό 5 ,

προκύπτει πάντα ο αριθμός 2002 ». Να υπολογίσετε το άθροισμα x y z w .

Λύση:

Υπάρχουν 4 περιπτώσεις :

x y z 5 2002 x y z 2007

x y w 5 2002 x y w 2007

y z w 5 2002 y z w 2007

x w z 5 2002 x w z 2007

Κάνοντας πρόσθεση κατά μέλη έχω ότι:

3x 3y 3z 3w 8028 3(x y z w) 8028 x y z w 2676 .

ΘΕΜΑ 123 (ΑΝΤΩΝΗΣ ΚΥΡΙΑΚΟΠΟΥΛΟΣ)

Να βρείτε τους ακέραιους αριθμούς x,y , για τις οποίος ισχύει: 2

5(x y) 2x 4y 5xy 9 .

Λύση:

Έστω (x,y) μια λύση της εξίσωσης.

Τότε 2

5x 5y 2 5x 4 5y 5x 5y 45 οπότε για a 5x και b 5y είναι

22 2 2

(a b) 2a 4b ab 45 2a 2ab 2b 4a 8b 90

2 2 2(a b) (a 2) (b 4) 110 (1).

Άρα 2(a b) 110 και έτσι a b 11 .

Page 112: ΠΡΟΤΕΙΝΟΜΕΝΕΣ ΑΣΚΗΣΕΙΣ ΓΙΑ ΜΑΘΗΤΙΚΟΥΣ ΔΙΑΓΩΝΙΣΜΟΥΣ_ALL

http://www.mathematica.gr/forum/viewtopic.php?f=109&t=15584

Επιμέλεια : xr.tsif Σελίδα 21

Ο a b είναι πολλαπλάσιο του 5 άρα a b 0,5, 5,10, 10 .

• Αν a b 0 τότε η (1) δίνει 2 2(a 2) (a 4) 110 που δεν έχει ακέραιες

λύσεις.

• Αν a b 5 τότε η (1) δίνει 2a 3a 40 0 οπότε a 5 ή a 7

(απορρίπτεται).

Για a 5 είναι b 10 οπότε x 1 και y 2 .

• Ανa b 5 τότε η (1) δίνει 2a 7a 0 οπότε a 0 ή a 8 (απορρίπτεται).

Για a 0 είναι b 5 οπότε x 0 και y 1 .

• Αν a b 10 τότε η (1) δίνει 2a 8a 15 0 οπότε a 5 ή a 3

(απορρίπτεται).

Για a 5 είναι b 5 οπότε x 1 και y 1 .

• Αν a b 10 τότε η (1) δίνει 2a 12a 95 0 που δεν έχει ακέραιες λύσεις.

Οι λύσεις είναι τα ζεύγη 1,2 , 0, 1 , 1,1 .

Β τρόπος Έχουμε:

2 2 2 25(x y 2xy) 2x 4y 5xy 9 5y (5x 4)y 5x 2x 9 0

2 2 2Δ (5x 4) 20(5x 2x 9) 75x 196

14 3 14 3 14 3Δ 0 x 3 x 3

15 15 15

Και αφού ο x είναι ακέραιος, θα πρέπει x 1 ή x 0 ή x 1

Αν x 1 τότε έχουμε:

2 25y 9y 5 2 9 0 5y 9y 2 0

Δ 81 40 121

Page 113: ΠΡΟΤΕΙΝΟΜΕΝΕΣ ΑΣΚΗΣΕΙΣ ΓΙΑ ΜΑΘΗΤΙΚΟΥΣ ΔΙΑΓΩΝΙΣΜΟΥΣ_ALL

http://www.mathematica.gr/forum/viewtopic.php?f=109&t=15584

Επιμέλεια : xr.tsif Σελίδα 22

Άρα 9 121 9 11

y 210 10

(εφ όσον ο y είναι ακέραιος

Αν x 0 τότε ομοίως βρίσκουμε y 1

Αν x 1 όμοια βρίσκουμε ότι y 1 .

ΘΕΜΑ 124 (DEMETRES)

Δίνεται ένας πραγματικός αριθμός x . Να δειχθεί ότι υπάρχουν ακέραιοι m,n με

1 n 4 ώστε 1

m nx3

.

Και για να την "δυσκολέψουμε" λίγο (*) δείξτε ότι αν N N* , τότε υπάρχουν

ακέραιοι m,n με 1 n N 1 ώστε1

m nxN

.

Λύση:

Έστω οι N 1 αριθμοί x [x],2x [2x],..., N 1 x [ N 1 x] . Όλοι τους

ανήκουν στο διάστημα 0,1 . Το 0,1 χωρίζεται σε N υποδιαστήματα

1 1 2 N 10, , , , ..., ,1

N N N N

.

Από την αρχή της περιστεροφωλιάς δύο από τους N 1 αριθμούς, έστω οι

kx [kx],sx [sx] με 1 k s N 1 θα ανήκουν στο ίδιο υποδιάστημα πλάτους

1

N. Άρα

1s k x [sx] [kx]

N .

Θέτουμε n s k και m [sx] [kx] . Τότε 1 n s k N .

Παρατήρηση:

Το σύμβολο λοιπόν [x]διαβάζεται:

Το ακέραιο μέρος του (πραγματικού) αριθμού x .

Page 114: ΠΡΟΤΕΙΝΟΜΕΝΕΣ ΑΣΚΗΣΕΙΣ ΓΙΑ ΜΑΘΗΤΙΚΟΥΣ ΔΙΑΓΩΝΙΣΜΟΥΣ_ALL

http://www.mathematica.gr/forum/viewtopic.php?f=109&t=15584

Επιμέλεια : xr.tsif Σελίδα 23

Για παράδειγμα, έχουμε: [3,254] 3 , [0,376] 0 ,[ 2,35] 3 , [7] 7 .

Δηλαδή το ακέραιο μέρος ενός δεκαδικού αριθμού είναι ο αμέσως προηγούμενος

ακέραιος από αυτόν τον αριθμό.

Αν ο αριθμός x είναι ακέραιος, τότε ισχύει [x] x .

Μάλιστα, ισχύει η εξής ανισότητα: [x] x [x] 1 .

Επίσης αν x είναι πραγματικός αριθμός και k ακέραιος, τότε [x k] [x] k .

ΘΕΜΑ 125 (ΣΩΤΗΡΗΣ ΛΟΥΡΙΔΑΣ)

Αν ισχύουν 2 2 2 2

a,b,c,d 0

a b c d 20

a b c d 200

τότε να δείξετε ότι a,b,c,d 5(1 3).

Λύση:

Γνωρίζουμε, ότι ισχύει 2 2 2 23(b c d ) (b c d) , για κάθε b,c,d .

Επομένως, λόγω των συνθηκών, θα έχουμε 2 23(200 a ) (20 a) δηλαδή

2a 10a 50 0 και από εδώ προκύπτει, ότι a 5(1 3).

Ομοίως και για τα b,c,d .

ΘΕΜΑ 126 (Socrates)

Αν a,b,c πραγματικοί αριθμοί τέτοιοι ώστε 4 4 4

1a b c να δείξετε ότι

5 5 5 1(ab) (bc) (ca)

4 .

Λύση:

Έχουμε 4 4 4 2 8 8 8 4 4 4 4 4 4 4 4 4 4 4 4

1 (a b c ) a b c 2(a b b c c a ) 3(a b b c c a )

Page 115: ΠΡΟΤΕΙΝΟΜΕΝΕΣ ΑΣΚΗΣΕΙΣ ΓΙΑ ΜΑΘΗΤΙΚΟΥΣ ΔΙΑΓΩΝΙΣΜΟΥΣ_ALL

http://www.mathematica.gr/forum/viewtopic.php?f=109&t=15584

Επιμέλεια : xr.tsif Σελίδα 24

Άρα 4 4 4 4 4 4 1a b b c c a

3 (*)

Έχουμε επίσης 4 4 4 3 12 12 12 8 4 4 8 8 4 8 41 (a b c ) a b c 3(a b a b ) 3(b c c b )

8 4 8 4 4 4 43(c a a c ) 6a b c .

Χρησιμοποιούμε ότι : 12 12 12 6 6 6 6 6 6a b c a b b c c a

ότι : 8 4 4 8 6 6a b a b 2a b κ.τ.λ. για να καταλήξουμε ότι

6 6 6 6 6 6 4 4 4 6 6 6 6 6 61 7(a b b c c a ) 6a b c 7(a b b c c a ) .

Δηλαδή 6 6 6 6 6 6 1a b b c c a

7 (**)

Όμως 5 5 4 4 6 62a b a b a b κ.τ.λ. άρα από τις (*) και (**) παίρνουμε

5 5 5 5 5 5 1 1 1 5 1a b b c c a .

2 3 7 21 4

Β τρόπος

Διαφορετικά:

Είναι 4 4 4 2 2

1 a b c 2a b , οπότε 2

ab .2

Άρα 5 5 5 4 4 4 4 4 4 4 4 4 22 2 2 1

(ab) (bc) (ca) (a b b c c a ) (a b c ) .2 6 6 4

Η άσκηση είναι από εδώ: http://forum.gil.ro/viewtopic.php?f=19&t=1080

ΘΕΜΑ 127 (ΣΩΤΗΡΗΣ ΛΟΥΡΙΔΑΣ)

Πόσες είναι οι παραγοντοποιήσεις του 441000 σε δύο παράγοντες μεγαλύτερους

της μονάδας που οι παράγοντες αυτοί να είναι πρώτοι μεταξύ τους;

(*) Δύο θετικοί ακέραιοι m,n 1 είναι πρώτοι μεταξύ τους, όταν ο Μέγιστος

Κοινός Διαιρέτης τους είναι 1 , συμβολίζουμε (m,n) 1 .

Page 116: ΠΡΟΤΕΙΝΟΜΕΝΕΣ ΑΣΚΗΣΕΙΣ ΓΙΑ ΜΑΘΗΤΙΚΟΥΣ ΔΙΑΓΩΝΙΣΜΟΥΣ_ALL

http://www.mathematica.gr/forum/viewtopic.php?f=109&t=15584

Επιμέλεια : xr.tsif Σελίδα 25

Λύση:

Έχουμε 3 2 3 2441.000 2 ·3 ·5 ·7 . Αν τώρα ο ένας από τους δυο παράγοντες διαιρείται

με το 2 , τότε ο άλλος δεν πρέπει να διαιρείται με το 2 . Δηλαδή αυτός που

διαιρείται με το 2 πρέπει να διαιρείται και με το 32 . Ομοίως και για τους άλλους

παράγοντες. Άρα αρκεί να βρούμε με πόσους τρόπους μπορούμε να χωρίσουμε τα

2,3,5,7 σε δυο μη κενές ομάδες.

Π.χ. αν η μια ομάδα περιέχει το 2 και η άλλη τα 3,5,7 οι παράγοντες θα είναι ο 3

2 και ο 2 3 23 5 7 . Επειδή οι αριθμοί είναι μικροί μπορούμε τον διαχωρισμό να το

κάνουμε με το χέρι. Βρίσκουμε

2 / 3,5,7 , 2,3 / 5,7 , 2,5 / 3,7 , 2,7 / 3,5 , 2,3,5 / 7 , 2,3,7 / 5 , 2,5,7 / 3

συνολικά δηλαδή 7 διαχωρισμούς.

Αλλιώς μπορούμε να το κάνουμε ως εξής. Αποφασίζουμε αν το 2 θα μπει στην

πρώτη ή στην δεύτερη ομάδα. Έχουμε συνολικά 2 τρόπους να αποφασίσουμε.

Μετά αποφασίζουμε αν το 3 θα μπει στην πρώτη ή στην δεύτερη ομάδα με

άλλους δυο τρόπους κ.τ.λ. Συνολικά έχουμε 42 16 τρόπους. Όμως δεν βρήκαμε

ακριβώς αυτό που ζητάμε. Για παράδειγμα μετρήσαμε δυο τρόπους με τους

οποίους είτε η πρώτη είτε η δεύτερη ομάδα δεν θα έχουν κανένα αριθμό. Αυτό

απαγορεύεται και πρέπει να τους αφαιρέσουμε. Άρα συνολικά μέχρι τώρα

16 2 14 τρόποι. Πάλι όμως δεν έχουμε την σωστή απάντηση. Αν π.χ. βάλουμε

τον 2 στην πρώτη ομάδα και τους 3,5,7 στην δεύτερη ή το αντίστροφο ο

διαχωρισμός είναι ο ίδιος. Μετρήσαμε λοιπόν κάθε διαχωρισμό δυο φορές. Άρα το

σωστό είναι 14

72 το οποίο συμφωνεί με την προηγούμενη απάντηση.

Εννοείται ότι η δεύτερη μέθοδος γενικεύεται την στιγμή που η πρώτη όχι. Π.χ. αν

ο Σωτήρης ήταν πιο κακός και μας ζητούσε

«Πόσες είναι οι παραγοντοποιήσεις του 100! σε δύο παράγοντες μεγαλύτερους

της μονάδας που οι παράγοντες αυτοί να είναι πρώτοι μεταξύ τους;

Τότε με το ίδιο σκεπτικό θα έπρεπε αρχικά να βρούμε πόσοι υπάρχουν από το 1

ως το 100 ( η απάντηση είναι 25 ) και μετά θα έπρεπε να βρούμε με πόσους

τρόπους μπορούμε να χωρίσουμε αυτούς τους 25 αριθμούς σε δύο ομάδες.

Page 117: ΠΡΟΤΕΙΝΟΜΕΝΕΣ ΑΣΚΗΣΕΙΣ ΓΙΑ ΜΑΘΗΤΙΚΟΥΣ ΔΙΑΓΩΝΙΣΜΟΥΣ_ALL

http://www.mathematica.gr/forum/viewtopic.php?f=109&t=15584

Επιμέλεια : xr.tsif Σελίδα 26

Ακολουθώντας το σκεπτικό της δεύτερης μεθόδου βρίσκουμε 25

242 22 1

2

τρόποι. Ο αριθμός αυτός είναι τεράστιος. και η πρώτη μέθοδο της καταγραφής

όλων των περιπτώσεων θα αποτύγχανε οικτρά.

ΘΕΜΑ 128 (DEMETRES)

α) Πόσοι αναγραμματισμοί υπάρχουν της λέξης ΣΗΜΕΡΑ ; (Π.χ. το ΗΜΡΑΕΣ

είναι ένας τέτοιος αναγραμματισμός. Δεν είναι απαραίτητο ο αναγραμματισμός να

έχει νόημα.)

β) Πόσοι αναγραμματισμοί υπάρχουν της λέξης ΜΑΘΗΜΑΤΙΚΑ ;

γ) Σε πόσους από τους αναγραμματισμούς της λέξης ΜΑΘΗΜΑΤΙΚΑ δεν

εμφανίζονται δυο συνεχόμενα όμοια γράμματα. (Π.χ. απαγορεύουμε τον

αναγραμματισμό ΜΑΑΘΗΜΑΤΙΚ , κ.τ.λ.)

Λύση:

Κάνω την αρχή βάζοντας την λύση για το (α): Ας προσπαθήσουμε να φτιάξουμε

ένα αναγραμματισμό.

Ως πρώτο γράμμα μπορούμε να διαλέξουμε ένα οποιοδήποτε εκ των

Σ,Η,Μ,Ε,Ρ,Α . Έχουμε δηλαδή 6 διαφορετικές επιλογές.

Αφ' ότου το επιλέξουμε για το δεύτερο γράμμα, όποιο και αν ήταν το πρώτο

γράμμα έχουμε 5 διαφορετικές επιλογές.

Π.χ. αν το πρώτο γράμμα ήταν το Σ , θα μπορούσαμε για δεύτερο γράμμα να

επιλέξουμε οποιοδήποτε από τα Η,Μ,Ε,Ρ,Α αλλά όχι το Σ .

Συνολικά για τα πρώτα δυο γράμματα του αναγραμματισμού έχουμε 6 5 30

επιλογές. Συνεχίζοντας με αυτόν τον τρόπο συνολικά θα έχουμε

6 5 ... 2 1 720 διαφορετικούς αναγραμματισμούς.

Επειδή τέτοιοι πολλαπλασιασμοί εμφανίζονται συχνά όταν θέλουμε να

μετρήσουμε κάτι χρησιμοποιούμε ένα ειδικό σύμβολο το οποίο ονομάζουμε

παραγοντικό.

Page 118: ΠΡΟΤΕΙΝΟΜΕΝΕΣ ΑΣΚΗΣΕΙΣ ΓΙΑ ΜΑΘΗΤΙΚΟΥΣ ΔΙΑΓΩΝΙΣΜΟΥΣ_ALL

http://www.mathematica.gr/forum/viewtopic.php?f=109&t=15584

Επιμέλεια : xr.tsif Σελίδα 27

Εν συντομία η απάντηση είναι 6! όπου γενικά το n! σημαίνει να

πολλαπλασιάσουμε όλους τους θετικούς ακεραίους από το 1 ως τοn .

Αν προσπαθήσουμε να κάνουμε κάτι παρόμοιο για το (β) θα συναντήσουμε

περισσότερες δυσκολίες.

Για το πρώτο γράμμα έχουμε 7 επιλογές. Αν όμως το πρώτο γράμμα είναι το M ή

το A μετά πάλι έχουμε 7 επιλογές ενώ αν είναι ένα από τα Θ,Η,Τ,Ι,Κ μετά θα

έχουμε μόνο 6 επιλογές.

Πρέπει λοιπόν να σκεφτούμε κάτι πιο έξυπνο...

Β τρόπος

Ας δώσω μια υπόδειξη: Θεωρούμε αρχικά ότι τα δύο M είναι διαφορετικά μεταξύ

του (ας τα πούμε 1 2

M ,M ), όπως επίσης ότι είναι διαφορετικά μεταξύ τους τα τρία

A (ας τα πούμε 31 2

A ,A ,A ). Δηλαδή ψάχνω (αρχικά) τους αναγραμματισμούς της

λέξης 1 1 2 2 3

M A ΘHM A TIKA . Αφού τους βρω, ξανακάνω τα M ίδια. Δηλαδή οι

λέξεις 1 2

...M ...M ... και 12

...M ...M ... είναι ίδιες. Άρα διαιρώ δια 2 , αφού τις

μέτρησα όλες δύο φορές.

Χμμμ, και με τα τρία A τι γίνεται; No problem. Θα διαιρέσω με τον αριθμό "εσύ

θα μου πεις" για τα μην διπλομετρήσω τίποτα.

Και μία υπόδειξη για το γ) . Βρες πρώτα με πόσους τρόπους τα δύο M και τα τρία

A είναι δίπλα – δίπλα και αφαίρεσε από το συνολικό πλήθος.

Πώς θα μετρήσουμε πόσες φορές τα δύο M και τα τρία A είναι διπλά δίπλα;

Καλή ερώτηση.

Να ένας τρόπος: "Κολλάμε" τα δύο M μεταξύ τους, και τα τρία A μεταξύ τους.

Δηλαδή αν M MM,A AAA , πόσοι αναγραμματισμοί υπάρχουν της λέξης

MAΘHTIK ; Χρειάζεται να συνεχίσω; Όχι βέβαια! Ας χαρούν οι μαθητές μας

την συμπλήρωση των λεπτομερειών.

ΘΕΜΑ 129 (ΜΠΑΜΠΗΣ ΣΤΕΡΓΙΟΥ)

Να βρείτε τον ακέραιο αριθμό x από την παρακάτω σχέση :

2008x 2009 2009x 2010

2010x 2011 2011x 2012

.

Page 119: ΠΡΟΤΕΙΝΟΜΕΝΕΣ ΑΣΚΗΣΕΙΣ ΓΙΑ ΜΑΘΗΤΙΚΟΥΣ ΔΙΑΓΩΝΙΣΜΟΥΣ_ALL

http://www.mathematica.gr/forum/viewtopic.php?f=109&t=15584

Επιμέλεια : xr.tsif Σελίδα 28

Λύση:

Κάνω χιαστί:

(2008x 2009)(2011x 2012) (2010x 2011)(2009x 2010)

(2008x 2009)(2009x 2x 2010 2) (2008x 2x 2009 2)(2009x 2010)

Έστω a 2008 , b 2009 , c 2010

(ax b)[bx c 2(x 1)] [ax b 2(x 1)](bx c)

2 2 2 2abx acx 2ax(x 1) b x bc 2b(x 1) abx b x 2bx(x 1)

acx cb 2c(x 1) 2ax(x 1) 2b(x 1) 2bx(x 1) 2c(x 1)

2ax(x 1) 2b(x 1) 2bx(x 1) 2c(x 1) 0 2(x 1)(ax b bx c) 0

Άρα x 1 0 x 1 ή

ax b bx c 0 2008x 2009 2009x 2010 0 x 1 .

Β τρόπος

Μια σπουδαία ιδιότητα των αναλογιών είναι η εξής:

Αν x z

y w τότε

x z x z

y w y w

(με την προϋπόθεση να μην μηδενίζεται κάποιος παρονομαστής)

Έχουμε λοιπόν

2008x 2009 2009x 2010 2008x 2009 2009x 2010 x 11

2010x 2011 2011x 2012 2010x 2011 2011x 2012 x 1

Άρα 2008x 2009

1 2008x 2009 2010x 2011 2x 2 x 12010x 2011

.

H τιμή όμως αυτή που βρήκαμε μηδενίζει τον παρονομαστή κάποιων από τα

παραπάνω κλάσματα και άρα δεν μπορούμε να την δεχτούμε. Μοιάζει λοιπόν η

εξίσωση αδύνατη. Όμως η ιδιότητα των αναλογιών που χρησιμοποιήσαμε ισχύει

Page 120: ΠΡΟΤΕΙΝΟΜΕΝΕΣ ΑΣΚΗΣΕΙΣ ΓΙΑ ΜΑΘΗΤΙΚΟΥΣ ΔΙΑΓΩΝΙΣΜΟΥΣ_ALL

http://www.mathematica.gr/forum/viewtopic.php?f=109&t=15584

Επιμέλεια : xr.tsif Σελίδα 29

με την προϋπόθεση ότι ο παρονομαστής του κλάσματος που προκύπτει να είναι

διάφορος του μηδενός.

Με άλλα λόγια αν οι παρονομαστές των δύο ίσων κλασμάτων είναι ίσοι τότε δεν

μπορούμε να εφαρμόσουμε την ιδιότητα.

Εξετάζοντας λοιπόν την περίπτωση να είναι ίσοι οι παρονομαστές της δοσμένης

εξίσωσης, βρίσκουμε x 1 . Η τιμή αυτή παρατηρούμε ότι επαληθεύει την

εξίσωση. Άρα τελικά είναι η λύση της.

ΘΕΜΑ 130 (ΑΝΤΩΝΗΣ ΚΥΡΙΑΚΟΠΟΥΛΟΣ)

Να βρείτε τους πραγματικούς αριθμούς x,y,z,w ,για τις οποίους ισχύουν:

x y z w 4 (1) και 8 8 8 8 4x y z w (2).

Λύση:

Με χρήση της

8 8 8 8 8a b c d a b c d

(*)4 4

για a,b,c,d 0 , με

ισότητα αν και μόνον αν a b c d (βλέπε παρακάτω) έχουμε

8 8 8 8 8 8x y z w | x | | y | | z | | w | | x | | y | | z | |

y |

1 14 4 4

άρα | x | | y | | z | | w | 1 . Συνεπώς x,y,z,w 1 . Εύκολα διαπιστώνουμε από την

πρώτη εξίσωση ότι η μόνη εφικτή είναι η x y z w 1 .

(*) βγαίνει με πολλαπλή χρήση της C – S:

2 2 2 2 4 4 4 4a b c d 1 1 1 1 a b c d 2 4 a b c d ... και

λοιπά.

Άλλος τρόπος: πολλαπλή χρήση της 2 2 2(a b) 2(a b ) .

Β τρόπος

Είναι 2

4 8 4x 1 x 2x 1

Page 121: ΠΡΟΤΕΙΝΟΜΕΝΕΣ ΑΣΚΗΣΕΙΣ ΓΙΑ ΜΑΘΗΤΙΚΟΥΣ ΔΙΑΓΩΝΙΣΜΟΥΣ_ALL

http://www.mathematica.gr/forum/viewtopic.php?f=109&t=15584

Επιμέλεια : xr.tsif Σελίδα 30

2

2 4 2x 1 x 2x 1

2 2

x 1 x 2x 1

Πολλαπλασιάζουμε τη 2η με 2 , την 3η με 4 και προσθέτουμε κατά μέλη.

Προκύπτει

2 2 24 2 8

x 1 2 x 1 4 x 1 x 8x 7 .

Επαναλαμβάνουμε τα ίδια και για τους y,z,w . Προσθέτοντας προκύπτει

2 2 2 22 24 2 4 2

x 1 2 x 1 4 x 1 y 1 2 y 1 4 y 1

2 2 2 22 24 2 4 2

z 1 2 z 1 4 z 1 w 1 2 w 1 4 w 1

8 8 8 8x y z w 8 x y z w 28 .

Από υπόθεση το 2ο μέλος ισούται με 4 8· 4 28 0 . Άρα και το 1ο μέλος

ισούται με 0 .

Το 1ο μέλος είναι άθροισμα τετραγώνων, άρα όλα αυτά τα τετράγωνα είναι 0 .

Επομένως x y z w 1 .

ΘΕΜΑ 131 (VZF)

Ανa,b,c είναι περιττοί ακέραιοι, αποδείξτε ότι η 2ax bx c 0 , δεν έχει ρητή

λύση.

Λύση:

Αρκεί να δείξουμε ότι η Διακρίνουσα του τριωνύμου δεν είναι τέλειο τετράγωνο.

Πριν πάμε όμως σε αυτό θα κάνουμε την εξής παρατήρηση: Τα δυνατά υπόλοιπα

ενός τέλειου τετραγώνου στην διαίρεση του με το 8 είναι 0,1 και 4 . Πράγματι,

κάνοντας τις πράξεις σε κάθε περίπτωση (n 8k 1 , n 8k 2 , ...) καταλήγουμε

σε αυτό το συμπέρασμα.

Page 122: ΠΡΟΤΕΙΝΟΜΕΝΕΣ ΑΣΚΗΣΕΙΣ ΓΙΑ ΜΑΘΗΤΙΚΟΥΣ ΔΙΑΓΩΝΙΣΜΟΥΣ_ALL

http://www.mathematica.gr/forum/viewtopic.php?f=109&t=15584

Επιμέλεια : xr.tsif Σελίδα 31

Επιστρέφουμε στο πρόβλημα και έχουμε 2b 4ac . Αφού όμως a,b,c

περιττοί ακέραιοι μπορούμε να γράψουμε:

2b 8 1 (ως τετράγωνο περιττού)

c 2m 1

a 2n 1

Άρα 8 1 4(2n 1)(2m 1) 8 1 4(2k 1) 8 1 8k 4

πoλ8 3 πoλ8 5

το οποίο δεν μπορεί να είναι τέλειο τετράγωνο σύμφωνα με τα παραπάνω.

Β τρόπος

Αν p

q ρητή ρίζα, μπορούμε χωρίς βλάβη να υποθέσουμε ότι οι p,q δεν είναι και

οι δύο άρτιοι.

Είναι 2 2ap bpq cq 0 .

Αν p,q περιττοί τότε 2 20 ap bpq cq =(περιττός) +(περιττός)

+(περιττός) =(περιττός) , άτοπο.

Αν p άρτιος, qπεριττός τότε 2 20 ap bpq cq =(άρτιος) + (άρτιος)

+(περιττός) = (περιττός), άτοπο.

Αν p περιττός, qάρτιος τότε 2 20 ap bpq cq = (περιττός) + (άρτιος) +

(άρτιος) =(περιττός), άτοπο.

ΘΕΜΑ 132 (VZF)

Οι τιμές των a,b,c,d είναι 1,2,3,4 όχι απαραίτητα με αυτή τη σειρά. Ποιά είναι

η μέγιστη πιθανή τιμή του ab bc cd da ;

Page 123: ΠΡΟΤΕΙΝΟΜΕΝΕΣ ΑΣΚΗΣΕΙΣ ΓΙΑ ΜΑΘΗΤΙΚΟΥΣ ΔΙΑΓΩΝΙΣΜΟΥΣ_ALL

http://www.mathematica.gr/forum/viewtopic.php?f=109&t=15584

Επιμέλεια : xr.tsif Σελίδα 32

Λύση:

Θα χρησιμοποιήσουμε τη βασική ανισότητα 2x yxy ( )

2

ως εξής:

2 2a b c d 10ab bc cd da (a c)(b d) ( ) ( ) 25

2 2

Επομένως η ζητούμενη τιμή είναι 25 και λαμβάνεται όταν a c b d 5 .

Β τρόπος

Αφού τα a,b,c,d παίρνουν διαφορετικές τιμές μεταξύ τους

έχουμε με δοκιμές ότι (a c) (b d) (4 1) (2 3) 25 .

ΘΕΜΑ 133 (VZF)

Αποδείξτε ότι αν r s t τότε 2 2 2 2r s t (r s t) .

Λύση:

Ισοδύναμα έχουμε ότι: 22 2

r s r s t r s 2t r s t

2

r s r s r s 2t r s 0

r s r s r s 2t 0 2 r s s t 0 .

Β τρόπος

Θα γράψω άλλη μία λύση γιατί χρησιμοποιεί μία τεχνική που είναι πάντα χρήσιμη.

Έχει το πλεονέκτημα ότι βγάζει "από μόνη της την ανισότητα χωρίς να

σκεφτούμε".

Αφού r s t , μπορούμε να θέσουμε s t u,r t u v όπουu, v 0 . Είναι

τότε

Αριστερό μέλος = 2 2 2

r s t

Δεξί μέλος 2 2 2 2(r s t) (t v) t 2vt v

Page 124: ΠΡΟΤΕΙΝΟΜΕΝΕΣ ΑΣΚΗΣΕΙΣ ΓΙΑ ΜΑΘΗΤΙΚΟΥΣ ΔΙΑΓΩΝΙΣΜΟΥΣ_ALL

http://www.mathematica.gr/forum/viewtopic.php?f=109&t=15584

Επιμέλεια : xr.tsif Σελίδα 33

Είναι τώρα προφανές ότι το αριστερό μέλος είναι από το δεξί (αφού η διαφορά

τους 2uv 0 ).

Το πλεονέκτημα της μεθόδου είναι ότι επειδή τα t,u,v είναι ελεύθερα (δεν

γνωρίζει το ένα το άλλο) το τελικό αποδεικτέο είναι πάντα "προφανές".

ΘΕΜΑ 134 (Socrates)

Βρείτε όλα τα ζεύγη φυσικών αριθμών (x,y) τέτοια ώστε

2x(xy 2y 3) (x y)(3x y) .

Λύση:

Μετά τις πράξεις η εξίσωση γίνεται 2 2y 2x x 4 y 3x 6x .

Συμπληρώνουμε το τετράγωνο στο 1ο μέλος:

2 22 2

y x x 4 x x 4 3x 6x.

Θέτουμε 22 2 4 3 2

P x x x 4 3x 6x x 8x 13x 6x.

Έχουμε λύση όταν για x φυσικό το P(x) είναι τέλειο τετράγωνο.

Είναι 2

2 4 3 2x 4x 1 x 8x 14x 8x 1.

Ακόμα 2

2 4 3 2x 4x 2 x 8x 12x 16x 4.

2

2 4 3 2x 4x 3 x 8x 10x 24x 9.

Παρατηρούμε ότι 2

2P(x) x 4x 1 για κάθε x φυσικό.

Ακόμα ισχύει ότι 2

2 2P(x) x 4x 3 x 10x 3 x x 10 3.

Άρα για κάθε φυσικό x 11 ισχύει 2 2

2 2x 4x 3 P(x) x 4x 1 .

Page 125: ΠΡΟΤΕΙΝΟΜΕΝΕΣ ΑΣΚΗΣΕΙΣ ΓΙΑ ΜΑΘΗΤΙΚΟΥΣ ΔΙΑΓΩΝΙΣΜΟΥΣ_ALL

http://www.mathematica.gr/forum/viewtopic.php?f=109&t=15584

Επιμέλεια : xr.tsif Σελίδα 34

Αν το P(x) είναι τέλειο τετράγωνο και x 11 τότε 2

2P(x) x 4x 2 .

Η τελευταία όμως δεν έχει ακέραιες λύσεις.

Μένει να εξετάσουμε τι γίνεται για x 10 Βρίσκουμε ότι τα μόνα τέλεια

τετράγωνα είναι τα

P(0) P(1) P(6) 0,P(8) 16·49.

• Για x 0 : 2y 0 y 0.

• Για x 1 : 2(y 3) 0 y 3. (απορρίπτεται)

• Για x 6 : 2(y 12) 0 y 12.

• Για x 8 : 2 2(y 32) 28 y 60 ή y 4 .

ΘΕΜΑ 135 (Socrates)

Αν a,b θετικοί ακέραιοι a b , τέτοιοι ώστε ο ab(a b) να διαιρεί τον 3 3

a b ab να δείξετε ότι ο αριθμός ab είναι τέλειος κύβος.

Λύση:

Έστω d (a,b) Τότε υπάρχουν θετικοί ακέραιοι x y με a xd ,b yd και

(x,y) 1 .

Άρα 3 3 3 3 3 2xy(x y)d / d x d y d xy οπότε 3 3

xy(x y)d / dx dy xy .

Επομένως 3x / dy και εφόσον (x,y) 1 θα είναι x / d .

Ομοίως 3y / dx και εφόσον (x,y) 1 θα είναι y / d .

Επομένως xy / d Άρα υπάρχει θετικός ακέραιος k ώστε d kxy .

Άρα 3 3xy(x y)d / kxyx kxyy xy οπότε 3 3

(x y)xyk / kx ky 1 .

Επομένως 3 3k / kx ky 1 οπότε k 1 . Έτσι

32ab xyd xy .

Page 126: ΠΡΟΤΕΙΝΟΜΕΝΕΣ ΑΣΚΗΣΕΙΣ ΓΙΑ ΜΑΘΗΤΙΚΟΥΣ ΔΙΑΓΩΝΙΣΜΟΥΣ_ALL

http://www.mathematica.gr/forum/viewtopic.php?f=109&t=15584

Επιμέλεια : xr.tsif Σελίδα 35

ΘΕΜΑ 136 (ΔΗΜΗΤΡΗΣ ΙΩΑΝΝΟΥ)

Αν a,b θετικοί ακέραιοι και αν 3a 4b 120 , να αποδείξετε ότι: 30 a b 40 .

Λύση:

για a 0 ,b 30 ικανοποιείται η εξίσωση οπότε οι a,b που είναι λύσεις της

διοφαντικής εξίσωσης έχουν τη μορφή a 4t,b 30 3t όπου ο t είναι ακέραιος.

Αφού οι a,b είναι θετικοί πρέπει να ισχύουν a 4t 0 t 0 και

b 0 30 3t 0 t 10 . οπότε πρέπει τελικά 0 t 10 .

Άρα η ζητούμενη ανισότητα γράφεται

30 a b 40 30 4t 30 3t 40 0 t 10 , που ισχύει.

Β τρόπος Αλλιώς: 3a 3b 3a 4b 120 , άρα a b 40 .

4a 4b 3a 4b 120 , άρα a b 30 .

ΘΕΜΑ 137 (ΔΗΜΗΤΡΗΣ ΙΩΑΝΝΟΥ)

Αν ο Μέγας Αλέξανδρος πέθαινε 9 χρόνια αργότερα, θα βασίλευε το μισό της

ζωής του. Αν όμως πέθαινε 9 χρόνια νωρίτερα, θα βασίλευε το1

8 της ζωής του.

Πόσα χρόνια έζησε;

Λύση:

Ας υποθέσουμε ότι ο Μέγας Αλέξανδρος ανέλαβε την βασιλεία σε ηλικία a ετών

και πέθανε x ετών τότε βασίλεψε x a έτη, αν πέθαινε μετά 9 έτη θα είχε ζήσει

x 9 χρόνια και θα βασίλευε x 9 a χρόνια άρα :

x 9x 9 a 2x 18 2a x 9 x 9 2a

2

.

Αν πέθαινε 9 χρόνια νωρίτερα θα είχε ζήσει x 9 χρόνια και θα είχε βασιλέψει

x 9 a χρόνια άρα

Page 127: ΠΡΟΤΕΙΝΟΜΕΝΕΣ ΑΣΚΗΣΕΙΣ ΓΙΑ ΜΑΘΗΤΙΚΟΥΣ ΔΙΑΓΩΝΙΣΜΟΥΣ_ALL

http://www.mathematica.gr/forum/viewtopic.php?f=109&t=15584

Επιμέλεια : xr.tsif Σελίδα 36

x 9x 9 a 8x 72 8α x 9 7x 63 8α 7x 63 4x 36

8

3x 99 x 33 .

επομένως έζησε 33 χρόνια (φαντάζομαι ότι συμφωνούν και οι ιστορικοί).

ΘΕΜΑ 138 (VZF)

Αποδείξτε ότι 1 1 1 1 1 1 1 1

1 ... ...2 3 4 2n 1 2n n 1 n 2 2n

.

Λύση:

1 1 1 1 1 1 1 1 11 ... (1 ... )

2 3 4 2n 1 2n 2 3 2n 1 2n

1 1 1 1 1 1 1 1 1 1 12 ( ... ) 1 ... 2 (1 ... )

2 4 2n 2 3 2n 1 2n 2 2 3 n

1 1 1 1 1 1 1 1 1 1 1 11 ... (1 ... ) ...

2 3 4 2n 1 2n 2 3 4 n n 1 n 2 2n

ΘΕΜΑ 139 (VZF)

Αποδείξτε ότι 1 3 5 9999 1

...2 4 6 10000 100 .

Λύση:

1 2

2 3

3 4

4 5

...

Page 128: ΠΡΟΤΕΙΝΟΜΕΝΕΣ ΑΣΚΗΣΕΙΣ ΓΙΑ ΜΑΘΗΤΙΚΟΥΣ ΔΙΑΓΩΝΙΣΜΟΥΣ_ALL

http://www.mathematica.gr/forum/viewtopic.php?f=109&t=15584

Επιμέλεια : xr.tsif Σελίδα 37

9999 10000

10000 10001

Οπότε:

2 4 10000 2 4 6 10000 1 1 1A ... ...

3 5 10001 1 3 5 9999 10001 A 10001

Πολλαπλασιάζοντας με (το θετικό) A :

2

2

1 1 1 1A A

10001 10000 100 100 .

ΘΕΜΑ 140 (VZF)

Αν ισχύει ότι n

n r n r

r 0

n(x y) x y

r

(*) να αποδείξετε ότι

2n

k 0

n 2n

k n

.

Λύση:

Υπάρχει ένα τρόπος με χρήση του n n 2n(1 x) (1 x) (1 x) και σύγκριση του

συντελεστή του n

x στις δύο παραστάσεις. Δεν θα το κάνω για να το χαρούν άλλοι.

Είναι άλλωστε γνωστό. Θα δώσω όμως μία διαφορετική απόδειξη.

Μετράμε με δύο τρόπους το πλήθος των επιτροπών n ατόμων που μπορούμε να

κάνουμε από n αγόρια και n κορίτσια. Προφανώς είναι 2n

n

.

Ο άλλος τρόπος μέτρησης είναι: Η κάθε επιτροπή έχει k αγόρια και n k

κορίτσια 0 k n . Για κάθε τέτοιο k , αυτό γίνεται με

2

n n n

k n k k

τρόπους. Σύνολο

2n

k 0

n

k

, και λοιπά.

Page 129: ΠΡΟΤΕΙΝΟΜΕΝΕΣ ΑΣΚΗΣΕΙΣ ΓΙΑ ΜΑΘΗΤΙΚΟΥΣ ΔΙΑΓΩΝΙΣΜΟΥΣ_ALL

http://www.mathematica.gr/forum/viewtopic.php?f=109&t=15584

Επιμέλεια : xr.tsif Σελίδα 38

ΘΕΜΑ 141 (VZF)

Να αποδείξετε το n

n r n r

r 0

n(x y) x y

r

με ή και χωρίς τη χρήση του

διωνύμου του Newton n

n r

r 0

n(1 x) x

r

.

Λύση:

Ο τύπος (*) της 140 δίνει την 141 αν στη θέση του x πάρουμεx

y. H απευθείας

απόδειξη και των δύο, άλλωστε, είναι ολόιδια. Υπάρχει σε όλα τα βιβλία που

έχουν το ανάπτυγμα του Newton. Η συνήθης απόδειξη είναι με επαγωγή.

Παρατήρηση: Τα n

r

και C(n,r) συμβολίζουν τον αριθμό των τρόπων που

μπορούν να επιλεγούν r αντικείμενα από n , χωρίς να έχει σημασία η σειρά που

επιλέγονται. Ένας τύπος για το n

r

είναι n n!

r! (n r)!r

.Το n! λέγεται n

παραγοντικό και είναι ο μη αρνητικός αριθμός

n! 1 2 3 ... (n 1) n , όπου n ακέραιος και 0! 1 .

ΘΕΜΑ 142 (Socrates)

Στον πίνακα είναι γραμμένοι οι αριθμοί 0,1,2,...,100 με κενά μεταξύ τους.

Ένας μαθητής, ο A , τοποθετεί στα κενά 50 και 50* και υπολογίζει την τιμή της

παράστασης που προκύπτει (με τη συνήθη προτεραιότητα πράξεων.)

Έστω ότι το αποτέλεσμα που βρίσκει είναι ο αριθμός a .

Στη συνέχεια ο μαθητής B αλλάζει όλα τα σε * και όλα τα * σε στην

παράσταση που σχημάτισε ο A και υπολογίζει την τιμή της νέας παράστασης,

έστω b .

Αν τα τέσσερα τελευταία ψηφία του αριθμού a b είναι 2011 , να δείξετε ότι

κάποιος από τους μαθητές έκανε λάθος στις πράξεις.

Page 130: ΠΡΟΤΕΙΝΟΜΕΝΕΣ ΑΣΚΗΣΕΙΣ ΓΙΑ ΜΑΘΗΤΙΚΟΥΣ ΔΙΑΓΩΝΙΣΜΟΥΣ_ALL

http://www.mathematica.gr/forum/viewtopic.php?f=109&t=15584

Επιμέλεια : xr.tsif Σελίδα 39

Λύση:

Πρώτα θα γίνουν οι πολλαπλασιασμοί. Μετά, θα μείνουν 51αριθμοί τους οποίους

πρέπει να προσθέσουμε. Μπορούμε να βρούμε πόσοι από αυτούς είναι περιττοί

και πόσοι άρτιοι; Σίγουρα όλοι οι καινούργιοι αριθμοί που εμφανίστηκαν μετά από

τον πολλαπλασιασμό είναι άρτιοι. Για να είναι κάποιος περιττός πρέπει να ήταν

περιττός και να μην χρησιμοποιήθηκε στον πολλαπλασιασμό. Δηλαδή να είχε και

αριστερά του και δεξιά του το σύμβολο .

Δηλαδή για να δούμε αν ο a είναι περιττός ή άρτιος χωρίς να κάνουμε τις πράξεις,

κοιτάζουμε όλους τους περιττούς αριθμούς 1,3,...,99 και πως τοποθέτησε τα

σύμβολα και * o μαθητής A . Ας γράψουμε x

για όσους από αυτούς έχουν και

αριστερά και δεξιά το σύμβολο , *

x

για όσους έχουν αριστερά και δεξιά *

και ομοίως ορίζουμε τους*

x

και **

x . Τότε ο a είναι περιττός αν και μόνο αν ο

x

είναι περιττός. Επειδή ο b προκύπτει με εναλλαγή των και των * , ο b

είναι περιττός αν και μόνο αν ο **

x είναι περιττός.

Γνωρίζουμε όμως ότι 2x x x 50 αφού έχουν χρησιμοποιηθεί 50 .

Ομοίως 2x x x 50 . Άρα x x

. Άρα οa είναι περιττός αν και μόνο

αν ο b είναι περιττός και άρα ο a b είναι άρτιος. Επομένως το τελευταίο ψηφίο

του a b δεν μπορεί να ισούται με 1 και άρα κάποιος μαθητής έκανε σίγουρα

λάθος στις πράξεις.

ΘΕΜΑ 143 (Socrates)

Να δείξετε ότι η εξίσωση 2 2 2

a b c 5 έχει άπειρες θετικές ακέραιες λύσεις.

Λύση:

H εξίσωση γράφεται 2a c b b c 5. Ας αναζητήσουμε λύσεις για τις

οποίες a 5k όπου k θετικός ακέραιος και c b 5

Τότε 2

5k b c 1. Εύκολα βρίσκουμε 2

5k 6b .

2

Για να απαλλαγούμε από το 2 του παρονομαστή διαλέγουμε k 2m

Page 131: ΠΡΟΤΕΙΝΟΜΕΝΕΣ ΑΣΚΗΣΕΙΣ ΓΙΑ ΜΑΘΗΤΙΚΟΥΣ ΔΙΑΓΩΝΙΣΜΟΥΣ_ALL

http://www.mathematica.gr/forum/viewtopic.php?f=109&t=15584

Επιμέλεια : xr.tsif Σελίδα 40

Τώρα 2b 10m 3 οπότε 2

c 10m 2

Άρα οι άπειρες τριάδες 2 210m,10m 3,10m 2 όπου m θετικός ακέραιος, είναι

όλες λύσεις της εξίσωσης.

ΘΕΜΑ 144 (DEMETRES)

Αποδείξτε ότι 9999

1

1 3 5 1...

2 4 00006 15000 .

Για να προλάβω μερικούς.

Απαγορεύεται η χρήση Stirling ή παρόμοιων προσεγγίσεων. Ζητάω στοιχειώδη

λύση.

Λύση:

Μπορούμε να δείξουμε επαγωγικά ότι 1 3 2n 1 1

... ...2 4 2n 3n 1

Επειδή οι "μικροί" δεν έχουν διδαχθεί την μέθοδο της τέλειας επαγωγής (αυτό

θα γίνει μάλλον στην Β Λυκείου), θα δώσω μια αναλυτική απόδειξη της άσκησης

144 που έθεσε ο Demetres με την υπόδειξη που έγραψε ο socrates.

Θέλουμε λοιπόν να αποδείξουμε ότι : 1 3 2n 1 1

...2 4 2n 3n 1

για κάθε n 1 .

Η απόδειξη με την μέθοδο της τέλειας επαγωγής, γίνεται ως εξής:

Αν θέλουμε να αποδείξουμε την αλήθεια μιας πρότασης P(n) , με n p

(n φυσικός αριθμός) τότε:

* Εξετάζουμε αν η πρόταση ισχύει για n p 1

*Υποθέτοντας ότι η πρόταση ισχύει για n k , αποδείχνουμε ότι θα ισχύει και για

n k 1 .

Αν αυτά τα αποδείξουμε, τότε η πρότασή μας θα αληθεύει για κάθε n p (n N )

Ερχόμαστε λοιπόν στην άσκηση που θέλουμε να αποδείξουμε:

Page 132: ΠΡΟΤΕΙΝΟΜΕΝΕΣ ΑΣΚΗΣΕΙΣ ΓΙΑ ΜΑΘΗΤΙΚΟΥΣ ΔΙΑΓΩΝΙΣΜΟΥΣ_ALL

http://www.mathematica.gr/forum/viewtopic.php?f=109&t=15584

Επιμέλεια : xr.tsif Σελίδα 41

*Εξετάζουμε αν ισχύει για n 2 .

Δηλαδή αν 1 3 1 3 1 9 1

63 642 4 8 64 73 2 1 7

πράγμα που

αληθεύει.

(Άρα αποδείξαμε ότι η πρόταση ισχύει για n 2 )

* Υποθέτουμε τώρα ότι η πρόταση αληθεύει για n k . Δηλαδή ότι

1 3 2k 1 1...

2 4 2k 3k 1

(1)

και θα αποδείξουμε ότι θα αληθεύει και για n k 1 .

Δηλαδή ότι 1 3 2k 1 2(k 1) 1 1

...2 4 2k 2(k 1) 3(k 1) 1

(2)

Πράγματι, έχουμε από την υπόθεση που κάναμε ότι

1 3 2k 1 1...

2 4 2k 3k 1

1 3 2k 1 2(k 1) 1 1 2(k 1) 1...

2 4 2k 2(k 1) 2(k 1)3k 1

1 3 2k 1 2(k 1) 1 1 2k 1...

2 4 2k 2(k 1) 2k 23k 1

Αρκεί λοιπόν να αποδείξουμε ότι 2

2

(21 2k 1 1

2k 23k 1 3(k 1)

k 1) 1

(3k 1)(2k 2) 41 3k

2 2

(4k 4k 1) (3k 4) (3k 1) (4k 8k 4) 19k 20k .

πράγμα που ισχύει, αφού k φυσικός αριθμός.

Άρα αποδείξαμε όλα τα βήματα της τέλειας επαγωγής και συνεπώς το

ζητούμενο θα αληθεύει για κάθε n φυσικό αριθμό (μεγαλύτερο του 1).

Page 133: ΠΡΟΤΕΙΝΟΜΕΝΕΣ ΑΣΚΗΣΕΙΣ ΓΙΑ ΜΑΘΗΤΙΚΟΥΣ ΔΙΑΓΩΝΙΣΜΟΥΣ_ALL

http://www.mathematica.gr/forum/viewtopic.php?f=109&t=15584

Επιμέλεια : xr.tsif Σελίδα 42

ΔΙΩΝΥΜΟ ΤΟΥ ΝΕΥΤΩΝΑ Έχεις μάθει ότι

2 2 2(a b) a 2ab b και 3 3 2 2 3

(a b) a 3a b 3ab b

Με παρόμοιο τρόπο (όπως έχεις δει τις πιο πάνω αποδείξεις), θα μπορούμε να

αποδείξουμε ότι

4 4 3 2 2 3 44 3 4 2 3 4(a b) a a b a b ab b

1 1 2 1 2 3

5 5 4 3 2 2 3 4 55 4 5 3 4 5 2 3 4 5(a b) a a b a b a b ab b

1 1 2 1 2 3 1 2 3 4

Και γενικά αποδείχνεται ότι

n n n 1 n 2 2 n 3 3 nn n(n 1) n(n 1)(n 2)(a b) a a b a b a b ... b

1 1 2 1 2 3

Αυτή η ταυτότητα ονομάζεται "το διώνυμο του Νεύτωνα"

Το σύμβολο Σ δηλώνει άθροισμα. Παράδειγμα:

nr

r 1

x y

δηλώνει το εξής άθροισμα: n

r 1 2 3 n

r 1

x y x y x y x y ... x y

.

ΘΕΜΑ 145 (Socrates)

Να προσδιορίσετε τους φυσικούς αριθμούς r για τους οποίους υπάρχουν πρώτοι

αριθμοί p,q τέτοιοι ώστε: 2 2 2p pq q r .

Λύση:

Υπόδειξη:

Η εξίσωση μπορεί να γραφεί ως εξής (p q r)(p q r) pq.

Όμως p q r p , p q r q οπότε p q r pq...

Έχουμε :

2 2 2 2 2 2 2p pq q r p 2pq q 1 pq (p q) r pq

Page 134: ΠΡΟΤΕΙΝΟΜΕΝΕΣ ΑΣΚΗΣΕΙΣ ΓΙΑ ΜΑΘΗΤΙΚΟΥΣ ΔΙΑΓΩΝΙΣΜΟΥΣ_ALL

http://www.mathematica.gr/forum/viewtopic.php?f=109&t=15584

Επιμέλεια : xr.tsif Σελίδα 43

(p q r)(p q r) pq .

Επειδή όμως τα p,q είναι αριθμοί πρώτοι, το γινόμενο pq δεν αναλύεται σε

γινόμενο περισσοτέρων παραγόντων και δεδομένου ότι

p q r p , p q r q θα πρέπει υποχρεωτικά να είναι p q r pq και

p q r 1 (*).

'Άρα με πρόσθεση κατά μέλη έχουμε

2(p q) 1 pq 2p 2q pq 1 (2 q)p 2q 1

Αν ήταν 2 q 0 q 2 τότε θα είχαμε 0 4 1 που είναι άτοπο. Άρα

βρίσκουμε 2q 1 2q 4 3 2(q 2) 3 3

p p p p 2 , (1)q 2 q 2 q 2 q 2

Για να είναι ο pφυσικός (και πρώτος) αριθμός, θα πρέπει το q 2 να διαιρεί το 3 .

Άρα q 2 1 ή q 2 3 ,

Δηλαδή θα πρέπει q 3 ή q 5 .

1η Περίπτωση: q 3 .

Τότε από την σχέση (1) βρίσκουμε p 5 και από την σχέση (*) βρίσκουμεr 7 .

2η Περίπτωση: q 5 .

Τότε πάλι βρίσκουμε p 3 και r 7 .

ΘΕΜΑ 146 (Socrates)

Δίνεται ορθογώνιο ABCD με AB BC . Η μεσοκάθετος της διαγωνίου AC

τέμνει την πλευρά CDστο σημείο E . Ο κύκλος με κέντρο το E και ακτίνα

AEτέμνει την πλευρά AB στο σημείο F . Αν O η προβολή του C στην ευθεία

EF να δείξετε ότι τα σημεία B,O και D είναι συνευθειακά.

Λύση:

Page 135: ΠΡΟΤΕΙΝΟΜΕΝΕΣ ΑΣΚΗΣΕΙΣ ΓΙΑ ΜΑΘΗΤΙΚΟΥΣ ΔΙΑΓΩΝΙΣΜΟΥΣ_ALL

http://www.mathematica.gr/forum/viewtopic.php?f=109&t=15584

Επιμέλεια : xr.tsif Σελίδα 44

Αρκεί να αποδείξουμε ότι η γωνία o

DOB 180

. Η γωνία o

EOC 90

και

oFOC 90

.Το τρίγωνο EAC είναι ισοσκελές ( Η ME είναι μεσοκάθετος της

AC ) , άρα EAC ECA φ

. Από τα εγγράψιμα τετράπλευρα EMOC , EMAD

και από τις ισότητες ABD CDB φ

(αφού AB / /CD) προκύπτουν τα όμοια

ισοσκελή τρίγωνα EDO και FOB . Τα ορθογώνια τρίγωνα CEO , AED είναι ίσα

(υποτείνουσες και μία κάθετη αντίστοιχα ίσες). Άρα έχουμε CO AD BC ,

δηλαδή CBO COB θ

με oφ θ 90 .

Άρα τα σημεία B,O και D είναι συνευθειακά.

ΣΗΜΕΙΩΣΗ:

Μας χρειάζονται κάποιες πληροφορίες για τα εγγράψιμα τετράπλευρα.

(1) Ένα τετράπλευρο λέγεται εγγράψιμο σε κύκλο, αν υπάρχει κύκλος που να

περνάει από όλες τις κορυφές του.

Page 136: ΠΡΟΤΕΙΝΟΜΕΝΕΣ ΑΣΚΗΣΕΙΣ ΓΙΑ ΜΑΘΗΤΙΚΟΥΣ ΔΙΑΓΩΝΙΣΜΟΥΣ_ALL

http://www.mathematica.gr/forum/viewtopic.php?f=109&t=15584

Επιμέλεια : xr.tsif Σελίδα 45

(2) Ένα τετράπλευρο είναι εγγράψιμο σε κύκλο, αν υπάρχουν δύο γωνίες ίσες που

"βλέπουν" την ίδια πλευρά και έχουν κορυφή μια από τις κορυφές του

τετραπλεύρου.

Π.χ αν για το τετράπλευρο ΑΒΓΔ οι γωνίες ΑΒΔ και ΑΓΔ είναι ίσες, τότε αυτό

είναι εγγράψιμο σε κύκλο.

Και φυσικά ισχύει και το αντίστροφο (που είναι προφανές το γιατί): Αν ένα

τετράπλευρο είναι εγγράψιμο σε κύκλο, τότε οι γωνίες που "βλέπουν" την ίδια

πλευρά και έχουν κορυφή κάποια από τις κορυφές του τετραπλεύρου, είναι ίσες.

Φυσικά υπάρχουν και άλλα θεωρήματα που αναφέρονται στα εγγράψιμα

τετράπλευρα.

ΘΕΜΑ 147 (ΔΗΜΗΤΡΗΣ ΙΩΑΝΝΟΥ)

Τα ψηφία ενός διψήφιου αριθμού έχουν άθροισμα 11. Αν μεταξύ των ψηφίων του

παρεμβληθεί το 5 , τότε βρίσκεται τριψήφιος αριθμός, ο οποίος με τον αρχικό

διψήφιο έχουν άθροισμα 396 . Ποιος είναι ο διψήφιος αριθμός;

Λύση:

Έστω ab ο αριθμός που ψάχνω, άρα a b 11 (1) και

ab a5b 396 10a b 100a 50 b 396

110a 2b 346 55a b 173 (2).

άρα κάνοντας αφαίρεση κατά μέλη έχω ότι :

(2) (1) 55a b a b 162 54a 162 a 3 άρα b 8 . Επομένως ο

αριθμός που ψάχνω είναι ο 38 .

ΘΕΜΑ 148 (ΔΗΜΗΤΡΗΣ ΙΩΑΝΝΟΥ)

Μια βρύση A μπορεί να γεμίσει μια δεξαμενή σε 6 ώρες ενώ μια άλλη βρύση B

μπορεί να την γεμίσει σε 4 ώρες. Στις 11 το πρωί, ανοίγουμε την βρύση A και

στις 12 το μεσημέρι, ανοίγουμε και την βρύση B (χωρίς να κλείσουμε την A ).

Τι ώρα θα έχει γεμίσει η δεξαμενή;

Page 137: ΠΡΟΤΕΙΝΟΜΕΝΕΣ ΑΣΚΗΣΕΙΣ ΓΙΑ ΜΑΘΗΤΙΚΟΥΣ ΔΙΑΓΩΝΙΣΜΟΥΣ_ALL

http://www.mathematica.gr/forum/viewtopic.php?f=109&t=15584

Επιμέλεια : xr.tsif Σελίδα 46

Λύση:

Υπόδειξη

Παρατηρείστε ότι αν η βρύση A λειτουργεί για x ώρες, τότε η B θα λειτουργεί

για x 1 ώρες. Τώρα, η βρύση A σε 1 ώρα γεμίζει το 1

6 της δεξαμενής και άρα

σε x ώρες θα γεμίσει τα x

6 αυτής.

Όμοια , η βρύση B γεμίζει σε 1 ώρα το 1

4 της δεξαμενής και άρα σε x 1 ώρες

θα γεμίσει τα x 1

4

της δεξαμενής. Και οι δύο μαζί θα πρέπει να γεμίσουν όλη την

δεξαμενή (δηλ. τα12

12 της δεξαμενής).

Φτιάξτε τώρα την εξίσωση και λύστε την άσκηση.

x x 1 122x 3(x 1) 12 5x 15 x 3

6 4 12

.

΄Αρα η δεξαμενή θα γεμίσει στις 2 το μεσημέρι.

ΘΕΜΑ 149 (Socrates)

Αν οι θετικοί πραγματικοί αριθμοί x,y,z είναι τέτοιοι ώστε

y z xx y z 2

z x y να προσδιορίσετε όλες τις δυνατές τιμές του x y z .

Λύση:

3

y z x y z x6 x y z 6 x y z 3 · · 6 x y z 3

z x y z x y

x y z 3  (1) .

Page 138: ΠΡΟΤΕΙΝΟΜΕΝΕΣ ΑΣΚΗΣΕΙΣ ΓΙΑ ΜΑΘΗΤΙΚΟΥΣ ΔΙΑΓΩΝΙΣΜΟΥΣ_ALL

http://www.mathematica.gr/forum/viewtopic.php?f=109&t=15584

Επιμέλεια : xr.tsif Σελίδα 47

Επιπλέον ισχύει:

xz y 2z

xy z 2x

zy x 2y

.

Αθροίζοντας τις τρεις αυτές σχέσεις κατά μέλη λαμβάνουμε ότι:

xz xy zy x y z  (*) .

Όμως:

2 2 2 (*) 2 (*)x y z x y z (x y z) 2(xz xy zy) x y z

(*) 2(x y z) 2(x y z) x y z x y z 3  (2) .

Από (1) , (2) προκύπτει πως x y z 3 , η οποία είναι και η μόνη δυνατή τιμή

της παράστασης.

ΘΕΜΑ 150 (Socrates)

Να λυθεί το σύστημα

2

2

2

2x 14 yz

2y 14 zx

2z 14 xy

.

Λύση:

2 2 22x 14 yz (1) 2y 14 zx (2) 2z 14 xy (3)

2 2(1) (2) 2(x y ) yz zx 2(x y)(x y) z(y x)

• Αν x y τότε έχουμε: z 2x 2y (4)

Όμοια αφαιρώντας απο την (1) την (3) έχουμε: 2(x z)(x z) z(z x)

• Αν x z τότε έχουμε: y 2x 2z (5)

Όμοια αφαιρώντας απο την (2) την (3) έχουμε: 2(y z)(y z) x(z y)

• Αν z y τότε έχουμε: x 2y 2z (6)

Page 139: ΠΡΟΤΕΙΝΟΜΕΝΕΣ ΑΣΚΗΣΕΙΣ ΓΙΑ ΜΑΘΗΤΙΚΟΥΣ ΔΙΑΓΩΝΙΣΜΟΥΣ_ALL

http://www.mathematica.gr/forum/viewtopic.php?f=109&t=15584

Επιμέλεια : xr.tsif Σελίδα 48

Το σύστημα των εξισώσεων (4), (5), (6) εύκολα βλέπουμε ότι έχει μοναδική λύση

την (x,y,z) (0,0,0) η οποία απορρίπτεται γιατί δεν επαληθεύει τις αρχικές.

• Αν x y τότε οι εξισώσεις (1) και (3) γίνονται:

2(1) 2x 14 xz

2 2(3) 2z 14 x Αφαιρώντας κατά μέλη τις παραπάνω έχουμε:

2z2(x z)(x z) x(z x) 2x 2z x x

3

και με αντικατάσταση στην παραπάνω έχουμε:

2

2 z2z 14 4 z 3

9 Άρα x y 2

• Αν y z δουλεύουμε ομοίως και βρίσκουμε x 3 οπότε και y z 2

• Αν x z δουλεύουμε ομοίως και βρίσκουμε y 3 οπότε και x z 2

• Αν x y z τότε η (1) γίνεται: 2 2 22x 14 x x 14 x 14

Οπότε: x y z 14 . Τελικά οι λύσεις του συστήματος είναι:

(x,y,z) {( 2, 2,3),(2,2, 3),(3, 2, 2),( 3,2,2),( 2,3, 2),,(2, 3,2)

,( 14, 14, 14)} .

ΘΕΜΑ 151 (ΔΗΜΗΤΡΗΣ ΙΩΑΝΝΟΥ)

Αν για τους μη μηδενικούς αριθμούς a,b,x,y ισχύει ay bx , να υπολογιστεί η

τιμή της παράστασης: 2 2

2 2 2 2

xA

b xa y

a

.

Λύση:

2 2 2 2

2 2 2 2 2 2

x x xay bx

y y

a a

x

a

b b a b y

.

Page 140: ΠΡΟΤΕΙΝΟΜΕΝΕΣ ΑΣΚΗΣΕΙΣ ΓΙΑ ΜΑΘΗΤΙΚΟΥΣ ΔΙΑΓΩΝΙΣΜΟΥΣ_ALL

http://www.mathematica.gr/forum/viewtopic.php?f=109&t=15584

Επιμέλεια : xr.tsif Σελίδα 49

Άρα η παράσταση A γίνεται 2 2

2 2 2 2

xA 1

b ya x

a

.

ΘΕΜΑ 152 (ΔΗΜΗΤΡΗΣ ΙΩΑΝΝΟΥ)

Αν ισχύουν x y z

a b c , a b c 1 , 2 2 2

a b c 1 και αν a,b,c 0 να δείξετε

ότι xy yz xz 0 .

Λύση:

Από το πρόβλημα γνωρίζω ότι : a b c 1 άρα

b c 1 a , a c 1 b , a b 1 c .

και έστω: x y z

ka b c , άρα x ak , y bk , z ck .

Το xy yz xz μπορώ να το γράψω και:

2xy yz zx ka·kb kb·kc kc·ka k (ab bc ca)

κάνοντας αφαίρεση κατά μέλη ανάμεσα στις σχέσεις a b c 1 και 2 2 2

a b c 1 έχω ότι:

2 2 2a b c a b c 0 a(1 a) b(1 b) c(1 c) 0

a(b c) b(a c) c(a b) 0 2ab 2ac 2ab 0 ab ac bc 0

άρα κάνοντας αντικατάσταση έχω ότι: 2 2k (ab bc ca) k ·0 0 .

άρα η σχέση xy yz xz 0 επαληθεύεται.

Β τρόπος

Από τη x y z

ka b c έχουμε

x y z x y z x y zk

a b c a b c 1

, άρα x y z k .

Page 141: ΠΡΟΤΕΙΝΟΜΕΝΕΣ ΑΣΚΗΣΕΙΣ ΓΙΑ ΜΑΘΗΤΙΚΟΥΣ ΔΙΑΓΩΝΙΣΜΟΥΣ_ALL

http://www.mathematica.gr/forum/viewtopic.php?f=109&t=15584

Επιμέλεια : xr.tsif Σελίδα 50

Επίσης έχουμε από την δοθείσα ότι 2 2 2 2 2 2 2 2 2

2

2 2 2 2 2 2

x y z x y z x y zk

a b c a b c 1

, άρα 2 2 2 2

x y z k .

Τελικά 2 2 2 2 2 21 1xy yz zx (x xy z) (x y z ) k k 0

2 2 .

ΘΕΜΑ 153 (ΧΡΗΣΤΟΣ ΤΣΙΦΑΚΗΣ)

Δίνεται μία γωνία οω 19

.

Μπορείτε να κατασκευάσετε μόνο με χάρακα και διαβήτη μία γωνία με μέτρο ο1 ;

Λύση:

Ναι, αφού μπορούμε να κατασκευάσουμε μια γωνία ο18 , (αφού το 18 είναι

πολλαπλάσιο του 3 )

Δείτε http://www.mathematica.gr/forum/viewtopic.php?f=6&t=6808&p=38729

Β τρόπος

Αφού 19 19 361 , σημαίνει ότι αν πάρουμε 19 φορές την γωνία μας, θα

ξεπεράσει τον κύκλο των ο

360 κατά ο1 . Άρα ...

ΠΑΡΑΤΗΡΗΣΗ

Το εξής δεν είναι πολύ γνωστό αλλά όφειλε να είναι περισσότερο γνωστό (*):

Ακέραιες γωνίες που κατασκευάζονται με κανόνα και διαβήτη είναι τα

πολλαπλάσια του 3 , και μόνον αυτά.

Ένα πόρισμα: Η γωνία ο60 μοιρών δεν τριχοτομείται με κανόνα και διαβήτη γιατί

τότε θα κατασκευαζόταν η γωνία ο

20 μοιρών η οποία δεν είναι πολλαπλάσιο του 3 .

(*) η απόδειξη, αν και όχι ιδιαίτερα δύσκολη, ξεφεύγει της Σχολικής ύλης.

Γρήγορη αιτιολόγηση για αυτούς που ξέρουν Θεωρία Galois:

Η γωνία ο

12 μοιρών κατασκευάζεται ως o ο

72 60 (η ο

72 προκύπτει από το

κανονικό πεντάγωνο και η ο

60 είναι βέβαια απλή). Με διχοτόμηση

Page 142: ΠΡΟΤΕΙΝΟΜΕΝΕΣ ΑΣΚΗΣΕΙΣ ΓΙΑ ΜΑΘΗΤΙΚΟΥΣ ΔΙΑΓΩΝΙΣΜΟΥΣ_ALL

http://www.mathematica.gr/forum/viewtopic.php?f=109&t=15584

Επιμέλεια : xr.tsif Σελίδα 51

κατασκευάζονται οι ο6 και ο

3 , και άρα όλα τα πολλαπλάσια του 3 . Από την άλλη

είναι γνωστό από Θεωρία Galois ότι δεν κατασκευάζεται η τριχοτόμος ο20 των

ο60 . Άρα δεν κατασκευάζεται η γωνία ο

1 μοίρας (αλλιώς θα φτιάχναμε την ο ο ο ο

20 18 1 1 ). Όμοια έπεται ότι δεν κατασκευάζονται οι 3k 1 , 3k 2 .

Μένουν οι 3k .

Επίσης, από την θεωρία Galois , ισχύει:

Μια γωνία θ μπορεί να τριχοτομηθεί με κανόνα και διαβήτη, αν και μόνο αν, το

πολυώνυμο 34x 3x συνθ είναι μη – ανάγωγο πάνω από το (συνθ) .

Πρόβλημα:

Να βρεθούν οι ρητές γωνίες m

n (m,n 1 , gcd(m,n) 1 ) που κατασκευάζονται

με κανόνα και διαβήτη.

ΛΥΣΗ

Για τις ακέραιες γωνίες γνώριζα. Για τις ρητές όχι. Για να δούμε:

Ισχυρίζομαι ότι η γωνία m

n όπου m,n με (m,n) 1 είναι κατασκευάσιμη αν

και μόνο αν

α) Το m είναι πολλαπλάσιο του 3 .

β) Το n είναι της μορφής r

1 k2 p p , όπου

1 kp , ,p είναι διακεκριμένοι πρώτοι

της μορφής s2

2 1 όλοι όμως διαφορετικοί του 3 και 5 .

Για το αποτέλεσμα θα χρησιμοποιήσω

Λήμμα 1: Αν n , τότε η γωνία n μοιρών είναι κατασκευάσιμη αν και μόνο αν

το n είναι πολλαπλάσιο του 3 .

Λήμμα 2: Έστω k,n με (k,n) 1 . Τότε η 3k

n είναι κατασκευάσιμη αν και

μόνο αν η 3

n είναι κατασκευάσιμη.

Page 143: ΠΡΟΤΕΙΝΟΜΕΝΕΣ ΑΣΚΗΣΕΙΣ ΓΙΑ ΜΑΘΗΤΙΚΟΥΣ ΔΙΑΓΩΝΙΣΜΟΥΣ_ALL

http://www.mathematica.gr/forum/viewtopic.php?f=109&t=15584

Επιμέλεια : xr.tsif Σελίδα 52

Απόδειξη:

Το «αν» είναι προφανές. Για το «μόνο αν», αφού η γωνία τριών μοιρών είναι

κατασκευάσιμη, τότε και η 3n 3k

n

είναι κατασκευάσιμη. Αλλά

(3n 3k,3k) 3(n k,k) 3(n,k) 3 . Άρα υπάρχουν ακέραιοι a,b ώστε

(3n k)a kb 3 και άρα η γωνία 3

n είναι κατασκευάσιμη.

Λήμμα 3 (Gauss): Το κανονικό n – γωνο είναι κατασκευάσιμο αν και μόνο αν ο

n είναι της μορφής r

1 k2 p p , όπου

1 kp , ,p είναι διακεκριμένοι πρώτοι της

μορφής s2

2 1 .

Από το Λήμμα 3, βρίσκουμε ότι η γωνία ο

360

n και άρα και η γωνία

ο45

n είναι

κατασκευάσιμη αν και μόνο αν o n είναι της πιο πάνω μορφής.

Αν ισχύουν λοιπόν τα (α) και (β), τότε η γωνία ο

45

n είναι κατασκευάσιμη άρα και

η 3

n είναι κατασκευάσιμη και άρα και η

m

n είναι κατασκευάσιμη.

Μένει να δείξουμε το αντίστροφο. Αν η m

n είναι κατασκευάσιμη τότε και η m

είναι κατασκευάσιμη άρα το m είναι πολλαπλάσιο του 3 , δηλαδή το (α) ισχύει.

Ας υποθέσουμε πως το (β) δεν ισχύει. Από το (α) και την συνθήκη (m,n) 1

σίγουρα 3 n . Θα εξετάσουμε δυο περιπτώσεις:

Α) 5 n : Η γωνία ο

45

n δεν είναι κατασκευάσιμη και αφού (45,n) 1 , ούτε

και η 3

n είναι κατασκευάσιμη, άρα ούτε και η

m

n είναι κατασκευάσιμη,

άτοπο.

Page 144: ΠΡΟΤΕΙΝΟΜΕΝΕΣ ΑΣΚΗΣΕΙΣ ΓΙΑ ΜΑΘΗΤΙΚΟΥΣ ΔΙΑΓΩΝΙΣΜΟΥΣ_ALL

http://www.mathematica.gr/forum/viewtopic.php?f=109&t=15584

Επιμέλεια : xr.tsif Σελίδα 53

Β) 5 / n : Αφού η m

n είναι κατασκευάσιμη, τότε και η

m

5 είναι και αφού

(m,5) 1 και 3

m τότε και η

3

5 είναι κατασκευάσιμη. Όμως γνωρίζουμε ότι

η 45 9

25 5 δεν είναι κατασκευάσιμη άρα ούτε και η

3

5.

ΘΕΜΑ 154 (DEMETRES)

Να βρεθούν όλες οι ακέραιες λύσεις τις εξίσωσης 2 2x y 15 .

Λύση:

Από το πρόβλημα έχω ότι:

2 2 2 2x y 15 x y 15 (x y)(x y) 3·5 1·15

1η περίπτωση

x y 3

x y 5

κάνοντας αφαίρεση κατά μέλη έχω ότι:

x y x y 2 y 1 , x 4 .

2η περίπτωση

x y 5

x y 3

κάνοντας αφαίρεση κατά μέλη έχω ότι:

x y x y 2 y 1 , x 4 .

3η περίπτωση

x y 15

x y 1

κάνοντας αφαίρεση κατά μέλη έχω ότι:

x y x y 14 y 7 , x 8 .

Page 145: ΠΡΟΤΕΙΝΟΜΕΝΕΣ ΑΣΚΗΣΕΙΣ ΓΙΑ ΜΑΘΗΤΙΚΟΥΣ ΔΙΑΓΩΝΙΣΜΟΥΣ_ALL

http://www.mathematica.gr/forum/viewtopic.php?f=109&t=15584

Επιμέλεια : xr.tsif Σελίδα 54

4η περίπτωση

x y 1

x y 15

κάνοντας αφαίρεση κατά μέλη έχω ότι:

x y x y 14 y 7 , x 8 .

Επειδή το x και το y είναι υψωμένα στο τετράγωνο ισχύουν και οι τιμές με

αντίθετο πρόσημο.

Άρα οι λύσεις της εξίσωσης είναι:

(y,x) ( 1,4)(1,4)(7,8)( 7,8)(1, 4)( 1, 4)( 7, 8)(7, 8) .

Κανονικά υπάρχουν οκτώ περιπτώσεις αφού πρέπει να υπολογίσουμε και τα

x y 1,x y 15 κ.τ.λ. Οι άλλες τέσσερις περιπτώσεις δίνουν ακριβώς τις

επιπλέον λύσεις που έγραψες στο τέλος. (Αφού θα έχουν σίγουρα τα αντίθετα

πρόσημα.)

Page 146: ΠΡΟΤΕΙΝΟΜΕΝΕΣ ΑΣΚΗΣΕΙΣ ΓΙΑ ΜΑΘΗΤΙΚΟΥΣ ΔΙΑΓΩΝΙΣΜΟΥΣ_ALL

http://www.mathematica.gr/forum/viewtopic.php?f=109&t=15584

Επιμέλεια : xr.tsif Σελίδα 55

ΘΕΜΑ 155 (DEMETRES)

Πέντε σημεία βρίσκονται μέσα σε ένα τετράγωνο πλευράς μήκους 1 . Να

αποδείξετε ότι δύο από αυτά τα σημεία απέχουν το πολύ 2

2 μεταξύ τους.

Λύση:

Χωρίζουμε το τετράγωνο σε τέσσερα ίσα τετράγωνα, που το καθένα από αυτά έχει

πλευρά ίση με 1

2. Αν σε ένα από τα τέσσερα τετράγωνα σημειώσουμε δύο σημεία,

τότε η μέγιστη απόστασή τους επιτυγχάνεται αν αυτά είναι τα άκρα μιας

διαγωνίου του τετραγώνου.

Δηλαδή η μέγιστη απόσταση των δύο σημείων ισούται με το μήκος της διαγωνίου,

για την οποία έχουμε:

2 2

2 1 1 2x x

2 2 2

.

Αφού έχουμε πέντε σημεία και τέσσερα τετράγωνα, δύο τουλάχιστον σημεία θα

βρίσκονται σε ένα τετράγωνο, άρα δύο τουλάχιστον από αυτά θα απέχουν το πολύ

2

2 μεταξύ τους.

ΘΕΜΑ 156

6

(ΑΝΤΩΝΗΣ ΚΥΡΙΑΚΟΠΟΥΛΟΣ)

Οι ακέραιοι αριθμοί x,y,z,w,t είναι διαφορετικοί ανά δύο και ισχύει:

(9 x)(9 y)(9 z)(9 w)(9 t) 12 . Να αποδείξετε ότι: x y z w t 42 .

Λύση:

Αφού οι x,y,z,w,t είναι ακέραιοι διαφορετικοί ανά δύο, το ίδιο ισχύει και για

τους 9 x,9 y,9 z,9 w,9 t.

Λόγω της σχέσης που δίνεται, θα πρέπει να γράψουμε το 12 ως το γινόμενο πέντε

διαφορετικών ακεραίων. Εύκολα βλέπουμε, ότι ο μοναδικός τρόπος για να γίνει

αυτό είναι ο 1 2 3 ( 1) ( 2).

Page 147: ΠΡΟΤΕΙΝΟΜΕΝΕΣ ΑΣΚΗΣΕΙΣ ΓΙΑ ΜΑΘΗΤΙΚΟΥΣ ΔΙΑΓΩΝΙΣΜΟΥΣ_ALL

http://www.mathematica.gr/forum/viewtopic.php?f=109&t=15584

Επιμέλεια : xr.tsif Σελίδα 56

Άρα είναι π.χ. 9 x 1,9 y 2,9 z 3,9 w 1,9 t 2 (ή οποιοσδήποτε

άλλος συνδυασμός), οπότε οι x,y,z,w,t είναι ίσοι με τους 8,7,6,10,11

αντίστοιχα. Το ζητούμενο έπεται.

ΘΕΜΑ 157 (Socrates)

Μπορούμε να μετασχηματίσουμε ένα ζεύγος (a,b) πραγματικών αριθμών στο

2(a ,b)

b , αν b 0 ή στο

2(a,b )

a αν a 0 .

α) Δείξτε ότι ξεκινώντας από το ζεύγος (2011,2) και χρησιμοποιώντας την

παραπάνω διαδικασία μπορούμε να πάρουμε το ζεύγος (2, 2011) .

β) Δείξτε ότι με όποιο τρόπο και να πραγματοποιηθεί το α), σε κάποια στιγμή θα

εμφανιστεί ζεύγος που περιέχει το 0 .

γ) Ξεκινώντας από το ζεύγος (2010,2)μπορούμε να πάρουμε το ζεύγος (1,2011) ;

Λύση:

α) Από πολλαπλή χρήση της πρώτης πράξης (με πλην) έχουμε

(2011,2) (2010,2) (2009,2) (2008,2) ... (3,2) (2,2)

Τώρα συνεχίζουμε με την δεύτερη πράξη (με πλην)

(2,2) (2,1) (2,0) (2, 1) ... (2, 2010) (2, 2011) , όπως θέλαμε

β) Παρατηρούμε ότι το γινόμενο των δύο αριθμών στο (a,b) πριν την πράξη σε

σύγκριση με το ίδιο γινόμενο μετά την πράξη, αλλάζει κατά 2 . Πράγματι

2a b ab 2

b

, και όμοια η άλλη περίπτωση.

Τώρα, αφού στην αρχή το γινόμενο των όρων στο (2011,2) είναι 4022 και στο

τέλος 4022 , υποχρεωτικά (με βήματα 2 ) θα περάσει από το 0 (βασικά, θα

περάσει από όλους τους άρτιους μεταξύ 4022 και 4022 . Αλλά τότε το ζεύγος

(a,b) θα έχει υποχρεωτικά τον ένα παράγοντα ίσο με 0 , αφού ab 0 .

Page 148: ΠΡΟΤΕΙΝΟΜΕΝΕΣ ΑΣΚΗΣΕΙΣ ΓΙΑ ΜΑΘΗΤΙΚΟΥΣ ΔΙΑΓΩΝΙΣΜΟΥΣ_ALL

http://www.mathematica.gr/forum/viewtopic.php?f=109&t=15584

Επιμέλεια : xr.tsif Σελίδα 57

γ) Από το (2010,2) δεν μπορούμε να πάμε στο (1,2011) γιατί το μεν πρώτο έχει

γινόμενο όρων άρτιο ενώ το δεύτερο, περιττό. Όμως οι αλλαγές είναι 2 , που

σημαίνει ότι από άρτιο δεν θα φτάσουμε ποτέ σε περιττό.

ΘΕΜΑ 158 (ΔΗΜΗΤΡΗΣ ΙΩΑΝΝΟΥ)

Οι μη παράλληλες πλευρές ισοσκελούς τραπεζίου έχουν μήκη 10m η κάθε μία,

ενώ η περίμετρός του είναι 152m . Αν το ύψος του είναι το 1

9 της μεγάλης βάσης

και οι βάσεις του είναι ανάλογες προς τους αριθμούς 6 και 5 , να υπολογίσετε το

εμβαδόν του.

Λύση:

Αν x,y είναι η μικρή και η μεγάλη βάση αντίστοιχα, Π η περίμετρος και υ το ύψος

του τραπεζίου έχουμε ότι:

Π 152 x y 132x 60

1 11υ y υ y

υ y 89 99

x y x y x y 132y 7212

5 6 5 6 11 11132

x y 132 E 8E υ E υ 2

2 2

2E 528m .

ΘΕΜΑ 159 (ΔΗΜΗΤΡΗΣ ΙΩΑΝΝΟΥ)

Δίνεται ο αριθμός 1! 2! ... 2004! .

α) Να βρείτε ποιο είναι το ψηφίο των μονάδων του.

β) Να αποδείξετε ότι ο αριθμός αυτός διαιρείται με το 3 .

(Υπενθυμίζουμε ότι συμβολίζουμε: n! 1 2 ... (n 1) n και διαβάζετε

ν – παραγοντικό).

Λύση:

α) Έχω ότι:

A 1! 2! 3! 4! 5! ... 2004! 1 1·2 1·2·3 1·2·3·4 1·2·3·4·5 ...

2004! 1 2 6 24 120 120·6 ... 2004!

Page 149: ΠΡΟΤΕΙΝΟΜΕΝΕΣ ΑΣΚΗΣΕΙΣ ΓΙΑ ΜΑΘΗΤΙΚΟΥΣ ΔΙΑΓΩΝΙΣΜΟΥΣ_ALL

http://www.mathematica.gr/forum/viewtopic.php?f=109&t=15584

Επιμέλεια : xr.tsif Σελίδα 58

Μετά το 5! Όλοι οι υπόλοιποι αριθμοί θα πολλαπλασιαστούν με το 120 άρα το

τελευταίο ψηφίο τους θα είναι πάντα 0 .

Επομένως τ(A) τ(1 2 6 24) 3 .

β) A 1 1·2 1·2·3 1·2·3·4 1·2·3·4·5 ... 2004!

Μετά το 2! όλα τα γινόμενα έχουν μέσα τους το 3 . Άρα αν βγάλω κοινό

παράγοντα το 3 έχω ότι : A 1 1·2 3k 3(1 k) . Άρα 3 / A .

ΘΕΜΑ 160 (ΔΗΜΗΤΡΗΣ ΙΩΑΝΝΟΥ)

Ένας επιστήμονας και ο βοηθός του ανέλαβαν μια έρευνα σε χημικό εργαστήριο

από την οποία θα εισπράξουν 85116ευρώ. Ο επιστήμονας θα απασχοληθεί για 42

μέρες και ο βοηθός του για 45 μέρες. Η ημερήσια αμοιβή του επιστήμονα είναι

κατά 40% μεγαλύτερη της ημερήσιας αμοιβής του βοηθού του. Πόσα χρήματα θα

εισπράξει ο καθένας στο τέλος της έρευνας;

Λύση:

Ας υποθέσουμε ότι ο βοηθός θα πάρει x ευρώ. Τότε ο επιστήμονας θα πάρει

140x

100 ευρώ. Άρα έχουμε την εξίσωση 1,4x 42 x 45 85116 x 820 .

Άρα ο βοηθός θα εισπράξει 820 45 36900 ευρώ και ο επιστήμονας θα εισπράξει

1,4 820 42 48216 ευρώ.

ΘΕΜΑ 161 (ΔΗΜΗΤΡΗΣ ΙΩΑΝΝΟΥ)

Αν a R, a 1 και αν οι πραγματικοί αριθμοί x,y είναι τέτοιοι ώστε: 2 4

2 2 2

x y 1 a a

1 a x xy y

και

2 4

2 2 2

x y 1 a a

1 a x xy y

, να προσδιορίσετε την τιμή του x .

Λύση:

Αναλύω τους όρους των δυο κλασμάτων οπότε προκύπτει ότι:

2 2 3 3(x y)(x xy y ) x y , 2 2 4 3 2 3

(1 a )(1 a a ) 1 (a ) και

2 2 3 3(x y)(x xy y ) x y , 2 2 4 2 3

(1 a )(1 a a ) 1 (a ) .

Page 150: ΠΡΟΤΕΙΝΟΜΕΝΕΣ ΑΣΚΗΣΕΙΣ ΓΙΑ ΜΑΘΗΤΙΚΟΥΣ ΔΙΑΓΩΝΙΣΜΟΥΣ_ALL

http://www.mathematica.gr/forum/viewtopic.php?f=109&t=15584

Επιμέλεια : xr.tsif Σελίδα 59

Άρα 3 3 6x y 1 a και 3 3 6

x y 1 a

Κάνοντας αφαίρεση κατά μέλη έχω ότι:

3 3 3 3 6 6 3 6x y x y 1 a 1 a y a (1)

Άρα από την 3 6 6 3(1) x a 1 a x 1 x 1 .

ΘΕΜΑ 162 (ΔΗΜΗΤΡΗΣ ΙΩΑΝΝΟΥ)

Οι πραγματικοί αριθμοί x,y είναι τέτοιοι ώστε x 0,y 1 και 2xy 1 y y x .

Να βρείτε την ελάχιστη δυνατή τιμή του x .

Λύση:

Έχουμε: 2

2 y y 1xy 1 y y x x

y 1

, άρα θέλουμε να βρούμε την

ελάχιστη τιμή τού 2

y y 1

y 1

.

Είναι: 2 2

y y 1 y y(y 1) y1 1

y 1 y 1 y 1

y 1 1 1 1y 1 1 1 y (y 1) ( ) 3 2 3 5

y 1 y 1 y 1

.

(Μένει μόνο μια εκκρεμότητα: Πρέπει να διαπιστώσουμε αν υπάρχει y τέτοιο

ώστε 2

y y 15

y 1

.

Λύνοντας την εξίσωση αυτή, βρίσκουμε y 2 και άρα πράγματι η ελάχιστη τιμή

του κλάσματος 2

y y 1

y 1

είναι το 5 .

Στην περίπτωση που η εξίσωση θα ήταν αδύνατη, θα έπρεπε να ακολουθούσαμε

άλλο δρόμο για να βρίσκαμε την ζητούμενη ελάχιστη τιμή.)

Page 151: ΠΡΟΤΕΙΝΟΜΕΝΕΣ ΑΣΚΗΣΕΙΣ ΓΙΑ ΜΑΘΗΤΙΚΟΥΣ ΔΙΑΓΩΝΙΣΜΟΥΣ_ALL

http://www.mathematica.gr/forum/viewtopic.php?f=109&t=15584

Επιμέλεια : xr.tsif Σελίδα 60

Β τρόπος

Έχω ότι : 2 2 2xy 1 y y x y y x xy 1 0 y y(1 x) x 1 0 .

για να έχει πραγματικές ρίζες πρέπει να ισχύει Δ 0 .

άρα 2 2(1 x) 4(x 1) 0 x 6x 5 0 .

Θα βρω τις ρίζες του 2x 6x 5 0 , για να το παραγοντοποιήσω

2

1 2

6 ( 6) 4·5 6 16 6 4x ,x 5,1

2 2 2

.

άρα 2x 6x 5 0 (x 5)(x 1) 0

οπότε έχει δυο περιπτώσεις

1η περίπτωση

x 5 0 και x 1 0 δηλαδή x 5 και x 1

οι κοινές λύσεις των δυο ανισώσεων είναι x 5 .

2η περίπτωση

x 5 0 και x 1 0 δηλαδή x 5 και x 1

οι κοινές λύσεις των δυο ανισώσεων είναι x 1 .

όμως έχω τον περιορισμό x 0 άρα κοινή λύση των ανισώσεων είναι: 0 x 1 .

Όμως αν κάνω τις πράξεις στην αρχική εξίσωση που μου δίνει το πρόβλημα έχω

ότι: 2

yx 1

y 1

Που σημαίνει ότι το x είναι σίγουρα μεγαλύτερο από το 1αφού ψ 1 ψ 1 0

Άρα η σχέση 0 x 1 δεν ισχύει.

Άρα η ελάχιστη τιμή του x είναι 5 δηλ. ισχύει η 1η περίπτωση x 5 .

Page 152: ΠΡΟΤΕΙΝΟΜΕΝΕΣ ΑΣΚΗΣΕΙΣ ΓΙΑ ΜΑΘΗΤΙΚΟΥΣ ΔΙΑΓΩΝΙΣΜΟΥΣ_ALL

http://www.mathematica.gr/forum/viewtopic.php?f=109&t=15584

Επιμέλεια : xr.tsif Σελίδα 61

ΘΕΜΑ 163 (ΔΗΜΗΤΡΗΣ ΙΩΑΝΝΟΥ)

Θεωρούμε τους πραγματικούς αριθμούς x,y,z,w . Αν αντικαταστήσουμε αυτούς

με τους αριθμούς 1 1 1 1

x 10, y 10, z 10,x y z w w 10 τότε το άθροισμα

των αριθμών 1 1 1 1

x y z w 1040 . Αν αντικαταστήσουμε τους x,y,z,w με

τους αριθμούς 2 2 2 2

10 x, 10 y , 10 z , 10 wx y z w , πόσο θα είναι το

άθροισμα 2 2 2 2

x y z w ;

Λύση:

Σύμφωνα με την εκφώνηση έχουμε ότι: 1 1 1 1

x y z w 1040 .

Άρα αντικαθιστώντας έχουμε:

x 10 y 10 z 10 w 10 1040 x y z w 40 1040

x y z w 1040 40 x y z w 1000 (*)

Μετά απο την επόμενη αντικατάσταση έχουμε:

10 x 20 y 30 z 40 w x y z w 100

(x y z w) 100

όμως από το (*) έχουμε: (1000) 100 1000 100 900

Επομένως, το άθροισμα 2 2 2 2

x y z 90w 0 .

ΘΕΜΑ 164 (ΣΩΤΗΡΗΣ ΛΟΥΡΙΔΑΣ)

Επιλύσατε στους θετικούς ακέραιους την εξίσωση που ακολουθεί:

(x 1)(y 1)(z 1) 3xyz .

Λύση:

Έχουμε:

x 1 y 1 z 1(x 1)(y 1)(z 1) 3xyz 3

x y z

1 1 1(1 )(1 )(1 ) 3

x y z

Αν υποθέσουμε ότι τα x,y,z είναι όλα μεγαλύτερα ή ίσα του 3 , τότε θα είχαμε:

Page 153: ΠΡΟΤΕΙΝΟΜΕΝΕΣ ΑΣΚΗΣΕΙΣ ΓΙΑ ΜΑΘΗΤΙΚΟΥΣ ΔΙΑΓΩΝΙΣΜΟΥΣ_ALL

http://www.mathematica.gr/forum/viewtopic.php?f=109&t=15584

Επιμέλεια : xr.tsif Σελίδα 62

1 1 1 4x 3 1

x 3 x 3 .

Όμοια έχουμε: 1 4 1 4

1 και 1y 3 z 3

.

Με πολλαπλασιασμό κατά μέλη παίρνουμε: 1 1 1 64

(1 )(1 )(1 ) 3x y z 27

Τούτο όμως είναι άτοπο.

Άρα τουλάχιστον ένα απο τα x,y,z θα είναι μικρότερο του 3 .

Ας υποθέσουμε ότι x 3 . Τότε θα είναι x 1 ή x 2 .

1η Περίπτωση: x 1 .

Τότε η δοσμένη εξίσωση γράφεται:

2(y 1)(z 1) 3 1 xy 2yz 2y 2z 2 3yz yz 2y 2z 2

2y 2(y 2)z 2y 2 z

y 2

(αφού προφανώς είναι y 2 )

Άρα y 1 y 2 3 y 2 3 3

z 2 2 2( ) 2(1 )y 2 y 2 y 2 y 2 y 2

.

Για να είναι θετικός ακέραιος ο z θα πρέπει y 2 1 ή y 2 3 και άρα

y 3 ή y 5 .

Οπότε έχουμε: x 1 , y 3 , z 8 ή x 1 , y 5 , z 4 .

Και οι τιμές αυτές εύκολα διαπιστώνουμε ότι επαληθεύουν την δοσμένη εξίσωση.

2η Περίπτωση: x 2

Τότε ομοίως έχουμε:

y 1 y 1 2 23(y 1)(z 1) 3 2yz yz z y 1 z 1

y 1 y 1 y 1

Page 154: ΠΡΟΤΕΙΝΟΜΕΝΕΣ ΑΣΚΗΣΕΙΣ ΓΙΑ ΜΑΘΗΤΙΚΟΥΣ ΔΙΑΓΩΝΙΣΜΟΥΣ_ALL

http://www.mathematica.gr/forum/viewtopic.php?f=109&t=15584

Επιμέλεια : xr.tsif Σελίδα 63

Και για να είναι ο z θετικός ακέραιος θα πρέπει y 1 1 ή y 1 2 δηλαδή

y 2 ή y 3 .

Άρα x 2 , y 2 , z 3 ή x 2 , y 3 , z 2 .

Και πάλι εύκολα διαπιστώνουμε ότι οι τιμές αυτές επαληθεύουν την δοσμένη

εξίσωση.

Με δεδομένο τώρα ότι η εξίσωση δεν αλλάζει αν αντιμεταθέσουμε τα γράμματα

x,y,z συμπεραίνουμε ότι οι ζητούμενες τριάδες (x,y,z) είναι οι εξής:

(2,2,3) , (2,3,2) , (3,2,2) , (1,3,8) , (1,8,3) , (3,8,1) , (3,1,8) , (8,3,1) , (8,1,3) ,

(1,4,5) , (1,5,4) , (4,1,5) , (4,5,1) , (5,1,4) , (5,4,1) .

Β τρόπος

Ας υποθέσουμε ότι: x y z .

Άρα οδηγούμαστε στην

3 3 3

1 1 1 643 1 z 3 1 1 3,

z z 3 27

άτοπο.

Άρα z 1 ή z 2 .

Για z 1 ,

1 1 3

1 1 xy 2x 2y 2 0 x 2 y 2 6x y 2

x,y 8,3

η

x,y 8,3

Για z 2

x 1 y 1 2 x 3 και y 2.

Page 155: ΠΡΟΤΕΙΝΟΜΕΝΕΣ ΑΣΚΗΣΕΙΣ ΓΙΑ ΜΑΘΗΤΙΚΟΥΣ ΔΙΑΓΩΝΙΣΜΟΥΣ_ALL

http://www.mathematica.gr/forum/viewtopic.php?f=109&t=15584

Επιμέλεια : xr.tsif Σελίδα 64

Μια παρόμοια, από τη φετινή τσεχική ολυμπιάδα:

Βρείτε όλες τις τριάδες πρώτων αριθμών (p,q,r) που ικανοποιούν την εξίσωση

(p 1)(q 2)(r 3) 4pqr.

www.artofproblemsolving.com/Forum/viewtopic.php?p=2344774&#p2344774

ΘΕΜΑ 165 (ΔΗΜΗΤΡΗΣ ΙΩΑΝΝΟΥ)

Γράφουμε στη σειρά τους αριθμούς από το 1990 έως και το 1997 . Να εξετάσετε

αν ο αριθμός που προκύπτει είναι πρώτος.

Λύση:

Ο αριθμός που προκύπτει είναι ο 19901991199219931994199519961997 .

Αν προσθέσουμε τα ψηφία του βρίσκουμε άθροισμα 180 . Και επειδή ο 180

διαιρείται τουλάχιστον με το 3 , άρα και ο παραπάνω αριθμός διαιρείται

τουλάχιστον με το 3 και άρα δεν μπορεί να είναι πρώτος.

ΘΕΜΑ 166 (ΔΗΜΗΤΡΗΣ ΙΩΑΝΝΟΥ)

Δίνεται ισοσκελές τρίγωνο ΑΒΓ ( ΑΒ ΑΓ ). Με διάμετρο την πλευρά ΑΓ

γράφουμε κύκλο που τέμνει την πλευρά ΒΓ στο Δ . Φέρνουμε ακόμα την Αx

κάθετη στην ΑΔ που τέμνει τον κύκλο στο Ε .

i) Να αποδείξετε ότι το ΑΔ είναι ύψος του τριγώνου ΑΒΓ .

ii) Να συγκρίνετε το εμβαδόν του τριγώνου ΑΒΓ προς το εμβαδόν του

τετραπλεύρου ΑΔΓΕ .

Λύση:

i) Η γωνία ΑΔΓ

είναι εγγεγραμμένη και βαίνει σε ημικύκλιο. Άρα είναι ορθή και

άρα το ΑΔ είναι ύψος του ισοσκελούς τριγώνου ΑΒΓ .

Page 156: ΠΡΟΤΕΙΝΟΜΕΝΕΣ ΑΣΚΗΣΕΙΣ ΓΙΑ ΜΑΘΗΤΙΚΟΥΣ ΔΙΑΓΩΝΙΣΜΟΥΣ_ALL

http://www.mathematica.gr/forum/viewtopic.php?f=109&t=15584

Επιμέλεια : xr.tsif Σελίδα 65

ii) Επειδή το τετράπλευρο

ΑΔΓΕ είναι ορθογώνιο

(αφού έχει 3 γωνίες ορθές)

η διαγώνιος ΑΓ το χωρίζει

σε δύο ίσα και άρα και

ισεμβαδικά τρίγωνα.

Επίσης το ύψος ΑΔ του

ισοσκελούς τριγώνου

ΑΒΓ (που είναι και

διάμεσος) χωρίζει το

τρίγωνο σε δύο ίσα άρα και

ισεμβαδικά τρίγωνα.

Άρα έχουμε: (ΑΔΓΕ) 2(ΑΓΔ) (ΑΒΓ) .

ΘΕΜΑ 167 (ΔΗΜΗΤΡΗΣ ΙΩΑΝΝΟΥ)

Στην ημιευθεία Οx θεωρούμε σημεία Α,Β,Γ ώστε (ΟΑ) 2μ , (ΟΒ) 6μ και

(ΟΓ) 12μ . Αν Δ,Ε,Ζ τα μέσα των ΑΒ,ΒΓ,ΓΑ αντίστοιχα, να υπολογίσετε τα

(ΔΖ) και (ΕΓ) . Τι παρατηρείτε;

Λύση:

ΑΒ ΑΓ ΑΒ ΑΓ ΑΒ 6μΔΖ ΔΒ ΒΖ ΑΖ ΑΒ 3μ

2 2 2 2 2

.

ΒΓ ΟΓ ΟΒΕΓ 3μ

2 2

.

ΘΕΜΑ 168 (ΔΗΜΗΤΡΗΣ ΙΩΑΝΝΟΥ)

Δίνεται τρίγωνο ΑΒΓ με πλευρές ΑΒ 6 , ΒΓ 8 και έστω ΑΜ διάμεσος

αυτού. Η μεσοκάθετη της διαμέσου ΑΜ τέμνει την πλευρά ΑΓ στο σημείο Ε . Αν

οι πλευρές ΑΒ,ΑΓ και ΒΓ είναι ανάλογες προς τις πλευρές ΕΜ,ΜΓ και ΕΓ του

τριγώνου ΕΜΓ αντίστοιχα, να βρεθεί το μήκος της πλευράς ΑΓ .

Page 157: ΠΡΟΤΕΙΝΟΜΕΝΕΣ ΑΣΚΗΣΕΙΣ ΓΙΑ ΜΑΘΗΤΙΚΟΥΣ ΔΙΑΓΩΝΙΣΜΟΥΣ_ALL

http://www.mathematica.gr/forum/viewtopic.php?f=109&t=15584

Επιμέλεια : xr.tsif Σελίδα 66

Λύση:

Έχω ότι : AB AΓ BΓ 6 AΓ 8

λEM MΓ EΓ EM 4 EΓ

Από το σχήμα γνωρίζω ότι :

ΕΜ ΕΑ , AE EΓ AΓ

Άρα :

EM·λ 6 EA·λ 6 ,

4λ AΓ

EΓ·λ 8

Άρα EA EΓ AΓ

2 26 8 14 14 14 144λ 4λ 4·λ 14 λ λ

λ λ λ 4 4 2 .

Επομένως 4· 14

AΓ 2 142

.

ΘΕΜΑ 169 (ΔΗΜΗΤΡΗΣ ΙΩΑΝΝΟΥ)

Στο τέλος του Β Παγκοσμίου πολέμου σε ένα στρατόπεδο βρίσκονται 1997

αιχμάλωτοι: Οι 998 είναι Ιταλοί και οι 999 είναι Γερμανοί. Ο Διοικητής του

στρατοπέδου αποφασίζει να απελευθερώσει σταδιακά τους κρατούμενους, εκτός

από έναν τον οποίο θα κρατήσει για λίγο καιρό ακόμα στο στρατόπεδο.

Η διαδικασία απόλυσης των κρατουμένων είναι η εξής:

Επιλέγονται τυχαία τρεις κρατούμενοι και φεύγουν οι δύο.

Αν και οι τρεις είναι της ίδιας εθνικότητας, ο ένας από αυτούς επιστρέφει, ενώ αν

είναι διαφορετικής εθνικότητας επιστρέφει αυτός που έχει διαφορετική εθνικότητα

από τους άλλους δύο.

Ποιας εθνικότητας θα είναι ο "άτυχος" κρατούμενος;

Page 158: ΠΡΟΤΕΙΝΟΜΕΝΕΣ ΑΣΚΗΣΕΙΣ ΓΙΑ ΜΑΘΗΤΙΚΟΥΣ ΔΙΑΓΩΝΙΣΜΟΥΣ_ALL

http://www.mathematica.gr/forum/viewtopic.php?f=109&t=15584

Επιμέλεια : xr.tsif Σελίδα 67

Λύση:

Έστω x ο αριθμό των γερμανών κρατουμένων , τότε

Αν επιλεγούν 3 Γερμανοί , επιστρέφει ο 1 , οπότε φεύγουν 2 .

Αν επιλεγούν 2 Γερμανοί και 1 Ιταλός, επιστρέφει ο Ιταλός, οπότε

φεύγουν 2 Γερμανοί.

Αντίστοιχα στις άλλες πειπτώσεις θα φεύγουν 2 Ιταλοί.

Άρα ο αριθμός των κρατουμένων θα μειώνεται κάθε φορά ανά 2 , και αφού οι

Γερμανοί είναι 999 , τελευταίος και άτυχος θα είναι Γερμανός.

1.Διοφαντική εξίσωση, ονομάζεται μια εξίσωση για την οποία μας ενδιαφέρουν

μόνο οι ακέραιες λύσεις της.

πχ.

Αν μας δώσουν την 1

yx

από μόνη της, τότε οι περισσότεροι από εμάς θα

κοιτάξουν ειρωνικά τον συνομιλητή τους και θα πουν πως αυτή είναι μια

συνάρτηση, και άρα από μόνη της αν την δούμε ως εξίσωση έχει άπειρες λύσεις.

Αν όμως μας έλεγαν πως είναι διοφαντική;

Τότε οι λύσεις θα ήταν μόνο τα (1,1) και ( 1, 1) .

2.Γραμμική διοφαντική εξίσωση, ονομάζεται η διοφαντική εξίσωση της

μορφής 1 1 2 2 n n

a x a x ... a x c , όπου οι αριθμοί i

a και το c είναι

σταθεροί, γνωστοί αριθμοί και τα i

x οι άγνωστοι.

Για να εξετάσουμε αν όντως υπάρχουν λύσεις, πρώτα υπολογίζουμε τον

μέγιστο κοινό διαιρέτη των i

a . Δηλαδή, τον αριθμό 1 2 n

d (a ,a ,...,a ) .

Αν d / c , υπάρχουν λύσεις. Αν όχι, τότε δεν υπάρχουν.

Ας συγκεντρωθούμε όμως στην περίπτωση n 2 .

Δηλαδή 1 1 2 2

a x a x c

Page 159: ΠΡΟΤΕΙΝΟΜΕΝΕΣ ΑΣΚΗΣΕΙΣ ΓΙΑ ΜΑΘΗΤΙΚΟΥΣ ΔΙΑΓΩΝΙΣΜΟΥΣ_ALL

http://www.mathematica.gr/forum/viewtopic.php?f=109&t=15584

Επιμέλεια : xr.tsif Σελίδα 68

ή, για να φαίνεται καλύτερα, ax by c , d (a,b) .

Πρώτα, βλέπουμε αν d / c όπως περιγράψαμε πιο πάνω.

Μετά, για ευκολία στους υπολογισμούς, διαιρούμε και τα δυο μέλη με d .

Οπότε προκύπτει η ex fy g , όπου a

ed

, b

fd

, c

gd

.

Προφανώς (e,f ) 1 και 1 / g , οπότε υπάρχει λύση.

Βρίσκουμε μια λύση, o

x x και o

y y .

Δεν είναι όμως η μοναδική. Συγκεκριμένα, υπάρχουν άπειρες λύσεις.

Για να τις βρούμε, απλά κάνουμε αντικατάσταση της λύσης μας στις

ox x f t και

oy y e t , t Z .

Δίνουμε ακέραιες τιμές στο t για να βρούμε τις λύσεις. (κάθε ακέραιη τιμή του t

δίνει μια λύση)

-------------------------------------------------------------------------------------------

Παράδειγμα.

Η διοφαντική 6x 15y 21

έχουμε (6,15) 3 και 3 / 21. Άρα υπάρχει λύση.

Διαιρούμε δια 3 και τα δυο μέλη, και έχουμε 2x 5y 7 .

Προφανώς μια λύση είναι η x 1 , y 1 .

Με τις αντικαταστάσεις, έχουμε x 1 5t και y 1 2t .

Δίνοντας ακέραιες τιμές στο t έχουμε τις υπόλοιπες, άπειρες λύσεις.

πχ, στο t 1 έχουμε x 6 , y 1 .

στο t 0 έχουμε x 1 , y 1 .

στο t 1 έχουμε x 4 , y 3 κτλ.

ΘΕΜΑ 170 (Ferma_96)

Ένας ερασιτέχνης ψαράς, αποφασίζει μια μέρα, με ευκαιρία το ψάρεμα του να

κάνει κάποιες επισκέψεις στους φίλους του. Ξεκινάει λοιπόν από την θάλασσα

όπου και ψάρευε, παίρνει μαζί του τα ψάρια που ψάρεψε στην θάλασσα, και πάει

να βρει τους φίλους του. Οι φίλοι του είναι 3 , μένουν σε 3 διαφορετικά σπίτια, και

πριν πάει σε κάθε σπίτι πρέπει να περάσει από ένα ποτάμι. Σε κάθε ποτάμι

Page 160: ΠΡΟΤΕΙΝΟΜΕΝΕΣ ΑΣΚΗΣΕΙΣ ΓΙΑ ΜΑΘΗΤΙΚΟΥΣ ΔΙΑΓΩΝΙΣΜΟΥΣ_ALL

http://www.mathematica.gr/forum/viewtopic.php?f=109&t=15584

Επιμέλεια : xr.tsif Σελίδα 69

ψαρεύει, και διπλασιάζει τα ψάρια που έχει εκείνη την στιγμή. Ξεκινάει λοιπόν για

το πρώτο σπίτι, και περνάει από το πρώτο ποτάμι όπου ψαρεύει. Φτάνει στον φίλο

του, στον οποίο και δίνει λίγα ψάρια. Φεύγει, και περνάει από ένα δεύτερο ποτάμι,

ψαρεύει, και καταλήγει στον δεύτερο φίλο του, στον οποίο δίνει μια ποσότητα

ψαριών, ίση με αυτή που έδωσε στον προηγούμενο. Τέλος, περνάει από το τρίτο

ποτάμι, ψαρεύει, και δίνει στον τρίτο φίλο του μια ποσότητα ψαριών ίση με αυτή

που ήδη έδωσε στον καθένα από τους δυο προηγούμενος. Αν του περίσσεψε

ακριβώς ένα ψάρι, να βρείτε πόσα ψάρια μπορεί να είχε στην αρχή.(όλες τις

πιθανές τιμές).

Λύση:

Έστω ότι ο ψαράς είχε ψαρέψει x ψάρια στην θάλασσα και ότι σε κάθε σπίτι,

άφηνε από y ψάρια.

Περνώντας από το πρώτο ποτάμι, τα ψάρια του γίνονται 2x , αφήνει y ψάρια στο

πρώτο σπίτι και άρα του μένουν 2x y ψάρια.

Περνώντας από το δεύτερο ποτάμι, τα ψάρια του γίνονται 2(2x y) , αφήνει y

ψάρια στο δεύτερο σπίτι και άρα του μένουν 4x 3y ψάρια.

Τέλος, περνώντας από το τρίτο ποτάμι, τα ψάρια του γίνονται 2(4x 3y) , αφήνει

y ψάρια στο τρίτο σπίτι και άρα του μένουν 8x 7y ψάρια.

Με βάση τώρα την εκφώνηση, θα πρέπει να ισχύει η εξίσωση: 8x 7y 1

Μια λύση προφανής της διοφαντικής αυτής εξίσωσης είναι η x 1 , y 1 .

Άρα με βάση τα στοιχεία θεωρίας , έχουμε τις εξής άπειρες λύσεις:

x 1 7t,y  1 8t όπου πρέπει ο t να είναι αρνητικός ακέραιος (αφού οι αριθμοί

x,y πρέπει να είναι θετικοί ακέραιοι).

ΘΕΜΑ 171 (Ferma_96)

Μια κυρία, πάει να αγοράσει μήλα, πορτοκάλια, και αχλάδια. Τα μήλα είναι 5

ευρώ το κιλό, τα πορτοκάλια 4 και τα αχλάδια 7 . Η κυρία κρατάει 100 ευρώ

ακριβώς μαζί της. Πηγαίνοντας όμως σπίτι της, διαπιστώνει ότι τόσο τα

πορτοκάλια όσο και τα αχλάδια ήταν κάτι παραπάνω από χαλασμένα. Πάει πίσω

Page 161: ΠΡΟΤΕΙΝΟΜΕΝΕΣ ΑΣΚΗΣΕΙΣ ΓΙΑ ΜΑΘΗΤΙΚΟΥΣ ΔΙΑΓΩΝΙΣΜΟΥΣ_ALL

http://www.mathematica.gr/forum/viewtopic.php?f=109&t=15584

Επιμέλεια : xr.tsif Σελίδα 70

στο κατάστημα, βρίσκει τον ιδιοκτήτη, κάνει θέμα. Αρχίζει να φωνάζει, να απειλεί

ότι θα το πει σε όλη την γειτονιά και δεν θα πατήσει ποτέ κανείς ξανά. Ο

ιδιοκτήτης τρομοκρατημένος από αυτό το τελευταίο, της ζητάει συγνώμη, την

παρακαλάει, και στο τέλος της λέει να κλείσουν μια συμφωνία. Θα της επέστρεφε

πίσω τα λεφτά που έδωσε για τα πορτοκάλια και για τα αχλάδια, και επιπλέον θα

της έδινε και 3 ευρώ για κάθε κιλό πορτοκαλιών και αχλαδιών πήρε. Αν το κέρδος

του ιδιοκτήτη, μετά από όλη αυτή την ιστορία ήταν μόνο17 ευρώ, να βρείτε πόσα

κιλά μήλα, πορτοκάλια, και αχλάδια μπορεί αγόρασε η κυρία αρχικά.

Λύση:

Έστω ότι η κυρία αγόρασε x κιλά μήλα και y πορτοκάλια και αχλάδια.

Άρα 5x 3y 17 .

Έχουμε ότι (5, 3) (5,3) 1 .

Μία προφανής λύση της διοφαντικής εξίσωσης είναι x 4 , y 1 .

Άρα οι απειρες λύσεις δίνονται από:

x 4 3t,y  1 5t , αφού οι αριθμοί x,y είναι θετικοί ακέραιοι , πρέπει ο

t να είναι αρνητικός ακέραιος.

ΘΕΜΑ 172 (KARANUS)

Αν ο n 18 διαιρεί τον αριθμό n 53 , n φυσικός, τότε να βρεθεί ο n ;

Λύση:

Έχουμε:

ν 53 ν 18 35 351

ν 18 ν 18 ν 18 ν 18

Για να είναι ο αριθμός αυτός ακέραιος, θα πρέπει ο n 18 να είναι διαιρέτης του

35 . Και δεδομένου ότι ο αριθμός ν 53

ν 18

είναι θετικός, θα πρέπει:

n 18 1 , πράγμα άτοπο, αφού n N ή n 18 5 , ομοίως άτοπο ή

n 18 7 , ομοίως άτοπο ή n 18 35 από όπου προκύπτει n 17 .

Page 162: ΠΡΟΤΕΙΝΟΜΕΝΕΣ ΑΣΚΗΣΕΙΣ ΓΙΑ ΜΑΘΗΤΙΚΟΥΣ ΔΙΑΓΩΝΙΣΜΟΥΣ_ALL

http://www.mathematica.gr/forum/viewtopic.php?f=109&t=15584

Επιμέλεια : xr.tsif Σελίδα 71

ΘΕΜΑ 173 (Socrates)

Βρείτε όλες τις τριάδες πρώτων αριθμών (x,y,z) που ικανοποιούν την εξίσωση

(x 1)(y 2)(z 3) 4xyz .

Λύση:

x 1 y 2 z 3(x 1)(y 2)(z 3) 4xyz 4

x y z

1 2 3(1 ) (1 ) (1 ) 4

x y z .

Αν υποθέσουμε ότι τα x,y,z είναι όλα μεγαλύτερα ή ίσα του 4 , έχουμε:

1 1 1 5x 4 1

x 4 x 4 .

Όμοια έχουμε: 1 και 1y

2 6 3 7

4 4z και με πολλαπλασιασμό κατά μέλη

παίρνουμε: 1 105

(1 )(1 )(1 ) 4x y z

2 3 210

64 32 . Τούτο όμως είναι άτοπο.

Άρα τουλάχιστον ένα απο τα x,y,z θα είναι μικρότερο του 4 .

Ας υποθέσουμε ότι x 4 . Τότε θα είναι x 1 ή x 2 ή x 3 .

1η Περίπτωση: x 1 .

Τότε η δοσμένη εξίσωση γράφεται:

2(y 2)(z 3) 4 1 yz (y 2)(z 3) 2yz yz 3y 2z 6 2yz

yz 3y 2z 6 y 2

(y 2)z 3(y 2) z 3y 2

(αφού προφανώς

είναι y 2 )

Άρα y 2 y 2 4 y 2

z 3 3 3( ) 3(1 )y 2 y 2 y

4

y 2

4

2 y 2

.

Page 163: ΠΡΟΤΕΙΝΟΜΕΝΕΣ ΑΣΚΗΣΕΙΣ ΓΙΑ ΜΑΘΗΤΙΚΟΥΣ ΔΙΑΓΩΝΙΣΜΟΥΣ_ALL

http://www.mathematica.gr/forum/viewtopic.php?f=109&t=15584

Επιμέλεια : xr.tsif Σελίδα 72

Για να είναι θετικός ακέραιος ο z θα πρέπει y 2 1 ή y 2 2 ή y 2 4 και

άρα y 3 ή y 4 ή y 6 . Άρα y 3 (πρώτος)

Οπότε έχουμε: x 1 , y 3 , z 15 (όχι πρώτος) ,Απορρίπτεται.

2η Περίπτωση: x 2

Τότε η δοσμένη εξίσωση γράφεται:

3(y 2)(z 3) 4 2 yz 3(y 2)(z 3) 8yz 3yz 9y 6z 18 8yz

9y 185yz 9y 6z 18 (5y 6)z 9y 18 z

5y 6

.

9y 18 4y 24 y 6z 1 1 4

5y 6 5y 6 5y 6

.

Για y 2 , z 9 Απορρίπτεται ή y 3 , z 5 . (δεκτή)

3η Περίπτωση: x 3

Τότε η δοσμένη εξίσωση γράφεται:

4(y 2)(z 3) 4 3 yz (y 2)(z 3) 3yz yz 3y 2z 6 3yz

2yz 3y 2z 6 3y 6

(2y 2)z 3(y 2) z2y 2

(αφού προφανώς είναι

y 1 ).

3y 6 y 8 1 9z 1 1

2y 2 2y 2 2 2y 2

. Απορρίπτεται.

Άρα x 2 , y 3 , z 5 .

ΘΕΜΑ 174 (ΣΩΤΗΡΗΣ ΛΟΥΡΙΔΑΣ)

Έστω ορθογώνιο τρίγωνο Α ΒC , o

A 90

. Αν ο εγγεγραμμένος κύκλος στο

τρίγωνο Α ΒC , εφάπτεται στην υποτείνουσα BC στο σημείο E και δίνονται τα

μήκη EB a , EC b υπολογίστε το εμβαδό του τριγώνου Α ΒC .

Page 164: ΠΡΟΤΕΙΝΟΜΕΝΕΣ ΑΣΚΗΣΕΙΣ ΓΙΑ ΜΑΘΗΤΙΚΟΥΣ ΔΙΑΓΩΝΙΣΜΟΥΣ_ALL

http://www.mathematica.gr/forum/viewtopic.php?f=109&t=15584

Επιμέλεια : xr.tsif Σελίδα 73

Λύση:

Τα τρίγωνα DFB και DEB επειδή είναι ορθογώνια, είναι ίσα ως πλευρά γωνία .

Άρα η πλευρά FB είναι ίση με το a . Το ίδιο συμβαίνει και στα τρίγωνα CG D και

CED και η πλευρά CG είναι ίση με το b . Έστω x AF επειδή τα τρίγωνα

G DA , DF Aείναι ορθογώνια και έχουν τη μια τους γωνία 45 ,είναι και ισοσκελή

άρα x AF FD DG GA . Από το πυθαγόρειο θεώρημα έχω ότι :

2 2 2 2 2 2 2 2 2(x b) (x a) (a b) x b 2xb x a 2ax a b 2ab

2 22x 2ax 2bx 2ab x x(b a) ab 0 .

2 2 2Δ (a b) 4·( ab) a b 6ab

Έστω 2 2a b 6ab k , (a b) k

x2

παίρνω μόνο τη θετική ρίζα

Άρα:

(a b) k (a b) k(a )(b )(a x)(b x) 2 2(ABC)

2 2

Page 165: ΠΡΟΤΕΙΝΟΜΕΝΕΣ ΑΣΚΗΣΕΙΣ ΓΙΑ ΜΑΘΗΤΙΚΟΥΣ ΔΙΑΓΩΝΙΣΜΟΥΣ_ALL

http://www.mathematica.gr/forum/viewtopic.php?f=109&t=15584

Επιμέλεια : xr.tsif Σελίδα 74

2a a b k 2b b a k( )( ) (a b k )(b a k )2 2

2 8

2 2 2 22ab a b a b 6ab 8abab

8 8

.

Το εμβαδόν του τριγώνου είναι E ab .

Β τρόπος

Για λόγους και μόνο πολυφωνίας, δίνω μία διαπραγμάτευση στηριγμένη στο

σχήμα-τετράγωνο που ακολουθεί λαμβάνοντας υπ’ όψη ότι,

2 2

AD b a. Toτε εχoυμε : 4 ABC b a a b 4 ABC 4ab

ABC ab .

(*) Προφανώς ισχύει ότι:

AD AC AB b ρ a ρ b a, όταν ρ είναι η ακτίνα του

εγγεγραμμένου κύκλου.

Γ τρόπος

Είναι : E τρ (a b ρ)ρ .

Page 166: ΠΡΟΤΕΙΝΟΜΕΝΕΣ ΑΣΚΗΣΕΙΣ ΓΙΑ ΜΑΘΗΤΙΚΟΥΣ ΔΙΑΓΩΝΙΣΜΟΥΣ_ALL

http://www.mathematica.gr/forum/viewtopic.php?f=109&t=15584

Επιμέλεια : xr.tsif Σελίδα 75

Επίσης :1 1 1

E (a ρ)(b ρ) [(ab (a b ρ)ρ] (ab E)2 2 2

, οπότε :

ab EE E ab

2 2 .

ΘΕΜΑ 175 (Socrates)

Αν για τους πραγματικούς αριθμούς a,b,c,d ισχύει ότι 2 2 2 2 2 2

a b (a b) c d (c d) , να δείξετε ότι

4 4 4 4 4 4a b (a b) c d (c d) , Ισχύει το αντίστροφο;

Λύση:

Από την υπόθεση, έχουμε ότι 2 2 2 2 2 2 2 2 2 2

(a b) (c d) c d a b 2ab 2cd 2c 2d 2a 2b 2 2 2 2 2 2 2 2

ab cd c d a b ab cd c d a b .

Δηλαδή βρήκαμε ότι: 2 2 2 2 2 2(a b) (c d) c d a b ab cd (1).

Τώρα έχουμε:

4 4 2 2 2 2(a b) (c d) [(a b) (c d) ][(a b) (c d) ]

2 2 2 2 2 2 2 2 2 2 2 2(c d a b )(c d a b ) 2(a b c d )

2 2 2 2 2 2 2 2 2 2 2 2[(c d ) (a b )][(c d ) (a b )] 2a b 2c d

2 2 2 2 2 2 2 2(ab cd)[a b c d 2(ab cd)] (ab cd)(a b c d )

2 2 2 2 2 2 2 2 2 2 4 4 4 42(ab cd)(ab cd) (c d ) (a b ) 2a b 2c d c d a b .

Άρα δείξαμε ότι:

4 4 4 4 4 4 4 4 4 4 4 4(a b) (c d) c d a b a b (a b) c d (c d)

ΘΕΜΑ 176 (Socrates)

Αν οι πραγματικοί αριθμοί x,y,z είναι τέτοιοι ώστε

| x | | y z |, | y | | z x |, | z | | x y | να δείξετε ότι x y z 0

Page 167: ΠΡΟΤΕΙΝΟΜΕΝΕΣ ΑΣΚΗΣΕΙΣ ΓΙΑ ΜΑΘΗΤΙΚΟΥΣ ΔΙΑΓΩΝΙΣΜΟΥΣ_ALL

http://www.mathematica.gr/forum/viewtopic.php?f=109&t=15584

Επιμέλεια : xr.tsif Σελίδα 76

Λύση:

Με ύψωση στο τετράγωνο, έχουμε:

2 2 2 2 2 2 2 2 2x y z 2yz , y z x 2zx , z x y 2xy

Με πρόσθεση αυτών κατά μέλη, παίρνουμε:

2 2 2 20 x y z 2xy 2yz 2xz (x y z) 0 x y z 0 .

ΘΕΜΑ 177 (Socrates)

Αν οι πραγματικοί αριθμοί x,y,z είναι τέτοιοι ώστε

| x y | | z |, | y z | | x |, | z x | | y | ,να δείξετε ότι ένας από αυτούς ισούται με το

άθροισμα των άλλων δύο.

Λύση:

Αν | x y | | z |, | y z | | x |, | z x | | y | τότε ένας από τους x,y,z θα ισούται με

το άθροισμα των δύο άλλων. Αν υποθέσουμε ότι z 0 τότε θα έχουμε από τις

υποθέσεις ότι: x y 0, y x , x y .

Από τις δύο πιο πάνω τελευταίες σχέσεις συμπεραίνουμε ότι x y x y .

Άρα x y από όπου προκύπτει ότι x y 0 δηλ. x y z ή

x y x y 0 x y 0 x y z .

Άρα αν z 0 τότε έχουμε το ζητούμενο.

Έστω z 0 .Τότε διαιρώντας με z παίρνουμε από τις υποθέσεις:

x y1

z z ,

y x1

z z ,

x y1

z z .

Θέτουμε x y

a, bz z .Τότε οι πιο πάνω σχέσεις γράφονται:

Page 168: ΠΡΟΤΕΙΝΟΜΕΝΕΣ ΑΣΚΗΣΕΙΣ ΓΙΑ ΜΑΘΗΤΙΚΟΥΣ ΔΙΑΓΩΝΙΣΜΟΥΣ_ALL

http://www.mathematica.gr/forum/viewtopic.php?f=109&t=15584

Επιμέλεια : xr.tsif Σελίδα 77

a b 1 (1)

b 1 a (2)

1 a b (3)

Ας υποθέσουμε ότι a b (όμοια θα εργασθούμε και αν a b (ενώ η περίπτωση

a b προφανώς απορρίπτεται

Διακρίνουμε τώρα τις παρακάτω περιπτώσεις:

1η Περίπτωση: a b 1 .

Τότε οι σχέσεις (1),(2),(3) γράφονται: a b 1 , b 1 a , a 1 b

Οι δύο πρώτες από αυτές τις σχέσεις δίνουν a b 1,a b 1 που μας οδηγούν

σε άτοπο και άρα η 1η Περίπτωση απορρίπτεται.

2η Περίπτωση: a 1 b .

Τότε οι σχέσεις (1),(2),(3) γράφονται: a b 11 b aa 1 b

Από τις δύο τελευταίες με πρόσθεση κατά μέλη, παίρνουμε b 0 . Οπότε οι

σχέσεις (1),(2),(3) γράφονται: a b 11 b aa 1 b .

Και από τις δύο τελευταίες, παίρνουμε: a b 1a b 1 ,από όπου προκύπτει ότι

x ya b 1 1 x y z

z z , και άρα το ζητούμενο.

3η Περίπτωση: 1 a b .

Τότε οι σχέσεις (1),(2),(3) γράφονται: a b 11 b a 1 a b .

Από την (1) και (3) σχέση, με πρόσθεση κατά μέλη, παίρνουμε b b b 0 .

Τότε οι σχέσεις (1),(2),(3) γράφονται: a b 11 b a 1 a b .

Από την (1) και (3) σχέση παίρνουμε a b 1,a b 1 .

Page 169: ΠΡΟΤΕΙΝΟΜΕΝΕΣ ΑΣΚΗΣΕΙΣ ΓΙΑ ΜΑΘΗΤΙΚΟΥΣ ΔΙΑΓΩΝΙΣΜΟΥΣ_ALL

http://www.mathematica.gr/forum/viewtopic.php?f=109&t=15584

Επιμέλεια : xr.tsif Σελίδα 78

από όπου προκύπτει ότι x y

a b 1 1 x y z x y zz z

, δηλαδή

πάλι το ζητούμενο.

Β τρόπος

Υψώνοντας στο τετράγωνο έχουμε:

(x y z)(x y z) 0

(y z x)(y z x) 0

(z x y)(z x y) 0.

Πολλαπλασιάζοντας τις παραπάνω: 2 2 2(x y z) (x z y) (y z x) 0

οπότε 2 2 2(x y z) (x z y) (y z x) 0.

Γ τρόπος

Από την συμμετρία του προβλήματος, χωρίς βλάβη της γενικότητας μπορούμε να

υποθέσουμε ότι x y z . (*)

Τότε έχουμε x y | z | και y z | x | άρα x z | x | | z | x z . Για να ισχύει

όμως αυτό πρέπει να έχουμε παντού ισότητες και άρα πρέπει να έχουμε y z | x |

και x | x | . Άρα y z x .

------------------------------------------

(*) Τι εννοώ εδώ; Κανονικά πρέπει να ελέγξουμε όλες τις περιπτώσεις. Π.χ.

x y z , y x z κ.τ.λ. Το πρόβλημα όμως είναι συμμετρικό. Αν λοιπόν ισχύει ότι

y x z και πάρω την παραπάνω απόδειξη και όπου x γράψω y και όπου y

γράψω x , θα πάρω την απόδειξη για αυτήν την περίπτωση. Ας το αντιγράψω

λοιπόν για να το δούμε:

Αν y x z , τότε έχουμε y x | z | και x z | y | άρα y z | y | | z | y z .

Για να ισχύει όμως αυτό πρέπει να έχουμε παντού ισότητες και άρα πρέπει να

έχουμε x z | y | και y | y | . Άρα x z y .

Page 170: ΠΡΟΤΕΙΝΟΜΕΝΕΣ ΑΣΚΗΣΕΙΣ ΓΙΑ ΜΑΘΗΤΙΚΟΥΣ ΔΙΑΓΩΝΙΣΜΟΥΣ_ALL

http://www.mathematica.gr/forum/viewtopic.php?f=109&t=15584

Επιμέλεια : xr.tsif Σελίδα 79

Το ίδιο ακριβώς θα έπρεπε να κάνουμε και για τις υπόλοιπες περιπτώσεις. Για να

αποφύγουμε αυτήν την δουλειά, επιτρέπεται να γράψουμε την πρόταση (*) και να

ελέγξουμε μόνο την μία περίπτωση. (Εννοείται ότι το πρόβλημα πρέπει όντως να

είναι συμμετρικό στα x,y,z αλλιώς η απόδειξη θα είναι ελλιπής.)

ΘΕΜΑ 178 (Socrates)

Έστω M ένα σύνολο ακεραίων, τέτοιο ώστε

2

2

• 0 M

• (x 6x 9) M x M

• x M x M.

. Να δείξετε ότι:

α) υπάρχουν μη μηδενικοί και διαφορετικοί ανά δύο a,b,c,d M τέτοιοι ώστε

a b c d 0

β) 2007 M.

Λύση:

α) Φαντάζομαι ότι τα a,b,c,d M .

Kατ’ αρχήν σημειώνουμε ότι 2 2x 6x 9 (x 3) . Έχουμε

2(x 3) 0 M x 3 M

Άρα θα είναι και 2 23 9 M ,9 81 M

Για 2(x 3) 9 M παίρνουμε ότι x 6 M επομένως θα ισχύει και

26 36 M .

Για 2(x 3) 36 παίρνουμε πως x 3 M .

Τέλος για 2(x 3) 81 βρίσκουμε ότι x 6 M .

Συνοψίζοντας έχουμε οτι 6 M, 6 M, 3 M, 3 M , το ζητούμενο έπεται.

Page 171: ΠΡΟΤΕΙΝΟΜΕΝΕΣ ΑΣΚΗΣΕΙΣ ΓΙΑ ΜΑΘΗΤΙΚΟΥΣ ΔΙΑΓΩΝΙΣΜΟΥΣ_ALL

http://www.mathematica.gr/forum/viewtopic.php?f=109&t=15584

Επιμέλεια : xr.tsif Σελίδα 80

β) Για να δείξουμε ότι 2007 M. αρκεί να δείξουμε ότι 2 2(2007 3) 2004 M .

Όμως, σύμφωνα με τον τελευταίο κανόνα για να δείξουμε ότι 22004 M αρκεί να

είναι: 2004 M. .

Αλλά για να δείξουμε ότι 2004 M. αρκεί να δείξουμε πώς 2 2

(2004 3) 2001 M δηλαδή ότι 2001 M .

Επαναλαμβάνοντας συνεχώς τα βήματα αυτά αρκεί να είναι

2001 3 3 ... 3 M . Το οποίο ισχύει, αφού 3 M και 2001 πολ3 .

ΘΕΜΑ 179 (spiros filippas)

Θεωρούμε το σύνολο M 1,2,3,...,2003 . Πόσα υποσύνολα του M υπάρχουν με

άρτιο πλήθος στοιχείων;

Λύση:

Αν A ένα υποσύνολο του M με άρτιο πλήθος στοιχείων, τότε το συμπλήρωμά του

M A έχει περιττό, και αντίστροφα. Έτσι ζευγαρώνουμε ένα προς ένα τα

υποσύνολα με άρτιο πλήθος με εκείνα που έχουν περιττό. Συνεπώς τα άρτιου

πλήθους υποσύνολα είναι ακριβώς τα μισά, δηλαδή 2003 20021·2 2

2 .

(Μέτρησα και το κενό σύνολο, που έχει "άρτιο" πλήθος (μηδέν) στοιχείων ως ένα

από τα υποσύνολά μας. Αν δεν μας αρέσει, το διώχνουμε, οπότε μένουν 2002

2 1

σύνολα)

ΘΕΜΑ 180 (ΣΩΤΗΡΗΣ ΛΟΥΡΙΔΑΣ)

Ένα σύνολο έχει 2n 1 στοιχεία. Ποιο το πλήθος των υποσυνόλων του με n το

πολύ στοιχεία;

Λύση:

Θεωρείται γνωστό για εκείνους τους Juniors που δίνουν στους Μαθηματικούς

διαγωνισμούς (πέραν, βέβαια, των πρώτων φάσεων) το τμήμα εκείνο των βασικών

εννοιών επί των συνόλων:

Page 172: ΠΡΟΤΕΙΝΟΜΕΝΕΣ ΑΣΚΗΣΕΙΣ ΓΙΑ ΜΑΘΗΤΙΚΟΥΣ ΔΙΑΓΩΝΙΣΜΟΥΣ_ALL

http://www.mathematica.gr/forum/viewtopic.php?f=109&t=15584

Επιμέλεια : xr.tsif Σελίδα 81

Συμπλήρωμα, τομή, ένωση, αρχή εγκλεισμού – αποκλεισμού, ότι το πλήθος των

υποσυνόλων ενός συνόλου με n στοιχεία είναι n2 απλή αρχή του περιστερώνα.

Έτσι ας δούμε την εξής λύση:

Κάθε υποσύνολο του συνόλου μας με στοιχεία το πολύ n έχει αντίστοιχο

συμπληρωματικό ένα σύνολο με το λιγότερο n 1 στοιχεία. Σε κάθε σύνολο

αντιστοιχίζεται ένα συμπληρωματικό (η ένωση τους δίνει το σύνολό μας και δεν

έχουν τομή δηλαδή είναι αυτό που λέμε ξένα μεταξύ τους).

Επομένως το ζητούμενο πλήθος είναι το μισό του πλήθους των στοιχείων του

συνόλου των υποσυνόλων του συνόλου μας που είναι 2n 12

,δηλαδή είναι 2n2 .

B τρόπος

O τρόπος που μπορούμε κάθε φορά να πάρουμε k στοιχεία από ένα σύνολο

στοιχείων με 2n 1 είναι 2n 1

k

, συνεπώς μας ενδιαφέρει να υπολογίσουμε το

άθροισμα είναι 2n 1 2n 1 2n 1 2n 1

...0 1 2 n

,(προκύπτει από την

αρχή του αθροίσματος), που είναι το πλήθος των συνόλων που έχουν ως

n στοιχεία.

Όμως από το δυωνυμικό θεώρημα

2n 1 2n 1 2n 1 2n 1 2n 1 2n 1 2n 1

1 1 ... ...0 1 2 n n 1 2n 1

. Όμως ισχύει n n

k n k

.

Άρα 2n 1 2n 1 2n 1 2n 1

,...,2n 1 0 n n

. Και έτσι το δεξί μέλος από την

παραπάνω σχέση ισούται με το διπλάσιο του αθροίσματος που θέλουμε άρα το

άθροισμα ισούται με 2n 1 n1

·2 42

.

Page 173: ΠΡΟΤΕΙΝΟΜΕΝΕΣ ΑΣΚΗΣΕΙΣ ΓΙΑ ΜΑΘΗΤΙΚΟΥΣ ΔΙΑΓΩΝΙΣΜΟΥΣ_ALL

http://www.mathematica.gr/forum/viewtopic.php?f=109&t=15584

Επιμέλεια : xr.tsif Σελίδα 82

...Μια στρατηγική επίλυσης προβλημάτων συνοψίζεται στο εξής: «Αν δεν

μπορείς να λύσεις ένα πρόβλημα, τότε βρες και λύσε ένα πιο εύκολο πρόβλημα

το οποίο σχετίζεται με το αρχικό.

ΘΕΜΑ 181 (Socrates)

α) Να βρείτε τους θετικούς ακέραιους a,b αν ισχύει ab 160 90(a,b)

β) Ομοίως, αν ab 300 7[a,b] 5(a,b) όπου [a,b] και (a,b) το ελάχιστο κοινό

πολλαπλάσιο και ο μέγιστος κοινός διαιρέτης των a,b αντίστοιχα.

Λύση:

α) Υπόδειξη: Αν (a,b) d τότε a dx και b dy όπου οι x,y είναι πρώτοι

μεταξύ τους.

Η εξίσωση γράφεται d(dxy 90) 160 . Τι γίνεται όταν ο d είναι άρτιος; Όταν

είναι περιττός;

Οι διαιρέτες του 160 είναι: 1,2,4,5,8,10,16,20,32,40,80,160 .

1η περίπτωση:

d 1 d 1 (x,y) 1 x 2

dxy 90 160 xy 250 xy 250 y 125

ή

y 2

x 125

.

Άρα a 2,b 125 ή a 125,b 2 .

2η περίπτωση:

d 160 d 160

dxy 90 1 160xy 91

,Αδύνατη.

3η περίπτωση:

d 2 d 2 (x,y) 1 x 5 x 17ή

dxy 90 80 2xy 170 xy 85 y 17 y 5

Άρα a 10,b 34 ή a 34,b 10 .

Page 174: ΠΡΟΤΕΙΝΟΜΕΝΕΣ ΑΣΚΗΣΕΙΣ ΓΙΑ ΜΑΘΗΤΙΚΟΥΣ ΔΙΑΓΩΝΙΣΜΟΥΣ_ALL

http://www.mathematica.gr/forum/viewtopic.php?f=109&t=15584

Επιμέλεια : xr.tsif Σελίδα 83

Όμοια βρίσκουμε τις άλλες πριπτώσεις.

β) Αν [a,b] x και (a,b) y , τότε αφού [a,b] (a,b) ab είναι ab xy και η

εξίσωση γράφεται (x 5)(y 7) 335 (x 5)(y 7) 5 67

x 5 5 και y 7 67 x 10 και y 74

η η

x 5 67 και y 7 5 x 72 και y 12

.

ΘΕΜΑ 182 (Socrates)

Θεωρούμε τρίγωνο Α ΒC ορθογώνιο στο C και έστω CH το ύψος από την

κορυφή C . Οι διχοτόμοι των γωνιών ACH

και BCH

τέμνουν την πλευρά AB

στα σημεία K και L αντίστοιχα. Αν η περίμετρος του τριγώνου είναι 30 μονάδες

και το μήκος του τμήματος KL 4 μονάδες, να βρείτε το μήκος της υποτείνουσας

AB .

Λύση:

Page 175: ΠΡΟΤΕΙΝΟΜΕΝΕΣ ΑΣΚΗΣΕΙΣ ΓΙΑ ΜΑΘΗΤΙΚΟΥΣ ΔΙΑΓΩΝΙΣΜΟΥΣ_ALL

http://www.mathematica.gr/forum/viewtopic.php?f=109&t=15584

Επιμέλεια : xr.tsif Σελίδα 84

Στο τρίγωνο HLC η γωνία

o o o oA A B A A BHLC 90 45 45 45 45

2 2 2 2 2 2

.

Άρα oZHC 45

, οπότε CH CZ KL 4 .

Άρα από ΠΘ ισχύει 2 2 2(x 4) (y 4) (x y 4) xy 8

Άρα Π 12 2x 2y 30 και AB 9 4 13 .

ΘΕΜΑ 183 (Socrates)

α) Βρείτε όλα τα ζεύγη (p,q) πρώτων αριθμών τέτοια ώστε 2 52p 1 q .

β) Ομοίως, αν 6 2 21p q (p q)

2 .

Λύση:

α) Πρώτα μας είναι απαραίτητο να δούμε πως αναπτύσσεται η παράσταση n

(a b) .

Διώνυμο του Νεύτωνα:

n n n 1 n 2 2 n 3 3n n(n 1) n(n 1)(n 2)(a b) a a b a b a b ...

1 1 2 1 2 3

n 1 nn(n 1)...2.ab b

1 2 ... (n 1)

.

Ερχόμαστε τώρα στην λύση της άσκησης.

Ας υποθέσουμε ότι q 2 . Τότε έχουμε 2 5 22p 1 2 2p 31 που είναι άτοπο.

Άρα ο q είναι περιττός και άρα q 2n 1 ,n N * .

Τότε έχουμε: 2 52p 1 (2n 1)

Page 176: ΠΡΟΤΕΙΝΟΜΕΝΕΣ ΑΣΚΗΣΕΙΣ ΓΙΑ ΜΑΘΗΤΙΚΟΥΣ ΔΙΑΓΩΝΙΣΜΟΥΣ_ALL

http://www.mathematica.gr/forum/viewtopic.php?f=109&t=15584

Επιμέλεια : xr.tsif Σελίδα 85

2 5 4 3 2 2 35 4 5 3 4 52p 1 (2n) (2n) 1 (2n) 1 (2n) 1

1 1 2 1 2 3

4 5 2 5 4 3 22 3 4 5(2n) 1 1 2p 32n 80n 80n 40n 10n

1 2 3 4

2 5 4 3 2 2 4 3 2

p 16n 40n 40n 20n 5n p n(16n 40n 40n 20n 5) .

Όμως αφού ο p είναι πρώτος, για να αληθεύει η παραπάνω ισότητα, θα πρέπει να

είναι p n και 4 3 2p 16n 40n 40n 20n 5 , πράγμα όμως που είναι

αδύνατο, αφού η εξίσωση 4 3 2 4 3 2

16n 40n 40n 20n 5 n 16n 40n 40n 19n 5 0, είναι

αδύνατη εφόσον n N* ), ή θα πρέπει να είναι n 1 και 2 4 3 2

p 16n 40n 40n 20n 5 , (από όπου προκύπτει p 11 )

ή θα πρέπει 4 2 316n 40n 40n 20n 5 1 και 2

p n .

(που όμως είναι άτοπο λόγω της πρώτης ισότητας).

Τελικά βρήκαμε ότι πρέπει p 11 οπότε εύκολα βρίσκουμε ότι q 3 .

Μια λύση για την 183(β):

Εύκολα βλέπουμε ότι p q κι άρα gcd(p,q) 1 . (1)

Η δοθείσα γράφεται 5 2p(2p 2q p) 3q (2), οπότε ο p διαιρεί τον 2

3q .

Συνεπώς, λόγω της (1), p 3 όποτε λύνοντας ως προς q τη δευτεροβάθμια

εξίσωση που προκύπτει από τη (2):

2 2 2483 2q q (q 1) 484 22 , εύκολα παίρνουμε q 23 .

Μια διαφορετική λύση για την 183(α):

Παρατηρούμε ότι 5 2 5p 2p 1 q , οπότε p q .

Συνεπώς, ο pείναι περιττός με gcd(p,q 1) 1 . Κι αφού

2 5 4 3 22p q 1 (q 1)(q q q q 1) , αναγκαστικά θα είναι q 1 2 .

Συνεπώς, q 3 κι άρα p 11 .

Page 177: ΠΡΟΤΕΙΝΟΜΕΝΕΣ ΑΣΚΗΣΕΙΣ ΓΙΑ ΜΑΘΗΤΙΚΟΥΣ ΔΙΑΓΩΝΙΣΜΟΥΣ_ALL

http://www.mathematica.gr/forum/viewtopic.php?f=109&t=15584

Επιμέλεια : xr.tsif Σελίδα 86

Άλλη μία πάλι για την 183(α)

Αν p 3 τότε 5 2q 2p 1 19 το οποίο είναι αδύνατον. Επομένως p 3 και

επειδή είναι πρώτος, θα υπάρχει n ώστε p 3n 1 ή p 3n 2 . Στην πρώτη

περίπτωση έχουμε 5 2 2 2q 2(3n 1) 1 18n 12n 3 3(6n 4n 1) . Άρα το

5q είναι πολλαπλάσιο του 3 και άρα πρέπει q 3 . Ομοίως στην δεύτερη

περίπτωση έχουμε 5 2 2 2q 2(3n 2) 1 18n 24n 9 3(6n 8n 3) και πάλι

συμπεραίνουμε ότι q 3 .

Αν λοιπόν υπάρχει λύση πρέπει οπωσδήποτε να έχουμε q 3 . Τότε 2 52p 1 3

και βρίσκουμε p 11 .

Υπάρχει πιο εύκολος τρόπος να γραφτεί το πιο πάνω και χρησιμοποιεί την

ορολογία των ισοδυναμιών (modulo).

Αντί να γράφω «Υπάρχει n ώστε p 3n 1 ή p 3n 2 » θα γράφω «p 1mod3

ή p 2mod3 ». Αυτό δεν είναι τίποτα άλλο παρά μια συντομογραφία. Όταν

έχουμε ακεραίους a,b,c και ισχύει ότι c / (a b) γράφουμε a cbmod . Είναι

όμως και κάτι παραπάνω από μια συντομογραφία. Ας υποθέσουμε ότι

p 2mod3 . Μπορούμε μήπως να πούμε ότι 22p 2 mod3 ; Η απάντηση είναι ναι!

(Για να το αποδείξουμε, πρέπει να δείξουμε ότι αν το p 2 είναι πολλαπλάσιο του

3 τότε και το 2 2p 2 είναι πολλαπλάσιο του 3 . Αυτό όμως ισχύει διότι

2 2(p 2 p 2)(p 2) . Αυτό δεν έτυχε απλώς αλλά συμβαίνει πάντα. Δοκιμάστε

να αποδείξετε ότι αν a ca΄mod καιb cb΄mod τότε a cb (a΄ b΄)mod

και a cb (a΄ b΄)mod .)

Ας ξαναγράψουμε λοιπόν την πιο πάνω λύση χρησιμοποιώντας αυτήν την

ορολογία.

Αν p 3 τότε 5 2q 2p 1 19 το οποίο είναι αδύνατον. Επομένως p 3 και

επειδή είναι πρώτος, θα υπάρχει n ώστε p 1mod3 ή p 2mod3 . Στην πρώτη

περίπτωση έχουμε 5 2q 2p 1 3 0mod3 και στην δεύτερη περίπτωση έχουμε

5 2q 2p 1 9 0mod3 . Άρα το

5q είναι πολλαπλάσιο του 3 και άρα πρέπει

q 3 .

Page 178: ΠΡΟΤΕΙΝΟΜΕΝΕΣ ΑΣΚΗΣΕΙΣ ΓΙΑ ΜΑΘΗΤΙΚΟΥΣ ΔΙΑΓΩΝΙΣΜΟΥΣ_ALL

http://www.mathematica.gr/forum/viewtopic.php?f=109&t=15584

Επιμέλεια : xr.tsif Σελίδα 87

Αν λοιπόν υπάρχει λύση πρέπει οπωσδήποτε να έχουμε q 3 . Τότε 2 52p 1 3

και βρίσκουμε p 11 .

ΠΑΡΑΤΗΡΗΣΕΙΣ

Ας δώσουμε τώρα μερικά στοιχεία θεωρίας τα οποία χρησιμοποιήθηκαν για τις

διάφορες αποδείξεις που έγιναν πιο πάνω, αλλά και μερικά ακόμα που ίσως

χρειαστούν σε επόμενες προτεινόμενες ασκήσεις.

1. Αν [a,b] είναι το ΕΚΠ και (a,b) ο ΜΚΔ των ακεραίων αριθμών a,b τότε

[a,b](a,b) a b .

2. Αν (a,b) 1 , δηλαδή οι αριθμοί a,b είναι πρώτοι μεταξύ τους, και αν 2a b c

τότε οι a,b είναι τετράγωνα ακεραίων.

3. Αν pπρώτος και np / a (δηλ. ο p διαιρεί τον n

a ) τότε n np / a .

4. a bmodm m / a b (δηλαδή ο m διαιρεί τον a b ).

5. a bmodm οι a,b αφήνουν το ίδιο υπόλοιπο όταν διαιρεθούν με τον m .

6. a bmodm b amodm .

7. a bmodm και b cmodm a cmodm .

8. Αν a bmodm και c dmodm ,τότε

i) a c (b d)modm .

ii) a c (b d)modm .

iii) n n

a b modm για κάθε n N .

9. Αν a c (b c)modmc και c 0 τότε a bmodm .

10. Αν a bmodm και n / m τότε a bmodn .

11. 2 2

a bmod2 a b mod4 .

Page 179: ΠΡΟΤΕΙΝΟΜΕΝΕΣ ΑΣΚΗΣΕΙΣ ΓΙΑ ΜΑΘΗΤΙΚΟΥΣ ΔΙΑΓΩΝΙΣΜΟΥΣ_ALL

http://www.mathematica.gr/forum/viewtopic.php?f=109&t=15584

Επιμέλεια : xr.tsif Σελίδα 88

12. 3 3a bmod3 a b mod9 .

13. 2 2 2a bmodm a b 2abmodm .

14. Για κάθε περιττό a ισχύει 2a 1mod8 ή 2

a 1mod2 ή 2a 1mod4 .

15. Αν pπρώτος τότε pa amodp και αν ο pδεν διαιρεί τον a , τότε

p 1a 1modp

.

ΘΕΜΑ 184 (Socrates)

Δείξτε ότι 2 2a b 3a 2b c

,a b b c 4

για όλους τους θετικούς πραγματικούς

αριθμούς a,b,c .

Λύση:

Λόγω της CS ( στη μορφή Αndreescu) αρκεί να δείξουμε την

2

2(a b) 3a 2b c4(a b) (a 2b c)(3a 2b c)

a 2b c 4

Μετά τις πράξεις έχουμε

2 2 2RHS 3a 4b c 8ab 2ac άρα μένει να δείξουμε ότι

2 2 2 2 2 2 24(a b) 4a 8ab 4b 3a 4b c 8ab 2ac (a c) 0 .

Β τρόπος

Αρκεί να δείξουμε ότι 2

x 3x y

x y 4

για κάθε θετικούς πραγματικούς x,y .

[Πράγματι, τότε θα έχουμε 2 2

a b 3a b 3b c 3a 2b c.

a b b c 4 4 4

]

Η απόδειξη τώρα είναι εύκολη. Αρκεί να δείξουμε ότι 2 2 2

4x (x y)(3x y) 3x 2xy y ή ισοδύναμα ότι 2 2x y 2xy το οποίο

ισχύει.

Page 180: ΠΡΟΤΕΙΝΟΜΕΝΕΣ ΑΣΚΗΣΕΙΣ ΓΙΑ ΜΑΘΗΤΙΚΟΥΣ ΔΙΑΓΩΝΙΣΜΟΥΣ_ALL

http://www.mathematica.gr/forum/viewtopic.php?f=109&t=15584

Επιμέλεια : xr.tsif Σελίδα 89

ΘΕΜΑ 185 (CARANUS)

Προεκτείνουμε την υποτείνουσα ΓΒ ενός ορθογωνίου τριγώνου ΑΒΓ ,( oA 90

)

κατά τμήμα ΒΔ ΒΑ . Η διχοτόμος της γωνίας Γ

τέμνει την ΑΔ στο Ε . Ο

κύκλος (γ) με κέντρο Α και ακτίνα ΑΕτέμνει ξανά την ΕΓ στο Ζ . Να

αποδειχθεί ότι η ΕΖ χωρίζει τον κύκλο (γ) σε δύο τόξα , από τα οποία το ένα είναι

τριπλάσιο του άλλου. (Διαγωνισμός ΕΜΕ – 2000)

Λύση:

Φέρνουμε την ΑΖ . Τώρα αρκεί να αποδείξουμε ότι oΕA Ζ 90

. Το ΕAΖ είναι

ισοσκελές, αφού ΑΕ ΑΖ ακτίνες. Άρα ΑΕΖ ΑΖΕ

.

Ισχύει επίσης ότι Γ

ΑΕΖ Δ2

και Γ

ΑΖΕ ΖΑΓ2

. Επομένως

Δ Ζ ΑΓ ΔΑΒ Ζ ΑΓ

Και τελικά ο

ΕΑΖ Δ ΑΒ Β ΑΖ Ζ ΑΓ Β ΑΖ 90

.

Page 181: ΠΡΟΤΕΙΝΟΜΕΝΕΣ ΑΣΚΗΣΕΙΣ ΓΙΑ ΜΑΘΗΤΙΚΟΥΣ ΔΙΑΓΩΝΙΣΜΟΥΣ_ALL

http://www.mathematica.gr/forum/viewtopic.php?f=109&t=15584

Επιμέλεια : xr.tsif Σελίδα 90

ΘΕΜΑ 186 (ΔΗΜΗΤΡΗΣ ΙΩΑΝΝΟΥ)

Αν a,b,c,d είναι πραγματικοί αριθμοί τέτοιοι ώστε (c d)(c d) 0 και

a b a b a b a b

c d c d c d c d

, να αποδείξετε ότι ένας τουλάχιστον από τους

a,b,c,d ισούται με 0 .

Λύση:

a b a b a b a b a b a b a b a b

c d c d c d c d c d c d c d c d

a b a b a b a b 2b 2b2b ( 2d) 0

c d c d c d c d

b 0 ή d 0 .

Αν a 0 και b 0 , τότε η παράσταση γίνεται 0 0 , που ισχύει.

Αν a d 0 , τότε η παράσταση γίνεται 0 0 , που ισχύει.

Αν b d 0 , τότε η παράσταση γίνεται a a

c c , που ισχύει.

ΘΕΜΑ 187 (ΔΗΜΗΤΡΗΣ ΙΩΑΝΝΟΥ)

Να αποδείξετε ότι κάθε εξαψήφιος φυσικός αριθμός της μορφής xyzxyz , όπου

x,y,z είναι ψηφία με x 0 διαιρείται με τους αριθμούς 7,11 και 13 .

Λύση:

5 4 3 2xyzxyz 10 x 10 y 10 z 10 x 10y z 100100x 10010y 1001z

1001(100x 10y z) 7 11 13(100x 10y z) .

Άρα κάθε εξαψήφιος φυσικός αριθμός της μορφής xyzxyz , διαιρείται με τους

αριθμούς 7,11 και 13 .

Page 182: ΠΡΟΤΕΙΝΟΜΕΝΕΣ ΑΣΚΗΣΕΙΣ ΓΙΑ ΜΑΘΗΤΙΚΟΥΣ ΔΙΑΓΩΝΙΣΜΟΥΣ_ALL

http://www.mathematica.gr/forum/viewtopic.php?f=109&t=15584

Επιμέλεια : xr.tsif Σελίδα 91

ΘΕΜΑ 188 (ΔΗΜΗΤΡΗΣ ΙΩΑΝΝΟΥ)

Κατά πόσους διαφορετικούς τρόπους μπορούμε να βάλουμε έναν κόκκινο, δύο

μπλε και τρείς πράσινους βώλους σε έξι τρύπες που βρίσκονται σε ευθεία γραμμή

και ισαπέχουν;

Λύση:

Τοποθετούμε πρώτα τους 3 πράσινους βώλους, αυτό μπορούμε να το κάνουμε με

6 6! 1 2 3 4 5 620

3! (6 3)! 1 2 3 1 2 33

τρόπους.

Τοποθετούμε πρώτα τους 2 μπλε βώλους , αυτό μπορούμε να το κάνουμε με

3 3! 1 2 33

2! (3 2)! 1 2 12

τρόπους.

Τέλος έχουμε 1τρόπο για να τοποθετήσουμε τον κόκκινο βώλο. Με την βασική

αρχή της απαρίθμησης συνολικά θα έχουμε 20 3 1 60 τρόπους.

Β τρόπος

Ο κόκκινος βόλος, μπορεί να τοποθετηθεί στις έξι τρύπες με 6 τρόπους.

Οι άλλοι δύο μαύροι βόλοι, μπορούν να τοποθετηθούν στις υπόλοιπες 5 τρύπες με

10 τρόπους. (Πράγματι, ας ονομάσουμε A,B,C,D,E τις 5 τρύπες. Τότε οι δύο

μαύροι βόλοι, μπορούν να τοποθετηθούν στις τρύπες αυτές με τους εξής τρόπους:

AB,AC,AD,AE,BC,BD,BE,CD,CE,DE δηλαδή με 10 τρόπους. Τέλος οι

υπόλοιποι 3 βόλοι, στις τρεις τρύπες που απομένουν , μπορούν να τοποθετηθούν

μόνο με έναν τρόπο.

Άρα συνολικά οι τρόποι είναι 6 10 1 60 .

ΣΗΜΕΙΩΣΕΙΣ

Τετράγωνο και κύβος αθροίσματος με περισσότερους από δύο προσθετέους

1. 2 2 2 2

1 2 3 n 1 2 n 1 2 1 3 1 n(a a a ... a ) a a ... a 2a a 2a a ... 2a a

Page 183: ΠΡΟΤΕΙΝΟΜΕΝΕΣ ΑΣΚΗΣΕΙΣ ΓΙΑ ΜΑΘΗΤΙΚΟΥΣ ΔΙΑΓΩΝΙΣΜΟΥΣ_ALL

http://www.mathematica.gr/forum/viewtopic.php?f=109&t=15584

Επιμέλεια : xr.tsif Σελίδα 92

2 3 2 4 2 n n 1 n

2a a 2a a ... 2a a ... 2a a

.

2. 3 3 3 3 2

1 2 n 1 2 n 1 2 3 n(a a ... a ) a a ... a 3a (a a ... a )

2 2

2 1 3 n n 1 2 n 13a (a a ... a ) ... 3a (a a ... a ) 6 Σ

όπου Σ , είναι το άθροισμα όλων των γινομένων των 1 2 n

a ,a ,...,a ανά τρεις.

ΘΕΜΑ 189 (Socrates)

Αν σε τρίγωνο Α ΒC ισχύει 2BC AB AC , να δείξετε ότι η κορυφή A , τα

μέσα των πλευρών AB και AC και το έκκεντρο I είναι ομοκυκλικά σημεία.

Λύση:

Θεωρούμε ότι η περιφέρεια (AMN) όταν M,N τα μέσα των AB,AC αντίστοιχα,

τέμνει την διχοτόμο της γωνίας A

στο σημείο I .

Έστω D η προβολή του I στην AC , αν AB AC .

Από το θεώρημα του Mac’ Laurin παίρνουμε:

AM AN AB AC BC BCAD και AE τ BC D E,

2 4 2 2

όταν E είναι το σημείο επαφής του εγγεγραμμένου κύκλου με την πλευρά AC .

Εδώ θεωρούμε ότι το πρόβλημα λύθηκε.

Page 184: ΠΡΟΤΕΙΝΟΜΕΝΕΣ ΑΣΚΗΣΕΙΣ ΓΙΑ ΜΑΘΗΤΙΚΟΥΣ ΔΙΑΓΩΝΙΣΜΟΥΣ_ALL

http://www.mathematica.gr/forum/viewtopic.php?f=109&t=15584

Επιμέλεια : xr.tsif Σελίδα 93

Στο σχήμα φαίνεται και η κατασκευή ενός τριγώνου Α ΒC , ώστε να πληρούνται

τα δεδομένα.

ΘΕΜΑ 190 (Socrates)

Αν οι πραγματικοί αριθμοί a,b,c είναι τέτοιοι ώστε 2 2a (b c) b (a c) 2011

και a b να βρείτε την τιμή της παράστασης 2c (a b).

Λύση:

Έχουμε από την υπόθεση:

2 2 2 2 2 2a (b c) b (a c) a b a c ab cb ab(a b) c(a b)(a b) 0

(a b)(ab ac cb) 0 ab ac bc 0 , αφού είναι a b .

Page 185: ΠΡΟΤΕΙΝΟΜΕΝΕΣ ΑΣΚΗΣΕΙΣ ΓΙΑ ΜΑΘΗΤΙΚΟΥΣ ΔΙΑΓΩΝΙΣΜΟΥΣ_ALL

http://www.mathematica.gr/forum/viewtopic.php?f=109&t=15584

Επιμέλεια : xr.tsif Σελίδα 94

Άρα a(b c) bc (1).

Επίσης, από την υπόθεση έχουμε ότι:

2a (b c) 2011 a a(b c) 2011 a( bc) 2011 abc 2011 (2).

Από την (1) έχουμε c(a b) ab (3).

Τώρα (λόγω και των σχέσεων (2) και (3)) έχουμε: 2

c (a b) c c(a b) c( ab) abc ( 2011) 2011 .

ΘΕΜΑ 191 (Socrates)

Αν για τους θετικούς πραγματικούς αριθμούς a,b,c ισχύει abc 1 να δείξετε ότι

a b c 3.

b(a b) c(b c) a(c a) 2

Λύση:

Θέτω 1

ax

, 1

by

, 1

cz

με xyz 1 .

Η ανισότητα μετά από τις αντικαταστάσεις γίνεται: 2 2 2

y z x 3

x y y z x z 2

.

Από Andreescu παίρνω:

22 2 2 x y zy z x

x y y z x z 2 x y z

.

Άρα τώρα θέλω: x y z 3 που ισχύει λόγω της ΑΜ – ΓΜ αφού xyz 1 .

ΘΕΜΑ 192 (ΔΗΜΗΤΡΗΣ ΙΩΑΝΝΟΥ)

Οι αριθμοί 1,2,3,4,5χωρίζονται σε δύο ομάδες A και B . Είναι αληθές ότι

υπάρχουν δύο αριθμοί πάντα που ανήκουν στην ίδια ομάδα και η διαφορά τους

ανήκει στην ίδια ομάδα; (ΕΜΕ 1995 – Γ Γυμνασίου)

Page 186: ΠΡΟΤΕΙΝΟΜΕΝΕΣ ΑΣΚΗΣΕΙΣ ΓΙΑ ΜΑΘΗΤΙΚΟΥΣ ΔΙΑΓΩΝΙΣΜΟΥΣ_ALL

http://www.mathematica.gr/forum/viewtopic.php?f=109&t=15584

Επιμέλεια : xr.tsif Σελίδα 95

Λύση:

Ναι είναι αληθές: Αν οι 2,4 ανήκουν στην ίδια ομάδα, τελειώσαμε. Άρα

μπορούμε να υποθέσουμε ότι είναι σε διαφορετικές. Κοιτάμε τον 1 . Αν είναι στην

ομάδα του 2 , τελειώσαμε. Άρα μπορούμε να υποθέσουμε ότι το 1 είναι με το 4 .

Συνεχίζοντας, μπορούμε να υποθέσουμε ότι το 3 είναι με το 2 (αν ήταν στην

ομάδα {1,4} , τελειώσαμε). Έτσι οι ομάδες είναι {2,3},{1,4} . Όπου και αν μπει το

5 , τελειώσαμε.

ΘΕΜΑ 193 (ΔΗΜΗΤΡΗΣ ΙΩΑΝΝΟΥ)

Αν ο αριθμός 12a3b διαιρείται με τους αριθμούς 4 και 9 , η μικρότερη τιμή του

ψηφίου a είναι: α) 0 , β) 1 , γ) 2 , δ) 8 , ε) 9

(ΕΜΕ για την Γ Γυμνασίου, 1995)

Λύση:

Για να βρούμε τη μικρότερη τιμή του a πρέπει να βρούμε και τη τιμή του b .

Για να διαιρείται ο 3b με το 4 , ο b πρέπει να είναι 2 ή 6 .

Αν ο b είναι 2 , τότε ο a θα πρέπει να είναι ένα ψηφίο, ώστε ο x να

διαιρείται με το 9 , (όπου x 12a3b ).

Για να διαιρείται με το 9 , πρέπει το άθροισμα των ψηφίων του x να διαιρείται με

το 9 . Έτσι: 12a3b 1 2 a 3 2 8 a

Άρα ο a είναι το ψηφίο 1 .

Αν ο b είναι 6 τότε ο a θα πρέπει να είναι:

12a36 1 2 a 3 6 12 a

Άρα ο a πρέπει να είναι 6 .

(προχώρησα πιο γρήγορα μιας και έχω εξηγήσει πιο πάνω το τρόπο)

Άρα η μικρότερη τιμή του a είναι το 1 και επομένως η σωστή απάντηση είναι το

b .

Page 187: ΠΡΟΤΕΙΝΟΜΕΝΕΣ ΑΣΚΗΣΕΙΣ ΓΙΑ ΜΑΘΗΤΙΚΟΥΣ ΔΙΑΓΩΝΙΣΜΟΥΣ_ALL

http://www.mathematica.gr/forum/viewtopic.php?f=109&t=15584

Επιμέλεια : xr.tsif Σελίδα 96

ΘΕΩΡΙΑ ΑΡΙΘΜΩΝ (ΘΕΩΡΙΑ)

ΘΕΩΡΗΜΑ 1. Αν ο a είναι φυσικός αριθμός μεγαλύτερος του 1 , τότε έχει έναν

τουλάχιστον πρώτο διαιρέτη d όπου 2 d a

ΘΕΩΡΗΜΑ 2. Αν ο p είναι πρώτος και a,b είναι φυσικοί αριθμοί με a,b p

τότε ο p δεν διαιρεί το γινόμενο a b .

ΘΕΩΡΗΜΑ 3. Αν a,b είναι φυσικοί αριθμοί , p πρώτος και αν ο p δεν διαιρεί

τους a,b ,τότε ο p δεν θα διαιρεί ούτε το γινόμενο a b .

ΘΕΩΡΗΜΑ 4. Αν a,b ακέραιοι και όχι ταυτόχρονα μηδέν, τότε υπάρχει ο ΜΚΔ

των a,b και μάλιστα αν a bk u με 0 u b τότε (a,b) (b,u) , όπου με

(a,b) συμβολίζουμε τον ΜΚΔ των a,b .

ΘΕΩΡΗΜΑ 5. Αν a,b είναι ακέραιοι όχι ταυτόχρονα μηδέν, τότε υπάρχουν k,n

ακέραιοι, ώστε (a,b) ka nb .

ΘΕΩΡΗΜΑ 6. Αν (a,b) d τότε:

i) a b

( , ) 1d d

ii) (ca,cb) cd όπου c είναι ακέραιος, διάφορος του μηδενός.

ΘΕΩΡΗΜΑ 7. Αν (a,b) 1 και b / ac τότε b / c .

ΘΕΩΡΗΜΑ 8. Αν (a,b) 1 και a / c , b / c τότε ab / c .

ΘΕΩΡΗΜΑ 9. Αν a,b,c ακέραιοι και (a,b) (a,c) 1 τότε (a,bc) 1 .

ΘΕΩΡΗΜΑ 10. Ανa,b,c ακέραιοι τότε (a,b,c) ((a,b),c) .

(Το παραπάνω θεώρημα γενικεύεται και για περισσότερους ακεραίους)

ΘΕΩΡΗΜΑ 11. Αν i

(a,a ) 1 με 1 i n , τότε 1 2 n

(a,a ,a ,...,a ) 1 .

ΘΕΩΡΗΜΑ 12. Αν a,b,n είναι φυσικοί αριθμοί τότε

Page 188: ΠΡΟΤΕΙΝΟΜΕΝΕΣ ΑΣΚΗΣΕΙΣ ΓΙΑ ΜΑΘΗΤΙΚΟΥΣ ΔΙΑΓΩΝΙΣΜΟΥΣ_ALL

http://www.mathematica.gr/forum/viewtopic.php?f=109&t=15584

Επιμέλεια : xr.tsif Σελίδα 97

i) Αν (a,b) 1 θα είναι και n n(a ,b ) 1 .

ii) Αν n na / b a / b .

ΘΕΩΡΗΜΑ 13. Έστω a είναι φυσικός αριθμός με 1 2 nk k k

1 2 na p p ... p

Τότε το πλήθος των θετικών διαιρετών του a είναι

1 2 3 ns(a) (1 k )(1 k )(1 k )...(1 k ) , ενώ το άθροισμα των θετικών διαιρετών

του a είναι 1 2 nk 1 k 1 k 1

1 2 n

1 2 n

p 1 p 1 p 1Σ(a) ...

p 1 p 1 p 1

.

ΣΗΜΕΙΩΣΗ: Σ(a) a 1 a πρώτος.

ΘΕΜΑ 194 (ΔΗΜΗΤΡΗΣ ΙΩΑΝΝΟΥ)

Να αποδείξετε ότι το υπόλοιπο της διαίρεσης του πρώτου αριθμού p με τον 30

δεν είναι σύνθετος.

Λύση:

Αρχικά προφανώς θα είναι 1 29 ( το υπόλοιπο) και 2k 1 . Επίσης θα

είναι p mod30(p 30y p 30y 30 / p ) .

Για 3 παίρνουμε 30 / p 3 30x p 3 p 3n άτοπο. Το ίδιο συμβαίνει

για τις περιπτώσεις 5,9,15,21,25,27 . Άρα τελικά ο μπορεί να είναι ίσος

μόνο με τους αριθμούς 1,7,11,13,17,19,23,29 δηλαδή μόνο με μη σύνθετους

αριθμούς.

Β τρόπος

Είναι p 30k u (1), όπου 0 u 30 . Αφού όμως p πρώτος, θα πρέπει u 0

οπότε τελικά είναι 0 u 30 .

Ας υποθέσουμε ότι ο u είναι σύνθετος. Τότε θα έχει έναν πρώτο διαιρέτη d με

2 d 30 .

Άρα d 2 ή d 3 ή d 5 .

Page 189: ΠΡΟΤΕΙΝΟΜΕΝΕΣ ΑΣΚΗΣΕΙΣ ΓΙΑ ΜΑΘΗΤΙΚΟΥΣ ΔΙΑΓΩΝΙΣΜΟΥΣ_ALL

http://www.mathematica.gr/forum/viewtopic.php?f=109&t=15584

Επιμέλεια : xr.tsif Σελίδα 98

Διακρίνουμε τις περιπτώσεις:

1η ΠΕΡ: d 2 .

Τότε από την (1) συμπεραίνουμε ότι πρέπει ο 2 να διαιρεί τον p , πράγμα που είναι

άτοπο, αφού pπρώτος.

Όμοια και για τις δύο άλλες περιπτώσεις.

ΘΕΜΑ 195 (ΔΗΜΗΤΡΗΣ ΙΩΑΝΝΟΥ)

Αν οι αριθμοί p και 28p 1 είναι πρώτοι, να βρεθεί ο p .

Λύση:

Μία διαπραγμάτευση με βάση το θεώρημα, ο 3 διαιρεί το γινόμενο τριών

διαδοχικών ακεραίων:

Αν p 3 και 2 2 2 2 23 / 8p 1 8p 8p 1 3 / 8p 1 8p 3k 1,

άτοπο από τις πιθανές περιπτώσεις p 3n ,p 3n 1 , p 3n 2 .

Άρα p 3 .

Β τρόπος

Έστω p 3 . Τότε ο 2p θα έχει τη μορφή 3 1 . Με αντικατάσταση παίρνουμε

28p 1 8(3 1) 1 3n , άτοπο. Άρα θα είναι p 3 .

ΘΕΜΑ 196 (ΔΗΜΗΤΡΗΣ ΙΩΑΝΝΟΥ)

Δίνονται οι φυσικοί αριθμοί a,b,c,d οι οποίοι συνδέονται με την σχέση: ab cd .

Να αποδείξετε ότι ο αριθμός 2011 2011 2011 2011

a b c d είναι σύνθετος.

Page 190: ΠΡΟΤΕΙΝΟΜΕΝΕΣ ΑΣΚΗΣΕΙΣ ΓΙΑ ΜΑΘΗΤΙΚΟΥΣ ΔΙΑΓΩΝΙΣΜΟΥΣ_ALL

http://www.mathematica.gr/forum/viewtopic.php?f=109&t=15584

Επιμέλεια : xr.tsif Σελίδα 99

Λύση:

Είναι γνωστό, ότι κάθε φυσικός αριθμός, μπορεί να αναλυθεί κατά μοναδικό

τρόπο σε γινόμενο πρώτων παραγόντων.

Έτσι, έχουμε ότι:

1 2 kα α α

1 2 ka p p ...p

1 2 λβ β β

1 2 λb q q ...q

μ1 2γγ γ

1 2 μc r r ...r

ρ1 2δδ δ

1 2 ρd s s ...s

(Όπου όλοι οι παραπάνω αριθμοί είναι πρώτοι)

Άρα , αφού ab cd , τότε κάποιοι από τους i j

p ,q θα είναι ίσοι με κάποιους από

τους n m

r ,r , όπου 1 i k , 1 j λ , 1 n μ , 1 m μ .

Άρα μπορούμε να βρούμε φυσικούς αριθμούς x,y,z,w διάφορους του μηδενός ,

ώστε να είναι : a xy,b zw,c xz,d yw , όπου

1 i k , 1 j λ , 1 n μ , 1 m μ .

Τότε έχουμε:

2011 2011 2011 2011 2011 2011 2011 2011 2011 2011 2011 2011a b c d x y z w x z y w

2011 2011 2011 2011 2011 2011 2011 2011 2011 2011x (y z ) w (z y ) (y z )(x w ).

Άρα ο αριθμός αυτός είναι σύνθετος, αφού κανένας από τους δύο παράγοντες δεν

είναι ίσος με την μονάδα

Β τρόπος

Έστω m gcd(a,c) και n gcd(b,d) .

Τότε a mp και c mq με p,q θετικούς ακεραίους με gcd(p,q) 1 , και

Page 191: ΠΡΟΤΕΙΝΟΜΕΝΕΣ ΑΣΚΗΣΕΙΣ ΓΙΑ ΜΑΘΗΤΙΚΟΥΣ ΔΙΑΓΩΝΙΣΜΟΥΣ_ALL

http://www.mathematica.gr/forum/viewtopic.php?f=109&t=15584

Επιμέλεια : xr.tsif Σελίδα 100

b nr και d ns με r,s θετικούς ακεραίους με gcd(r,s) 1 .

Αφού

p a d s

q c b r και gcd(p,q) 1 , gcd(r,s) 1 , έπεται ότι p s και q r .

Τότε 2011 2011 2011 2011 2011 2011 2011 2011a b c d (m n )(p q ) .

με 2011 2011m n 1 και 2011 2011

p q 1 . Άρα ο αριθμός είναι σύνθετος.

ΘΕΜΑ 197 (ΜΑΚΗΣ ΧΑΤΖΟΠΟΥΛΟΣ)

Να προσδιορίσετε τις τιμές του πρώτου αριθμού n , ώστε οι αριθμοί

n,n 10,n 14 να είναι όλοι τους πρώτοι. (Ρωσία 1998)

Λύση:

Υποθέτουμε n 3 . Τότε ο n μπορεί να έχει μόνο τις μορφές 3 1 ή 3 2 .

Για n 3 1 παίρνουμε n 14 πoλ3 , άτοπο.

Για n 3 2 παίρνουμε n 10 πoλ3 , άτοπο. Άρα n 3 .

ΘΕΜΑ 198 (ΜΑΚΗΣ ΧΑΤΖΟΠΟΥΛΟΣ)

Με τα ψηφία 1,4,6,9 και μόνο γράφουμε δύο τυχαίους αριθμούς με αυθαίρετο

πλήθος στοιχείων (π.χ. 4169 , 964419 κλπ). Να αποδείξετε ότι ανάμεσα σε όλους

αυτούς τους αριθμούς που δημιουργούνται δεν υπάρχουν δύο έτσι ,ώστε ο ένας να

διαιρείται με τον άλλο και το πηλίκο να είναι ίσο με 17 . (Βουλγαρία)

Λύση:

Θέλουμε να αποδείξουμε ότι δεν μπορεί να ισχύει η σχέση x 17y , όπου x,y

τυχαίοι αριθμοί αποτελούμενοι από αυθαίρετο πλήθος των ψηφίων 1,4,6,9 .

Διακρίνουμε 4 περιπτώσεις:

το τελευταίο ψηφίο του y είναι το 1 οπότε ο x τελειώνει σε 7

Page 192: ΠΡΟΤΕΙΝΟΜΕΝΕΣ ΑΣΚΗΣΕΙΣ ΓΙΑ ΜΑΘΗΤΙΚΟΥΣ ΔΙΑΓΩΝΙΣΜΟΥΣ_ALL

http://www.mathematica.gr/forum/viewtopic.php?f=109&t=15584

Επιμέλεια : xr.tsif Σελίδα 101

το τελευταίο ψηφίο του y είναι το 4 οπότε ο x τελειώνει σε 8

το τελευταίο ψηφίο του y είναι το 6 οπότε ο x τελειώνει σε 2

το τελευταίο ψηφίο του y είναι το 9 οπότε ο x τελειώνει σε 3 .

ΘΕΜΑ 199 (ΜΑΚΗΣ ΧΑΤΖΟΠΟΥΛΟΣ)

Να αποδειχθεί ότι ο αριθμός 2 89A 1 3 3 ... 3 διαιρείται με το 13 .

( Ρουμανία 1997)

Λύση:

Η παράσταση A έχει 90 όρους άρα μπορούμε να την χωρίσουμε σε τριάδες (με τη

σειρά).

Παρατηρούμε ότι οι τριάδες που έχουν το 3 με εκθέτες διαδοχικούς αριθμούς, το

άθροισμά των όρων κάθε τέτοιας τριάδας είναι διαιρετό με το 13 .

ΑΠΟΔΕΙΞΗ

Έχουμε: Το 33 αφήνει υπόλοιπο 1 διαιρούμενο με το 13 : 3

3 27 13 2 1 .

Αυτό σημαίνει ότι τα 3 άρια υψωμένα σε εκθέτη που είναι πολλαπλάσιο του 3

αφήνουν υπόλοιπο 1 διαιρούμενα με το 13 .

Με την ίδια λογική βρίσκουμε ότι το 43 αφήνει υπόλοιπο 3 αν διαιρεθεί με το 13 ,

ενώ το 5

3 αφήνει υπόλοιπο 9 διαιρούμενο με το 13 .

Παρατηρούμε ότι αν προσθέσουμε τα υπόλοιπα των αριθμών 3

3 , 4

3 και 5

3 το

αποτέλεσμα είναι 13 .

το ίδιο γίνεται και για όμοιες τριάδες με μεγαλύτερο εκθέτη, όπως η 6 6 63 0 , 3 1 , 3 2

το 63 0 αφήνει υπόλοιπο 1 αφού το 60 είναι πολλαπλάσιο του 3 . Το 6

3 1 αφήνει

υπόλοιπο 3 και το 6

3 2 αφήνει υπόλοιπο 9 .

Άρα κάθε τριάδα πρέπει να είναι της μορφής : 3ν 3ν 1 3ν 2

3 ,3 ,3

.

Επίσης η πρώτη τριάδα 2

1 3 3 13 είναι διαιρετή με το 13 .

Page 193: ΠΡΟΤΕΙΝΟΜΕΝΕΣ ΑΣΚΗΣΕΙΣ ΓΙΑ ΜΑΘΗΤΙΚΟΥΣ ΔΙΑΓΩΝΙΣΜΟΥΣ_ALL

http://www.mathematica.gr/forum/viewtopic.php?f=109&t=15584

Επιμέλεια : xr.tsif Σελίδα 102

Άρα αν προσθέσουμε τις τριάδες που οι εκθέτες του 3 είναι διαδοχικοί αριθμοί

και την πρώτη προκύπτει η παράσταση A που είναι διαιρετή με το 13 .

Μπορούμε πιο απλά να πούμε ότι κάθε παρένθεση που περιέχει τους 3 όρους θα

είναι της μορφής x x 1 x 2 x 2 x3 3 3 3 (1 3 3 ) 3 ·13

άρα πολλαπλάσιο του 13 .

Β τρόπος

Είναι 2 89 90 3 302A (3 1)(1 3 3 ... 3 ) 3 1 (3 ) 1 0 mod13 (*),

αφού 33 27 1 mod13 .

Αφού gcd(2,13) 1 έπεται ότι a 0 mod13 .

(διαφορετικά, πολλαπλασιάζοντας την (*) με 7 παίρνουμε

13a a 14a 0 mod13 , κι άρα a 0 mod13 ).

ΘΕΜΑ 200 (ΜΑΚΗΣ ΧΑΤΖΟΠΟΥΛΟΣ)

Να βρεθεί ο πρώτος αριθμός p αν 27p 1 n , όπου n φυσικός μη μηδενικός.

(Γερμανία 1991)

Λύση:

Είναι p 2 αφού ο αριθμός 7·2 1 15 δεν είναι τετράγωνο ακεραίου.

Άρα, ο p είναι περιττός και επομένως ο 2

n , άρα και ο n είναι άρτιος. Ας είναι

n 2m με m θετικό ακέραιο.

Τότε, είναι 27p 1 4m , άρα 7p (2m 1)(2m 1). Επειδή είναι, προφανώς,

2m 1 1 και οι 7,p είναι πρώτοι, θα πρέπει

( 2k 1 7 και 2k 1 p ) ή ( 2k 1 p και 2k 1 7 ).

Στην πρώτη περίπτωση βρίσκουμε k 4 άρα p 9 , άτοπο, αφού p πρώτος.

Στη δεύτερη περίπτωση είναι k 3 και p 5 , δεκτό!

Page 194: ΠΡΟΤΕΙΝΟΜΕΝΕΣ ΑΣΚΗΣΕΙΣ ΓΙΑ ΜΑΘΗΤΙΚΟΥΣ ΔΙΑΓΩΝΙΣΜΟΥΣ_ALL

http://www.mathematica.gr/forum/viewtopic.php?f=109&t=15584

Επιμέλεια: xr.tsif Σελίδα 1

Page 195: ΠΡΟΤΕΙΝΟΜΕΝΕΣ ΑΣΚΗΣΕΙΣ ΓΙΑ ΜΑΘΗΤΙΚΟΥΣ ΔΙΑΓΩΝΙΣΜΟΥΣ_ALL

http://www.mathematica.gr/forum/viewtopic.php?f=109&t=15584

Επιμέλεια: xr.tsif Σελίδα 2

ΠΡΟΤΕΙΝΟΜΕΝΕΣ

ΑΣΚΗΣΕΙΣ ΓΙΑ ΜΑΘΗΤΙΚΟΥΣ

ΔΙΑΓΩΝΙΣΜΟΥΣ

ΤΕΥΧΟΣ 3ο

ΑΣΚΗΣΕΙΣ 201 - 300

Αφιερωμένο σε κάθε μαθητή που ασχολείται ή πρόκειται να ασχοληθεί με

Μαθηματικούς διαγωνισμούς

Τσιφάκης Χρήστος : xr.tsif

Page 196: ΠΡΟΤΕΙΝΟΜΕΝΕΣ ΑΣΚΗΣΕΙΣ ΓΙΑ ΜΑΘΗΤΙΚΟΥΣ ΔΙΑΓΩΝΙΣΜΟΥΣ_ALL

http://www.mathematica.gr/forum/viewtopic.php?f=109&t=15584

Επιμέλεια: xr.tsif Σελίδα 3

ΘΕΜΑ 201 (Socrates)

Αν x,y,z 0 να δείξετε ότι x y y z z x

x y z .y 2 z 2 x 2

Λύση:

Αρκεί να δείξουμε ότι x y

xy 2

,

y zy

z 2

,

z xz

x 2

, αφού τότε με πρόσθεση

κατά μέλη θα πάρουμε τη ζητούμενη.

Πράγματι

x yx xy 2x x y x y xy 0

y 2

y zy y z yz 0

z 2

z xz z x zx 0

x 2

, που ισχύουν.

ΘΕΜΑ 202 (Socrates)

Αν οι πραγματικοί αριθμοί a,b,c είναι τέτοιοι ώστε 2 2 2 3 3 3

a b c a b c 1 , να

βρείτε την τιμή της παράστασης 2 9 2011

P a b c .

(Από τα παραπάνω παραδείγματα, έχει γίνει φανερό το τι προσπαθούμε να

πετύχουμε για να λύνουμε τέτοιου είδους ασκήσεις)

Λύση:

(Α) Έστω 0 a,b,c 1 τότε

3 2a 1 a a

3 2b 1 b b

Page 197: ΠΡΟΤΕΙΝΟΜΕΝΕΣ ΑΣΚΗΣΕΙΣ ΓΙΑ ΜΑΘΗΤΙΚΟΥΣ ΔΙΑΓΩΝΙΣΜΟΥΣ_ALL

http://www.mathematica.gr/forum/viewtopic.php?f=109&t=15584

Επιμέλεια: xr.tsif Σελίδα 4

3 2c 1 c c

και με πρόσθεση κατά μέλη προκύπτει 3 3 3 2 2 2a b c a b c 1 1 άτοπο.

(Β) Έστω a,b,c 0 τότε 3 3 3a b c 0 1 0 άτοπο.

(Γ) Έστω ένα τουλάχιστον από τους αριθμούς a,b,c είναι μεγαλύτερο της μονάδας,

υποθέτουμε χωρίς βλάβη της γενικότητας ότι

a 1 τότε 3 2a a 1 όμως 2 2 2 2 2 2

a b c 1 a 1 b c .

Άρα, 2 2 2 2 2a 1 1 b c 1 b c 0 άτοπο.

(Δ) Έστω ένα τουλάχιστον από τους αριθμούς a,b,c είναι αρνητικός, υποθέτουμε

χωρίς βλάβη της γενικότητας ότι a 0 τότε πρέπει κάποιο από τα b,c να είναι

μεταξύ του μηδέν και ένα, (αν είναι μεγαλύτερο του ένα, τότε από την Γ περίπτωση

θα καταλήγαμε σε άτοπο, αν και οι δύο ήσαν αρνητικοί αριθμοί τότε από την Β

περίπτωση θα ήταν άτοπο), έστω 0 b 1 , τότε: 3a 0 και 2

a 0 άρα 3 2a a ,

επίσης 3 2b b και με πρόσθεση κατά μέλη παίρνουμε: 3 3 2 2

a b a b δηλαδή *c 0

3 2 2 31 c 1 c c c 1 c

που λόγω της Γ περίπτωσης είναι άτοπο.

* Θα δείξουμε ότι c 0 .

Έστω ότι c 0 , τότε η δεδομένη σχέση γίνεται 3 3 2 2

a b a b άτοπο γιατί έχουμε

αποδείξει ότι: 3 3 2 2

a b a b .

Σημείωση: Δεν χρειάζεται, αλλά το δίνω...Θα δείξουμε ότι c 0 .

3a 1 1

3 3 3 3 3 3 3 3 3 3 3a b c 1 a 1 b c 0 b c 1 c 1 b

3 2 3c 1 b 1 b b 0 c 0 c 0 .

και εδώ τελειώνει η διερεύνηση.

Page 198: ΠΡΟΤΕΙΝΟΜΕΝΕΣ ΑΣΚΗΣΕΙΣ ΓΙΑ ΜΑΘΗΤΙΚΟΥΣ ΔΙΑΓΩΝΙΣΜΟΥΣ_ALL

http://www.mathematica.gr/forum/viewtopic.php?f=109&t=15584

Επιμέλεια: xr.tsif Σελίδα 5

Οι περιπτώσεις όλοι οι αριθμοί να είναι μηδέν ή ένα , εύκολα απορρίπτονται και

μένει ένας από αυτούς να είναι 1 και οι άλλοι μηδέν, πχ. a 1 , b c 0 τότε 2 9 2011

P a c 1b .

Β τρόπος

Είναι 2 2 2 2a a b c 1 άρα 2

a 1 οπότε και a 1 . Όμοια b 1 , c 1 . Άρα

2 2 2 3 3 3 2 2 20 1 1 (a b c ) (a b c ) a (1 a) b (1 b) c (1 c) άθροισμα

θετικών.

Συνεπώς 2 2 20 a (1 a) b (1 b) c (1 c) δηλαδή τα a,b,c παίρνουν μόνο τις τιμές

0ή 1 . Με έλεγχο διαπιστώνουμε ότι οι μόνες αποδεκτές είναι οι

(1,0,0),(0,1,0),(0,0,1) και η τελική παράσταση είναι 1 σε όλες τις περιπτώσεις.

ΘΕΜΑ 203 (ΔΗΜΗΤΡΗΣ ΙΩΑΝΝΟΥ )

a) Να αποδειχθεί ότι υπάρχουν άπειρες τριάδες θετικών ακεραίων m,n,k έτσι, ώστε

να ισχύει ότι: 2

4mn m n k 1

b) Να αποδείξετε ότι δεν υπάρχουν θετικοί ακέραιοι m,n,k έτσι, ώστε να ισχύει ότι: 2

4mn m n k .

Λύση:

Π.χ. 2(m,n,k) (1,3 ,3 )

Είναι 22(4m4mn m 1)(4n 1)n k 4k 1 . Υπάρχει πρώτος p της μορφής

p 4a 3 τέτοιος ώστε p / 4m 1 ,οπότε και 2p / 4k 1 .

Αυτό όμως είναι άτοπο, δείτε π.χ.

www.mathematica.gr/forum/viewtopic.php?p=88420#p88420.

Page 199: ΠΡΟΤΕΙΝΟΜΕΝΕΣ ΑΣΚΗΣΕΙΣ ΓΙΑ ΜΑΘΗΤΙΚΟΥΣ ΔΙΑΓΩΝΙΣΜΟΥΣ_ALL

http://www.mathematica.gr/forum/viewtopic.php?f=109&t=15584

Επιμέλεια: xr.tsif Σελίδα 6

ΘΕΜΑ 204 (Socrates)

Να βρείτε τους πρώτους p,q,r έτσι ώστε οι αριθμοί pq qr rp και

3 3 3p q r 2pqr να διαιρούνται με τον p q r .

Λύση:

Έχουμε 2 2 2 3 3 3(p q r)(p q r pq pr qr) p q r 3pqr . Άρα p q r

διαιρεί το 2 2 2(p q r)(p q r pq pr qr) pqr και άρα διαιρεί το pqr .

Επειδή οι p,q,r είναι πρώτοι, χωρίς βλάβη της γενικότητας μπορούμε να υποθέσουμε

ότι ο pδιαιρεί τον pqr .

[Αυτό ισχύει διότι πρέπει ο pqr να ισούται με ένα από τους p,q,r,pq,pr,qr,pqr ].

Επειδή όμως ο pqr διαιρεί και τον pq qr rp και ο pδιαιρεί αυτόν τον αριθμό

και άρα διαιρεί και τον qr . Επειδή όμως ο p είναι πρώτος πρέπει να διαιρεί είτε τον

q είτε τον r , χωρίς βλάβη της γενικότητας τον q . Αλλά και ο q είναι πρώτος και

άρα p q .

Περίπτωση 1η:

Ο r διαιρεί τον p q r . Τότε (αφού ο p q r διαιρεί τον pq qr rp ) ο r

διαιρεί και το 2pq p και άρα r p . Σε αυτήν την περίπτωση οι συνθήκες γίνονται

33p / 3p (η οποία ισχύει για όλα τα p ) και 3

3p / p η οποία ισχύει αν και μόνο αν

p 3 .

Περίπτωση 2η:

Ο r δεν διαιρεί τον p q r . Τότε (αφού ο p q r διαιρεί τον 2pqr p r ) πρέπει

ο p q r να ισούται είτε με p είτε με 2p με το πρώτο να απορρίπτεται. Αλλά τότε

2r p 2p p(p 2) ο οποίος δεν είναι πρώτος εκτός από την περίπτωση που

r p 3 . (Οπότε αναγόμαστε πάλι στην 1η περίπτωση.)

Άρα τελικά η μοναδική λύση είναι η q r p 3 .

Page 200: ΠΡΟΤΕΙΝΟΜΕΝΕΣ ΑΣΚΗΣΕΙΣ ΓΙΑ ΜΑΘΗΤΙΚΟΥΣ ΔΙΑΓΩΝΙΣΜΟΥΣ_ALL

http://www.mathematica.gr/forum/viewtopic.php?f=109&t=15584

Επιμέλεια: xr.tsif Σελίδα 7

ΘΕΜΑ 205 (Socrates)

Πόσοι τετραψήφιοι αριθμοί abcd είναι τέτοιοι ώστε a b c d και 2 2 2 2

a b c d ;

Λύση:

Έχουμε a b c d (1), και 2 2 2 2a b c d (2)

Άρα 2 2(a b) 2ab (c d) 2cd 2ab 2cd (3)

Με πρόσθεση κατά μέλη των σχέσεων (2) και (3) παίρνουμε

2 2(a b) (c d) a b c d a b c d

1η περίπτωση: a b c d (4)

Τότε από τις σχέσεις (1),(4) έπεται ότι a c, b d και άρα ο τετραψήφιος αριθμός

είναι της μορφής abab

Και τέτοιας μορφής τετραψήφιοι είναι στο πλήθος 90 (αφού το a μπορεί να πάρει

τις τιμές 1,2,3,...,9 και το b τις τιμές 0,1,2,3,...,9 .

2η περίπτωση: a b c d (5)

Τότε από τις σχέσεις (1),(5) έπεται ότι a d, b c και άρα ο τετραψήφιος αριθμός

είναι της μορφής abba

Και τέτοιας μορφής τετραψήφιοι είναι στο πλήθος πάλι 90 . Όμως από αυτούς θα

πρέπει να αφαιρέσουμε όσοι ήδη υπάρχουν στην πρώτη περίπτωση δηλαδή όσοι

έχουν a b . Δηλαδή πρέπει να αφαιρέσουμε τους αριθμούς 1111,2222,...,9999 οι

οποίοι είναι 9 στο πλήθος.

Από τα παραπάνω, συμπεραίνουμε ότι όλοι οι ζητούμενοι τετραψήφιοι είναι

90 90 9 171 .

Page 201: ΠΡΟΤΕΙΝΟΜΕΝΕΣ ΑΣΚΗΣΕΙΣ ΓΙΑ ΜΑΘΗΤΙΚΟΥΣ ΔΙΑΓΩΝΙΣΜΟΥΣ_ALL

http://www.mathematica.gr/forum/viewtopic.php?f=109&t=15584

Επιμέλεια: xr.tsif Σελίδα 8

ΘΕΜΑ 206 (ΔΗΜΗΤΡΗΣ ΙΩΑΝΝΟΥ )

Να αναλυθεί ο αριθμός 20102 1 σε γινόμενο δύο παραγόντων, έτσι ώστε ο καθένας

να είναι μεγαλύτερος του 10042 .

Λύση:

2010 1005 503 1005 5032 1 2 1 2 2 1 2 A·B , όπου 1005 503 1004

A 2 1 2 2 και

1005 503 1004 503 1004 1004 503 1004B 2 1 2 2·2 2 1 2 2 2 1 2 .

ΘΕΜΑ 207 (Cretanman )

Να δείξετε ότι υπάρχουν τουλάχιστον 5 ζευγάρια από διαδοχικούς θετικούς

ακεραίους που είναι τέτοιοι ώστε το άθροισμα των τετραγώνων των αριθμών σε κάθε

ζευγάρι να διαιρεί τον αριθμό 20062 1 .

Λύση:

Προς το παρόν έχω βρει τέσσερα ζευγάρια. Δίνω το σκεπτικό: Αν x,x 1 είναι οι

δυο διαφορετικοί ακέραιοι, τότε 2 2 2x (x 1) 2x 2x 1 . Αυτό μοιάζει πολύ με

την παραγοντοποίηση 2006 1005 503 1005 5032 1 (2 2 1)(2 2 1) που έκανε ο

Αλέξανδρος στο πρόβλημα 206. Κάνοντας το ίδιο για το 2006

2 1 έχουμε 2006 1003 502 1003 502

2 1 (2 2 1)(2 2 1) . Τώρα παρατηρούμε ότι για 501x 2 έχουμε

2 1003 502 20062x 2x 1 2 2 1/ 2 1 .

Βρήκαμε λοιπόν το πρώτο ζευγάρι. Είναι το 501 501(2 ,2 1) .

Αφού αυτό δούλεψε ας προσπαθήσουμε να συνεχίσουμε με παρόμοιο τρόπο. Για

κάθε μη αρνητικό ακέραιο n έχουμε 4n 2 2n 1 n 1 2n 1 n 12 1 (2 2 1)(2 2 1)

.

Επίσης για n

x 2 έχουμε 2 2n 1 n 1

2x 2x 1 2 2 1 . Άρα αν

4n 2 20062 1 / 2 1

,

τότε παίρνουμε και το ζευγάρι n n(2 , 2 1) .

Μπορούμε να βρούμε τέτοιους αριθμούς;

Page 202: ΠΡΟΤΕΙΝΟΜΕΝΕΣ ΑΣΚΗΣΕΙΣ ΓΙΑ ΜΑΘΗΤΙΚΟΥΣ ΔΙΑΓΩΝΙΣΜΟΥΣ_ALL

http://www.mathematica.gr/forum/viewtopic.php?f=109&t=15584

Επιμέλεια: xr.tsif Σελίδα 9

Βασικό λήμμα: Αν a,b φυσικοί αριθμοί με b περιττό, τότε το a2 1 διαιρεί το

ab2 1 .

Απόδειξη: a a 2a 3a (b 1)a a 2a 3a (b 1)a(1 2 )(1 2 2 2 2 ) (1 2 2 2 2 )

a 2a 3a 4a ba ab(2 2 2 2 2 ) 2 1 .

[Η υπόθεση b περιττός χρειάζεται για να έχουμε πλην στον συντελεστή του (b 1)a2

στο αριστερό μέλος.]

Επειδή 2006 2 17 59 από το βασικό λήμμα συμπεραίνουμε ότι m 20062 1 | 2 1 για

κάθε m {2,2 17,2 59,2 17 59} {2,34,118,2006} . Επειδή όλοι αυτοί οι αριθμοί

είναι της μορφής 4n 2 παίρνουμε τα ζευγάρια 8 8 29 29 501 501

(1,2),(2 ,2 1),(2 ,2 1),(2 ,2 1) .

Μας λείπει ακόμη ένα ζευγάρι. Αν και ο Αλέξανδρος ζητάει θετικούς ακεραίους, θα

κλέψω και θα πω ότι είναι το (0,1) . Αν μας πει ότι θέλει ακόμη ένα θα πρέπει να

ξανασκεφτούμε τι να κάνουμε...

Είναι 1005 503 1004 502 22 2 1 2 (2 1) , οπότε άλλο ένα ζεύγος είναι

502 502(2 1,2 ).

Παρόμοια, μπορούμε να βρούμε και άλλα ζεύγη.

ΘΕΜΑ 208 (Socrates)

Οι θετικοί ακέραιοι a,b,n είναι τέτοιοι ώστε ο αριθμός 2 2

a 2nb να είναι τέλειο

τετράγωνο ακεραίου. Να δείξετε ότι ο αριθμός 2 2

a nb γράφεται ως άθροισμα δύο

τέλειων τετραγώνων.

Λύση:

Έχουμε: 2 2 2 2 2 2 2 2 2 2 2 2

a 2nb k a 2nb a k a 2a 2nb k b 2 2

2 2 2 2 2 2k a k a k aa nb a nb ( ) ( )

2 2 2

.

Page 203: ΠΡΟΤΕΙΝΟΜΕΝΕΣ ΑΣΚΗΣΕΙΣ ΓΙΑ ΜΑΘΗΤΙΚΟΥΣ ΔΙΑΓΩΝΙΣΜΟΥΣ_ALL

http://www.mathematica.gr/forum/viewtopic.php?f=109&t=15584

Επιμέλεια: xr.tsif Σελίδα 10

Μένει να αποδείξουμε ότι οι αριθμοί k a k a

,2 2

είναι ακέραιοι.

Πράγματι, αν a άρτιος, τότε επειδή 2 2 2a 2nb k θα πρέπει και ο k να είναι άρτιος

(γνωστό) οπότε και k a άρτιος και k a άρτιος και άρα οι αριθμοί k a k a

,2 2

είναι ακέραιοι.

Αν πάλι είναι a περιττός τότε πρέπει και ο k να είναι περιττός οπότε οι αριθμοί

k a και k a είναι άρτιοι και έχουμε και πάλι το ζητούμενο.

ΘΕΜΑ 209 (Socrates)

Αν S(n) το άθροισμα των ψηφίων του φυσικού n να δείξετε ότι ο αριθμός

2S(2n 3) δεν είναι τέλειο τετράγωνο ακεραίου.

Λύση:

Είναι γνωστό ότι ένας αριθμός n είναι ισότιμος mod9 με το άθροισμα των

ψηφίων του, δηλαδή 2 2S(2n 3) 2n 3 .Επομένως αρκεί να απο δείξουμε ότι ο

αριθμός 22n 3 δεν είναι τέλειο τετράγωνο.

Τα τετραγωνικά υπόλοιπα mod9 είναι τα 0,1,4,7 .

Άρα 2 22n 0,2,5,8(mod9) 2n 3 2,3,5,8(mod9) ,που δεν είναι τετραγωνικά

υπόλοιπα mod9 .

Συνεπώς ο αριθμός 2S(2n 3) δεν είναι τέλειο τετράγωνο.

ΘΕΜΑ 210 (Socrates)

Να βρείτε τους πρώτους p,q,r ώστε οι αριθμοί

2 2 2pq r,pq r ,qr p,qr p ,rp q,rp q να είναι επίσης πρώτοι.

Page 204: ΠΡΟΤΕΙΝΟΜΕΝΕΣ ΑΣΚΗΣΕΙΣ ΓΙΑ ΜΑΘΗΤΙΚΟΥΣ ΔΙΑΓΩΝΙΣΜΟΥΣ_ALL

http://www.mathematica.gr/forum/viewtopic.php?f=109&t=15584

Επιμέλεια: xr.tsif Σελίδα 11

Λύση:

Απάντηση: Μόνο η περίπτωση p 2,q 3,r 5 ή κυκλικά.

Λύση: Δεν μπορεί να είναι και οι τρεις από τους p,q,r περιττοί γιατί θα ήταν άρτιος

2 ο pq r . Άρα κάποιος είναι άρτιος πρώτος, και χωρίς βλάβη p 2 . Δηλαδή η

υπόθεση τώρα γίνεται "οι αριθμοί 2 22q r,2q r ,qr 2,qr 4,2r q,2r q είναι

όλοι πρώτοι".

Λόγω του qr 2 , οι q,r είναι και οι δύο περιττοί.

Χρησιμοποιούμε το γεγονός ότι οι περιττοί πρώτοι από τον 5 και πέρα είναι της

μορφής 6k 1 . Θα δούμε τώρα ότι δεν μπορεί και οι δύο από τους q,r να είναι 5 .

Πράγματι, αποκλείεται η περίπτωση q 6Q 1,r 6R 1 γιατί τότε

qr 2 (6Q 1)(6R 1) 2 6S 3 σύνθετος.

Όμοια αποκλείεται η περίπτωση q 6Q 1,r 6R 1 καθώς και η

q 6Q 1,r 6R 1 λόγω του qr 4 .

Επίσης αποκλείεται η q 6Q 1,r 6R 1 λόγω του qr 2 .

Συμπεραίνουμε ότι τουλάχιστον ένας από τους περιττούς πρώτους q,r είναι 5 .

Χωρίς βλάβη q 3 .

H υπόθεση τώρα γίνεται "οι αριθμοί 26 r,6 r ,3r 2,3r 4,2r 3,2r 9 είναι

όλοι πρώτοι". (*)

Με το χέρι ελέγχουμε ότι η r 5 δίνει λύση αλλά οι r 3,7,11,13,17,19,23,29 δεν

δίνουν λύση (άμεσο).

Θα δούμε ότι δεν έχουμε άλλη λύση. Οι πρώτοι που περισσεύουν είναι της μορφής

r 30k 1,30k 7,30k 11,30k 13,30k 17,30k 19,30k 23,30k 29 .

Ελέγχοντας με το χέρι διαπιστώνουμε ότι κανένας από αυτούς δεν δίνει όλους τους

(*) πρώτους.

Page 205: ΠΡΟΤΕΙΝΟΜΕΝΕΣ ΑΣΚΗΣΕΙΣ ΓΙΑ ΜΑΘΗΤΙΚΟΥΣ ΔΙΑΓΩΝΙΣΜΟΥΣ_ALL

http://www.mathematica.gr/forum/viewtopic.php?f=109&t=15584

Επιμέλεια: xr.tsif Σελίδα 12

Συγκεκριμένα, σε ΚΑΘΕ περίπτωση τουλάχιστον ένας από τους (*) βγαίνει

πολλαπλάσιο του 5 .

Π.χ. ο r 30k 1 δίνει πολλαπλάσιο του 5 τον 2r 3 , ο r 30k 7 δίνει

πολλαπλάσιο του 5 τον 3r 4 , και ούτω καθ' εξής (ρουτίνα).

Δείξαμε ότι οι μόνες αποδεκτές περιπτώσεις είναι οι παραπάνω p 2,q 3,r 5 ή

κυκλικά.

ΘΕΜΑ 211 (Socrates)

Να δείξετε ότι a(a 1) b(b 1) c(c 1) (a b c 4)(a b c 5 4 ) , αν

a,b,c 2.

Λύση:

Αρκεί να αποδείξουμε ότι:

2 2 2a b c (a b c) (a b c 4)(a b c 5) 4

2(a b c) 2(ab ac cb) (a b c) (a b c 4)(a b c 5) 4

Θέτουμε a b c t . Τότε αρκεί να αποδείξουμε ότι:

2t 2(ab ac bc) t (t 4)(t 5) 4

4t ab ac cb 12 4(a b c) ab ac cb 12 (1).

Θέτουμε x a 2,y b 2,z c 2 . Τότε από την υπόθεση έχουμε ότι

x,y,z 0 και η σχέση (1) ισοδύναμα γράφεται:

4(x 2 y 2 z 2) (x 2)(y 2) (x 2)(z 2) (y 2)(z 2) 12

0 xy xz zy η οποία είναι αληθής, αφού x,y,z 0 .

Page 206: ΠΡΟΤΕΙΝΟΜΕΝΕΣ ΑΣΚΗΣΕΙΣ ΓΙΑ ΜΑΘΗΤΙΚΟΥΣ ΔΙΑΓΩΝΙΣΜΟΥΣ_ALL

http://www.mathematica.gr/forum/viewtopic.php?f=109&t=15584

Επιμέλεια: xr.tsif Σελίδα 13

ΘΕΜΑ 212 (Socrates)

Θεωρούμε σκακιέρα 50x50 . Αρχικά όλα τα τετράγωνα 1x1έχουν μαύρο χρώμα. Μια

κίνηση συνίσταται στο να αλλάξουμε το χρώμα όλων των τετραγώνων μιας στήλης ή

μιας γραμμής (αν είναι μαύρο γίνεται άσπρο και αν είναι άσπρο γίνεται μαύρο.)

α) Δείξτε ότι δεν είναι δυνατόν, μετά από ένα αριθμό κινήσεων, να έχουμε ακριβώς

2011 άσπρα τετράγωνα στη σκακιέρα .

β) Μπορεί να προκύψει σκακιέρα με ακριβώς 2010 άσπρα τετράγωνα;

Λύση:

Έστω ότι εφαρμόζουμε την κίνηση σε n οριζόντιες και m κάθετες

γραμμές(n,m 50 ).Τότε το πλήθος των λευκών τετραγώνων που έχουν προκύψει

από τις κινήσεις είναι 50n 50m μείον το πλήθος των μαύρων τετραγώνων που

έχουν προκύψει από τις κινήσεις. Το πλήθος αυτό ισούται με το εμβαδό του

ορθογωνίου που δημιουργείται από τις τομές των γραμμών. Αυτό είναι ίσο με 2nm

σε περίπτωση που οι γραμμές δεν εφάπτονται αυτό δεν αλλάζει το αποτέλεσμα διότι

απλά μεταφέρουμε την γραμμή όσα τετράγωνα χρειάζονται έτσι ώστε οι γραμμές να

εφάπτονται. Έτσι το πλήθος των λευκών τετραγώνων που έχουν προκύψει από τις

κινήσεις είναι ίσο με 50n 50m 2nm 2(25m 25n nm) (1).

α) Για να έχει η σκακιέρα 2011 λευκά τετραγωνάκια αρκεί η εξίσωση

2(25m 25n nm) 2011 να έχει θετικές ακέραιες λύσεις μικρότερες ή ίσες του

50 . Αυτή είναι αδύνατη αφού (2011,2) 1 .Συνεπώς δεν γίνεται να έχουμε 2011

λευκά τετραγωνάκια.

β) Όπως και πριν για να έχει η σκακιέρα 2010 λευκά τετράγωνα αρκεί η εξίσωση

2(25m 25n nm) 2010 να έχει θετικές ακέραιες λύσεις μικρότερες ή ίσες του

50 . Η εξίσωση γράφεται (25 n)(25 m) 380 που έχει λύσεις, πχ (m,n) (40,1) .

Συνεπώς μπορούμε να έχουμε 2010 λευκά τετραγωνάκια στη σκακιέρα.

Page 207: ΠΡΟΤΕΙΝΟΜΕΝΕΣ ΑΣΚΗΣΕΙΣ ΓΙΑ ΜΑΘΗΤΙΚΟΥΣ ΔΙΑΓΩΝΙΣΜΟΥΣ_ALL

http://www.mathematica.gr/forum/viewtopic.php?f=109&t=15584

Επιμέλεια: xr.tsif Σελίδα 14

Β τρόπος στο πρώτο ερώτημα.

Έστω σε ένα στάδιο έχουμε M μαύρα και A άσπρα τετράγωνα. Έστω ότι στην

επόμενη κίνηση (αλλαγή χρωματισμού μιας γραμμής ή στήλης) περιέχονται n μαύρα

και (φυσικά) 50 n άσπρα τετράγωνα. Τότε τα μαύρα θα γίνουν

M' M n (50 n) (δηλαδή τα n που χάσαμε συν τα 50 n που κερδίσαμε) και

τα άσπρα θα γίνουν A΄ A (50 n) n .

Εξετάζουμε τώρα την διαφορά M΄ Α΄ . Είναι M΄ Α΄ ... Μ Α 100 4n .

Παρατηρούμε ότι ο αριθμός 100 4n είναι πολλαπλάσιο του 4 . Συμπέρασμα, σε

κάθε βήμα ο αριθμός M A αλλάζει κατά πολλαπλάσιο του 4 . Στην αρχή είναι

M A 2500 0 πολλαπλάσιο του 4 , ενώ αν γίνουν 2011 τα άσπρα (και 489 τα

μαύρα), τότε M A 489 2011 1522 που δεν είναι πολλαπλάσιο του 4 .

Συνεπώς δεν θα φτάσουμε ποτέ σε 2011 άσπρα τετράγωνα.

ΘΕΜΑ 213 (Socrates)

Έστω A ένα υποσύνολο του συνόλου {1,2,...,2009} με 1005 στοιχεία, τέτοιο ώστε

το άθροισμα δύο οποιονδήποτε στοιχείων του να μην ισούται με 2009 , ούτε με

2010 . Πόσα τέτοια σύνολα A υπάρχουν;

Λύση:

Χωρίζουμε το σύνολο {1,2,...,2009} σε δύο υποσύνολα B και Γ όπου το B

περιέχει τα στοιχεία {1,2,...,1004} και το Γ περιέχει τα στοιχεία

{1005,1006,...,2009} . Είναι προφανές ότι το σύνολο Γ είναι ένα από τα ζητούμενα

σύνολα.

Θα αποδείξουμε ότι είναι το μοναδικό. Παρατηρούμε ότι αν μετακινήσουμε ένα

στοιχείο του Γ ( π.χ. το 1005 ) στο σύνολο B τότε πρέπει να αφαιρέσουμε ένα άλλο

στοιχείο (το1004 ) και έτσι το σύνολο θα έχει πάλι 1004 στοιχεία . Έτσι είναι

αδύνατο το B να αποκτήσει 1005 στοιχεία. Αν μετακινήσουμε ένα στοιχείο του B

στο Γ (π.χ. το 1004 ) πρέπει να αφαιρέσουμε δύο άλλα στοιχεία (το 1005 και το

1006 ) και έτσι και αυτό το σύνολο είναι αδύνατο να ξανααποκτήσει 1005 στοιχεία.

Page 208: ΠΡΟΤΕΙΝΟΜΕΝΕΣ ΑΣΚΗΣΕΙΣ ΓΙΑ ΜΑΘΗΤΙΚΟΥΣ ΔΙΑΓΩΝΙΣΜΟΥΣ_ALL

http://www.mathematica.gr/forum/viewtopic.php?f=109&t=15584

Επιμέλεια: xr.tsif Σελίδα 15

ΘΕΜΑ 214 (ΣΩΤΗΡΗΣ ΛΟΥΡΙΔΑΣ )

Υπολογίστε τετραψήφιο αριθμό xyzw του δεκαδικού συστήματος που να είναι

τέλειο τετράγωνο όταν επιπλέον ισχύουν οι σχέσεις: xy 3zw 1 και w y 1 .

Λύση:

Η πρώτη δοθείσα σχέση γράφεται: 10x y 30z 3w 1 και αν αντικαταστήσουμε

όπου w y 1 , έχουμε 10x 30z 2y 4 (1).

Παρατηρούμε ότι το πρώτο μέλος τελειώνει σε 0 , άρα και το άθροισμα στο δεύτερο

μέλος πρέπει να τελειώνει σε 0 , αυτό γίνεται μόνο για y 3 ή y 8 . Οπότε έχουμε

δύο περιπτώσεις:

1η περίπτωση: y 3 άρα και w 4 .

η (1) γράφεται 10x 30z 10 δηλαδή x 3z 1 .

για z 0 έχουμε x 1 ,

για z 1 έχουμε x 4 ,

για z 2 έχουμε x 7 ,

δηλαδή τους αριθμούς:1304 , 4314 , 7324 που κανένας δεν είναι τέλειο τετράγωνο

οι υπόλοιπες τιμές για το z απορρίπτονται, γιατί δεν θα υπάρχει αντίστοιχο x .

2η περίπτωση: y 8 άρα w 9 .

η (1) γράφεται x 3z 2

παίρνουμε πάλι περιπτώσεις για το z που μπορεί να πάρει τις τιμές z 0 , z 1 ,

z 2 και βρίσκουμε τους αριθμούς: 2809 , 5819 , 8829 από τους οποίου μόνο ο

αριθμός 2809 είναι τέλειο τετράγωνο ,άρα είναι ο ζητούμενος.

Page 209: ΠΡΟΤΕΙΝΟΜΕΝΕΣ ΑΣΚΗΣΕΙΣ ΓΙΑ ΜΑΘΗΤΙΚΟΥΣ ΔΙΑΓΩΝΙΣΜΟΥΣ_ALL

http://www.mathematica.gr/forum/viewtopic.php?f=109&t=15584

Επιμέλεια: xr.tsif Σελίδα 16

ΘΕΜΑ 215 (ΣΩΤΗΡΗΣ ΛΟΥΡΙΔΑΣ )

Να αποδειχθεί ότι η παράσταση:

2 2 2

1 1 1

b c c a a b

, είναι τέλειο

τετράγωνο.

Λύση:

Θέτουμε : x a b , y b c , w c a , oπότε x y w 0 .

Επίσης από την ταυτότητα 2 2 2 2(x y w) x y w 2xy 2yw 2xw έχουμε

2 2 2x y w 2xy 2yw 2xw (1) αφού x y w 0 .

υψώνουμε την (1) στο τετράγωνο οπότε 2 2 2 2 2 2 2

4x y 4y w 4x w 8xyw(x y w)A (2) , όπου A το πρώτο μέλος της

(1). Αφού x y w 0 η (2) γίνεται 2A 4B (3) , όπου 2 2 2 2 2 2

B x y y w x w .

Διαιρούμε τώρα την (3) με 2 2 24x y w και προκύπτει το ζητούμενο,

γιατί 2 2 2B / x y w ισούται με τη δοθείσα παράσταση.

ΘΕΜΑ 216 (ΣΩΤΗΡΗΣ ΛΟΥΡΙΔΑΣ )

Σε τρίγωνο ABC δίνονται: AB 5 , AC 7 και το ύψος AD 4 . Υπολογίστε την

BC , το εμβαδόν του και την ακτίνα ρ του εγγεγραμμένου του κύκλου.

Λύση:

Εύκολα με Πυθαγόρειο βρίσκουμε ότι BC 3 33 , άρα

4(3 33)E 6 2 33

2

.

Page 210: ΠΡΟΤΕΙΝΟΜΕΝΕΣ ΑΣΚΗΣΕΙΣ ΓΙΑ ΜΑΘΗΤΙΚΟΥΣ ΔΙΑΓΩΝΙΣΜΟΥΣ_ALL

http://www.mathematica.gr/forum/viewtopic.php?f=109&t=15584

Επιμέλεια: xr.tsif Σελίδα 17

Γνωρίζουμε επίσης ότι 15 33

E τρ 6 2 33 ρ2

, από όπου παίρνουμε και

το ρ , ( τ ημιπερίμετρος και ρ ακτίνα εγγεγραμμένου κύκλου).

ΘΕΜΑ 217 (ΣΩΤΗΡΗΣ ΛΟΥΡΙΔΑΣ )

Δίνεται τρίγωνο ABC και σημείο

Ρ στο εσωτερικό του. Θεωρούμε

D το σημείο τομής της CP με την

AB .( Συμβολικά D CP AB και

E BP AC (σχήμα 1))

1) Αιτιολογείστε γιατί οι

περιγεγραμμένοι κύκλοι στα

τρίγωνα PDB , PCE δεν είναι

δυνατόν να εφάπτονται .

2) Αν οι κύκλοι αυτοί είναι ίσοι

τότε αποδείξτε ότι ισχύει η σχέση

AE AD AC AB και αντίστροφα.

Υπόδειξη για το (1) (σχήμα 2) Η

εγγεγραμμένη γωνία KML

είναι ίση με

την γωνία LK P

(γιατί;)

Page 211: ΠΡΟΤΕΙΝΟΜΕΝΕΣ ΑΣΚΗΣΕΙΣ ΓΙΑ ΜΑΘΗΤΙΚΟΥΣ ΔΙΑΓΩΝΙΣΜΟΥΣ_ALL

http://www.mathematica.gr/forum/viewtopic.php?f=109&t=15584

Επιμέλεια: xr.tsif Σελίδα 18

(*) Μεθοδολογία: Όταν έχεις τεμνόμενους κύκλους θεώρησε την κοινή τους χορδή,

μπορεί να χρειαστεί. Επίσης όταν έχεις εφαπτόμενους κύκλους θεώρησε την κοινή

τους εφαπτομένη στο σημείο επαφής τους, μπορεί να χρειαστεί.

Λύση:

1) Έστω ότι οι κύκλοι εφάπτονται. Επίσης θέτουμε CEP x

και BDP y

.Τότε η

γωνία BPC x y

, όμως αυτό είναι άτοπο, γιατί το άθροισμα των γωνιών x και

ECP

δεν μπορεί να ισούται με x y , αφού ECP

δεν μπορεί να έχει μέτρο y . Αυτό

γιατί ECP A y

.

2) Αφού οι κύκλοι είναι ίσοι από το νόμο των ημιτόνων και τις ίσες γωνίες DPB

και

EPC

έχουμε ότι DB EC .

Έτσι: AC AB AE EC AD DB AE AD .

Β τρόπος

1) Στην περίπτωση που οι κύκλοι

αυτοί εφάπτονταν, αυτό θα συνέβαινε

στο σημείο P , αφού ανήκει και στους

δύο κύκλους. Έτσι για αυτούς θα

είχαμε την κάτω σχηματική απόδοση.

Με βάση τώρα και την θεωρία που

ήδη αναφέραμε παίρνουμε:

DBP DPK LPC PEC DB EC, με A DB και A EC,

που είναι άτοπο.

Επομένως οι κύκλοι αυτοί θα έχουν και άλλο ένα κοινό σημείο διαφορετικό του P .

Page 212: ΠΡΟΤΕΙΝΟΜΕΝΕΣ ΑΣΚΗΣΕΙΣ ΓΙΑ ΜΑΘΗΤΙΚΟΥΣ ΔΙΑΓΩΝΙΣΜΟΥΣ_ALL

http://www.mathematica.gr/forum/viewtopic.php?f=109&t=15584

Επιμέλεια: xr.tsif Σελίδα 19

2) Στην περίπτωση της ισότητας των κύκλων αυτών θα είχαμε:

DPB CPE τoξ.DB τoξ.CE DB CE.

Από εδώ προκύπτει άμεσα το ζητούμενο.

ΘΕΜΑ 218 (Socrates)

Βρείτε όλους τους διψήφιους αριθμούς AB τέτοιους ώστε AB / A0B .

Λύση:

Απάντηση στο ερώτημα: Αν δεν ξέχασα άλλες τόσες περιπτώσεις γιατί δεν είμαι

καλός στα Λογιστικά, έχουμε τις λύσεις

10 / 100, 15 / 105, 18 / 108, 20 / 200, 30 / 300, 40 / 400, 45 / 405,

50 / 500, 60 / 600, 70 / 700, 80 / 800, 90 / 900 . Εξ υποθέσεως 10A B / 100A B

άρα 10A B / 90A . Τρέχουμε τώρα τον A 1,2,3,4,5,6,7,8,9 .

Π.χ. για τον A 1 ψάχνουμε τους αριθμούς της μορφής 1B / 90 . Αναλογιζόμενοι ότι

οι διαιρέτες του 90 είναι 1,2,3,5,6,9,10,15,18,30,45,90 (δεν θα χρειαστούν όλοι

Page 213: ΠΡΟΤΕΙΝΟΜΕΝΕΣ ΑΣΚΗΣΕΙΣ ΓΙΑ ΜΑΘΗΤΙΚΟΥΣ ΔΙΑΓΩΝΙΣΜΟΥΣ_ALL

http://www.mathematica.gr/forum/viewtopic.php?f=109&t=15584

Επιμέλεια: xr.tsif Σελίδα 20

εδώ, αλλά τους καταγράφω γιατί χρειάζονται στις επόμενες περιπτώσεις), βρίσκουμε

τις λύσεις B 0 ή B 5 ή Β 8 .

Και λοιπά, και λοιπά και λοιπά.

Β τρόπος

Για b 0 το ζητούμενο ισχύει.

Για b 0 γράφουμε 100a b k(10a b) 10a(10 k) (k 1)b.

Είναι 10a 10 b οπότε k 1 10 k 10 k 6.

-Αν k 6 έχουμε b 8a οπότε a 1, b 8

-Αν k 7 έχουμε b 5a οπότε a 1, b 5

-Αν k 8 έχουμε 7b 20a αδύνατη.

-Αν k 9 έχουμε 4b 5a οπότε a 4, b 5 .

ΘΕΜΑ 219 (Socrates)

Αν οι θετικοί αριθμοί x,y,z είναι τέτοιοι ώστε

2 2 2

1 1 1 1

x 1 y 1 z 1 2

να δείξετε ότι

3 3 3

1 1 1 1.

x 2 y 2 z 2 3

Λύση:

Αρκεί να δειχθεί ότι 3 2

3 21

x 2 x 1

.

Όμως 3 2 3 3 2

3 2

3 22x 4 3x 3 x x 1 3x

x 2 x 1

,

που ισχύει από ΑΜ – ΓΜ. Το ζητούμενο έπεται.

Page 214: ΠΡΟΤΕΙΝΟΜΕΝΕΣ ΑΣΚΗΣΕΙΣ ΓΙΑ ΜΑΘΗΤΙΚΟΥΣ ΔΙΑΓΩΝΙΣΜΟΥΣ_ALL

http://www.mathematica.gr/forum/viewtopic.php?f=109&t=15584

Επιμέλεια: xr.tsif Σελίδα 21

Για να ισχύει η ισότητα πρέπει x 1 . Ομοίως προκύπτει ότι για να ισχύουν οι

επιμέρους ισότητες πρέπει y 1, z 1 . Όμως η δοθείσα σχέση δεν επαληθεύεται για

x y z 1 και συνεπώς η ισότητα δεν μπορεί να ισχύει.

ΘΕΜΑ 220 (Socrates)

Αν οι πραγματικοί αριθμοί a,b,c είναι τέτοιοι ώστε οι αριθμοί ab,bc,ca να είναι

ρητοί, να δείξετε ότι υπάρχουν ακέραιοι x,y,z όχι όλοι μηδέν, τέτοιοι ώστε

ax by cz 0 .

Λύση:

Αν δύο ή τρεις από τους a,b,c είναι 0 , απλό (αν π.χ. a b 0 c παίρνουμε

x y 1,z 0 .

Χωρίς βλάβη της γενικότητας υποθέτουμε ότι ab,bc,ca δεν είναι όλοι 0 . Παίρνουμε

x 2Mbc , y Mac , z Mab όπου M οποιοδήποτε πολλαπλάσιο των

παρονομαστών των ab,bc,ca .

Ελέγχουμε ότι ισχύουν τα ζητούμενα.

ΘΕΜΑ 221 (Socrates)

Σε ένα διαγωνισμό συμμετέχουν 90 μαθητές. Κάθε μαθητής γνώρισε τουλάχιστον

60 άλλους συμμετέχοντες μαθητές. Να αποδείξετε ότι τέσσερις τουλάχιστον μαθητές

έχουν τον ίδιο αριθμό γνωριμιών.

(Η γνωριμία είναι συμμετρική. Αν ο A γνωρίζει τον B , τότε και ο B γνωρίζει τον A .)

Λύση:

Για κάθε 60 n 89 ας γράψουμε n

A για το σύνολο των μαθητών που γνώρισαν

ακριβώς n άλλους μαθητές. Στόχος μας είναι να δείξουμε ότι κάποιο από τα n

A

περιέχει τουλάχιστον τέσσερις μαθητές. Ας υποθέσουμε πως αυτό δεν ισχύει. Άρα

Page 215: ΠΡΟΤΕΙΝΟΜΕΝΕΣ ΑΣΚΗΣΕΙΣ ΓΙΑ ΜΑΘΗΤΙΚΟΥΣ ΔΙΑΓΩΝΙΣΜΟΥΣ_ALL

http://www.mathematica.gr/forum/viewtopic.php?f=109&t=15584

Επιμέλεια: xr.tsif Σελίδα 22

nA 3 για κάθε 60 n 89 . Επειδή όμως συνολικά έχουμε 90 μαθητές, τότε

60 8990 | A | | A | 3 3 90 και άρα, για να ισχύει η ισότητα πρέπει

nA 3 για κάθε 60 n 89 . Δηλαδή ακριβώς 3 μαθητές γνώρισαν άλλους 60 ,

ακριβώς 3 γνώρισαν άλλους 61 , ... , ακριβώς 3 γνώρισαν άλλους 89 .

Ας μετρήσουμε τώρα όλες τις γνωριμίες. Έχουμε συνολικά 3(60 61 89)

2

γνωριμίες αφού στο άθροισμα 3(60 61 89) μετράμε κάθε γνωριμία διπλά,

από μία για κάθε ένα από τα δυο άτομα που γνωρίστηκαν μεταξύ τους. Όμως

3(60 61 89) 3 149 30

2 4

ο οποίος δεν είναι ακέραιος. Αυτό είναι αδύνατον

και άρα το ζητούμενο αποδείχθηκε.

Θα μπορούσαμε να γράψουμε την πιο πάνω λύση χρησιμοποιώντας την ορολογία της

θεωρίας γραφημάτων. Δείτε π.χ.

«Ο Γιώργος ισχυρίζεται ότι χθες βρισκόταν σε μια συνάντηση με άλλα εννιά άτομα οι

οποίοι (συμπεριλαμβανομένου του Γιώργου) έκαναν 3,4,4,4,5,5,6,6,7,7 χειραψίες

αντίστοιχα. Ο Ανδρέας είναι σίγουρος πως ο Γιώργος έχει κάνει λάθος στο μέτρημα.

Πως το ξέρει;»

Λύση

Οι χειραψίες γίνονται σε "ζευγάρια",(αν ο x κάνει χειραψία με τον y τότε αυτόματα

έχει κάνει και ο y με τον ) x , άρα συνολικά ζητάμε άρτιο αριθμό χειραψιών, κάτι

που δεν ισχύει στην δική μας περίπτωση.

Ωραία.

Να αναφέρω σχετικά με τον τίτλο ότι μπορούμε να παραστήσουμε αυτήν την

κατάσταση με ένα "γράφημα" το οποίο αποτελείται από 10 σημεία τα οποία

ονομάζουμε κορυφές (μια κορυφή για κάθε άτομο) και στο οποίο ενώνουμε δυο

κορυφές μεταξύ τους με μια γραμμή την οποία ονομάζουμε ακμή αν και μόνο αν τα

αντίστοιχα άτομα έχουν κάνει χειραψία μεταξύ τους.

Page 216: ΠΡΟΤΕΙΝΟΜΕΝΕΣ ΑΣΚΗΣΕΙΣ ΓΙΑ ΜΑΘΗΤΙΚΟΥΣ ΔΙΑΓΩΝΙΣΜΟΥΣ_ALL

http://www.mathematica.gr/forum/viewtopic.php?f=109&t=15584

Επιμέλεια: xr.tsif Σελίδα 23

Για κάθε κορυφή v ενός γραφήματος G συμβολίζουμε με d(v) τον αριθμό των

ακμών που περνάνε από αυτήν την κορυφή. (Στην περίπτωσή μας αυτό ισούται με

τον αριθμό των χειραψιών που έκανε το αντίστοιχο άτομο.)

Αυτό που έχει αποδείξει ο Μάριος είναι ότι για κάθε γράφημα G αν αθροίσουμε όλα

τα d(v)θα πάρουμε ένα άρτιο αριθμό. Πιο συγκεκριμένα αυτός ο αριθμός θα ισούται

με 2e(G) όπου με e(G) συμβολίζουμε τον αριθμό των ακμών του γραφήματος.

ΘΕΜΑ 222 (Socrates)

Ένας επταψήφιος αριθμός αποτελείται από διαφορετικά μεταξύ τους ψηφία και είναι

πολλαπλάσιο καθενός από αυτά(τα ψηφία). Να βρεθεί από ποια ψηφία αποτελείται ο

αριθμός.

Λύση:

Ο επταψήφιος 3548160 είναι πολλαπλάσιο όχι μόνο των ψηφίων του αλλά όλων των

0,1,2,...,9 .

Για την κατασκευή του δεν πάμε τελείως στα τυφλά: Αν κρατήσουμε τα ψηφία

1,3,4,5,6,8,0 έχουν άθροισμα 27 οπότε εξασφαλίζουμε ότι φτιάχνουν πολλαπλάσιο

του 3 και 9 . Αν βάλω το 160 στο τέλος του, εξασφαλίζω πολλαπλάσιο των 2,4,5,8 .

Αυτόματα έχω του 6 2 3 και φυσικά των 1και 0 . Μένει ο 7 . Ανακατεύοντας τα

ψηφία πλην των τριών τελευταίων όλο και κάποιο πολλαπλάσιο του 7 θα βρίσκω.

Π.χ. δεν μου κάνει ο 5348160 , αλλά δεν στεναχωριέμαι: δοκιμάζω εναλλάσσοντας

τα 3,5 και, ω του θαύματος, βρήκα πολλαπλάσιο του 7 (αν ξέρω και κριτήριο

διαιρετότητας με το 7 , τόσο το καλύτερο.

ΛΥΣΗ

Το 0 απαγορεύεται να επιλεγεί ως ψηφίο. Επειδή τουλάχιστον ένα από τα 2,4,6,8

θα πρέπει να επιλεγεί (αλλιώς θα είχαμε πενταψήφιο αριθμό) ο αριθμό πρέπει να

είναι πολλαπλάσιο του δύο. Αν ήταν πολλαπλάσιο του πέντε, τότε θα ήταν

πολλαπλάσιο του δέκα και θα έληγε σε 0 , άτοπο. Άρα ούτε το 5 επιτρέπεται να

επιλεγεί. Άρα ο αριθμός περιέχει εφτά ψηφία από τα 1,2,3,4,6,7,8,9 . Αν δεν

Page 217: ΠΡΟΤΕΙΝΟΜΕΝΕΣ ΑΣΚΗΣΕΙΣ ΓΙΑ ΜΑΘΗΤΙΚΟΥΣ ΔΙΑΓΩΝΙΣΜΟΥΣ_ALL

http://www.mathematica.gr/forum/viewtopic.php?f=109&t=15584

Επιμέλεια: xr.tsif Σελίδα 24

περιέχει το 9 , τότε το άθροισμα των ψηφίων του αριθμού θα ισούται με

1 2 3 4 6 7 8 31 το οποίο δεν είναι πολλαπλάσιο του 3 . Άρα ούτε και ο

αριθμός θα είναι πολλαπλάσιο του 3 , άτοπο αφού περιέχει το 3 . Επομένως ο

αριθμός περιέχει το ψηφίο 9 και άρα το άθροισμα των ψηφίων του είναι

πολλαπλάσιο του 9 . Επομένως το άθροισμα των ψηφίων πρέπει να ισούται με 36

(το 27 είναι πολύ μικρό και το 45 πολύ μεγάλο) και άρα το άλλο ψηφίο που λείπει

είναι το 4 . Επομένως ο αριθμός περιέχει τα ψηφία 1,2,3,6,7,8,9 .

Η άσκηση δεν ζητάει να βρούμε τον αριθμό αλλά μόνο τα ψηφία οπότε επιτρέπεται

να σταματήσουμε εδώ.

ΘΕΜΑ 223 (Socrates)

Στο επίπεδο δίνονται 51 σημεία με ακέραιες συντεταγμένες και τέτοια ώστε η

απόσταση μεταξύ δύο οποιονδήποτε από αυτά να είναι φυσικός αριθμός. Να δείξετε

ότι τουλάχιστον το 49% αυτών των αποστάσεων είναι άρτιοι αριθμοί.

Λύση:

Χωρίζουμε τα σημεία σε δύο ομάδες.

Α ομάδα: Όσα σημεία έχουν και τις δύο συντεταγμένες άρτιες ή και τις δύο περιττές.

Β ομάδα: Όσα σημεία έχουν μια συντεταγμένη άρτια και μια περιττή.

Παρατηρούμε ότι αν πάρουμε δυο σημεία της A ομάδας, το τετράγωνο της

απόστασής τους είναι άρτιος. Άρα και η απόστασή τους (αφού είναι ακέραιος) είναι

και αυτή άρτιος αριθμός. Το ίδιο συμβαίνει και για κάθε δυο σημεία της B ομάδας.

Αν γράψουμε x για τον αριθμό των σημείων της A ομάδας και y 51 x για τον

αριθμό των σημείων της B ομάδας, τότε έχουμε το πολύ 2

y4

(x y)x

αποστάσεις οι

Page 218: ΠΡΟΤΕΙΝΟΜΕΝΕΣ ΑΣΚΗΣΕΙΣ ΓΙΑ ΜΑΘΗΤΙΚΟΥΣ ΔΙΑΓΩΝΙΣΜΟΥΣ_ALL

http://www.mathematica.gr/forum/viewtopic.php?f=109&t=15584

Επιμέλεια: xr.tsif Σελίδα 25

οποίες είναι περιττές. Αυτό δίνει ποσοστό 2

51 1 51

514 100

2

περιττών αποστάσεων.

Άρα τουλάχιστον το 49% των αποστάσεων είναι άρτιες.

ΘΕΜΑ 224 (ΣΩΤΗΡΗΣ ΛΟΥΡΙΔΑΣ )

Έστω τρίγωνο ABC με n 1 n 2 n 1 n n 1 n n 1AB 3·2 3,BC 2 2 2 ,AC 2 2 2 .

1) Προσδιορίστε τις γωνίες του τριγώνου,

2) Υπολογίστε τοn στην περίπτωση που η περίμετρος του τριγώνου είναι:

24 3( 3 1) .

Λύση:

1) Ας θέσουμε για ευκολία nx 2 Τότε, είναι

n 2 na 2 (2 2 1) 3·2 3x, n n1 3 3

b 2 2 1 2 x,2 2 2

( ) n3 3 3 3

c 2 x.2 2

Παρατηρούμε, ότι 2 2 2 2 2 29 27b c x x 9x a ,

4 4 άρα το τρίγωνο είναι ορθογώνιο

με 0

A 90 .

Επίσης είναι a

b2

, άρα 0

B 30 .

οπότε 0

C 60 .

2) Για την περίμετρο του τριγώνου έχουμε

3 3 3 9 3 3 3 3(1 3)a b c 3x x x x x.

2 2 2 2

Άρα πρέπει x 16 δηλαδή n

2 16 οπότε x 4 .

Άλλες λύσεις εδώ :http://www.mathematica.gr/forum/viewtopic.php?f=69&t=12119

Page 219: ΠΡΟΤΕΙΝΟΜΕΝΕΣ ΑΣΚΗΣΕΙΣ ΓΙΑ ΜΑΘΗΤΙΚΟΥΣ ΔΙΑΓΩΝΙΣΜΟΥΣ_ALL

http://www.mathematica.gr/forum/viewtopic.php?f=109&t=15584

Επιμέλεια: xr.tsif Σελίδα 26

ΘΕΜΑ 225 (ΣΩΤΗΡΗΣ ΛΟΥΡΙΔΑΣ )

Δίνεται κύκλος (O,R) . Να βρεθεί τρόπος να χωριστεί η επιφάνεια του σε τρία

ισοδύναμα μέρη (δηλαδή μέρη του ίδιου εμβαδού) που για το κάθε ένα από αυτά τα

μέρη να ισχύει: υπάρχει το πολύ ένα τμήμα της γραμμής, από την οποία περιορίζεται,

που να ορίζει ημικύκλιο με την διάμετρο του, εντός του μέρους.

(*) Απλά αναφέρουμε ότι το πρόβλημα γενικεύεται και για χωρισμό σε n ισοδύναμα

μέρη με βάση την ίδια δέσμευση.

Λύση:

Δεν είμαι τόσο βέβαιος ότι καταλαβαίνω τι ακριβώς θα πει «... με την διάμετρο του

εντός του μέρους », αλλά κάνω μία προσπάθεια: Η διάμετρος του μεγάλου κύκλου

έχει χωριστεί σε τρία ίσα μέρη. Εύκολα βλέπουμε ότι τα χρωματισμένα εμβαδά είναι

ίσα μεταξύ τους.

Όμως έβαλα την προϋπόθεση με την

διάμετρο (...που για το κάθε ένα από

αυτά τα μέρη να ισχύει: υπάρχει το πολύ

ένα τμήμα της γραμμής, από την οποία

περιορίζεται, που να ορίζει ημικύκλιο με

την διάμετρο του εντός του μέρους) ,

ώστε να αποκλειστούν άλλες

περιπτώσεις, όπως εκείνη (για

παράδειγμα) του σχήματος που

ακολουθεί και όπου ο μικρός μέσα

κύκλος (με Εμβαδό το ένα τρίτο (1

3) του

Μεγάλου κύκλου), αποτελείται από δύο

τουλάχιστον ημικύκλια μου η διάμετρός

τους να βρίσκεται εντός του (άρα απαγορευμένη αυτή η περίπτωση).

Page 220: ΠΡΟΤΕΙΝΟΜΕΝΕΣ ΑΣΚΗΣΕΙΣ ΓΙΑ ΜΑΘΗΤΙΚΟΥΣ ΔΙΑΓΩΝΙΣΜΟΥΣ_ALL

http://www.mathematica.gr/forum/viewtopic.php?f=109&t=15584

Επιμέλεια: xr.tsif Σελίδα 27

Επομένως αν αντικαθιστούσα την αντίστοιχη έκφραση της εκφώνησης από την:

….που για τουλάχιστον ένα από αυτά τα μέρη να υπάρχουν τουλάχιστον δύο

τμήματα της γραμμής από την οποία περιορίζεται που να αποκόπτουν από αυτήν

ημικύκλια με την διάμετρο τους να βρίσκεται εντός του μέρους , τότε θα αποκλείαμε

την περίπτωση «γιν γιάν» που ανέφερες και θα δεχόμασταν εκείνη του σχήματος

που ακολουθεί (κάτω).

Αυτές ήταν οι σκέψεις μου που με οδήγησαν στην συγκεκριμένη εκφώνηση.

(*) Η ακτίνα του μικρού κύκλου είναι: AE R 3

r ,3 3

όπου R είναι η ακτίνα του

μεγάλου κύκλου.

(**) Θα παρακαλούσα τους μικρούς (μόνο σε ηλικία) συναδέλφους να ασχοληθούν

με την απόδειξη της ισότητας των τριών αυτών εμβαδών τόσο στο σχήμα του

Μιχάλη, όσο και σε αυτό που ακολουθεί για την Γεωμετρική τους προπονησούλα.

Page 221: ΠΡΟΤΕΙΝΟΜΕΝΕΣ ΑΣΚΗΣΕΙΣ ΓΙΑ ΜΑΘΗΤΙΚΟΥΣ ΔΙΑΓΩΝΙΣΜΟΥΣ_ALL

http://www.mathematica.gr/forum/viewtopic.php?f=109&t=15584

Επιμέλεια: xr.tsif Σελίδα 28

ΘΕΜΑ 226 (ΣΩΤΗΡΗΣ ΛΟΥΡΙΔΑΣ )

Δίνεται τρίγωνο ABC και σημείο P στο εσωτερικό του. Θεωρούμε D το σημείο

τομής της CP με την AB ( Συμβολικά D CP AB και E BP AC . Αποδείξτε

μία ειδική περίπτωση (βασικότατη) του θεωρήματος Miquel :

Αν το τετράπλευρο ADPE είναι εγγράψιμο σε κύκλο τότε το άλλο σημείο τομής T

των δύο κύκλων θα είναι σημείο της BC .

Μπορείτε να διατυπώσετε το αντίστροφο τού παραπάνω προβλήματος και να το

επιλύσετε;

Λύση:

Έστω ότι ο περιγεγραμμένος κύκλος του τριγώνου BDP τέμνει την πλευρά BCστο

σημείο T .

Θα αποδείξουμε ότι το τετράπλευρο TPEC είναι εγγράψιμο σε κύκλο, (είναι η

ισοδύναμη πρόταση της ζητούμενης).Στο αρχικό σχήμα έχουμε:

Αν η γωνία ADP x

, τότε ισχύουν τα εξής σύμφωνα με το γνωστό θεώρημα (για

το οποίο ισχύει και το αντίστροφό του)

Page 222: ΠΡΟΤΕΙΝΟΜΕΝΕΣ ΑΣΚΗΣΕΙΣ ΓΙΑ ΜΑΘΗΤΙΚΟΥΣ ΔΙΑΓΩΝΙΣΜΟΥΣ_ALL

http://www.mathematica.gr/forum/viewtopic.php?f=109&t=15584

Επιμέλεια: xr.tsif Σελίδα 29

"Σε κάθε εγγράψιμο τετράπλευρο σε κύκλο, κάθε εξωτερική του γωνία ισούται με

την απέναντι εσωτερική" έχουμε τα εξής:

Η γωνία oBDP 180 x

, η γωνία PEC x

και η γωνία oPTC 180 x

Άρα, το τετράπλευρο TPEC είναι εγγεγραμμένο σε κύκλο, άρα οι δύο κύκλοι για

τους οποίους συζητάμε διέρχονται από σημείο της πλευράς BC .

ΘΕΜΑ 227 (ΔΗΜΗΤΡΗΣ ΙΩΑΝΝΟΥ )

Μπορεί η διαφορά δύο τριψήφιων αριθμών A,B οι οποίοι έχουν τα ίδια ψηφία αλλά

με αντίθετη διάταξη, να είναι τετράγωνο κάποιου φυσικού αριθμού (διάφορου από το

μηδέν);

Λύση:

Έστω ότι υπάρχουν τριψήφιοι αριθμοί A,B οι οποίοι έχουν τα ίδια ψηφία αλλά με

αντίθετη διάταξη . Πρέπει 2abc cba n με a,c {1,2,...,9} και b {0,1,2,...,9} .

2 2 2abc cba n 100a 10b c 100c 10b a n 99a 99c n

2 299(a c) n 9 11(a c) n .

Πρέπει το a c 11 . Άτοπο γιατί τα a,c {1,2,...,9} .

ΘΕΜΑ 228 (Socrates)

Βρείτε όλους τους φυσικούς n για τους οποίους ο αριθμός 1! 4! 7! ... (3n 1)!

είναι τέλειο τετράγωνο ακεραίου.

Λύση:

Οι 1! 1,1! 4! 25 είναι τέλεια τετράγωνα. Θα δούμε ότι δεν υπάρχουν άλλες

περιπτώσεις.

Page 223: ΠΡΟΤΕΙΝΟΜΕΝΕΣ ΑΣΚΗΣΕΙΣ ΓΙΑ ΜΑΘΗΤΙΚΟΥΣ ΔΙΑΓΩΝΙΣΜΟΥΣ_ALL

http://www.mathematica.gr/forum/viewtopic.php?f=109&t=15584

Επιμέλεια: xr.tsif Σελίδα 30

Το 1! 4! 7! 5065 δεν είναι τέλειο τετράγωνο. Από κει και πέρα, αφού οι αριθμοί

της μορφής 11!,15!,... λήγουν σε 00 έχουν τουλάχιστον δύο πεντάρια ο καθένας, ως

παράγοντες), τα παραπάνω αθροίσματα είναι της μορφής 100b 65 . Αν ήσαν τέλεια

τετράγωνα, θα ήσαν της μορφής 2100b 65 (10c d) ,0 d 9 .

Παρατηρούμε ότι οι αριθμοί 1! 4! 7! ... είναι "μεγάλοι" (μεγαλύτεροι του 510 )

οπότε c "μεγάλο" (μεγαλύτερο του 50) . Άρα 2 2 2 2 2 2

100c 20cd d 100100b c 1006 0d d 1005 ( c10c d) d (*),

που σημαίνει ότι τα δύο τελευταία ψηφία του είναι ένας από τους 2 2 20 , 1 ,...,9 .

Κανένα όμως από αυτά δεν είναι 65 , οπότε η ισότητα (*) είναι αδύνατη.

ΘΕΜΑ 229 (DEMETRES )

Δίνεται ένα θετικός ακέραιος αριθμός n . Αναδιατάσσουμε τα ψηφία του για να

πάρουμε ένα άλλο ακέραιο αριθμό m . Να εξεταστεί αν το άθροισμά τους μπορεί να

ισούται με 99...9 όπου εμφανίζονται ακριβώς 2011 εννιάρια.

Λύση:

Θα εξετάσουμε περιπτώσεις με λιγότερα εννιάρια και θα δούμε αν μπορεί να βγει

κάποιο συμπέρασμα.

Για δύο εννιάρια, αρκεί τα ψηφία των διψήφιων αριθμών να έχουν άθροισμα 9 .

Για τρία εννιάρια έστω οι αριθμοί abc και η αναδιάταξη του cab .

Τότε 110a 11b 101c 999 Από εδώ συμπεραίνουμε ότι θα πρέπει να είναι

b c 9 . Χρησιμοποιώντας αυτό παίρνουμε 110a 90c 900 11a 9c 90 , που

είναι αδύνατο.

Για τέσσερα εννιάρια, έστω ο αριθμός abcd .

Τότε θα πρέπει τα ψηφία ανά ζευγάρια να έχουν άθροισμα 9 , π.χ. έστω ότι a c 9

και b d 9 . Τότε τοποθετώντας το c ως ψηφίο χιλιάδων, το d ως ψηφίο

εκατοντάδων, το a ως ψηφίο δεκάδων και το bως ψηφίο μονάδων θα έχουμε το

Page 224: ΠΡΟΤΕΙΝΟΜΕΝΕΣ ΑΣΚΗΣΕΙΣ ΓΙΑ ΜΑΘΗΤΙΚΟΥΣ ΔΙΑΓΩΝΙΣΜΟΥΣ_ALL

http://www.mathematica.gr/forum/viewtopic.php?f=109&t=15584

Επιμέλεια: xr.tsif Σελίδα 31

απαιτούμενο αποτέλεσμα, αφού

1000(a c) 100(b d) 10(c a) d b 1000·9 100·9 10·9 9 9999

Το συμπέρασμα είναι ότι:

Γενικά οι αριθμοί που θα έχουν άρτιο πλήθος ψηφίων θα έχουν πάντα μια

αναδιάταξη που να δίνει το απαιτούμενο αποτέλεσμα αφού θα μπορούμε πάντα να

τους χωρίζουμε σε ζευγάρια ώστε αυτά να έχουν άθροισμα 9 . Αντίθετα όσοι αριθμοί

θα έχουν περιττό πλήθος ψηφίων δεν θα έχουν αναδιάταξη που να δίνει το

απαιτούμενο αποτέλεσμα αφού δεν θα μπορούμε τα ψηφία τους να τα χωρίσουμε σε

ζευγάρια ώστε να έχουν άθροισμα 9 . Έτσι τελικά δεν γίνεται το άθροισμα ενός

αριθμού και μιας αναδιάταξης των ψηφίων του να είναι ο 99...9 με 2011 εννιάρια

(μήπως χρειάζεται μια πιο ισχυρή απόδειξη..; )

Καλά έκανες και κοίταξες τις μικρές περιπτώσεις και σωστά υποψιάζεσαι ότι δεν

γίνεται να έχουμε περιττό αριθμό εννιαριών. Πρέπει όμως να αποδειχθεί. Προς το

παρόν έχει δώσει μια απόδειξη (*) για τρία εννιάρια αλλά γιατί αυτή η απόδειξη να

γενικεύεται; Πρέπει να το εξηγήσεις.

(*) Και στην απόδειξη με τα τρία εννιάρια υπάρχει ένα πρόβλημα. Έχεις ελέγξει

μόνο μια αναδιάταξη την cab . Υπάρχουν όμως άλλες πέντε αναδιατάξεις που πρέπει

να ελεγχθούν. Οι abc,acb,bac,bca,cba . Για κάθε μία από αυτές πρέπει να δειχθεί

ότι το άθροισμα με το abc δεν μπορεί να ισούται με 999 . Και αν οι περιπτώσεις εδώ

είναι λίγες και μπορούν να ελεγχθούν μία προς μία, για τα 2011 εννιάρια πρέπει να

σκεφτούμε κάτι διαφορετικό. Κάτι που θα δουλεύει για όλες τις περιπτώσεις...

ΠΑΡΑΤΗΡΗΣΕΙΣ

Μια σπουδαία μέθοδος για την απόδειξη μιας συνεπαγωγής είναι η μέθοδος της

αντιθετοαντιστροφής.

Ας πούμε πρώτα τι ονομάζουμε λογική πρόταση.

Page 225: ΠΡΟΤΕΙΝΟΜΕΝΕΣ ΑΣΚΗΣΕΙΣ ΓΙΑ ΜΑΘΗΤΙΚΟΥΣ ΔΙΑΓΩΝΙΣΜΟΥΣ_ALL

http://www.mathematica.gr/forum/viewtopic.php?f=109&t=15584

Επιμέλεια: xr.tsif Σελίδα 32

Λογική πρόταση, ονομάζουμε κάθε έκφραση που έχει νόημα και δέχεται έναν και

μόνο από τους χαρακτηρισμούς αληθής (Α) ή ψευδής (Ψ).

ΑΝΤΙΘΕΤΟΑΝΤΙΣΤΡΟΦΗ

Έστω ότι έχουμε δύο λογικές προτάσεις p και q και ας συμβολίσουμε με p,q τις

αρνήσεις των προτάσεων αυτών. Τότε η παρακάτω ισοδυναμία είναι πάντα αληθής:

(p q) (q p) .

Συνεπώς, όταν θέλουμε να αποδείξουμε μια συνεπαγωγή p q , αρκεί να

αποδείχνουμε (αν αυτό είναι ευκολότερο) την συνεπαγωγή:

q p (δηλαδή ότι η άρνηση της qσυνεπάγεται την άρνηση της p .

ΘΕΜΑ 230 (ΔΗΜΗΤΡΗΣ ΙΩΑΝΝΟΥ )

Δίνονται οι ακέραιοι x,y . Αν η παράσταση 2 2x y δεν διαιρείται με το 3 , να

αποδείξετε ότι ακριβώς ένα από τα x,y διαιρείται με το 3 .

Λύση:

Αν υποθέσουμε ότι x 0(mod3) και y 0(mod3) τότε

2 2 2 2x 1(mod3) και y 1(mod3) ,άρα x y 0(mod3) , άτοπο.

ΘΕΜΑ 231 (Socrates)

Να βρεθούν οι φυσικοί αριθμοί n για τους οποίους οι αριθμοί n

n 1 και 2n(2n) 1

είναι πρώτοι.

Λύση:

Θα χρησιμοποιήσουμε το εξής Λήμμα :

"Αν n 1 και a 2 είναι ακέραιοι τέτοιοι ώστε ο n

a 1 είναι πρώτος, τότε (ο a είναι

άρτιος, και) r

n 2 για κάποιο θετικό ακέραιο r ."

Page 226: ΠΡΟΤΕΙΝΟΜΕΝΕΣ ΑΣΚΗΣΕΙΣ ΓΙΑ ΜΑΘΗΤΙΚΟΥΣ ΔΙΑΓΩΝΙΣΜΟΥΣ_ALL

http://www.mathematica.gr/forum/viewtopic.php?f=109&t=15584

Επιμέλεια: xr.tsif Σελίδα 33

ΑΠΟΔΕΙΞΗ ΛΗΜΜΑΤΟΣ

Το ότι o a είναι άρτιος είναι προφανές, αφού κάθε πρώτος αριθμός είναι η δύναμη

ενός πολλαπλάσιου του δύο αυξημένη κατά την μονάδα. Δηλαδή εάν p είναι πρώτος,

τότε ισχύει ότι jp (2k) 1 όπου j,k Z .

Αυτό που μένει προς απόδειξη είναι το ότι mj 2 .

Είναι επίσης προφανές αφού εάν είχαμε έναν περιττό αριθμό π.χ. j 3 τότε θα

είχαμε p 9 (για k 1 , ή για άλλη τιμή του k κάποιον άλλο σύνθετο αριθμό) που

δεν είναι πρώτος. Άρα θέλουμε να ισχύει ότι jp (2k) 1 όπου j,k Z και ο j

είναι άρτιος, δηλαδή παίρνει τη μορφή 2m .

ΑΠΟΔΕΙΞΗ

Για n 1 , είναι nn 1 2 και 2n

(2n) 1 5 , άρα η λύση n 1 γίνεται δεκτή.

Βασιζόμενοι στο βοηθητικό πρόβλημα, βλέπουμε ότι αν n 1 , τότε θα είναι rn 2 ,

(με r 0 ) και οι αριθμοί

rn r 2

n 1 2 1 και

r 12n (r 1) 2(2n) 1 2 1

θα είναι πρώτοι.

Από το βοηθητικό πρόβλημα, ξανά θα είναι r k

r 2 2 και r 1 μ(r 1)2 2

, για

κάποιους θετικούς ακέραιους k,μ σαφώς με μ k .

Διαιρώντας κατά μέλη παίρνουμε μ k 1r 1

2r

, κι άρα r 1 που δίνει n 2 .

Εύκολα ελέγχεται ότι το n 2 γίνεται δεκτό, αφού δίνει

nn 1 5 και 2n

(2n) 1 257 , που είναι πρώτοι αριθμοί.

Συνεπώς, οι μόνες λύσεις είναι n 1 και n 2 .

Page 227: ΠΡΟΤΕΙΝΟΜΕΝΕΣ ΑΣΚΗΣΕΙΣ ΓΙΑ ΜΑΘΗΤΙΚΟΥΣ ΔΙΑΓΩΝΙΣΜΟΥΣ_ALL

http://www.mathematica.gr/forum/viewtopic.php?f=109&t=15584

Επιμέλεια: xr.tsif Σελίδα 34

ΘΕΜΑ 232 (Socrates)

Αν οι θετικοί ακέραιοι a,b,c,d * είναι τέτοιοι ώστε 2 2ad b bc c , να δείξετε

ότι ο αριθμός 2 2 2 2a b c d είναι σύνθετος.

Λύση:

Είναι

2 2 2 2 2 2 2 2 2 2 2 2a b c d (a d) 2ad b c (a d) 2(b bc c ) b c

2 2(a d) (b c) (a b c d)(a b c d) .

Για να ολοκληρωθεί η απόδειξη χρειάζεται να εξηγήσουμε γιατί καμία από τις

παρενθέσεις δεν ισούται με 1 . Για την πρώτη αυτό είναι φανερό, αφού μιλάμε για

θετικούς ακέραιους. Αν τώρα, υποθέσουμε, ότι η δεύτερη παρένθεση ισούται με 1 ,

θα είναι 2 2 2 2a b c d a b c d .

Καθώς όμως, είναι a,b,c,d * , θα έπρεπε τότε να ισχύει a b c d 1 το

οποίο αντίκειται στην 2 2

ad b bc c .

ΘΕΜΑ 233 (ΣΩΤΗΡΗΣ ΛΟΥΡΙΔΑΣ )

Θεωρούμε ευθεία (ε) και σημεία της Α,B,C ώστε AB BC .Έστω ότι δύο άνισοι

κύκλοι με κέντρα τα σημεία A,C τέμνουν ένα κύκλο με κέντρο το σημείο B στα

σημεία D,E . Να συγκριθούν οι χορδές οι αποκοπτόμενες από τις περιφέρειες με

κέντρα τα σημεία A,B από την ευθεία DE .

Page 228: ΠΡΟΤΕΙΝΟΜΕΝΕΣ ΑΣΚΗΣΕΙΣ ΓΙΑ ΜΑΘΗΤΙΚΟΥΣ ΔΙΑΓΩΝΙΣΜΟΥΣ_ALL

http://www.mathematica.gr/forum/viewtopic.php?f=109&t=15584

Επιμέλεια: xr.tsif Σελίδα 35

Λύση:

Οι ΑΜ ,ΒΝ και CG είναι παράλληλες (γιατί;) και αφού AB BC έπεται ότι

MN NG . Αλλά DN NE οπότε MD EG και επομένως ZD EH .

ΘΕΜΑ 234 (ΣΩΤΗΡΗΣ ΛΟΥΡΙΔΑΣ )

Α) Αν a,b και ab 0 , να αποδειχθεί ότι: 2 2 2(a b) a b ,

επίσης αν a,b,c και a b c 0 , να αποδειχθεί ότι: 3 3 3a b c 3abc .

Β) Έστω *m,n

και 0 t 2 . Αποδείξτε ότι:

23 3

3 2

m n 31.

tmnt

Μια υπενθύμιση (Ταυτότητα Euler):

3 3 3 2 2 2a b c 3abc a b c a b c ab bc ca ή

2 2 23 3 3 1

a b c 3abc a b c a b b c c a .2

Page 229: ΠΡΟΤΕΙΝΟΜΕΝΕΣ ΑΣΚΗΣΕΙΣ ΓΙΑ ΜΑΘΗΤΙΚΟΥΣ ΔΙΑΓΩΝΙΣΜΟΥΣ_ALL

http://www.mathematica.gr/forum/viewtopic.php?f=109&t=15584

Επιμέλεια: xr.tsif Σελίδα 36

Λύση:

Α) Η πρώτη ισοδυναμεί με την αληθή 2ab 0 . Ας σημειωθεί ότι η ανισότητα είναι

γνήσια αν ab 0 (*).

Η δεύτερη είναι άμεση από την ταυτότητα Euler.

B) Έχουμε 2 2 2 2m n ( mnt) m n 2mn 0 , οπότε από το προηγούμενο

2 3 2 3 3 2 2(m ) (n ) ( mnt) 3(m )(n )( mnt) .

Διαιρώντας με το 3(mnt) έχουμε

6 6

3 2

m n 31

(mnt) t

.

Τελειώνουμε παρατηρώντας (χρήση του Α) και του (*)) ότι 3 3 2 6 6(m n ) m n .

ΘΕΜΑ 235 (ΔΗΜΗΤΡΗΣ ΙΩΑΝΝΟΥ )

Να αποδείξετε ότι ανάμεσα από 79 διαδοχικούς φυσικούς αριθμούς, μπορούμε να

βρούμε έναν, που το άθροισμα των ψηφίων του να είναι διαιρετό με 13 . Ισχύει η

πρόταση για 78 διαδοχικούς φυσικούς;

Λύση:

Εξετάζουμε τον μικρότερο από τους δοθέντες 79 αριθμούς.

Είναι της μορφής A00,A01,...,A99 .

α) Αν ο μικρότερος αυτός αριθμός είναι στο διάστημα A00,A01,...,A20 τότε οι 79

αριθμοί συμπεριλαμβάνουν σίγουρα τους A20,A21,...,A78 και ειδικά

συμπεριλαμβάνουν τους {A20,A21,...,A29} {A75,A76,A77} . Τα υπόλοιπα

μόντουλο 13 των δύο τελευταίων ψηφίων των (δεκατριών) αυτών αριθμών είναι,

αντίστοιχα, {2,3,...,10,11} {12,0,1} . Δηλαδή περιλαμβάνουν όλα τα δυνατά

υπόλοιπα δια 13 , συνεπώς κάποιος από αυτούς είναι πολλαπλάσιο του 13 (ποιος

ακριβώς εξαρτάται από το άθροισμα των ψηφίων του A ).

Page 230: ΠΡΟΤΕΙΝΟΜΕΝΕΣ ΑΣΚΗΣΕΙΣ ΓΙΑ ΜΑΘΗΤΙΚΟΥΣ ΔΙΑΓΩΝΙΣΜΟΥΣ_ALL

http://www.mathematica.gr/forum/viewtopic.php?f=109&t=15584

Επιμέλεια: xr.tsif Σελίδα 37

β) Αν ο μικρότερος αυτός αριθμός είναι στο διάστημα A21,...,A60 τότε οι 79

αριθμοί συμπεριλαμβάνουν σίγουρα τους A60,A61,...,A99 . Ειδικά

συμπεριλαμβάνουν τους {A60,A61,...,A66} {A67,A68,A69} {A97,A98,A99} .

Τα υπόλοιπα μόντουλο 13 των δύο τελευταίων ψηφίων των (δεκατριών) αυτών

αριθμών είναι, αντίστοιχα, {6,7,...,12} {0,1,2} {3,4,5} . Δηλαδή, όπως πριν,

περιλαμβάνουν όλα τα δυνατά υπόλοιπα δια 13 και συνεπώς κάποιος από αυτούς

είναι πολλαπλάσιο του 13 .

γ) Αν ο μικρότερος αυτός αριθμός είναι στο διάστημα A61,...,A99 , τότε το πολύ

39 από τους 79 αριθμούς βρίσκονται στο A61,...,A99 . Οι υπόλοιποι, που είναι

τουλάχιστον 40 το πλήθος, συμπεριλαμβάνουν τους B00,B01,...,B39 (ο B00 είναι

ο επόμενος του A99). Ειδικά περιλαμβάνουν τους 13 αριθμούς

{B00,B01,...,B09} {B19,B29,B39} . Τα υπόλοιπα μόντουλο 13 των δύο

τελευταίων ψηφίων των αριθμών αυτών είναι, αντίστοιχα, {0,1,2,...,9} {10,11,12} .

Δηλαδή περιλαμβάνουν όλα τα δυνατά υπόλοιπα δια 13 και συνεπώς κάποιος από

αυτούς είναι πολλαπλάσιο του 13 .

Οι τρεις περιπτώσεις τελειώνουν την απόδειξη.

Δεν έφτιαξα ακόμη παράδειγμα 78 αριθμών που δεν έχουν την παραπάνω ιδιότητα.

Είναι σαφές και προφανές πώς θα κατασκευαστούν με βάση την περίπτωση γ).

Κανείς από τους 78 διαδοχικούς αριθμούς

9999999961,9999999962,...,9999999999,10000000000,...,10000000038 , δεν έχει

άθροισμα ψηφίων ένα πολλαπλάσιο του 13 .

Μπορεί να κάνει κανείς απευθείας έλεγχο: Οι μεν της μορφής 9

9999999961,9999999962,...,9999999999 έχουν άθροισμα ψηφίων ίσα με

{79,80,...,85} {79,80,...,88} {80,81,...,89} {81,82,...,90} και οι

10000000000,...,10000000038 έχουν

{1,2,...,10} {2,3,...,11} {3,4....,12} {4,5,...,12} . Κανένας από αυτούς δεν είναι

πολλαπλάσιο του 13 , και επαληθεύσαμε .

Page 231: ΠΡΟΤΕΙΝΟΜΕΝΕΣ ΑΣΚΗΣΕΙΣ ΓΙΑ ΜΑΘΗΤΙΚΟΥΣ ΔΙΑΓΩΝΙΣΜΟΥΣ_ALL

http://www.mathematica.gr/forum/viewtopic.php?f=109&t=15584

Επιμέλεια: xr.tsif Σελίδα 38

Πως σκεφτόμαστε; Με οδηγό πάντα το γ) της απόδειξης , εξετάζουμε τους 78

αριθμούς της μορφής A61,...,A99,B00,B01,...,38 . Το A είναι να το επιλέξουμε.

Το άθροισμα των δύο τελευταίων ψηφίων των αριθμών με " A " έχουν υπόλοιπο

μόντουλο 13 (ελέγχουμε με το χέρι) τους {0,1,...,5,7,...,12} (δηλαδή χάνουν τον 6 ).

Όμοια, οι αριθμοί "B " χάνουν το 12 . Οπότε ψάχνουμε αριθμό A με άθροισμα

s(A) ψηφίων ίσο με 7mod13 (ώστε οι " A " να χάνουν το 0mod13 ) και συγχρόνως

s(A 1) να είναι ίσο με 1mod13 (ώστε και οι "B " να χάνουν το 0mod13 ). Το

προφανές είναι να ψάξουμε αριθμό της μορφής 9999...9 γιατί η δεύτερη συνθήκη

γίνεται τετριμμένη ενώ με δοκιμές επιλέγουμε όσα εννιάρια χρειαζόμαστε για να

ισχύει η πρώτη συνθήκη (πήρα 8 μια και 8 9 7mod13 ).

ΘΕΜΑ 236 (ΔΗΜΗΤΡΗΣ ΙΩΑΝΝΟΥ )

Έστω 1 2 7

a ,a ,...,a δοσμένα ευθύγραμμα τμήματα με 1 2 7

1 a a ... a 10 .

Να αποδείξετε ότι 3 τουλάχιστον από αυτά είναι πλευρές τριγώνου.

Λύση:

Αν δεν υπήρχαν τρία που κατασκευάζουν τρίγωνο, τότε για οποιαδήποτε τρία με

a b c θα ίσχυε a b c .

Τώρα, αφού 2 1

a a 1 θα έχουμε 3 2 1

a a a 1 1 2 , 4 3 2

a a a 2 1 3 ,

5 4 3a a a 3 2 5 ,

6 5 4a a a 5 3 8 και τέλος

7 6 5a a a 8 5 13 ,

άτοπο αφού 7

a 10 .

ΘΕΜΑ 237 (Socrates)

Να δείξετε ότι κάθε τέλειος κύβος γράφεται ως διαφορά τετραγώνων ακεραίων.

Λύση:

Αν ο n είναι περιττός τότε

2 23 3

3 n 1 n 1n

2 2

Page 232: ΠΡΟΤΕΙΝΟΜΕΝΕΣ ΑΣΚΗΣΕΙΣ ΓΙΑ ΜΑΘΗΤΙΚΟΥΣ ΔΙΑΓΩΝΙΣΜΟΥΣ_ALL

http://www.mathematica.gr/forum/viewtopic.php?f=109&t=15584

Επιμέλεια: xr.tsif Σελίδα 39

Αν ο n είναι άρτιος τότε

2 23 3

3 n 4 n 4n

4 4

Β τρόπος:

2 22 2

3 n n n nn

2 2

Πως το βρήκαμε; Θέλουμε 3 2 2n x y (x y)(x y) . Την πιο πάνω λύση την

παίρνουμε αν λύσουμε το σύστημα 2x y n και x y n . (Ο Αλέξανδρος λύνει το

σύστημα 3x y n και x y 1 στην περίπτωση που ο n είναι περιττός και το

σύστημα 2

nx y

2 και x y 2 στην περίπτωση που ο n είναι άρτιος. Θέλει μόνο

λίγη προσοχή αφού οι λύσεις που θα βρούμε είναι δεκτές μόνο αν είναι ακέραιες.)

ΘΕΜΑ 238 (Socrates)

Να βρεθεί ο μέγιστος αριθμός διαδοχικών ακεραίων που μπορούν να γραφούν στη

μορφή 2 2n 2m , m,n .

Λύση:

Ανάμεσα σε 6 διαδοχικούς φυσικούς κάποιος είναι της μορφής 8k 5 ή της μορφής

8k 7 . Όμως ο αριθμός 2 2

n 2m δεν είναι ποτέ αυτής της μορφής.

Άρα ο μεγαλύτερος αριθμός διαδοχικών ακεραίων που μπορούν να γραφτούν με

αυτή τη μορφή είναι 5 .

Όμως 2 2

0 0 2·0 , 2 2

1 1 2·0 , 2 2

2 0 2·1 , 2 2

3 1 2·1 , 2 2

4 2 2·0 .

Άρα είναι και ο μέγιστος.

Page 233: ΠΡΟΤΕΙΝΟΜΕΝΕΣ ΑΣΚΗΣΕΙΣ ΓΙΑ ΜΑΘΗΤΙΚΟΥΣ ΔΙΑΓΩΝΙΣΜΟΥΣ_ALL

http://www.mathematica.gr/forum/viewtopic.php?f=109&t=15584

Επιμέλεια: xr.tsif Σελίδα 40

ΘΕΜΑ 239 (Socrates)

Να βρείτε όλες τις τριάδες θετικών ακεραίων (a,b,c)τέτοιες ώστε [a,b,c] a b c

όπου [a,b,c] το ελάχιστο κοινό πολλαπλάσιο των a,b,c .

Λύση:

Ας υποθέσουμε ότι c b a .

Προφανώς, c a , αφού διαφορετικά θα ήταν c b a , και [a,b,c] a a b c .

Εξ ορισμού, είναι [a,b,c] xa yb zc για κάποιους θετικούς ακέραιους x,y,z .

Από την υπόθεση, λοιπόν, είναι (z 1)c a b , οπότε a b

0 z 1 2c c

,

κι άρα z 1 1 , δηλαδή z 2 και c a b .

Αν ήταν b a , τότε c 2a και [a,b,c] [a,a,2a] 2a a b c , άτοπο.

Συνεπώς, είναι b a .

Αλλά, από yb 2(a b) έπεται ότι 2a (y 2)b , κι άρα το b , που είναι a

διαιρεί το 2a 2b .

Συνεπώς, θα είναι b 2a , κι άρα c 3a . Προφανώς, τότε ικανοποιείται η

[a,b,c] a b c .

Αποδείξαμε, λοιπόν, ότι αν c b a και [a,b,c] a b c , τότε c 3a και b 2a .

Θεωρώντας όλες τις μεταθέσεις των τριάδων αυτών, συμπεραίνουμε ότι οι

ζητούμενες λύσεις είναι (a,b,c) ~ (t,2t,3t) για κάποιο θετικό ακέραιο t .

ΘΕΜΑ 240 (Atemlos )

Να δείξετε ότι ο 20k 4 10k 2

a a 1 είναι σύνθετος για

*a N,a 1,k N .

Page 234: ΠΡΟΤΕΙΝΟΜΕΝΕΣ ΑΣΚΗΣΕΙΣ ΓΙΑ ΜΑΘΗΤΙΚΟΥΣ ΔΙΑΓΩΝΙΣΜΟΥΣ_ALL

http://www.mathematica.gr/forum/viewtopic.php?f=109&t=15584

Επιμέλεια: xr.tsif Σελίδα 41

Λύση:

Άμεση από την 4 2 2 2x x 1 (x x 1)(x x 1) . Εδώ έχουμε 5k 1

x a .

Δηλαδή 20k 4 10k 2 4(5k 1) 2(5k 1) (5k 1) 5 )4 2( k 1a a 1 a a 1 ( )a 1)(a

.

ΘΕΜΑ 241 (ΔΗΜΗΤΡΗΣ ΙΩΑΝΝΟΥ )

Έστω 1

a Z , a R 0a

. Να αποδείξετε ότι n

n

1a Z

a , για κάθε n N .

Λύση:

n 2 n n 1

n 2 n n 1

1 1 1 1a a (a )(a )

a a a a

(1).

Έστω ότι αν ο 1

aa

είναι ακέραιος τότε m

m

1a

a είναι επίσης ακέραιος για κάθε

m n τότε απο την ταυτότητα (1) θα ισχύει και για m n , άρα θα ισχύει για κάθε

φυσικό n .

ΘΕΜΑ 242 (ΔΗΜΗΤΡΗΣ ΙΩΑΝΝΟΥ )

Να βρείτε τα έξι τελευταία ψηφία του αριθμού 1095 , όταν ο αριθμός αυτός γραφτεί

στο δυαδικό σύστημα αρίθμησης.

Λύση:

Αρκεί να βρούμε το 109

5 mod64

Αφού το 36

5 το υπολογίζει ... το κομπιουτεράκι μου = 49mod64 ,

στη συνέχεια μου υπολογίζει το 3

49 5 21mod64 και μετά πατώντας bin

"γλυτώνω" τις διαιρέσεις: 010101 .

Page 235: ΠΡΟΤΕΙΝΟΜΕΝΕΣ ΑΣΚΗΣΕΙΣ ΓΙΑ ΜΑΘΗΤΙΚΟΥΣ ΔΙΑΓΩΝΙΣΜΟΥΣ_ALL

http://www.mathematica.gr/forum/viewtopic.php?f=109&t=15584

Επιμέλεια: xr.tsif Σελίδα 42

Β τρόπος

Αν δεν θέλαμε κομπιουτεράκι, θα μπορούσαμε να πούμε: Από Euler είναι φ(64)

5 1mod64 και αφού (από τον τύπο για την φ ) είναι φ(64) 32 , ισχύει

325 1mod64 . Άρα 109 13 32 3 13

5 5 (5 ) 5 mod64 και λοιπά.

Αν δεν θέλαμε καθόλου Euler αλλά να μείνουμε σε "πολύ στοιχειώδη" Μαθηματικά,

θα μπορούσαμε να κατεβάζαμε σταδιακά τον μεγάλο εκθέτη λέγοντας, π.χ.,

109 3 36 36 36 95 5 (5 ) 5 (125) 5 ( 3) 5 81

9 4 2 25 17 5 17 17 5 17 (17 ) 5 17 21mod64 , και λοιπά.

Γ τρόπος

Ας επανέλθω σε αυτήν την άσκηση με ακόμη μια μέθοδο. Χρειάζεται να γνωρίζουμε

το http://en.wikipedia.org/wiki/Binomial_theorem (διωνυμικό θεώρημα).

Όπως και στις προηγούμενες λύσεις, ψάχνουμε το 1095 mod64 . Έχουμε

109 109109 109

5 (1 4) 1 4 16 mod641 2

Όμως 109 3mod16 και 109

2mod42

επομένως

1095 1 12 32 21mod64 .

Η ίδια μέθοδος δίνει επίσης ότι n 25 8n 4n 1mod64 .

Δ τρόπος

Θα χρησιμοποιήσω την ταυτότητα

n n n 1 n 2 2 n 3 3 nn(n 1) n(n 1)(n 2)(x y) x nx y x y x y ... y

1 2 2 3

Έχουμε τώρα: 109 n n 1 6 5

n n 1 6 5 1 05 a 2 a 2 ... a 2 a 2 ... a 2 a

Page 236: ΠΡΟΤΕΙΝΟΜΕΝΕΣ ΑΣΚΗΣΕΙΣ ΓΙΑ ΜΑΘΗΤΙΚΟΥΣ ΔΙΑΓΩΝΙΣΜΟΥΣ_ALL

http://www.mathematica.gr/forum/viewtopic.php?f=109&t=15584

Επιμέλεια: xr.tsif Σελίδα 43

όπου τα n n 1 0

a ,a ,...,a

παίρνουν τιμές 0 ή 1 .

Άρα έχουμε:

109 n 6 6 n 7 6 6 5

n n 1 6 5 1 0 05 a 2 2 a 2 2 ... a 2 a 2 ... a 2 a

109 n 6 n 7 6 5 4

n n 1 6 5 4 05 (a 2 a 2 ... a ) 2 a 2 a 2 ... a

Συνεπώς ο αριθμός 5 4

5 4a 2 a 2 ... a είναι το υπόλοιπο της διαίρεσης του 109

5 με

το 62 .

Και άρα τα τελευταία ψηφία του αριθμού που ζητάμε είναι τα 5 4 0

a ,a ,...,a .

Έχουμε τώρα:

109 2 109 109 108 2 107 2 2109 1085 (1 2 ) 1 109 1 (2 ) 1 (2 )

2

106 2 3 2 109 2 4 6109 108 1071 (2 ) ... (2 ) 1 109 2 109 54 2 k 2

2 3

, όπου k

φυσικός αριθμός

Άρα: 109 6 5 3 2 2 6 5 4 65 1 (2 2 2 2 1) 2 (2 45)(2 22) 2 k 2

8 7 5 4 2 15 10 9 4 61 2 2 2 2 2 2 2 22 2 45 45 22 2 k 2

4 2 5 4 61 2 2 2 45 22 2 m 2 , όπου m N .

Άρα 109 2 4 5 5 4 4 65 1 2 2 2 (2 13)(2 6) 2 m 2

2 4 5 13 9 8 4 61 2 2 2 2 6 2 13 2 78 2 m 2

2 4 6 2 5 3 2 4 61 2 79 2 m 2 1 2 (2 2 2 1) 2 m 2

2 9 7 6 4 6 2 4 61 2 2 2 2 2 m 2 1 2 2 t 2 όπου t είναι φυσικός αριθμός.

Άρα το υπόλοιπο που ζητάμε είναι 5 4 3 2 1 0

0 2 1 2 0 2 1 2 0 2 1 2

και τα τελευταία 6 ψηφία που ζητάμε είναι τα 010101 .

Page 237: ΠΡΟΤΕΙΝΟΜΕΝΕΣ ΑΣΚΗΣΕΙΣ ΓΙΑ ΜΑΘΗΤΙΚΟΥΣ ΔΙΑΓΩΝΙΣΜΟΥΣ_ALL

http://www.mathematica.gr/forum/viewtopic.php?f=109&t=15584

Επιμέλεια: xr.tsif Σελίδα 44

ΘΕΜΑ 243 (Socrates)

Αν οι θετικοί ακέραιοι x,y,z,t,a,b είναι τέτοιοι ώστε xt yz 1 και x a z

y b t να

δείξετε ότι ab (x z)(y t) .

Λύση:

Έχουμε

1 x z x a a z bx ay at bz 1 1 t y

yt y t y b b t yb bt yb

bt byt

,

οπότε b t y (1).

Ομοίως, είναι

1 t y t b b y ta bz bx ay 1 1 x z

xz z x z a a x az ax az ax axz

,

οπότε a x z (2).

Πολλαπλασιάζοντας τις (1), (2) κατά μέλη παίρνουμε τη ζητούμενη ανισότητα.

ΘΕΜΑ 244 (Socrates)

Αν x και y είναι ακέραιοι, τότε ο αριθμός x y είναι επίσης ακέραιος.

Η πράξη ικανοποιεί τα ακόλουθα:

• x (y z) (x y) z, για όλους τους ακέραιους x,y,z .

• (y z) x (y x) 2z, για όλους τους ακέραιους x,y,z .

• 1 1 1.

Να υπολογίσετε τον αριθμό 25 10 .

Page 238: ΠΡΟΤΕΙΝΟΜΕΝΕΣ ΑΣΚΗΣΕΙΣ ΓΙΑ ΜΑΘΗΤΙΚΟΥΣ ΔΙΑΓΩΝΙΣΜΟΥΣ_ALL

http://www.mathematica.gr/forum/viewtopic.php?f=109&t=15584

Επιμέλεια: xr.tsif Σελίδα 45

Bonus: Δείξτε ότι: a b 2a b .

Λύση:

25 10 (24 1) 24 10 2 24 1 9) 2 210 ( 14 1 7 (1 23) 7

1 2 23 7 1 46 7 401 .

Βonus

1 1 1 (a 1) a 1 1 (a 1) 1 2a 1( )

(a 1) 1 2a 1 a 1 2 2a 1 a 1 2a 1

a b (1 b) 2a 1 a b (1 b) 2a 1 a b 2a b( ) .

ΘΕΜΑ 245 (Socrates)

Σε ένα διαγωνισμό τοξοβολίας συμμετέχουν 30 αθλητές. Ο στόχος χωρίζεται σε δύο

ζώνες, τις A και B . Αν το βέλος χτυπήσει στη ζώνη A , ο αθλητής κερδίζει 10

βαθμούς ενώ αν χτυπήσει στη ζώνη B , κερδίζει 5 βαθμούς. Δεν δίνονται βαθμοί για

τα βέλη που δεν βρίσκουν το στόχο. Κάθε αθλητής ρίχνει 16 βέλη. Στο τέλος του

διαγωνισμού, παρατηρήθηκε ότι πάνω από το 50% των βελών χτύπησαν στη ζώνη

B , ενώ ο αριθμός των βελών που χτύπησαν στη ζώνη A είναι ίσος με τον αριθμό

αυτών που δεν βρήκαν το στόχο.

Να δείξετε ότι υπάρχουν δύο αθλητές με την ίδια βαθμολογία.

Λύση:

Ονομάζουμε x,y,z τον αριθμό των βελών που σαν σύνολο θα χτυπήσουν τις ζώνες

A,Β,Γ αντίστοιχα (όπου Γ η εκτός στόχου "ζώνη").

Προφανώς είναι x y z 30 16 480 και επιπλέον ισχύει y 240 και x z με

maxx 119 .

Τώρα καλούμε k k k

x ,y ,z τον αριθμό των βελών που ένας τοξοβόλος k , με

k {1,2,3,...,30} Δ θα ρίξει στις ζώνες A,Β,Γ αντίστοιχα. Αφού κάθε τοξοβόλος

Page 239: ΠΡΟΤΕΙΝΟΜΕΝΕΣ ΑΣΚΗΣΕΙΣ ΓΙΑ ΜΑΘΗΤΙΚΟΥΣ ΔΙΑΓΩΝΙΣΜΟΥΣ_ALL

http://www.mathematica.gr/forum/viewtopic.php?f=109&t=15584

Επιμέλεια: xr.tsif Σελίδα 46

ρίχνει 16 βέλη έχουμε k k k

x y z 16 (*) . Δεδομένου του ότι ο καθένας παίρνει

10 πόντους για τη ζώνη A , 5 για τη Β και 0 για τη Γ , το συνολικό σκορ κάθε

τοξοβόλου είναι (*)

k k k k k k k kS 10x 5y 0z 10x 5y 80 5(x z ) .

Εξ αυτού συμπεραίνουμε ότι για να έχουν 2 συμμετέχοντες k,k΄ το ίδιο σκορ είναι:

k k k΄ k΄S(k) S(k΄) x z x z .

Θα υποθέσουμε ότι κανένας τοξοβόλος δεν έχει ίδιο σκορ με κάποιον άλλο και

συνεπώς k k k΄ k΄

S(1) S(2) ... S(30) x z x z για κάθε k,k΄ Δ με k k΄ .

Από περιστεροφωλιά υπάρχουν 15 ομόσημες παραστάσεις k k

x z , έστω θετικές με

διαφορετικό ακέραιο αποτέλεσμα και άρα το ελάχιστο άθροισμά τους είναι:

1 2 ... 15 120 όμως k k k

x x z και άρα το ελάχιστο άθροισμα των 15 αυτών

kx είναι 120 τη στιγμή που

maxx 119 και καταλήγουμε σε αντίφαση ( υπόψη ότι

30

kk 1

x x 120

). Συνεπώς 2 αθλητές θα έχουν σίγουρα ίδιο σκορ.

ΘΕΜΑ 246 (ΔΗΜΗΤΡΗΣ ΙΩΑΝΝΟΥ )

Να λυθεί η εξίσωση: (x 4)(x 5)(x 6)(x 7) 1680 .

Λύση:

Αν τεθεί x 7 y , η εξίσωση γράφεται y(y 1)(y 2)(y 3) 1 1681 , δηλαδή

2 2(y 3y 1) 1681 .

Επομένως αναγόμαστε στη λύση των εξισώσεων

2y 3y 1 41 και 2

y 3y 1 41 .

Προφανώς, η δεύτερη εξίσωση είναι αδύνατη, ενώ η πρώτη έχει τις ρίζες 8 , 5

Άρα η αρχική έχει τις ρίζες 1 , 12 .

Page 240: ΠΡΟΤΕΙΝΟΜΕΝΕΣ ΑΣΚΗΣΕΙΣ ΓΙΑ ΜΑΘΗΤΙΚΟΥΣ ΔΙΑΓΩΝΙΣΜΟΥΣ_ALL

http://www.mathematica.gr/forum/viewtopic.php?f=109&t=15584

Επιμέλεια: xr.tsif Σελίδα 47

Φυσικά, για να λύσει κάποιος την αρχική εξίσωση δεν είναι απαραίτητο να θυμάται

την ταυτότητα 2 2x(x 1)(x 2)(x 3) 1 (x 3x 1) .

Είναι καλό όμως, να θυμόμαστε ότι το γινόμενο τεσσάρων διαδοχικών ακεραίων

αυξημένο κατά ένα είναι τέλειο τετράγωνο!

Β τρόπος

Στο πρώτο μέλος της εξίσωσης έχουμε τέσσερις διαδοχικούς αριθμούς, άρα

υποψιαζόμαστε για τέλειο τετράγωνο. Κάνουμε τις πράξεις ως εξής: Επιμεριστική

την πρώτη με την τέταρτη και τις άλλες δύο μαζί.

Θέτουμε: 2 2x 11x 28 a , x 11x 30 a 2 .

Η εξίσωση γίνεται 2a 2a 1680 . Προσθέτουμε 1 και έτσι

2 2 2(x 11x 28 1) 41 . Οπότε αρκεί να λύσουμε τις εξισώσεις:

2x 11x 29 41 και την 2

x 11x 29 41 .

Η πρώτη δίνει λύσεις: x 12 ή x 1 , ενώ η δεύτερη είναι αδύνατη.

ΘΕΜΑ 247 (ΔΗΜΗΤΡΗΣ ΙΩΑΝΝΟΥ )

Χρωματίζουμε όλα τα σημεία του επιπέδου με 2 χρώματα. Να δείξετε ότι υπάρχουν

δύο τουλάχιστον σημεία του επιπέδου που έχουν το ίδιο χρώμα και απέχουν μεταξύ

τους απόσταση 1 .

Λύση:

Εξετάζουμε οποιοδήποτε ισόπλευρο τρίγωνο πλευράς 1 . Δύο κορυφές του

οπωσδήποτε έχουν το ίδιο χρώμα.

Page 241: ΠΡΟΤΕΙΝΟΜΕΝΕΣ ΑΣΚΗΣΕΙΣ ΓΙΑ ΜΑΘΗΤΙΚΟΥΣ ΔΙΑΓΩΝΙΣΜΟΥΣ_ALL

http://www.mathematica.gr/forum/viewtopic.php?f=109&t=15584

Επιμέλεια: xr.tsif Σελίδα 48

ΘΕΜΑ 248 (ΔΗΜΗΤΡΗΣ ΙΩΑΝΝΟΥ )

Χρωματίζουμε όλα τα σημεία του επιπέδου με τρία χρώματα. Να αποδειχθεί ότι

υπάρχουν δύο τουλάχιστον σημεία του επιπέδου που έχουν το ίδιο χρώμα και

απέχουν μεταξύ τους απόσταση ίση με 1 .

Λύση:

Θεωρούμε πάνω στο επίπεδο δύο ρόμβους ABCD , AEZH οι οποίοι έχουν κοινή

την κορυφή τους A , πλευρά ίση με 1 και την γωνία της κορυφής A ίση με o60

μοίρες. Επίσης είναι με τέτοιον τρόπο τοποθετημένοι ώστε η απόσταση ZC να είναι

ίση με μονάδα.

Από την αρχή του περιστερώνα, (αφού έχουμε 7 κορυφές και 3 χρώματα) θα

υπάρχουν 3 τουλάχιστον κορυφές που θα έχουν το ίδιο χρώμα. Επίσης από την ίδια

αρχή, δύο τουλάχιστον από αυτά θα ανήκουν στον ίδιο ρόμβο. Εύκολα τώρα

διαπιστώνουμε ότι όπως και να θεωρήσουμε τα τρία σημεία του αυτού χρώματος,

δύο τουλάχιστον, θα απέχουν μεταξύ τους απόσταση 1 .

Β τρόπος

Υπάρχει και εδώ (με επιπλέον ερώτημα).

ΘΕΜΑ 249 (ΔΗΜΗΤΡΗΣ ΙΩΑΝΝΟΥ )

Να αποδείξετε ότι η παράσταση 25

A (n 2 2n 3)2

είναι τέλειο τετράγωνο

ακεραίου, αν γνωρίζουμε ότι η A είναι ακέραιος αριθμός (n N )

Λύση:

Αφού η παράσταση είναι ακέραιος τότε η ρίζα είναι ρητός !

Έστω a

2n 3b

όπου a,b ακέραιοι με bμη μηδενικό τότε 2 2

2

(a 3b )n

2b

.

Page 242: ΠΡΟΤΕΙΝΟΜΕΝΕΣ ΑΣΚΗΣΕΙΣ ΓΙΑ ΜΑΘΗΤΙΚΟΥΣ ΔΙΑΓΩΝΙΣΜΟΥΣ_ALL

http://www.mathematica.gr/forum/viewtopic.php?f=109&t=15584

Επιμέλεια: xr.tsif Σελίδα 49

Τώρα αν στην αρχική παράσταση θέσουμε όπου 2 2

2

(a 3b )n

2b

και A m ακέραιο

τότε μετά από πράξεις καταλήγουμε ότι 2 225(a b) m(2b) , άρα m τέλειο

τετράγωνο.

Β τρόπος

Πρέπει λοιπόν ο αριθμός 2n 3 να είναι τέλειο τετράγωνο και επειδή είναι περιττός

(ως άθροισμα άρτιου και περιττού), θα πρέπει να είναι τέλειο τετράγωνο περιττού.

Άρα υπάρχει k φυσικός αριθμός ώστε να είναι

2 22n 3 (2k 1) n 2k 2k 1 . Άρα

2 2 225 25A [2k 2k 1 2 (2k 1) ] A (2k 2k 1 2 2k 1)

2 2

2 2A 25k (5k) , και η απόδειξη ολοκληρώθηκε.

Γ τρόπος

Προφανώς το 2n 3 είναι περιττός φυσικός, οπότε

2

2n 3 1A 5

2

με

2n 3 1

2

φυσικό.

ΘΕΜΑ 250 (ΔΗΜΗΤΡΗΣ ΙΩΑΝΝΟΥ )

Αν x,y και αν ισχύει ότι: 2 2 2 2x y 3x y 8(x y 2xy) 43 , να βρείτε την

αριθμητική τιμή της παράστασης: 4 4A x y .

Λύση:

Σε τέτοιες ασκήσεις που δίνεται εξίσωση στους πραγματικούς και ζητάει να

υπολογιστεί μια παράσταση όπως στην περίπτωση μας με ακριβές νούμερο τότε

προσπαθούμε να την φέρουμε σε μορφή 2 2 2

1 1 2 2 k ka x a x ....a x 0 .

Page 243: ΠΡΟΤΕΙΝΟΜΕΝΕΣ ΑΣΚΗΣΕΙΣ ΓΙΑ ΜΑΘΗΤΙΚΟΥΣ ΔΙΑΓΩΝΙΣΜΟΥΣ_ALL

http://www.mathematica.gr/forum/viewtopic.php?f=109&t=15584

Επιμέλεια: xr.tsif Σελίδα 50

και i

a 0 οπότε θα έχει λύση μόνο για i

x 0 με i 1,2,...k .

Στην περίπτωση μας τώρα μετά από πράξεις η εξίσωση γίνεται:

2 2(x y 4) 3(xy 3) 0 άρα όπως είπαμε παραπάνω θα έχει λύση μόνο για

x y 4 , xy 3 .

Οπότε 2 2x y 10 και 4 4

x y 82 .

ΘΕΜΑ 251 (Socrates)

Σε ένα ενυδρείο υπάρχουν ψάρια και των δύο φύλων. Επιλέγουμε στη τύχη δύο

ψάρια. Αν η πιθανότητα να είναι και τα δύο του ιδίου φύλου είναι 1

2 , να δείξετε ότι

ο αριθμός των ψαριών στο ενυδρείο είναι τέλειο τετράγωνο ακεραίου.

Λύση:

Έστω t, τα αρσενικά και τα θηλυκά αντίστοιχα τότε C(t,2) C( ,2) 1

C(t,2) C( ,2) t 2

.

Μετά από πράξεις καταλήγουμε ότι 2(t ) t άρα αποδείχθηκε.

Αφού t, ακέραιοι τότε οι τιμές των t, ανήκουν στο σύνολο

k(k 1) k(k 1) (k 2)(k 1)A , ,

2 2 2

.

ΘΕΜΑ 252 (Socrates)

Αν x και y είναι μη αρνητικοί ακέραιοι, τότε ο αριθμός x y είναι επίσης μη

αρνητικός ακέραιος. Η πράξη ικανοποιεί τη συνθήκη: (x y)(y z) x z (1).

Αν 23 47 0 να υπολογίσετε τον αριθμό 61 89 .

Page 244: ΠΡΟΤΕΙΝΟΜΕΝΕΣ ΑΣΚΗΣΕΙΣ ΓΙΑ ΜΑΘΗΤΙΚΟΥΣ ΔΙΑΓΩΝΙΣΜΟΥΣ_ALL

http://www.mathematica.gr/forum/viewtopic.php?f=109&t=15584

Επιμέλεια: xr.tsif Σελίδα 51

Λύση:

Από υπόθεση έχουμε: 61 89 μη αρνητικός ακέραιος.

Αν y z , τότε

(1) (x z)(z z) (x z) x z 0   ή z z 1

το πρώτο απορρίπτεται αφού 23 47 0 ,άρα z z 1 .

Αν z x , τότε

 (1) (x y)(y x) 1 (61 89)(89 61) 1 με 61 89 0 , 89 61 0 μη

αρνητικοί ακέραιοι με γινόμενο ίσο 1 ,άρα 61 89 1 .

ΘΕΜΑ 253 (Socrates)

Ένας αριθμός ατόμων παίζουν το παρακάτω παιχνίδι: Κάθε παίκτης έχει αρχικά

300 €. Στην αρχή κάθε γύρου, ο κάθε παίκτης δίνει 10 € στη "μάνα". Τα χρήματα

αυτά δεν επιστρέφουν ξανά στο παιχνίδι. Στο τέλος κάθε γύρου, κάποιος παίκτης

(π.χ. ο χαμένος ) μοιράζει τα χρήματά του στους υπόλοιπους παίκτες σε ίσα ποσά και

αποχωρεί από το παιχνίδι. Το παιχνίδι τελειώνει όταν μείνει μόνο ένας παίκτης, ο

οποίος είναι και ο νικητής. Αν ο νικητής έχει, στο τέλος του παιχνιδιού, τόσα

χρήματα όσα ξεκίνησε, δηλαδή 300 €, να βρείτε τον αρχικό αριθμό των παικτών.

Λύση:

Έστω n οι παίκτες, οπότε παίχτηκαν n 1 γύροι. Όλα τα λεφτά ήταν (υπήρχαν ...)

300 n τα οποία είναι ίσα με αυτά που κράτησε η μάνα, και έφυγαν από το παιχνίδι,

συν τα 300 του νικητή:

n·10 (n 1)10 (n 2)10 ... 2·10 300 n·300

2n(n 1)10 290 300n n 59n 58 0 n 1,n 58

2

, κ.λπ.

Page 245: ΠΡΟΤΕΙΝΟΜΕΝΕΣ ΑΣΚΗΣΕΙΣ ΓΙΑ ΜΑΘΗΤΙΚΟΥΣ ΔΙΑΓΩΝΙΣΜΟΥΣ_ALL

http://www.mathematica.gr/forum/viewtopic.php?f=109&t=15584

Επιμέλεια: xr.tsif Σελίδα 52

ΘΕΜΑ 254 (ΔΗΜΗΤΡΗΣ ΙΩΑΝΝΟΥ )

Να αποδείξετε ότι αν pπρώτος, με p 3 , τότε ο αριθμός 2p 1 διαιρείται με το 24 .

Λύση:

Έχουμε ότι 2p 1 (p 1)(p 1) . Επειδή οι αριθμοί (p 1),(p 1) είναι δύο

διαδοχικοί άρτιοι (αφού p 2 ), έπεται εύκολα ότι 8 / (p 1)(p 1) .

Επίσης, οι (p 1),p,(p 1) είναι 3 διαδοχικοί φυσικοί αριθμοί. Κάποιος λοιπόν από

αυτούς θα είναι πολλαπλάσιο του 3 . Επειδή p πρώτος με p 3 , κάποιος από τους

(p 1),(p 1) θα είναι πολλαπλάσιο του 3 .

Άρα, 3 / (p 1)(p 1) .

Οπότε, αφού (3,8) 1 , έχουμε ότι ο αριθμός 3 8 / (p 1)(p 1) .

Β τρόπος

μπορούμε να δείξουμε το ζητούμενο για κάθε p περιττό και μη διαιρετό από το 3

(και όχι απαραίτητα πρώτο):

Είναι 2 2p 1(mod8) p 1 0(mod8)   (1) , αφού p περιττός, και

2 2p 1(mod3) p 1 0(mod3) (2) ,αφού p 0(mod3)

Το ζητούμενο έπεται απο τις (1) και (2).

Γ τρόπος

Επειδή ο pείναι πρώτος, θα έχει την μορφή p 6k 1 ή p 6k 5

1η ΠΕΡΙΠΤΩΣΗ: 2p 6k 1 p 1 12k(3k 1)

* Αν k άρτιος, δηλ. k 2n τότε 2p 1 24n(6n 1) και άρα έχουμε το ζητούμενο.

Page 246: ΠΡΟΤΕΙΝΟΜΕΝΕΣ ΑΣΚΗΣΕΙΣ ΓΙΑ ΜΑΘΗΤΙΚΟΥΣ ΔΙΑΓΩΝΙΣΜΟΥΣ_ALL

http://www.mathematica.gr/forum/viewtopic.php?f=109&t=15584

Επιμέλεια: xr.tsif Σελίδα 53

* Αν k περιττός, δηλ. k 2n 1 τότε 2

p 1 12(2n 1)(6n 4) 24(2n 1)(3n 2) οπότε και πάλι έχουμε το ζητούμενο.

2η ΠΕΡΙΠΤΩΣΗ: p 6k 5

Εργαζόμαστε ομοίως.

ΘΕΜΑ 255 (ΔΗΜΗΤΡΗΣ ΙΩΑΝΝΟΥ )

Στο επίπεδο θεωρούμε ένα 2011– γωνο. Να εξετάσετε αν είναι δυνατόν να φέρουμε

μια ευθεία πάνω σε αυτό το επίπεδο που να τέμνει όλες τις πλευρές του.

Λύση:

Ας υποθέσουμε ότι υπάρχει τέτοια ευθεία.

Ονομάζουμε το πολύγωνο 1 2 2011

A A ...A . Τα σημεία 1

A και 2

A θα ανήκουν σε

διαφορετικά ημιεπίπεδα ως προς την ευθεία. Το 3

A όπως επίσης και όλα τα σημεία

με περιττό δείκτη θα ανήκουν στο ίδιο ημιεπίπεδο. Όμως το τμήμα 1 2011

A A είναι

πλευρά του πολυγώνου, άρα τα 1 2011

A ,A ανήκουν σε διαφορετικά ημιεπίπεδα ως

προς την ευθεία, άτοπο.

ΘΕΜΑ 256 (ΔΗΜΗΤΡΗΣ ΙΩΑΝΝΟΥ )

Αν για τους πραγματικούς αριθμούς a,b,c ισχύει:

2 2ab ac b ac c ab και οι ρίζες της εξίσωσης

2ax bx c 0 είναι

πραγματικές, τότε να αποδείξετε μόνο η μία ρίζα της εξίσωσης θα περιέχεται μεταξύ

0 και 2 (δηλαδή μόνο μια ρίζα της θα ανήκει στο διάστημα (0,2) .

Λύση:

Λόγω της ιδιότητας | x | | y | | x y |, ισχύει

Page 247: ΠΡΟΤΕΙΝΟΜΕΝΕΣ ΑΣΚΗΣΕΙΣ ΓΙΑ ΜΑΘΗΤΙΚΟΥΣ ΔΙΑΓΩΝΙΣΜΟΥΣ_ALL

http://www.mathematica.gr/forum/viewtopic.php?f=109&t=15584

Επιμέλεια: xr.tsif Σελίδα 54

2 2 2 2| a || b c | | b ac | | c ab | | b c ab ac | | b c || a b c |, οπότε είναι

| a | | a b c | , άρα b c

1 1a a

| | .

Από τους τύπους Vieta, αυτή γράφεται ως 1 2 1 2

|1 (x x ) x x | 1 , δηλαδή

1 2| x 1 || x 1 | 1 . Τότε, μία τουλάχιστον από τις παρενθέσεις είναι 1

Ας είναι π.χ. 1

| x 1 | 1 , από εδώ βρίσκουμε 1

x (0,2) .

Θα αποδείξουμε τώρα, ότι η άλλη ρίζα πρέπει να είναι μη θετική.

Ας είναι 1 2

as x x

b και

1 2

cp x x

a . Αν ήταν

2x 0 , θα είχαμε s,p 0 . Με

χρήση των s,p η αρχική συνθήκη γράφεται ως

2 2 2 2 2 2 2 2| a s a p | | a s a p | | a p a s | , δηλαδή 2 2

| s p | | s p | | p s | . Λόγω της

υπόθεσης και της ανισότητας 2s 4p p , όλα μέσα στις απόλυτες τιμές είναι θετικά,

οπότε λαμβάνουμε

2 2 2 2 2 2s p s p p s 2p s p 2p 4p p p 2p 0 , άτοπο.

Β τρόπος

Αρκεί να δειχτεί ότι: f (0)f (2) 0 c(4a 2b c) 0 (1).

Μπορούμε να υποθέσουμε a 0 , διαφορετικά αλλάζουμε τα πρόσημα κ.λπ. Είναι 2

b 4ac , οπότε και 2

b ac 0 , αφού 2 2

b ac 0 b ac 4ac , άτοπο.

Είναι 2 2 2 2

ab ac b ac c ab a b c b ac c ab

a b c (b c)(a b c) a a b c a

2a b c 0, (2) ( 2a b c 0)

και 2 2 2 2

ab ac b ac ab ac b ac ab ac b ac ab a c b ac

Page 248: ΠΡΟΤΕΙΝΟΜΕΝΕΣ ΑΣΚΗΣΕΙΣ ΓΙΑ ΜΑΘΗΤΙΚΟΥΣ ΔΙΑΓΩΝΙΣΜΟΥΣ_ALL

http://www.mathematica.gr/forum/viewtopic.php?f=109&t=15584

Επιμέλεια: xr.tsif Σελίδα 55

2 2ab b , (3) b ab 2ac, (4) .

Αν αληθεύει η (3) , τότε b 0 a b (2) c 0 και η (1) αληθεύει, αφού

c(2a b (2a b c)) 0 .

Αν αληθεύει η (4) , τότε ab ac 0 b c , και

2b ab 2ac 4ac ab 2ac 2c b 0 c 0 ,

και η (1) αληθεύει, αφού c(2(2a b c) c) 0 .

ΘΕΜΑ 257 (Cretanman )

Να αποδείξετε ότι ο αριθμός 2n n 1 δεν διαιρείται από το 25 .

Λύση:

Αν συνέβαινε αυτό τότε το 100 θα έπρεπε να διαιρεί τον 2 24n 4n 4 (2n 1) 3 ,

που σημαίνει ότι ο αριθμός 2 24n 4n 4 (2n 1) 3 λήγει σε δύο μηδενικά.

Εξετάζουμε λοιπόν το τελευταίο ψηφίο του αριθμού 2 24n 4n 4 (2n 1) 3 .

Κάθε περιττός στο τετράγωνο έχει τελευταίο ψηφίο 1,5,9 αν τώρα προσθέσουμε 3

προκύπτει ότι το τελευταίο ψηφίο του 2 24n 4n 4 (2n 1) 3 θα είναι 4 ή 8 ή

2 άρα όχι 0 .

Οπότε το 25 δεν διαιρεί τον αριθμό .

Β τρόπος

Θα αποδείξουμε, ότι ο αριθμός 2

n n 1 δεν διαιρείται με το 5 .

Ας είναι n 5k u , με u 0,1,2,3,4 . Τότε, 2 2 2

n n 1 25k 10ku 5k u u 1 .

Απομένει να διαπιστώσουμε ότι το 2

u u 1 δεν διαιρείται με το 5 .

Page 249: ΠΡΟΤΕΙΝΟΜΕΝΕΣ ΑΣΚΗΣΕΙΣ ΓΙΑ ΜΑΘΗΤΙΚΟΥΣ ΔΙΑΓΩΝΙΣΜΟΥΣ_ALL

http://www.mathematica.gr/forum/viewtopic.php?f=109&t=15584

Επιμέλεια: xr.tsif Σελίδα 56

Πράγματι, έχουμε 2

1,u 0

3,u 1

u u 1 7,u 2

13,u 3

21,u 4.

.

ΘΕΜΑ 258 (ΔΗΜΗΤΡΗΣ ΙΩΑΝΝΟΥ )

Θεωρούμε ένα κυρτό 100 – γωνο 1 2 100

A A ...A . Φέρνουμε την διαγώνιο 42 81

A A που το

χωρίζει σε δύο κυρτά πολύγωνα A,B . Πόσες κορυφές και πόσες διαγώνιες έχει

καθένα από τα δύο αυτά πολύγωνα;

Λύση:

Το πολύγωνο A θα έχει 42 20 62 κορυφές και το πολύγωνο B θα έχει

81 42 1 40 κορυφές.

Οι διαγώνιοι σε ένα πολύγωνο με n κορυφές ενώνουν κάθε κορυφή με όλες τις

υπόλοιπες κορυφές εκτός αυτές που βρίσκονται εκατέρωθεν της κορυφής. Από κάθε

κορυφή ενός πολυγώνου φέρνουμε n 3 ευθύγραμμα τμήματα (διαγώνιες).

Άρα το συνολικό πλήθος των διαγωνίων είναι n(n 3)

2

, γιατί έχουμε μετρήσει δύο

φορές την καθεμία (την ΑΓ και την ΓΑ ).

Έτσι το A έχει 62(62 3) 62 59

31 59 18292 2

διαγώνιες και το B

40(40 3) 40 3720 37 740

2 2

διαγώνιες.

Page 250: ΠΡΟΤΕΙΝΟΜΕΝΕΣ ΑΣΚΗΣΕΙΣ ΓΙΑ ΜΑΘΗΤΙΚΟΥΣ ΔΙΑΓΩΝΙΣΜΟΥΣ_ALL

http://www.mathematica.gr/forum/viewtopic.php?f=109&t=15584

Επιμέλεια: xr.tsif Σελίδα 57

ΘΕΜΑ 259 (ΔΗΜΗΤΡΗΣ ΙΩΑΝΝΟΥ )

Να λυθεί η εξίσωση: x x 2 327 27 1 3cos x

.

(ΣΗΜΕΙΩΣΗ: Με το σύμβολοcosx συμβολίζουμε το συνx ).

Λύση:

Πράγματι έχουμε:

Από την εξίσωση και για xt 27 ισοδύναμα παίρνουμε:

2 2 3 2 3t 1 3cos x t 1 0, Δ 0 ... cos x 0 x 0...

Β τρόπος

Το α' μέλος είναι 3 , ενώ το β' 3 .....

x x27 27 1 3

2 33 3cos x

Άρα : x x27 27 1 3

και παίρνω x 0 .

ΑΝΑΛΥΤΙΚΑ

Έχουμε ότι: 2

2 3 2 3 33cos x 3 cos x 3 cosx 3 1 3 (1).

Επίσης από την ταυτότητα του Euler έχουμε:

x 3 x 3 3 x x(3 ) (3 ) 1 3 3 3 1

x x x x 2 x 2 x 21(3 3 1)[(3 3 ) (3 1) (3 1) ] 0

2

x x x x27 27 1 3 0 27 27 1 3

(2).

Από τις σχέσεις (1) και (2) έχουμε ότι:

x x27 27 1 3

(*) και 2 3

3cos x 3 (**)

Page 251: ΠΡΟΤΕΙΝΟΜΕΝΕΣ ΑΣΚΗΣΕΙΣ ΓΙΑ ΜΑΘΗΤΙΚΟΥΣ ΔΙΑΓΩΝΙΣΜΟΥΣ_ALL

http://www.mathematica.gr/forum/viewtopic.php?f=109&t=15584

Επιμέλεια: xr.tsif Σελίδα 58

Από την (*) έχουμε ότι:

x x 2 x x 2 x

x

127 1 3 (27 ) 1 2 27 0 (27 1) 0 27 1 x 0

27 .

Εύκολα τώρα διαπιστώνουμε ότι η x 0 επαληθεύει τόσο την (**) όσο και την

δοσμένη εξίσωση και άρα είναι η λύση της.

ΘΕΜΑ 260 (ΔΗΜΗΤΡΗΣ ΙΩΑΝΝΟΥ )

Αν πολλαπλασιάσουμε δύο οποιουσδήποτε αριθμούς του συνόλου S {2,5,13}

και ύστερα αφαιρέσουμε τον αριθμό 1 , τότε ο νέος αριθμός που θα προκύψει θα

είναι τέλειο τετράγωνο. Να αποδείξετε ότι το σύνολο S δεν μπορεί να επεκταθεί με

την προσθήκη ενός άλλου ακεραίου χωρίς να παραβιασθεί η παραπάνω συνθήκη.

Λύση:

Έστω ότι υπάρχει τέτοιο q , τότε υπάρχουν ακέραιοι x,y,z ώστε

2 2 22q 1 x ,5q 1 y ,13q 1 z .

Από την πρώτη, φαίνεται ότι x περιττός. Είναι ακόμα 2 2 2x z 3y 1 (I).

Από εδώ βλέπουμε, ότι οι y,z είναι και οι δύο περιττοί ή και οι δύο άρτιοι.

Στην πρώτη περίπτωση, θέτοντας στην (Ι) x 2k 1,y 2l 1,z 2m 1 ,

βρίσκουμε 2 2 24k 4k 4m 4m 12 12 2, δηλαδή 4 / 2 , άτοπο.

Ας είναι, λοιπόν, y 2 ,z 2m .

Επειδή είναι 2 25z 13y 8 , βρίσκουμε

2 25 13m 2 . Από εδώ είναι ,m άρτιοι ή

,m περιττοί.

Αν 2p,m 2r , βρίσκουμε 2 24(5p 13r ) 2 , άτοπο.

Page 252: ΠΡΟΤΕΙΝΟΜΕΝΕΣ ΑΣΚΗΣΕΙΣ ΓΙΑ ΜΑΘΗΤΙΚΟΥΣ ΔΙΑΓΩΝΙΣΜΟΥΣ_ALL

http://www.mathematica.gr/forum/viewtopic.php?f=109&t=15584

Επιμέλεια: xr.tsif Σελίδα 59

Αν 2p 1,m 2r 1 , βρίσκουμε 2 220p 20p 52r 52r 10 , δηλαδή 4 / 10 ,

πάλι άτοπο.

Για την ιστορία, είναι το πρόβλημα 1 της διεθνούς ολυμπιάδας το 1986!

ΘΕΜΑ 261 (Socrates)

Σε ένα μαγικό τετράγωνο 4x4 το άθροισμα των αριθμών κάθε στήλης, γραμμής και

διαγωνίου είναι το ίδιο και έστω ίσο με s . Να δείξετε ότι το άθροισμα των αριθμών

στα τετράγωνα των τεσσάρων κορυφών του τετραγώνου είναι επίσης ίσο με s .

Λύση:

2(A D E F) (A B C D) (E F G H) (A a d H)

(E c b D) (B a c F) (C b d G) s s s s s s 2s .

Με παρόμοιο τρόπο δείχνουμε π.χ.

B C F G a b c d s και, εννοείται,

ισχύει το ίδιο για τα "συμμετρικά" τους

τετραγωνάκια.

Β τρόπος

Ονομάζουμε τα τετραγωνάκια ως εξής:

1 2 3 4a   a   a   a

1 2 3 4b   b b b

1 2 3 4c   c c   c

1 2 3 4d   d d d

Θέλουμε να δείξουμε ότι: 1 4 1 4

a a d d s  . Το άθροισμα όλων των αριθμών

είναι ίσο με 4s . Επίσης είναι 2 2 2 2 3 3

a b c d a ... d 2s , άρα αρκεί να

δείξουμε ότι: 1 1 4 4

b c b c s (1).

Page 253: ΠΡΟΤΕΙΝΟΜΕΝΕΣ ΑΣΚΗΣΕΙΣ ΓΙΑ ΜΑΘΗΤΙΚΟΥΣ ΔΙΑΓΩΝΙΣΜΟΥΣ_ALL

http://www.mathematica.gr/forum/viewtopic.php?f=109&t=15584

Επιμέλεια: xr.tsif Σελίδα 60

Αρκεί τώρα ,να παρατηρήσουμε ότι αφού οι 2 διαγώνιοι έχουν άθροισμα 2s θα είναι

2 3 3 4 1 1 4 4a a d d b c b c 2s , άρα αντί της (1) αρκεί να δείξουμε ότι

2 3 3 4 1 1 4 4a a d d b c b c , το οποίο ισχύει αφού είναι:

2 3 3 4 2 3 2 3a a d d 2s (b b c c ) και

1 1 4 4 2 3 2 3b c b c 2s (b b c c ) .

ΘΕΜΑ 262 (Socrates)

Να βρείτε τις ακέραιες λύσεις της εξίσωσης 11

x 2x 1 3y 4y 1 25

.

Λύση:

Επειδή ο 4y 1 είναι ισότιμος με 3mod4 , δεν είναι ποτέ τέλειο τετράγωνο. Άρα ο

4y 1 είναι άρρητος. Συνεπώς η ισότητα 11

x 2x 1 3y 4y 1 25

δείχνει

ότι και ο 2x 1 είναι άρρητος. Από αυτό και την δοθείσα εύκολα βλέπουμε ότι

ισχύει 11x 3y 2 2x 1 4y 1

5

(*). Λύνοντας το σύστημα των

εξισώσεων αυτών, ισοδύναμα 11

x 3y 2 2x 1 4y 15

, θα βρούμε

x 5,y 3 .

(*) Αυτό βασίζεται στο εξής:

Έστω p,q ρητοί και a, b άρρητες ρίζες ακεραίων με p a q b .

Υψώνοντας στο τετράγωνο την (p q) a b , εύκολα διαπιστώνουμε ότι ισχύει

(p q) ( a b) .

ΘΕΜΑ 263 (Socrates)

Αν m,n N {0,1} τέτοιοι ώστε m

6 0n

, να δείξετε ότι m 1

6n 2mn

.

Page 254: ΠΡΟΤΕΙΝΟΜΕΝΕΣ ΑΣΚΗΣΕΙΣ ΓΙΑ ΜΑΘΗΤΙΚΟΥΣ ΔΙΑΓΩΝΙΣΜΟΥΣ_ALL

http://www.mathematica.gr/forum/viewtopic.php?f=109&t=15584

Επιμέλεια: xr.tsif Σελίδα 61

Λύση:

Η ζητούμενη είναι πιο ισχυρή από τη δοσμένη ανισότητα (δηλαδή η δοσμένη

προκύπτει αμέσως από την ζητούμενη) αλλά εδώ έχουμε να κάνουμε με φυσικούς

αριθμούς και αυτό ίσως μας βοηθήσει. Για φυσικούς ισχύει το εξής ωραίο.

Αν x y τότε x y 1 που είναι πιο ισχυρό. Θα προσπαθήσουμε να το

χρησιμοποιήσουμε. Κοιτάμε την δοσμένη. Την γράφουμε m

6n

. Υψώνουμε στο

τετράγωνο (για να έχουμε μόνο φυσικούς αριθμούς) και παίρνουμε 2 26n m .

Άρα 2 26n m 1 (*). Τώρα κοιτάμε την ζητούμενη.

Την πάμε στη ισοδύναμη μορφή 2

2 2 2 2

m 1 16

n 4m n n .

Άρα βλέπουμε ότι είναι πάλι πιο ισχυρή από την (*) .

Μήπως μπορεί η (*) να βελτιωθεί κι άλλο? Αν ήταν λίγο πιο ισχυρή θα λύναμε την

άσκηση αφού ,αν προς στιγμήν ξέραμε ότι 2 26n m 2 (**) , τότε

2 2

2 2 2 2 2

m 2 m 1 16

n n 4m n n

(αυτή η τελευταία βγαίνει εύκολα με πράξεις).

Μένει λοιπόν να δείξουμε την (**). Αν κοιτάξουμε την (*) βλέπουμε ότι πρέπει να

δείξουμε ότι 2 26x y 1 για όλους τους φυσικούς x,y .

Απόδειξη του τελευταίου:

Έστω ότι 2 26x y 1 . Το πρώτο μέλος είναι πολλαπλάσιο του 6 . Θέτουμε

y 6a b με b 0,1,2,3,4,5 .

Τότε το δεύτερο μέλος γίνεται πολλαπλάσιο του 6 συν 1,2,5,4,5,2 δηλαδή όχι

πολλαπλάσιο του 6 , άτοπο.

Άρα δεν υπάρχουν τέτοιοι φυσικοί. Άρα ισχύει η (**) που λύνει την άσκηση όπως

είδαμε.

Page 255: ΠΡΟΤΕΙΝΟΜΕΝΕΣ ΑΣΚΗΣΕΙΣ ΓΙΑ ΜΑΘΗΤΙΚΟΥΣ ΔΙΑΓΩΝΙΣΜΟΥΣ_ALL

http://www.mathematica.gr/forum/viewtopic.php?f=109&t=15584

Επιμέλεια: xr.tsif Σελίδα 62

Β τρόπος

Νομίζω βρήκα μία λύση με CS.

Αρχικά έχουμε ότι n m 1 και m 6n .

Θέλουμε να δείξουμε ότι

2

m 16

n 2mn

ή 2 2 2 224m n (2m 1) .

Από CS έχουμε ότι 2 2 3(2m 1) (m 1)(4m 1) . Ισχύει

4m 4 6n 4m 1 4 6n και 3 2 2m 1 (m 1)(m m 1) 6n(m m 1) .

Άρα, έχουμε δείξει ότι 2 2 2 2 2(2m 1) 4 6n· 6n·(m m 1) 24n (m m 1)

Οπότε, αρκεί να δείξουμε ότι 2 2 2 224n (m m 1) 24n m ή

2 2m m 1 m m 1 , που ισχύει από την υπόθεση.

ΘΕΜΑ 264 (ΔΗΜΗΤΡΗΣ ΙΩΑΝΝΟΥ )

Το υποσύνολο A των πραγματικών αριθμών, έχει τις ιδιότητες:

Z A

2 3  A  

Αν a,b  A a b   A , ab  A . Να αποδείξετε ότι 1

  A2 3

.

(όπου Z είναι το σύνολο των ακεραίων)

Λύση:

Παρατηρούμε πρώτα ότι 1

3 22 3

. Άρα αρκεί να δείξουμε ότι ο

τελευταίος αριθμός είναι στοιχείο του συνόλου A .

Τώρα 2 3  A   οπότε ( 2 3)( 2 3) A , δηλαδή 5 2 6 A .

Page 256: ΠΡΟΤΕΙΝΟΜΕΝΕΣ ΑΣΚΗΣΕΙΣ ΓΙΑ ΜΑΘΗΤΙΚΟΥΣ ΔΙΑΓΩΝΙΣΜΟΥΣ_ALL

http://www.mathematica.gr/forum/viewtopic.php?f=109&t=15584

Επιμέλεια: xr.tsif Σελίδα 63

Επίσης 5 A , αφού είναι ακέραιος. Άρα (5 2 6) ( 5) 2 6 A .

Τότε (2 6)( 2 3) 6 2 4 3 A .

Τώρα ( 5)( 2 3) 5 2 5 3 A .

Άρα (6 2 4 3) ( 5 2 5 3) 2 3 A .

Τέλος αφού 1 A έχουμε ( 1)( 2 3) 3 2 A , όπως θέλαμε.

ΘΕΜΑ 265 (ΔΗΜΗΤΡΗΣ ΙΩΑΝΝΟΥ )

Σε ένα ισοσκελές τρίγωνο ABC θεωρούμε ένα σημείο D στην βάση BC και ένα

σημείο E στην πλευρά ACέτσι ώστε να είναι γωνία B A D 2 CDE

. Να

αποδείξετε ότι το τρίγωνο ADE είναι ισοσκελές. (Για την Γ Γυμνασίου)

Λύση:

Αφού το τρίγωνο ABC είναι ισοσκελές Β C φ

.Στο τρίγωνο ADC η ω

είναι

εξωτερική , άρα ο ο

ω Α 2x φ 180 2φ 2x φ 180 φ 2x

(1).

Αλλά (1)

ο ο oβ 180 ω x β 180 180 φ 2x x φ x

και η α

είναι εξωτερική στο EDC , άρα α φ x

, δηλαδή β α

.

Page 257: ΠΡΟΤΕΙΝΟΜΕΝΕΣ ΑΣΚΗΣΕΙΣ ΓΙΑ ΜΑΘΗΤΙΚΟΥΣ ΔΙΑΓΩΝΙΣΜΟΥΣ_ALL

http://www.mathematica.gr/forum/viewtopic.php?f=109&t=15584

Επιμέλεια: xr.tsif Σελίδα 64

ΘΕΜΑ 266 (ΔΗΜΗΤΡΗΣ ΙΩΑΝΝΟΥ )

Θεωρούμε τα πολυώνυμα 4 3 2P(x) x 3x x 3,Q(x) x 2x 3 και

2R(x) x 5x a .

α) Να ορίσετε το a έτσι ώστε το πολυώνυμο R(x) να διαιρείται από το x 2 .

β) Να αναλύσετε σε γινόμενο παραγόντων τα πολυώνυμα R(x),P(x),Q(x) .

γ) Να αποδείξετε ότι η παράσταση 2 P(x)x x 15

Q(x) είναι τέλειο τετράγωνο.

(Για την Γ Γυμνασίου)

(ΣΗΜΕΙΩΣΗ: Η διαίρεση πολυωνύμου διά πολυώνυμο δεν είναι μέσα στην

διδακτέα ύλη, νομίζω όμως ότι όποιος λαβαίνει μέρος σε τέτοιου είδους

διαγωνισμούς, πρέπει να την μελετήσει.)

Λύση:

α) Πρέπει το x 2 να είναι ρίζα του R(x) . Άρα έχουμε 2

2 5 2 a 0 a 14 .

β) 4 3 3 3P(x) x 3x x 3 x (x 3) (x 3) (x 3)(x 1)

2(x 1)(x 3)(x x 1) .

2 2Q(x) x 2x 3 x 2x 2 1 (x 1)(x 1) 2(x 1) (x 1)(x 3) .

2 2R(x) x 5x 14 (x 5x 14) (x 2)(x 7) .

γ) 2

2 2P(x) (x 1)(x 3)(x x 1)x x 15 x x 15

Q(x) (x 1)(x 3)

2 2 2x x x x 1 15 16 4 .

Page 258: ΠΡΟΤΕΙΝΟΜΕΝΕΣ ΑΣΚΗΣΕΙΣ ΓΙΑ ΜΑΘΗΤΙΚΟΥΣ ΔΙΑΓΩΝΙΣΜΟΥΣ_ALL

http://www.mathematica.gr/forum/viewtopic.php?f=109&t=15584

Επιμέλεια: xr.tsif Σελίδα 65

ΘΕΜΑ 267 (ΔΗΜΗΤΡΗΣ ΙΩΑΝΝΟΥ )

α) Αν 2 2 2b c a ,b c να υπολογίσετε την παράσταση:

3 3 3 3b c b c

b c b c

.

β) Αν 1

a k,a 0,a

να βρεθεί η παράσταση: 4

4

1a

a σαν έκφραση του k .

(Για την Γ Γυμνασίου)

Λύση:

α) 3 3 3 3 3 3 3 3

2 2

b c b c (b c )(b c) (b c )(b c)

b c b c b c

2 2 2 2

2 2

[(b c)(b bc c )(b c)] [(b c)(b bc c )(b c)]

b c

2 2 2 2 2 2 2 2

2 2

(b c )(b bc c ) (b c )(b bc c )

(b c )

2 2 2 2 2 2

2 2 2 2 2

2 2

(b c )[(b bc c ) (b bc c )](b bc c b bc c ) 2a

(b c )

.

Σημείωση: χρησιμοποίησα τις ταυτότητες:

3 3 2 2a b (a b)(a ab b ) και 3 3 2 2

a b (a b)(a ab b ) .

β) 2 2 2 2 2 2

2 2

1 1 1 1a k (a ) k a 2 k a k 2

a a a a .

2 2 2 2 2 2 4 4 2

2 2 4

1 1 1a k 2 (a ) (k 2) a 2 k 4k 4

a a a

4 4 2

4

1a k 4k 2

a .

Page 259: ΠΡΟΤΕΙΝΟΜΕΝΕΣ ΑΣΚΗΣΕΙΣ ΓΙΑ ΜΑΘΗΤΙΚΟΥΣ ΔΙΑΓΩΝΙΣΜΟΥΣ_ALL

http://www.mathematica.gr/forum/viewtopic.php?f=109&t=15584

Επιμέλεια: xr.tsif Σελίδα 66

ΘΕΜΑ 268 (ΔΗΜΗΤΡΗΣ ΙΩΑΝΝΟΥ )

Σε τρίγωνο ABC τα μήκη των πλευρών του, είναι διαδοχικοί ακέραιοι και ισχύει

AB BC CA . Αν η διχοτόμος AD είναι κάθετη στην διάμεσο BE , να βρεθούν τα

μήκη των πλευρών του τριγώνου. (Για την Β Γυμνασίου)

Λύση:

Αφού τα μήκη των πλευρών του, είναι διαδοχικοί ακέραιοι και ισχύει

AB BC CA , έστω AB x , BC x 1 , AC x 2 . Στο τρίγωνο ABE η

διχοτόμος AZ είναι και ύψος , άρα το τρίγωνο είναι ισοσκελές. Άρα

AB AE EC δηλαδή x 2 2x x 2 . Άρα AB 2 , BC 3 , AC 4 .

ΘΕΜΑ 269 (ΔΗΜΗΤΡΗΣ ΙΩΑΝΝΟΥ )

Να βρείτε τους πραγματικούς αριθμούς 1 2 3 4

x ,x ,x ,x για τους οποίους ισχύουν

ταυτόχρονα οι σχέσεις 2 2 2 2

1 2 3 4x x x x 1 και 4 4 4 4

1 2 3 4x x x x 1 .

(Για την Γ Γυμνασίου)

Page 260: ΠΡΟΤΕΙΝΟΜΕΝΕΣ ΑΣΚΗΣΕΙΣ ΓΙΑ ΜΑΘΗΤΙΚΟΥΣ ΔΙΑΓΩΝΙΣΜΟΥΣ_ALL

http://www.mathematica.gr/forum/viewtopic.php?f=109&t=15584

Επιμέλεια: xr.tsif Σελίδα 67

Λύση:

2 2 2 2 2 2 2 2

1 2 3 4 1 2 3

2

4) ( )x x x x 1 [ x ]( x x 1x

2 2 2 2 2 2 2 2

1 2 1

2 2

2

4 4

3 4 4 23

2 2

1 1 2) 2 )( ) ( )(x x (x x x x x x 1 x x 2x x

4 4 2 2 2 2 2 2

3 4 3 4

2 2 2 2 2 2 2 2

1 2 3 4 1 2 3 41 2 3 42 )( ) x x 2x x 1 2x x 2 )((x x x x (x x x x ) 2x x 0 .

Πρέπει 2 2

1 2x x 0 και 2 2 2 2

1 2 3 4(x x x( 0x) ) και 2 2

3 4x x 0 και άρα

1 2 3 4x x x x 0 , Άτοπο.

Άρα δεν υπάρχουν 1 2 3 4

x ,x ,x ,x που ικανοποιούν τις σχέσεις.

ΘΕΜΑ 270 (ΔΗΜΗΤΡΗΣ ΙΩΑΝΝΟΥ )

Αν x x

x x

a ay ,a 0,a 1,x 0

a a

και αν

4x 4x

4x 4x

a az

a a

, να εκφράσετε το z σαν

συνάρτηση του y . (Για την Γ Γυμνασίου)

Λύση:

x x 2x

2y 1

x 2x 2x 2x

x x 2x

a a a 1y ya a a a

a a a 1

y 1y 1 (y 1) y 1

y 1

4

4x 4x 8x 4 44

44x 4x 8x 4 4

4

(y 1)1

a a a 1 (y 1) (y 1)(y 1)z

(y 1)a a a 1 (y 1) (y 1)1

(y 1)

.

Page 261: ΠΡΟΤΕΙΝΟΜΕΝΕΣ ΑΣΚΗΣΕΙΣ ΓΙΑ ΜΑΘΗΤΙΚΟΥΣ ΔΙΑΓΩΝΙΣΜΟΥΣ_ALL

http://www.mathematica.gr/forum/viewtopic.php?f=109&t=15584

Επιμέλεια: xr.tsif Σελίδα 68

ΘΕΜΑ 271 (ΔΗΜΗΤΡΗΣ ΙΩΑΝΝΟΥ )

Ποιο είναι το τελευταίο ψηφίο του αριθμού: 999999999A 99999 .

(Για την Β και την Γ Γυμνασίου)

Λύση:

Θέλουμε να βρούμε το τελευταίο ψηφίο του αριθμού 999999999A 99999

Επειδή ο αριθμός 99 είναι περιττός, θα έχουμε ότι 99 2k 1 με k φυσικό αριθμό.

Τότε 99 2k 1 2k 2 k999 999 999 999 (999 ) 999

.

Όμως ο αριθμός 2999 λήγει σε 1 . Άρα και ο αριθμός 2(999 )k θα λήγει και αυτός σε

1 και συνεπώς ο 2 k(999 ) 999 θα λήγει σε 9 . Άρα θα είναι περιττός και θα έχει την

μορφή 2n 1 με n φυσικό αριθμό.

Άρα θα έχουμε: 99999 2n 1 2 n

9999 9999 (9999 ) 9999 . Βλέπουμε ότι και αυτός ο

αριθμός λήγει σε 9 και άρα είναι περιττός. Άρα θα γράφεται στην μορφή 2m 1 με

m φυσικό αριθμό.

Άρα θα έχουμε: 2m 1 2 mA 99999 (99999 ) 99999

.

Και από εδώ φαίνεται ότι ο αριθμός αυτός λήγει σε9 .

ΘΕΜΑ 272 (ΔΗΜΗΤΡΗΣ ΙΩΑΝΝΟΥ )

Να εξετάσετε αν ο αριθμός 1968 78

7 1968 3 68A

2011 2001

είναι ακέραιος.

(Για την Β και την Γ Γυμνασίου)

Page 262: ΠΡΟΤΕΙΝΟΜΕΝΕΣ ΑΣΚΗΣΕΙΣ ΓΙΑ ΜΑΘΗΤΙΚΟΥΣ ΔΙΑΓΩΝΙΣΜΟΥΣ_ALL

http://www.mathematica.gr/forum/viewtopic.php?f=109&t=15584

Επιμέλεια: xr.tsif Σελίδα 69

Λύση:

Παρατηρούμε ότι 1968 78 1968 78

7 1968 3 68 7 1968 3 68A

2011 2001 10

.

Αρκεί να βρούμε το τελευταίο ψηφίο του 1968 787 1968 3 68 .

Οι δυνάμεις του 8 τελειώνουν σε: 8,4,2,6 και επαναλαμβάνονται ανά 4 .

Άρα 1968:4 492 και 1968 4 4921968 (1968 ) , το 4

1968 τελειώνει σε 6 και όλες οι

δυνάμεις του 6 τελειώνουν και αυτές σε 6 . Άρα το 19687 1968 θα τελειώνει σε 2 .

Όμοια το 4 19 2 47 18 968 36 (68 )68

. Το 468 τελειώνει σε 6 και όλες οι δυνάμεις του

6 τελειώνουν και αυτές σε 6 , άρα το 7868 τελειώνει σε 6 και το 78

3 68 τελειώνει

σε 8 . Άρα 1968 787 1968 3 68 τελειώνει σε 4 που δεν διαιρείται με το 10 .

Άρα ο αριθμός A δεν είναι ακέραιος.

ΘΕΜΑ 273 (ΔΗΜΗΤΡΗΣ ΙΩΑΝΝΟΥ )

Ισόπλευρο τρίγωνο και τετράγωνο έχουν την ίδια περίμετρο. Μπορεί να έχουν και το

ίδιο εμβαδόν; (Για την Β Γυμνασίου)

Λύση:

Έστω a η πλευρά του τριγώνου και x η πλευρά του τετραγώνου. Τα δύο σχήματα

έχουν την ίδια περίμετρο κι έστω ότι έχουν και το ίδιο εμβαδόν. Τότε το σύστημα

2

2

3a 4x

a 3x

4

πρέπει να έχει λύση στο R . Έτσι έχουμε

Page 263: ΠΡΟΤΕΙΝΟΜΕΝΕΣ ΑΣΚΗΣΕΙΣ ΓΙΑ ΜΑΘΗΤΙΚΟΥΣ ΔΙΑΓΩΝΙΣΜΟΥΣ_ALL

http://www.mathematica.gr/forum/viewtopic.php?f=109&t=15584

Επιμέλεια: xr.tsif Σελίδα 70

222 222

22

3a3a 3a3a 4x xx x

44 4a 0a 3

9aa 3 9aa 3 3axa 3( )4

44 164 4

. Άτοπο

Άρα δεν μπορεί να έχουν και το ίδιο εμβαδόν.

ΘΕΜΑ 274 (ΔΗΜΗΤΡΗΣ ΙΩΑΝΝΟΥ )

Αν 1 1 1 1

a b c k,a b c k

τότε ένας τουλάχιστον από τους a,b,c θα είναι

ίσος με k , (όπου οι αριθμοί a,b,c,k 0 ). (Για την Γ Γυμνασίου)

Λύση:

a b c kab bc ac 1

(ab bc ac)(a b c) abc1 1 1 1abc a b c

a b c k

2 2 2 2 2 2a b abc a c ab b c abc abc bc ac abc

2 2 2 2 2 2a b abc a c ab b c abc bc ac 0

2a (b c) ab(b c) bc(b c) ac(b c) 0

2(b c)(a ab bc ac) 0 (a c)(a b)(b c) 0

a c 0 a b 0 b c 0 . Άρα b k c k a k .

ΘΕΜΑ 275 (ΔΗΜΗΤΡΗΣ ΙΩΑΝΝΟΥ )

Να βρείτε την ελάχιστη και την μέγιστη τιμή της παράστασης: A 6x 8y αν

γνωρίζουμε ότι 2 2x y 1, x,y R . (Για την Γ Γυμνασίου)

Page 264: ΠΡΟΤΕΙΝΟΜΕΝΕΣ ΑΣΚΗΣΕΙΣ ΓΙΑ ΜΑΘΗΤΙΚΟΥΣ ΔΙΑΓΩΝΙΣΜΟΥΣ_ALL

http://www.mathematica.gr/forum/viewtopic.php?f=109&t=15584

Επιμέλεια: xr.tsif Σελίδα 71

Λύση:

2 2A 6x 8y 8y A 6x (8y A) 36x (1).

(1)362 2 2 2 2 2

x y 1 36x 36y 36 (8y A) 36y 36

2 2100y 16Ay A 36 0 , για κάθε y R .

Άρα η εξίσωση έχει λύση για κάθε y R και άρα

2 2Δ 0 144A 14400 0 A 100 A 10 10 A 10 .

Άρα η ελάχιστη τιμή της παράστασης είναι το 10 και η μέγιστη τιμή το 10 .

ΘΕΜΑ 276 (ΔΗΜΗΤΡΗΣ ΙΩΑΝΝΟΥ )

Να αποδείξετε ότι ο αριθμός n n n156 12 13 1 διαιρείται με το 132 για κάθε n

φυσικό αριθμό. (Για την Γ Γυμνασίου)

Λύση:

Το 156 αναλύεται σε γινόμενο παραγόντων: 156 2 2 3 13 12 13 .Έτσι

n n n n n n n n n n156 12 13 1 (12 13) 12 13 1 12 13 12 13 1

n n n n n12 (13 1) (13 1) (12 1) (13 1) πολ12 πολ11

πολ(12 11) πολ132 , για κάθε n φυσικό αριθμό.

ΘΕΜΑ 277 (ΔΗΜΗΤΡΗΣ ΙΩΑΝΝΟΥ )

Αν ο ακέραιος a γράφεται ως άθροισμα τετραγώνων δύο ακεραίων, τότε και ο

αριθμός 10a γράφεται επίσης ως άθροισμα τετραγώνων δύο ακεραίων.

(Για την Γ Γυμνασίου)

Page 265: ΠΡΟΤΕΙΝΟΜΕΝΕΣ ΑΣΚΗΣΕΙΣ ΓΙΑ ΜΑΘΗΤΙΚΟΥΣ ΔΙΑΓΩΝΙΣΜΟΥΣ_ALL

http://www.mathematica.gr/forum/viewtopic.php?f=109&t=15584

Επιμέλεια: xr.tsif Σελίδα 72

Λύση:

Έστω 2 2a x y , το 2 2

10 3 1 άρα από την ταυτότητα του Lagrange έχουμε:

2 2 2 2 2 2 2 210a (x y )(3 1 ) (3x 1y) (1x 3y) (3x y) (x 3y) .

ΘΕΜΑ 278 (ΔΗΜΗΤΡΗΣ ΙΩΑΝΝΟΥ )

Αν οι πραγματικοί αριθμοί x,y είναι καθένας τους άθροισμα δύο τετραγώνων, τότε

και ο xy είναι επίσης άθροισμα δύο τετραγώνων. (Για την Γ Γυμνασίου)

Λύση:

Έστω 2 2x a b και 2 2

y c d . Από την ταυτότητα του Lagrange έχουμε:

2 2 2 2 2 2xy (a b )(c d ) (ac bd) (ad bc) .

ΘΕΜΑ 279 (ΔΗΜΗΤΡΗΣ ΙΩΑΝΝΟΥ )

Να αποδείξετε ότι η παράσταση n 2 n 2 n nA 3 2 3 2

διαιρείται με το 10 για

κάθε n θετικό ακέραιο. (Για την Β Γυμνασίου)

Λύση:

n 2 n 2 n n n 2 n 2 n n n nA 3 2 3 2 3 (3 1) 2 (2 1) 3 ·10 2 5 2·5·3 2 ·5

n n 1 n n 12·5(3 2 ) 10(3 2 )

.

ΘΕΜΑ 280 (ΔΗΜΗΤΡΗΣ ΙΩΑΝΝΟΥ )

Μπορεί το άθροισμα πέντε διαδοχικών φυσικών αριθμών να είναι πρώτος;

Το ίδιο να εξετάσετε και για το άθροισμα των τετραγώνων πέντε διαδοχικών

φυσικών αριθμών. (Για την Γ Γυμνασίου)

Page 266: ΠΡΟΤΕΙΝΟΜΕΝΕΣ ΑΣΚΗΣΕΙΣ ΓΙΑ ΜΑΘΗΤΙΚΟΥΣ ΔΙΑΓΩΝΙΣΜΟΥΣ_ALL

http://www.mathematica.gr/forum/viewtopic.php?f=109&t=15584

Επιμέλεια: xr.tsif Σελίδα 73

Λύση:

Έστω a , a 1 , a 2 , a 3 , a 4 οι αριθμοί με a N . Τότε

a a 1 a 2 a 3 a 4 5a 10 5(a 2) . Για να είναι το άθροισμα πρώτος

πρέπει a 2 1 a 1 , άτοπο.

Όμοια για το άθροισμα τετραγώνων,

2 2 2 2 2 2 2a (a 1) (a 2) (a 3) (a 4) 5a 20a 30 5(a 4a 4) .

Για να είναι το άθροισμα πρώτος πρέπει 2 2

a 4a 4 1 a 4a 3 0 a 1 a 3 , άτοπο.

ΘΕΜΑ 281 (ΔΗΜΗΤΡΗΣ ΙΩΑΝΝΟΥ )

Να αποδείξετε ότι δεν υπάρχουν ακέραιοι a,b,c,d τέτοιοι ώστε να ικανοποιούν όλες

τις παρακάτω ισότητες:

abcd a 111...1 (το πλήθος των άσων είναι 2011)

abcd b 111...1 (το πλήθος των άσων είναι 2011)

abcd c 111...1 (το πλήθος των άσων είναι 2011)

abcd d 111...1 (το πλήθος των άσων είναι 2011)

(Για την Β και την Γ Γυμνασίου)

Λύση:

Έστω ότι υπάρχουν ακέραιοι a,b,c,d τέτοιοι ώστε να ικανοποιούν όλες τις

παραπάνω ισότητες. Τότε έχουμε:

( )abcd a 111...1

abcd b 111.b a

..1

(1) και

( )abcd c 111...1

abcd d 111.c d

..1

(2) .

Page 267: ΠΡΟΤΕΙΝΟΜΕΝΕΣ ΑΣΚΗΣΕΙΣ ΓΙΑ ΜΑΘΗΤΙΚΟΥΣ ΔΙΑΓΩΝΙΣΜΟΥΣ_ALL

http://www.mathematica.gr/forum/viewtopic.php?f=109&t=15584

Επιμέλεια: xr.tsif Σελίδα 74

Από (1) και (2) προκύπτει 2 2 2a c a 111...1 a(ac 1) 111...1 (2011 άσοι) άρα

a 1 και 2 2ac 1 111...1 c 1 111...1 (c 1)(c 1) 111...1

c 1 1 απορρίπτεται και c 1 111...1 c 111...10 (2010 άσοι)

Τότε 2 2 2 2a c a 111...1 (11...10) 1 11...1 (11...10) 11...12 Άτοπο.

ΘΕΜΑ 282 (ΔΗΜΗΤΡΗΣ ΙΩΑΝΝΟΥ )

Αν 1 2 n

a ,a ,...,a είναι θετικοί αριθμοί και έχουν γινόμενο ίσο με μονάδα, να

αποδείξετε ότι n

1 2 n(1 a )(1 a )...(1 a ) 2 . (Για την Γ Γυμνασίου)

Λύση:

Από την ανισότητα x y 2 xy έχουμε

n n

1 2 n 1 2 n 1 2 n(1 a )(1 a )...(1 a ) 2 a 2 a ... 2 a 2 a a a 2 .

ΘΕΜΑ 283 (ΔΗΜΗΤΡΗΣ ΙΩΑΝΝΟΥ )

Οι εξωτερικές γωνίες ενός τριγώνου ABC είναι ανάλογες προς τους αριθμούς 2,3,4 .

Να υπολογισθούν οι εσωτερικές γωνίες του. (Για την Β και Γ Γυμνασίου)

Λύση:

Οι εξωτερικές γωνίες του τριγώνου είναι οι o o o

180 A , 180 B , 180 C

.

Ισχύει

o o o o o o o

o180 A 180 B 180 C 180 A 180 B 180 C 36040

2 3 4 2 3 4 9

.

Άρα o o o

A 100 ,B 60 ,C 20 .

Page 268: ΠΡΟΤΕΙΝΟΜΕΝΕΣ ΑΣΚΗΣΕΙΣ ΓΙΑ ΜΑΘΗΤΙΚΟΥΣ ΔΙΑΓΩΝΙΣΜΟΥΣ_ALL

http://www.mathematica.gr/forum/viewtopic.php?f=109&t=15584

Επιμέλεια: xr.tsif Σελίδα 75

ΘΕΜΑ 284 (ΔΗΜΗΤΡΗΣ ΙΩΑΝΝΟΥ )

Να βρεθούν τα δύο τελευταία ψηφία του αριθμού 702 .

(Για την Β και Γ Γυμνασίου)

Λύση:

Είναι 92 512 12mod100 , άρα 63 7

2 12 mod100 και επομένως

70 72 24 mod100 (I).

Όμως, 224 576 24mod100 άρα 7

24 24mod100 (II).

Από (Ι) και (ΙΙ) προκύπτει ότι τα δύο τελευταία ψηφία του 702 είναι 24 .

Β τρόπος

Έχουμε 102 1025 1 25k 1 για κάποιο ακέραιο k . Άρα

70 7 72 (25k 1) 25k ( 1) 25 1 για κάποιο ακέραιο .

Πρέπει να υπάρχει ακέραιος r ώστε 4r 1 ή 4r 2 ή 4r 3 ή 4r 4

οπότε παίρνουμε 702 100r 24 ή 70

2 100r 49 ή 702 100r 74 ή

702 100r 99 . Όμως ο 70

2 είναι πολλαπλάσιο του 4 και από τους πιο πάνω

αριθμούς μόνο ο 100r 24 είναι πολλαπλάσιο του 4 . Επομένως ο 70

2 λήγει σε 24 .

Γ τρόπος

Ας βρούμε το τελευταίο ψηφίο του 70

2 . Είναι 70 17

2 16 4 . Ο αριθμός 17

16 τελειώνει

σε 6 αφού αν πολλαπλασιάσουμε δύο αριθμούς που τελειώνουν σε 6 προκύπτει

αριθμός που τελειώνει σε 6 . Άρα ο 70

2 τελειώνει σε 4 .

Ας βρούμε τώρα το προτελευταίο ψηφίο. Αφαιρούμε το 4 από το 70

2 και διαιρούμε

με το 10 . Το τελευταίο ψηφίο αυτού του αριθμού είναι το προτελευταίο ψηφίο του

702 . Έχουμε

70 34 17

16 152 4 2(4 1) 2(16 1)2·3·(16 16 ... 16 1)

10 5 5

.

Όλες οι δυνάμεις του 16 τελειώνουν σε 6 .

Page 269: ΠΡΟΤΕΙΝΟΜΕΝΕΣ ΑΣΚΗΣΕΙΣ ΓΙΑ ΜΑΘΗΤΙΚΟΥΣ ΔΙΑΓΩΝΙΣΜΟΥΣ_ALL

http://www.mathematica.gr/forum/viewtopic.php?f=109&t=15584

Επιμέλεια: xr.tsif Σελίδα 76

Άρα ο αριθμός 16 152·3·(16 16 ... 16 1) τελειώνει σε 2 αφού ο

2·3· 16·6 1 6·97 τελειώνει σε 2 .

Επομένως ο 702 τελειώνει σε 70 10 7 7 7 7

2 (2 ) 1024 (1000 24) 1000k 2424 .

Δ τρόπος

Έχω 70 10 7 7 7 72 (2 ) 1024 (1000 24) 1000k 24 .

Αρκεί λοιπόν να βρούμε τα δύο τελευταία ψηφία του 724 .

Έχουμε

7 7 6 5 4 3 224 24 24 24 24(24 1) 24 24(24 1)(24 24 24 24 24 1) 24

24 25 m 24 6 4 25 m 24 600m 24 πολ100 24 .

Άρα ο αριθμός 724 λήγει σε 24 και άρα σε 24 λήγει και ο δοσμένος αριθμός.

Σημείωση:

Να προσθέσω ότι ο 24 είναι ο μικρότερος αριθμός που αν υψωθεί σε οποιαδήποτε

περιττή δύναμη το τελευταίο διψήφιο τμήμα του μένει αναλλοίωτο δηλαδή 24 .

Άλλοι αριθμοί είναι 25 (για κάθε δύναμη) , 49 , 51 , 75 (για κάθε δύναμη), 76 , 99 .

ΘΕΜΑ 285 (ΔΗΜΗΤΡΗΣ ΙΩΑΝΝΟΥ )

Θεωρούμε τρίγωνο ABC με AB AC . Πάνω στην ημιευθεία AC παίρνουμε

σημείο T τέτοιο ώστε AT AB και πάνω στην ημιευθεία AB παίρνουμε σημείο P

ώστε AC AP . Έστω I το σημείο τομής των ευθειών BC,TP . Να αποδείξετε ότι η

AI είναι διχοτόμος της γωνίας A

. (Για την Γ Γυμνασίου)

Λύση:

Τα τρίγωνα ABC και ATP

είναι ίσα , γιατί έχουν AT AB , AC AP και A

κοινή. Άρα P C

και B T

.

Page 270: ΠΡΟΤΕΙΝΟΜΕΝΕΣ ΑΣΚΗΣΕΙΣ ΓΙΑ ΜΑΘΗΤΙΚΟΥΣ ΔΙΑΓΩΝΙΣΜΟΥΣ_ALL

http://www.mathematica.gr/forum/viewtopic.php?f=109&t=15584

Επιμέλεια: xr.tsif Σελίδα 77

Όμοια τα τρίγωνα BPI και TCI

είναι ίσα και άρα τα API και

ACI είναι ίσα γιατί έχουν PI IC ,

AC AP και P C

. Άρα ω φ

.

ΘΕΜΑ 286 (ΔΗΜΗΤΡΗΣ ΙΩΑΝΝΟΥ )

Η αρχή του Περιστερώνα, είναι πολύ χρήσιμη σε αρκετές ασκήσεις.

Έστω 99 ακέραιοι αριθμοί 1 2 99

a ,a ,...,a . Να αποδείξετε ότι υπάρχουν ακέραιοι k,m

με 0 k m 99 , ώστε το άθροισμα k 1 k 2 λ

a a .. π 99. λa ο

.

Λύση:

Θεωρούμε τα 99 αθροίσματα 1 1 2 1 2 3 1 2 99

a ,a a ,a a a ,...,a a ... a .

Αν ένα τουλάχιστον από αυτά είναι πολλαπλάσιο του 99 , τότε έχουμε τελειώσει

Υποθέτουμε ότι κανένα από αυτά δεν διαιρείται ακριβώς με το 99 . Τότε το εκάστοτε

υπόλοιπο θα παίρνει από τις τιμές 1,2,3,...,98 . Άρα τα 99 αθροίσματα μπαίνουν σε

98 θέσεις. Από την αρχή του περιστερώνα, τουλάχιστον δύο από τα αθροίσματα

αυτά δίνουν το ίδιο υπόλοιπο. Δηλαδή υπάρχουν ακέραιοι k,m ώστε

1 2 ka a ... a 99a b και

1 2 ma a ... a 99c b όπου k m .

Αφαιρούμε κατά μέλη τις δύο αυτές ισότητες και έχουμε:

k 1 k 2 ma a ... a 99(c a)

.

Page 271: ΠΡΟΤΕΙΝΟΜΕΝΕΣ ΑΣΚΗΣΕΙΣ ΓΙΑ ΜΑΘΗΤΙΚΟΥΣ ΔΙΑΓΩΝΙΣΜΟΥΣ_ALL

http://www.mathematica.gr/forum/viewtopic.php?f=109&t=15584

Επιμέλεια: xr.tsif Σελίδα 78

ΘΕΜΑ 287 (ΔΗΜΗΤΡΗΣ ΙΩΑΝΝΟΥ )

Αν a,b θετικοί αριθμοί και m ακέραιος, να αποδείξετε ότι:

m m

m 1a b1 1 2

b a

.

Λύση:

Μόλις χθες συζητήθηκε εδώ

Β τρόπος

Με την προϋπόθεση ότι ο m είναι θετικός ακέραιος

Έχουμε ότι 2 2(kx z) (nx v) 0 για κάθε k,z,n, v 0 .

Άρα 2 2 2 2 2(k n )x 2(kz nv)x z v 0 .

Άρα πρέπει 2 2Δ 0 (kv) (zn) 2(kv)(zn) .

Με την προϋπόθεση ότι kv,zn 0 , θέτοντας στην πιο πάνω σχέση m m2 2

a bkv (1 ) , zn (1 )

b a , έχουμε:

m m mm m

2 2 21 b a b a b1 1 2 1 1 2 2

b a b a b a

mm

m 1222 2 2 2 4 2 .

ΣΗΜΕΙΩΣΗ:

Χρησιμοποιήσαμε την γνωστή ανισότητα: a b

2b a για κάθε a,b 0 .

Page 272: ΠΡΟΤΕΙΝΟΜΕΝΕΣ ΑΣΚΗΣΕΙΣ ΓΙΑ ΜΑΘΗΤΙΚΟΥΣ ΔΙΑΓΩΝΙΣΜΟΥΣ_ALL

http://www.mathematica.gr/forum/viewtopic.php?f=109&t=15584

Επιμέλεια: xr.tsif Σελίδα 79

ΘΕΜΑ 288 (socratis lyras )

Δίνονται οι θετικοί αριθμοί a,b,c με abc 1 . Να αποδείξετε ότι:

68

a b c a b b c c a729

.

Λύση:

Παίρνουμε την τρίτη ρίζα στα 2 μέλη έτσι η ανισότητα γίνεται: 1

2

1 1

3 3 32

(a b c) (a b) (b c) (c a)9

.

Από την ΑΜ – ΓΜ το δεύτερο μέλος είναι μικρότερο ή ίσο με 2

(a b c)3

.Με

αυτόν τον μετασχηματισμό και μετά τις πράξεις η ανισότητα γράφεται

1(a b c) 1

3 που αποδεικνύεται με χρήση της ΑΜ – ΓΜ πάλι.

Η ισότητα ισχύει για a b c 1 .

Β τρόπος

Και μια λύση με κατασκευαστικό τρόπο.

3 6 3

a b c a b c a b c a b b c c a1 · ·

3 3 3 2 2 2

οι παραπάνω είναι εφαρμογές της AM – GM.

ΘΕΜΑ 289 (socratis lyras )

Αν a,b,c θετικοί πραγματικοί αριθμοί με a b c 1 να αποδείξετε ότι:

2 2 21 a b c 2 3abc .

Page 273: ΠΡΟΤΕΙΝΟΜΕΝΕΣ ΑΣΚΗΣΕΙΣ ΓΙΑ ΜΑΘΗΤΙΚΟΥΣ ΔΙΑΓΩΝΙΣΜΟΥΣ_ALL

http://www.mathematica.gr/forum/viewtopic.php?f=109&t=15584

Επιμέλεια: xr.tsif Σελίδα 80

Λύση:

2 2 2 2 2 2 2(a b c) a b c 2(ab bc ca) a b c 2 3abc(a b c) ...

ΘΕΜΑ 290 (ΔΗΜΗΤΡΗΣ ΙΩΑΝΝΟΥ )

Να αποδείξετε ότι ο αριθμός 2222 11119589 6051 διαιρείται ακριβώς με το 17 .

Λύση:

Παρατηρώ ότι: 9589 17 564 1 και 6051 355 17 16 .

Άρα o 9589 είναι ισουπόλοιπος με 1mod17 .

Άρα ο 22229589 είναι ισουπόλοιπος με 1mod17 (1). Επίσης ο 6051 είναι

ισουπόλοιπος με 1mod17 .

Άρα ο 11116051 είναι ισουπόλοιπος με 1mod17 (2).

Mε πρόσθεση των (1) και (2) προκύπτει το ζητούμενο.

ΘΕΜΑ 291 (ΔΗΜΗΤΡΗΣ ΙΩΑΝΝΟΥ )

Να αποδειχθεί ότι για κάθε x N , ο αριθμός 23x 1991 είναι άρρητος.

Λύση:

Η υπόριζη ποσότητα είναι 2mod3 , άρα μη τέλειο τετράγωνο, όπως θέλαμε.

Β τρόπος

Έστω ότι ο αριθμός αυτός είναι ρητός. Τότε θα ισχύει: 2 23x 1991 y .

Από την παραπάνω σχέση παίρνω ότι ο 2y είναι ισουπόλοιπος με 2mod3 το οποίο

είναι άτοπο.

Page 274: ΠΡΟΤΕΙΝΟΜΕΝΕΣ ΑΣΚΗΣΕΙΣ ΓΙΑ ΜΑΘΗΤΙΚΟΥΣ ΔΙΑΓΩΝΙΣΜΟΥΣ_ALL

http://www.mathematica.gr/forum/viewtopic.php?f=109&t=15584

Επιμέλεια: xr.tsif Σελίδα 81

Γ τρόπος

Έστω 23x 1991 y,y N . Τότε 2 2

3x 1991 y .

Αλλά y 3k ή y 3k 1 ή y 3k 2 .

1η ΠΕΡΙΠΤΩΣΗ: y 3k .

Τότε έχουμε 2 2 2 23x 1991 9k 3(3k x ) 1991 . Άρα θα πρέπει ο 3 να διαιρεί

τον 1991 πράγμα που είναι άτοπο.

Ακριβώς με τον ίδιο τρόπο, καταλήγουμε σε άτοπο και από τις δύο άλλες

περιπτώσεις.

Άρα ο δοσμένος αριθμός είναι άρρητος.

ΘΕΜΑ 292 (ΔΗΜΗΤΡΗΣ ΙΩΑΝΝΟΥ )

Αν x R και 1 2 3 4

a ,a ,a ,a είναι διαδοχικοί ακέραιοι αριθμοί και αν

1 2 3 4(x a )(x a )(x a )(x a ) k , να αποδείξετε ότι k 1 .

(Την άσκηση αυτή, είχε προτείνει το 1989 ο Αχιλλέας Συννεφακόπουλος, μαθητής

τότε της Β Λυκείου, στο περιοδικό "ΕΥΚΛΕΙΔΗΣ Β", μαζί με άλλες αξιόλογες

ασκήσεις)

Λύση:

Οι αριθμοί στο α' μέλος είναι τέσσερις διαδοχικοί άρα αν προσθέσουμε ένα

προκύπτει τέλειο τετράγωνο, έτσι: 2y k 1 .Όπου y το τέλειο τετράγωνο που

προκύπτει. Ξέρουμε επίσης ότι τα τέλεια τετράγωνα είναι μεγαλύτερα ή ίσα του 0 ,

άρα k 1 0 , άρα k 1 .

Β τρόπος

Αν 1

a m , επειδή οι 1 2 3 4

a ,a ,a ,a είναι διαδοχικοί ακέραιοι, είναι

Page 275: ΠΡΟΤΕΙΝΟΜΕΝΕΣ ΑΣΚΗΣΕΙΣ ΓΙΑ ΜΑΘΗΤΙΚΟΥΣ ΔΙΑΓΩΝΙΣΜΟΥΣ_ALL

http://www.mathematica.gr/forum/viewtopic.php?f=109&t=15584

Επιμέλεια: xr.tsif Σελίδα 82

1 2 3 4(x a )(x a )(x a )(x a ) (x m)(x m 1)(x m 2)(x m 3)

n(n 1)(n 2)(n 3) όπου n x m .

Τότε, το ζητούμενο προκύπτει άμεσα, από την ταυτότητα

2n(n 1)(n 2)(n 3) 1 [n(n 3) 1] .

ΘΕΜΑ 293 (Socrates )

Έστω A μη κενό υποσύνολο του έτσι ώστε: αν x,y και x y A τότε

xy A . Να αποδείξετε ότι A .

Λύση:

Έστω a ένα στοιχείο του A .Τότε προφανώς για κάθε t , t(a t) A . Άρα

2 3 2f (t) t a t t t[t(a t) t] A . Αλλά επειδή η f είναι πολυώνυμο 3ου

βαθμού, Imf ,άρα A .

Β τρόπος

'Έστω a ένα στοιχείο του A . Αν 2

ax

4 και

2a a 4x

y2

, τότε y(a y) x .

Αφού y (a y) a A , από τη δοθείσα ιδιότητα έπεται ότι x A . Συνεπώς,

2a

( , ] A4

(1).

Αρκεί, λοιπόν, να δείξουμε ότι το A περιέχει όλους τους φυσικούς αριθμούς.

Πράγματι, με

Με x 1 και y a 1 , έπεται ότι a 1 1 (a 1) A (*).

Με x 1 και y a , έπεται από (*) και τη δοθείσα ιδιότητα ότι a 1 a A (**)

Page 276: ΠΡΟΤΕΙΝΟΜΕΝΕΣ ΑΣΚΗΣΕΙΣ ΓΙΑ ΜΑΘΗΤΙΚΟΥΣ ΔΙΑΓΩΝΙΣΜΟΥΣ_ALL

http://www.mathematica.gr/forum/viewtopic.php?f=109&t=15584

Επιμέλεια: xr.tsif Σελίδα 83

*****************************************************

Από την (1) και τη (**) το πρόβλημα λύθηκε, αφού έπεται ότι 2

a[ , ) A

4 που σε

συνδυασμό με την (1) μας δίνει A .

*****************************************************

Με x 1 και y a 1 , έπεται ότι a 1 1·(a 1) A , που σε συνδυασμό με

την (**) μας δίνει ότι a 1 A (2).

Αφού 0 A (π.χ. από (1)), χρησιμοποιώντας τη (2) επαγωγικά δείχνουμε εύκολα ότι

A .

Η απόδειξη ολοκληρώθηκε.

Γ τρόπος

x A x 0 A 0 A (1) ,

(x 1) 1 A x 1 A (2) ,

1 ( 1) 0 A 1 A (3) .

(2) x A , άρα από την (3) έχουμε 1 A (4).

Έστω 2 33 33 3 3

0 0 0 0 0 0 0x x x 1 1 A ( x x ) A x x A . Και η

απόδειξη τελείωσε.

ΘΕΜΑ 294 (Socrates )

Βρείτε όλους τους θετικούς ακεραίους n τέτοιους ώστε: d , d / n d 1 / n 1 .

Λύση:

Ισχύουν: n kd και n 1 (d 1) τότε d(k ) 1 (k ή ίσον όταν n d )

k a με a 0 ή ίσον όταν n d .

Page 277: ΠΡΟΤΕΙΝΟΜΕΝΕΣ ΑΣΚΗΣΕΙΣ ΓΙΑ ΜΑΘΗΤΙΚΟΥΣ ΔΙΑΓΩΝΙΣΜΟΥΣ_ALL

http://www.mathematica.gr/forum/viewtopic.php?f=109&t=15584

Επιμέλεια: xr.tsif Σελίδα 84

Τότε da 1 , k da a 1 και n d(ad a 1) .

Άρα 2(n,d) (d a da d,d) με a 0 ή ίσον όταν n d .

ΘΕΜΑ 295 (Socrates )

Να δείξετε ότι υπάρχει n N τέτοιος ώστε 1 1 1 1 n

... 201121 2 3 n

.

Λύση:

Θα δείξουμε επαγωγικά ότι 1 1 1 1

... n1 2 3 n . Αυτό αρκεί για το

ζητούμενο γιατί τότε η δοθείσα παράσταση είναι n n

n2 2

, οπότε μπορούμε

να πάρουμε 2n 4022 .

Η περίπτωση n 1 είναι άμεση.

Για το επαγωγικό βήμα, έστω 1 1 1 1

... k1 2 3 k . Τότε

1 1 1 1 1 1

... k k 11 2 3 k k 1 k 1

, διότι

1 1k 1 k

k 1 k k 1

, όπως θέλαμε.

ΘΕΜΑ 296 (Socrates )

Θεωρούμε ένα πλήθοςn 1 ατόμων. Κάθε δύο άτομα συνδέονται με αμοιβαία φιλία

ή αμοιβαία έχθρα. Κάθε φίλος φίλου και κάθε εχθρός εχθρού είναι φίλος.

Αν τα άτομα A και B είναι φίλοι – εχθροί τότε αυτό το μετράμε ως 1 φιλία – έχθρα.

Αν μεταξύ των ατόμων αυτών υπάρχει ίσος αριθμός από φιλίες και έχθρες ,να βρείτε

όλες τις δυνατές τιμές του n .

Page 278: ΠΡΟΤΕΙΝΟΜΕΝΕΣ ΑΣΚΗΣΕΙΣ ΓΙΑ ΜΑΘΗΤΙΚΟΥΣ ΔΙΑΓΩΝΙΣΜΟΥΣ_ALL

http://www.mathematica.gr/forum/viewtopic.php?f=109&t=15584

Επιμέλεια: xr.tsif Σελίδα 85

Λύση:

Έστω x τυχόν άτομο και έστω A το σύνολο όλων των φίλων του x και B το σύνολο

όλων των εχθρών του x . Από τα δεδομένα του προβλήματος κάθε δύο μέλη του

A' A {x} είναι φίλοι μεταξύ τους (φίλος φίλου = φίλος) και κάθε δυο μέλη του

B είναι φίλοι μεταξύ τους (εχθρός εχθρού = εχθρός). Τέλος αν πάρουμε ένα άτομο

y από το A' και ένα άτομο z από το B τότε αυτοί είναι εχθροί. Πράγματι αυτό

ισχύει εξ ορισμού αν y x . Αν y x τότε δεν μπορεί ο y να είναι φίλος με τον z

αφού τότε ο x ως φίλος του y θα ήταν φίλος και του z , άτοπο.

Αν λοιπόν γράψουμε a | A' | και b | B | τότε a b n , έχουμε ακριβώς a b

2 2

φιλίες και ακριβώς abέχθρες. Άρα 2 2a a b b 2ab και άρα

2 2 2n a b a b 2ab (a b) είναι τέλειο τετράγωνο.

Επίσης αν 2n m είναι τέλειο τετράγωνο, τότε μπορούμε να χωρίσουμε τα άτομα σε

δυο ομάδες μεγέθους 2

(m m)

2

και

2(m m)

2

ώστε σε κάθε ομάδα ξεχωριστά να

είναι όλοι φίλοι μεταξύ τους και μεταξύ των ομάδων να είναι όλοι εχθροί μεταξύ

τους. Αυτό ικανοποιεί τα δεδομένα του προβλήματος.

ΘΕΜΑ 297 (ΔΗΜΗΤΡΗΣ ΙΩΑΝΝΟΥ )

Μια συλλογή διηγημάτων του Α. Παπαδιαμάντη περιέχει 70 διηγήματα, ένα μιας

σελίδας, ένα δύο σελίδων,..., ένα 70 σελίδων και όχι αναγκαστικά με αυτή την

σειρά. Κάθε διήγημα αρχίζει από καινούρια σελίδα και η αρίθμηση των σελίδων του

βιβλίου αρχίζει από την πρώτη σελίδα. Ποιος είναι ο μέγιστος αριθμός διηγημάτων

που αρχίζουν από σελίδα με περιττό αριθμό; ΕΜΕ – Α Λυκείου

Λύση:

Παρατηρούμε ότι αν ένα διήγημα αρχίζει από περιττή(άρτια) σελίδα και έχει άρτιο

αριθμό σελίδων, τότε το επόμενο θα ξεκινά κι αυτό από περιττή(άρτια) σελίδα.

Page 279: ΠΡΟΤΕΙΝΟΜΕΝΕΣ ΑΣΚΗΣΕΙΣ ΓΙΑ ΜΑΘΗΤΙΚΟΥΣ ΔΙΑΓΩΝΙΣΜΟΥΣ_ALL

http://www.mathematica.gr/forum/viewtopic.php?f=109&t=15584

Επιμέλεια: xr.tsif Σελίδα 86

Αντίστοιχα, αν ένα διήγημα αρχίζει από περιττή(άρτια) σελίδα και έχει περιττό

αριθμό σελίδων, τότε το επόμενο θα ξεκινά κι αυτό από άρτια(περιττή) σελίδα.

Θα υπάρξουν, λοιπόν, 34 ή 35 αλλαγές του τύπου:

άρτιος περιττός ή περιττός άρτιος (ανάλογα αν το τελευταίο διήγημα έχει

περιττό ή άρτιο αριθμό σελίδων). Επειδή ξεκινάμε από περιττή σελίδα (1η) θα

έχουμε τουλάχιστον 17 του τύπου περιττός άρτιος και άρα τουλάχιστον 17

διηγήματα αρχίζουν από άρτια σελίδα. Συνεπώς, τα διηγήματα που αρχίζουν από

περιττή σελίδα γίνονται μέγιστα όταν δεν έχουμε καθόλου άρτιος άρτιος και τότε

είναι 35 18 53 διηγήματα.

Αυτό είναι δυνατόν, αν π.χ. τα τοποθετήσουμε με τη σειρά 2,4,6,...,70,1,3,...,69 .

ΠΑΡΑΤΗΡΗΣΕΙΣ:

Με αφορμή την άσκηση 284 , όπου δόθηκαν λύσεις και χωρίς την χρήση των mod

είναι χρήσιμα μερικά θεωρητικά στοιχεία, που παραθέτω:

Έστω a,b φυσικοί αριθμοί

(1) από την ταυτότητα n n n 1 n 1 n(a b) α nα b ... nαb b βγάζοντας κοινό

παράγοντα το a έχουμε:

n n 1 n 2 n 1 n n n(a b) a(a na b ... nb ) b (a b) πολa b

.

(2) Για κάθε θετικό ακέραιο n ισχύει ότι n n n 1 n 2 n 3 2 n 1

a b (a b)(a a b a b ... b )

Άρα έχουμε ότι για κάθε θετικό ακέραιο n : n nπολ( )a bb a .

(3) Για κάθε περιττό θετικό ακέραιο n ισχύει ότι n n n 1 n 2 n 3 2 n 1

a b (a b)(a a b a b ... b ) πoλ(a b) .

(4) Για κάθε άρτιο και θετικό ακέραιο n ισχύει ότι n n n 1 n 2 n 3 2 n 1

a b (a b)(a a b a b ... b ) πoλ(a b) .

Page 280: ΠΡΟΤΕΙΝΟΜΕΝΕΣ ΑΣΚΗΣΕΙΣ ΓΙΑ ΜΑΘΗΤΙΚΟΥΣ ΔΙΑΓΩΝΙΣΜΟΥΣ_ALL

http://www.mathematica.gr/forum/viewtopic.php?f=109&t=15584

Επιμέλεια: xr.tsif Σελίδα 87

Ας δούμε μερικά παραδείγματα:

ΠΑΡΑΔΕΙΓΜΑ 1: Να βρεθούν τα δύο τελευταία ψηφία του αριθμού nA 4 (326) .

ΛΥΣΗ

n n nA 4 (300 26) 4 (πoλ300 26 ) 4 πoλ300 4 26

n nπoλ1200 4 (25 1) πoλ100 4(πoλ25 1 )

πoλ100 πoλ100 4 πoλ100 4 .

Άρα ο δοσμένος αριθμός λήγει σε 04 .

ΠΑΡΑΔΕΙΓΜΑ 2: Να βρεθούν τα δύο τελευταία ψηφία του αριθμού 2n 1B 524 .

ΛΥΣΗ

2n 1 2n 1 2n 1B (500 24) πoλ500 24 πoλ100 24 24 24

2nπoλ100 24(24 1) 24 πoλ100 24(πoλ(24 1)) 24

πoλ100 24 πoλ25 24 πoλ100 6 4 πoλ25 24

πoλ100 πoλ100 24 πoλ100 24 .

Άρα ο δοσμένος αριθμός λήγει σε 24 .

ΠΑΡΑΔΕΙΓΜΑ 3: Να βρεθούν τα δύο τελευταία ψηφία του αριθμού 100

C 902 .

ΛΥΣΗ

100 100 10 10C (900 2) πoλ900 2 πoλ100 (2 )

10 10πoλ100 1024 πoλ100 (1000 24)

10 10πoλ100 πoλ1000 24 πoλ100 πoλ100 24 24 24

9πoλ100 24(24 1) 24 πoλ100 6 4 πoλ(24 1) 24

Page 281: ΠΡΟΤΕΙΝΟΜΕΝΕΣ ΑΣΚΗΣΕΙΣ ΓΙΑ ΜΑΘΗΤΙΚΟΥΣ ΔΙΑΓΩΝΙΣΜΟΥΣ_ALL

http://www.mathematica.gr/forum/viewtopic.php?f=109&t=15584

Επιμέλεια: xr.tsif Σελίδα 88

πoλ100 πoλ100 24 πoλ100 24

Άρα ο δοσμένος αριθμός λήγει σε 76 .

ΠΑΡΑΔΕΙΓΜΑ 4: Να βρεθούν τα δύο τελευταία ψηφία του αριθμού 1689D 6 .

ΛΥΣΗ

1689 3 663 663 663 663(6 ) 216 (200 16) πολ 200 16D 6

4 663 2652 2 2650 10 265πολ100 (2 ) πολ100 2 πολ100 2 2 πολ100 4 (2 )

265 265πολ100 4 1024 πολ100 4 (1000 24)

265 265 2 2πολ100 4 (πολ1000 24 ) πολ100 4 πολ1000 4 (24 24 24 )

2 263 2πολ100 πολ100 4 [24 (24 1) 24 ]

2 2πολ100 4 [24 24 πολ(24 1) 24 ] πολ100 πολ100 4 24

πολ100 2304 . Άρα ο δοσμένος αριθμός λήγει σε 96 .

ΘΕΜΑ 298 (Socrates)

Σε μια μαθητική κατασκήνωση παρατηρήθηκε ότι:

α) Κάθε μαθητής γνώριζε έναν τουλάχιστον άλλο μαθητή. (Θεωρούμε ότι αν ο

μαθητής A γνωρίζει τον B , τότε και ο B γνωρίζει τον A .)

β) Αν δύο μαθητές έχουν τον ίδιο αριθμό γνωστών, τότε δεν έχουν κοινό γνωστό

μαθητή.

Να δείξετε ότι ένας τουλάχιστον από τους μαθητές έχει μόνο ένα γνωστό.

Λύση:

Έστω Aο μαθητής με τον μέγιστο αριθμό γνωστών. Ας υποθέσουμε ότι έχει ακριβώς

k γνωστούς, έστω τους 1 k

B , ,B . Οι 1 k

B , ,B έχουν κοινό γνωστό οπότε πρέπει

ανά δύο να έχουν διαφορετικό αριθμό γνωστών. Όμως κάθε ένας από αυτούς έχει

τουλάχιστον 1 και το πολύ k γνωστούς. Όμως έχουμε ακριβώς k άτομα άρα από

Page 282: ΠΡΟΤΕΙΝΟΜΕΝΕΣ ΑΣΚΗΣΕΙΣ ΓΙΑ ΜΑΘΗΤΙΚΟΥΣ ΔΙΑΓΩΝΙΣΜΟΥΣ_ALL

http://www.mathematica.gr/forum/viewtopic.php?f=109&t=15584

Επιμέλεια: xr.tsif Σελίδα 89

την αρχή του περιστερώνα για κάθε 1 i k ακριβώς ένας από τους j

B έχει ακριβώς

i γνωστούς. Ιδιαίτερα, υπάρχει j ώστε ο j

B να έχει ακριβώς ένα γνωστό, όπως

θέλαμε να δείξουμε.

ΘΕΜΑ 299 (Socrates)

Μπορούμε να τοποθετήσουμε στα τετράγωνα μιας σκακιέρας 6x6 αριθμούς από το

σύνολο { 1,0,1} έτσι ώστε σε κάθε γραμμή, στήλη και διαγώνιο το άθροισμα των

αριθμών να είναι διαφορετικό;

Λύση:

Δεν μπορούμε: Το μέγιστο άθροισμα των 6 αριθμών κάθε γραμμής, στήλης ή

διαγωνίου είναι, φυσικά, 1 1 1 1 1 1 6 και το ελάχιστο είναι 6 . Δηλαδή

έχουμε το πολύ 13 δυνατά αθροίσματα, τους αριθμούς 6, 5,...,4,5,6 . Αλλά το

πλήθος των γραμμών, στηλών και διαγωνίων της 6x6 σκακιέρας είναι

6 6 2 14 13 . Άρα δεν μπορεί όλα τα αθροίσματα να είναι διαφορετικά.

ΘΕΜΑ 300 (Socrates)

Οι θετικοί ακέραιοι 1 2 n

a ,a ,...,a ,... ικανοποιούν τη σχέση n 2 n 1 n

a a a 1 για κάθε

θετικό ακέραιο n . Να δείξετε ότι αν n 5 τότε ο αριθμός n

a 7 είναι σύνθετος.

Λύση:

Έχουμε ότι k

a 1 για κάθε k 3 .

Επίσης, για κάθε k 2 έχουμε (k

moda ):

k k 1a 0, a 1

και από τον αναδρομικό τύπο βλέπουμε ότι

k 6 ka 22moda

. Άρα ο

k 6a 22

διαιρείται με τον

ka και, αφού όλοι οι αριθμοί

ka με k 3 είναι

μεγαλύτεροι του 1 , όλοι οι αριθμοί k

a 22 με k 9 είναι σύνθετοι.

Page 283: ΠΡΟΤΕΙΝΟΜΕΝΕΣ ΑΣΚΗΣΕΙΣ ΓΙΑ ΜΑΘΗΤΙΚΟΥΣ ΔΙΑΓΩΝΙΣΜΟΥΣ_ALL

http://www.mathematica.gr/forum/viewtopic.php?f=109&t=15584

Επιμέλεια: xr.tsif Σελίδα 1

Page 284: ΠΡΟΤΕΙΝΟΜΕΝΕΣ ΑΣΚΗΣΕΙΣ ΓΙΑ ΜΑΘΗΤΙΚΟΥΣ ΔΙΑΓΩΝΙΣΜΟΥΣ_ALL

http://www.mathematica.gr/forum/viewtopic.php?f=109&t=15584

Επιμέλεια: xr.tsif Σελίδα 2

ΠΡΟΤΕΙΝΟΜΕΝΕΣ

ΑΣΚΗΣΕΙΣ ΓΙΑ ΜΑΘΗΤΙΚΟΥΣ

ΔΙΑΓΩΝΙΣΜΟΥΣ

ΤΕΥΧΟΣ 4ο

ΑΣΚΗΣΕΙΣ 301 - 400

Αφιερωμένο σε κάθε μαθητή που ασχολείται ή πρόκειται να ασχοληθεί με

Μαθηματικούς διαγωνισμούς

Τσιφάκης Χρήστος : xr.tsif

Page 285: ΠΡΟΤΕΙΝΟΜΕΝΕΣ ΑΣΚΗΣΕΙΣ ΓΙΑ ΜΑΘΗΤΙΚΟΥΣ ΔΙΑΓΩΝΙΣΜΟΥΣ_ALL

http://www.mathematica.gr/forum/viewtopic.php?f=109&t=15584

Επιμέλεια: xr.tsif Σελίδα 3

Να αποδείξετε ότι η παράσταση 5 3

n 5n 4nA

n 2

διαιρείται με το 24 για κάθε

φυσικό αριθμό n .

Λύση:

Μετά τις παραγοντοποιήσεις και τις απλοποιήσεις έχουμε ότι

n(n 1)(n 1)(n 2) . Αυτοί είναι τέσσερις διαδοχικοί αριθμοί που ανάμεσά

τους θα υπάρχει ένα πολλαπλάσιο του 4 , ένας ακόμα άρτιος και τουλάχιστον

ένα πολλαπλάσιο του 3 .

Χρησιμοποιώντας αυτά τελικά ο αριθμός είναι πολλαπλάσιο του 24 .

Δηλαδή 5 3

4 3 2n 5n 4nA n 2n n 2n (n 2)(n 1)n(n 1)

n 2

(γινόμενο τεσσάρων διαδοχικών ακεραίων). Άρα κάποιος από τους τέσσερεις

αριθμούς είναι πολλαπλάσιο του 2 , άλλος του 4 και ένας του 3 . Όλοι μαζί

πολλαπλάσιο του 2·3·4 24 .

ΘΕΜΑ 302 (ΔΗΜΗΤΡΗΣ ΙΩΑΝΝΟΥ )

Να βρείτε τα δύο τελευταία ψηφία του αριθμού 99 51A 99 51 .

Λύση:

9999 1(mod100) και επίσης 50

51 1(mod100) άρα 5151 51(mod100) .

Με αφαίρεση κατά μέλη έχουμε 99 5199 51 52(mod100) .

Όμως 52 48(mod100) . Άρα ο αριθμός τελειώνει σε 48 .

Β τρόπος

Έχουμε:

ΘΕΜΑ 301 (ΔΗΜΗΤΡΗΣ ΙΩΑΝΝΟΥ )

Page 286: ΠΡΟΤΕΙΝΟΜΕΝΕΣ ΑΣΚΗΣΕΙΣ ΓΙΑ ΜΑΘΗΤΙΚΟΥΣ ΔΙΑΓΩΝΙΣΜΟΥΣ_ALL

http://www.mathematica.gr/forum/viewtopic.php?f=109&t=15584

Επιμέλεια: xr.tsif Σελίδα 4

99 51 99 51 2 2599 51 (100 1) 51 51 πoλ100 1 51 (51 )

25 25πoλ100 1 51 (2601) πoλ100 1 51 2600 1) πoλ100 1 51 (πoλ2600 1) πoλ100 1 51 πoλ100 51

πoλ100 52 .

Άρα ο δοσμένος αριθμός θα λήγει σε 48 .

ΘΕΜΑ 303 (ΔΗΜΗΤΡΗΣ ΙΩΑΝΝΟΥ )

Να αποδειχθεί ότι το άθροισμα των τετραγώνων πέντε διαδοχικών ακεραίων

δεν είναι ίσο με το τετράγωνο ακεραίου.

Λύση:

Είναι 2 2 2 2 2 2(n 2) (n 1) n (n 1) (n 2) 5n 10 πολ5 . Άρα, αν

ήταν τέλειο τετράγωνο, θα ήταν της μορφής 2(5m) , δηλαδή 2 2

5n 10 25m ,

που σημαίνει ή 2 2n 2 5m πολ5 .

Όμως οι αριθμοί 2n 2 δεν είναι ποτέ πολλαπλάσια του 5 .

(το n αφήνει υπόλοιπα 0,1,2,3,4 αντίστοιχα, αν διαιρεθεί με το 5 , οπότε το 2

n 2 αφήνει υπόλοιπα 20 2 2 ή, αντίστοιχα, 3,1,1,3 κανένα από τα οποία

δεν είναι 0 .)

ΘΕΜΑ 304 (ΔΗΜΗΤΡΗΣ ΙΩΑΝΝΟΥ )

Να αποδειχθεί ότι το τετράγωνο κάθε πρώτου αριθμού μεγαλύτερου από το 3 ,

αν διαιρεθεί με το 12 , δίνει υπόλοιπο 1 .

Λύση:

2(6k 1) 12n 1 .Το ίδιο βγαίνει αν ο πρώτος είναι της μορφής 6k 1 .

Page 287: ΠΡΟΤΕΙΝΟΜΕΝΕΣ ΑΣΚΗΣΕΙΣ ΓΙΑ ΜΑΘΗΤΙΚΟΥΣ ΔΙΑΓΩΝΙΣΜΟΥΣ_ALL

http://www.mathematica.gr/forum/viewtopic.php?f=109&t=15584

Επιμέλεια: xr.tsif Σελίδα 5

Σημείωση:

Για να γίνει κατανοητός ο τρόπος που λύθηκε η άσκηση αυτή, χρησιμοποιούμε

την πρόταση:

Κάθε πρώτος αριθμός που έχει απόλυτη τιμή μεγαλύτερη του 3 γράφεται με την

μορφή 6k 1 ή 6k 1 .

Η απόδειξη αυτής της πρότασης δεν είναι δύσκολο να γίνει:

Πράγματι, κάθε ακέραιος γράφεται με την μορφή a 6k m όπου

m 0,1,2,3,4,5 .

Αν m 0 , τότε a 6k και άρα ο a δεν είναι πρώτος.

Αν m 3 τότε a 6k 3 3(2k 1) και άρα πάλι ο a δεν είναι πρώτος.

Όμοια αν m 4 ο a δεν είναι πρώτος.

Άρα για να είναι ο a πρώτος, πρέπει m 1 οπότε a 6k 1 ή m 5 οπότε

a 6k 5 6k 6 1 6(k 1) 1 πολ6 1 6k 1 .

ΘΕΜΑ 305 (ΔΗΜΗΤΡΗΣ ΙΩΑΝΝΟΥ )

Να εξετάσετε αν υπάρχουν ακέραιοι x,y τέτοιοι ώστε 2 23x y 7 .

Λύση:

Γράφουμε την εξίσωση σε αυτή τη μορφή 2 23x y 7 . Το αριστερό μέλος

διαιρείται με το 3 ενώ το δεξί όχι, γιατί τα τετραγωνικά υπόλοιπα του 3 είναι

το 0 και το 1 και το 27 y 1 a(mod3) όπου a ένας από τους αριθμούς 1

και 0 . Αυτό είναι άτοπο άρα δεν υπάρχουν ακέραιοι.

Page 288: ΠΡΟΤΕΙΝΟΜΕΝΕΣ ΑΣΚΗΣΕΙΣ ΓΙΑ ΜΑΘΗΤΙΚΟΥΣ ΔΙΑΓΩΝΙΣΜΟΥΣ_ALL

http://www.mathematica.gr/forum/viewtopic.php?f=109&t=15584

Επιμέλεια: xr.tsif Σελίδα 6

Β τρόπος (Γενίκευση)

Γενικότερα δεν υπάρχουν ακέραιοι x,y τέτοιοι ώστε ,

2 2 2 23x y 3k 1 3x 3k 1 y   ,k N αφού LHS 0(mod3) ενώ

RHS 1(mod3) ή 2(mod3) .

Β τρόπος

Θα γράψω μια πιο αναλυτική λύση για να γίνει κατανοητή από τους μαθητές του

Γυμνασίου:

Έχουμε 2 2 2 2 23x y 7 3x y 7 3 / y 7 .

Όμως y 3k ή y 3k 1 ή y 3k 2 .

Αν y 3k τότε 2 2y 7 9k 7 που όμως ο αριθμός αυτός δεν διαιρείται με το

3 . [Πράγματι, αν υπήρχε ακέραιος n τέτοιος ώστε

29k 7 3n τότε 2 27 7

3k n n 3k3 3

πράγμα που είναι άτοπο]

Όμοια βρίσκουμε άτοπο και για τις άλλες περιπτώσεις.

Άρα τέτοιοι ακέραιοι δεν υπάρχουν.

Σημείωση:

Η απόδειξη της γενίκευσης, γίνεται όπως και της ΑΣΚΗΣΗΣ 305

ΘΕΜΑ 306 (ΔΗΜΗΤΡΗΣ ΙΩΑΝΝΟΥ )

Οι διαιρέσεις του 253 και 525 με έναν φυσικό αριθμό a δίνουν υπόλοιπο 15 .

Να βρεθούν οι δυνατές τιμές του a .

Λύση:

Εξ υποθέσεως ο a διαιρεί τον 253 15 238 2·7·17 και τον

525 15 510 2·3·5·17 . Συνεπώς ο a , ως κοινός διαιρέτης των 2·7·17 και

Page 289: ΠΡΟΤΕΙΝΟΜΕΝΕΣ ΑΣΚΗΣΕΙΣ ΓΙΑ ΜΑΘΗΤΙΚΟΥΣ ΔΙΑΓΩΝΙΣΜΟΥΣ_ALL

http://www.mathematica.gr/forum/viewtopic.php?f=109&t=15584

Επιμέλεια: xr.tsif Σελίδα 7

2·3·5·17 είναι ένας από τους 1,2,17,34 . Οι δύο πρώτοι απορρίπτονται ως

μικρότεροι του υπολοίπου 15 . Μένουν οι 17,34 , που βέβαια επαληθεύουν τα

δεδομένα.

ΘΕΜΑ 307 (ΔΗΜΗΤΡΗΣ ΙΩΑΝΝΟΥ )

Με ποιο φυσικό αριθμό πρέπει να διαιρεθούν οι 1268 και 1802 για να πάρουμε

αντίστοιχα υπόλοιπα 8 και 17 .

Λύση:

1268 a π 8 1260 a π

1802 a k 17 1785 a k

με

a 8a 17

a 17

.

Αλλά

2 21260 2 3 5 7

1785 3 5 119

, ο ΜΚΔ είναι το a 3 5 15 17 . Άτοπο.

ΘΕΜΑ 308 (ΜΠΑΜΠΗΣ ΣΤΕΡΓΙΟΥ )

Σε ένα παραλληλόγραμμο ABCD η γωνία CBD

είναι ορθή. Από το μέσο του

OC OC, όπου O είναι το μέσο της BD , φέρνουμε ευθεία παράλληλη στην BD

που τέμνει τις ευθείες AB,AD στα σημεία E και Z αντίστοιχα. Να αποδειχθεί

ότι AE 3 OZ .

Λύση:

Το σημείο τομής των EZκαι DC , G είναι το μέσο της DC αφού στο τρίγωνο

DOC είναι GH / /DO και H μέσο της OC .

Από το παραλληλόγραμμο BEDG είναι DG BE x και άρα AE 3x .

Page 290: ΠΡΟΤΕΙΝΟΜΕΝΕΣ ΑΣΚΗΣΕΙΣ ΓΙΑ ΜΑΘΗΤΙΚΟΥΣ ΔΙΑΓΩΝΙΣΜΟΥΣ_ALL

http://www.mathematica.gr/forum/viewtopic.php?f=109&t=15584

Επιμέλεια: xr.tsif Σελίδα 8

Αρκεί τώρα να δειχθεί ότι OZ x . Στο ορθογώνιο DBCφέρνουμε την BG η

οποία θα είναι διάμεσος και άρα θα ισούται με DC

x2

.

Το ZOBG είναι παραλληλόγραμμο. Λόγω ισότητας των τριγώνων ZGD και

GCI , το G είναι μέσο της ZI . Μετά από ορθογώνιο DZBI , ZI

ZG OB2

.

Και βέβαια ZG / /OB .

Και τελικά από το παραλληλόγραμμο ZOBG προκύπτει το ζητούμενο.

Β τρόπος (Στο ίδιο σχήμα)

Προφανώς η COδιέρχεται από το A . Επίσης H είναι το μέσο της ZE γιατί

DB / /ZE και AO διάμεσος του τριγώνου ADB .

Έστω AO OC 2a από το θεώρημα του Θαλή ισχύει ότι AO 2a AB

2OH a BE

που αυτή γίνεται 2 BE AB .Αν θέσουμε BE x,AB 2x τότε AE 3x .

Αρκεί τώρα να αποδείξουμε ότι OZ x , όμως το τετράπλευρο ZOEC είναι

παραλληλόγραμμο (οι διαγώνιες διχοτομούνται) άρα CE OZ . Είναι επίσης

DG GC BG x (BG διάμεσος ορθογωνίου τριγ.) άρα BE / / GC και το

τετράπλευρο BGCE είναι παραλληλόγραμμο, επομένως CE GB x και η

απόδειξη ολοκληρώθηκε.

Page 291: ΠΡΟΤΕΙΝΟΜΕΝΕΣ ΑΣΚΗΣΕΙΣ ΓΙΑ ΜΑΘΗΤΙΚΟΥΣ ΔΙΑΓΩΝΙΣΜΟΥΣ_ALL

http://www.mathematica.gr/forum/viewtopic.php?f=109&t=15584

Επιμέλεια: xr.tsif Σελίδα 9

ΘΕΜΑ 309 (ΔΗΜΗΤΡΗΣ ΙΩΑΝΝΟΥ )

Να αποδείξετε ότι 2 2 104444444445 1111111111 4444444444 10 .

Λύση:

Για ευκολία θέτουμε 1111111111 x , άρα 4444444445 4x 1

Άρα είναι:

2 2 2 24444444445 1111111111 4444444444 (4x 1) x (4x)

2 2 1016x 8x 1 x 16x 9x 1 9·1111111111 1 10 .

ΘΕΜΑ 310 (ΔΗΜΗΤΡΗΣ ΙΩΑΝΝΟΥ )

Να συγκριθούν οι αριθμοί: 183 307 306 305a 3 , b 2 2 2 .

Λύση:

Το πλήθος των ψηφίων του ba είναι [alogb] 1 .

(Yπόδειξη: Λύσε την b xa 10 )

O 1833 έχει 88 ψηφία ενώ ο δεύτερος 92 ψηφία άρα είναι μεγαλύτερος ο

δεύτερος.

Β τρόπος

Αλλιώς: 307 306 305 305 305 5·61 61 61 3·61 1832 2 2 2 (4 2 1) 2 2 32 27 3 3 .

ΘΕΜΑ 311 (ΔΗΜΗΤΡΗΣ ΙΩΑΝΝΟΥ )

Να αποδείξετε ότι: 1 1 1 1

1 ... 25 6 7 17

.

Page 292: ΠΡΟΤΕΙΝΟΜΕΝΕΣ ΑΣΚΗΣΕΙΣ ΓΙΑ ΜΑΘΗΤΙΚΟΥΣ ΔΙΑΓΩΝΙΣΜΟΥΣ_ALL

http://www.mathematica.gr/forum/viewtopic.php?f=109&t=15584

Επιμέλεια: xr.tsif Σελίδα 10

Λύση:

Η ιδέα είναι να κρατήσουμε τους δύο πρώτους όρους, και να ασχοληθούμε μόνο

με τους υπόλοιπους 11.

Έχουμε 1 1 1 1 1 1 11 403

... 25 6 7 17 5 6 7 210 και

1 1 1 1 1 1 11 517... 1

5 6 7 17 5 6 17 510 .

ΘΕΜΑ 312 (ΔΗΜΗΤΡΗΣ ΙΩΑΝΝΟΥ )

Να αποδείξετε ότι

1 1 1 1... 2012 1

1 2 2 3 3 4 2011 2012

.

Λύση:

Είναι: 1 n 1 n

n 1 nn 1 nn 1 n

Άρα LHS 2 1 3 2 4 3 ... 2012 2011 2012 1 .

ΘΕΜΑ 313 (Socrates)

Βρείτε όλα τα ζεύγη πρώτων αριθμών (p,q) τέτοια ώστε οι αριθμοί 2 3p q και

2 3q p να είναι τέλεια τετράγωνα ακεραίων.

Λύση:

http://www.artofproblemsolving.com/Foru ... &t=443715&

Εάν p q 2 τότε 4 8 12 δεν είναι τέλειο τετράγωνο, άρα απορρίπτονται.

Page 293: ΠΡΟΤΕΙΝΟΜΕΝΕΣ ΑΣΚΗΣΕΙΣ ΓΙΑ ΜΑΘΗΤΙΚΟΥΣ ΔΙΑΓΩΝΙΣΜΟΥΣ_ALL

http://www.mathematica.gr/forum/viewtopic.php?f=109&t=15584

Επιμέλεια: xr.tsif Σελίδα 11

Εάν wlogp 2 και q 2 , τότε έχουμε 3 2q x 4 (x 2)(x 2) , οπότε

gcd(x 2,x 2) 1 με x περιττό. Αλλά τότε x 2 1 και 3q 5 , άτοπο

Έτσι πρέπει οι p,q και οι δύο να είναι περιττοί , οπότε gcd(x p,x p) 1 ή

gcd(x p,x p) p . Εάν ισχύει gcd(x p,x p) 1 , τότε x p 1 και

3q 2p 1 και όμοια 3

p 2q 1 , αλλά wlogp q οπότε 3q 2p 1 , άτοπο.

Εάν ισχύει gcd(x p,x p) p , τότε p q και x kp οπότε

3 2p p (k 1)(k 1) , άρα k 2 και έτσι p q 3 και πράγματι

29 27 36 6 .

Β τρόπος

Εάν p q , τότε 2p 1 x , έτσι p (x 1)(x 1) x 1 1 και p 3 .

Εάν p q , μπορούμε να υποθέσουμε ότι p q και q 3 .

Άρα 2 3 2 3a q (p )(a )q a p p με (a p,a p) (a p,2p) 1 και

(2p,q) 1 . Έτσι έχουμε a p 1 και 3a p q , οπότε

3 22p q 1 (q 1)(q q 1) 2q , άτοπο.

Άρα η (p,q) (3,3) είναι η μοναδική λύση.

ΘΕΜΑ 314 (Socrates)

Προσδιορίστε όλους τους θετικούς ακεραίους d με την ιδιότητα: αν ο d διαιρεί

τον n τότε διαιρεί και κάθε αριθμό που προκύπτει με αναδιάταξη των ψηφίων

του n .

Λύση:

Έστω1 2 n

a 12x x ...x ένα πολλαπλάσιο του d το οποίο αρχίζει με 12 , (που

υπάρχει προφανώς) . Τότε έχουμε ότι: 1 2 n

d / a d / b x x ...x 21 και

1 2 nd / c x x ...x 12 , άρα ο d / b c 9 , δηλαδή d / 9 . Άρα d {1,3,9} .

Page 294: ΠΡΟΤΕΙΝΟΜΕΝΕΣ ΑΣΚΗΣΕΙΣ ΓΙΑ ΜΑΘΗΤΙΚΟΥΣ ΔΙΑΓΩΝΙΣΜΟΥΣ_ALL

http://www.mathematica.gr/forum/viewtopic.php?f=109&t=15584

Επιμέλεια: xr.tsif Σελίδα 12

Β τρόπος

Έστω N ένα πολλαπλάσιο του d το οποίο περιέχει το ψηφίο 1 . Τότε ο

d / 10N .Δηλαδή 1 1 2 n

d / N x x ...x 10 και 2 1 2 n

d / N x x ...x 01 , άρα

1 2d / N N 10 1 9 . Άρα d {1,3,9} .

ΘΕΜΑ 315 (Socrates)

Υπάρχουν ακέραιοι x,y,z τέτοιοι ώστε 2 2 2 2011x y z 2007 ;

Λύση:

Είναι 2011 2011RHS 2007 ( 1) 1(mod8) . Για να ισχύει η αρχική θα πρέπει

LHS RHS 7(mod8) όμως, εύκολα βλέπουμε ότι ´

2k 0   1  4(mή )ή od8

και επομένως το ζητούμενο δεν μπορεί να ισχύει.

ΘΕΜΑ 316 (Socrates)

Αν ο αριθμός 2xy977z διαιρείται με τον 792 , να βρείτε τα ψηφία x,y,z .

Λύση:

Επειδή 792 8·9·11 έχουμε

• o 2xy977z διαιρείται με το 8 8 / 77z . Από εδώ εύκολα βλέπουμε ότι

z 6 . Άρα μιλάμε για τον αριθμό 2xy9776 .

Αυτός διαιρείται με το 9 , άρα το ίδιο συμβαίνει και με τον

2 x y 9 7 7 6 x y 31 , άρα και με τον x y 4 . Επειδή τα x,y

είναι ψηφία, τα μόνα ενδεχόμενα είναι x y 4 9 ή x y 4 18 άρα

x y 5 ή x y 14 . Από εδώ, φαίνεται ότι x 1 , y 4 ή x 2 , y 3 ή

x 3 , y 2 ή x 4 , y 1 ή x 9 , y 5 ή x 8 , y 6 ή x 7 , y 7 ή

x 6 , y 8 ή x 5 , y 9 ( † ).

Page 295: ΠΡΟΤΕΙΝΟΜΕΝΕΣ ΑΣΚΗΣΕΙΣ ΓΙΑ ΜΑΘΗΤΙΚΟΥΣ ΔΙΑΓΩΝΙΣΜΟΥΣ_ALL

http://www.mathematica.gr/forum/viewtopic.php?f=109&t=15584

Επιμέλεια: xr.tsif Σελίδα 13

Επίσης, 11 / 2xy9776 11 / 2 y 7 6 (x 9 7) 11 / (y x 1)( ) (†† )

Τελικά, βλέπουμε ότι από τα ζευγάρια της († ), το μόνο που ικανοποιεί την (†† )

είναι το x 2 , y 3 . Άρα, ο αριθμός είναι ο 2239776 .

ΘΕΜΑ 317 (Socrates)

Πόσοι θετικοί ακέραιοι, μικρότεροι ή ίσοι του 500 , διαιρούνται με έναν

τουλάχιστον από τους αριθμούς 2,3,5 ;

Λύση:

Με το 2 διαιρούνται όλοι οι άρτιοι αριθμοί που προφανώς είναι 250 στο

πλήθος.

Στη συνέχεια θα βρούμε πόσοι αριθμοί διαιρούνται με το 3 αλλά που δεν είναι

άρτιοι, (αφού τους άρτιους τους έχουμε ήδη υπολογίσει)

Τα πολλαπλάσια του 3 που είναι ανάμεσα στο 1 και στο 500 , είναι:

3,6,9,12,...,498 δηλαδή είναι στο πλήθος 166 αριθμοί. Από αυτούς θα

αφαιρέσουμε το πλήθος των άρτιων που είναι πολλαπλάσια του 3 . Δηλαδή τους

αριθμούς: 6,12,18,...,498 και άρα είναι στο πλήθος 83 ( 498:6 83 ).

Άρα οι αριθμοί που διαιρούνται με το 3 και δεν είναι άρτιοι είναι 166 83 83

στο πλήθος.

Τέλος πρέπει να βρούμε τους αριθμούς που διαιρούνται με το 5 αλλά που δεν

είναι άρτιοι και δεν διαιρούνται με το 3 .

Με το 5 διαιρούνται 100 αριθμοί (εύκολο).

Από αυτούς άρτιοι είναι οι αριθμοί 10,20,30,40,...,500 δηλαδή 50 στο πλήθος

αριθμοί.

Και αυτοί που διαιρούνται με το 3 χωρίς να είναι άρτιοι είναι οι αριθμοί

15,45,75,...,495 δηλαδή 17 στο πλήθος αριθμοί.

Page 296: ΠΡΟΤΕΙΝΟΜΕΝΕΣ ΑΣΚΗΣΕΙΣ ΓΙΑ ΜΑΘΗΤΙΚΟΥΣ ΔΙΑΓΩΝΙΣΜΟΥΣ_ALL

http://www.mathematica.gr/forum/viewtopic.php?f=109&t=15584

Επιμέλεια: xr.tsif Σελίδα 14

(Πράγματι οι αριθμοί 15,45,75,...,495 γράφονται

15,15 1 30,15 2 30,...,15 k 30 , όπου πρέπει 15 k 30 495 και άρα

k 16 . Άρα πράγματι είναι 17 στο πλήθος).

Άρα: Οι αριθμοί που διαιρούνται με το 5 αλλά δεν είναι άρτιοι και δεν

διαιρούνται με το 3 είναι 100 50 17 33 στο πλήθος

Από τα παραπάνω βρίσκουμε ότι το πλήθος των αριθμών που διαιρούνται με

έναν τουλάχιστον από τους 2,3,5 είναι 250 83 33 366 .

Β τρόπος

Χωρίζουμε τους αριθμούς 1 έως 500 σε ομάδες των 30 . Συγκεκριμένα

α) στην ομάδα 1 έως 30 ,

β) την 31 έως 60 ,

γ) την 61 έως 90 και λοιπά, μέχρι την 451 έως 480 ( σύνολο μέχρι εδώ 16

ομάδες)

δ) Τέλος μία ομάδα οι 20 αριθμοί 481,482,...,500 που περίσσεψαν.

Μετράμε με το χέρι πόσα πολλαπλάσια του 2,3,5 περιέχονται στην πρώτη

ομάδα. Θα βρούμε ότι είναι 22 αριθμοί , οι 2,3,4,5,6,8,9,... και λοιπά). Τώρα

κάθε άλλη ομάδα 30 αριθμών έχει ακριβώς το ίδιο πλήθος (δηλαδή 22 ) από

αριθμούς που είναι πολλαπλάσια του 2,3,5 . (Αυτό είναι απλό γιατί αν ο a είναι

πολλαπλάσιο του 2,3,5 , τότε και ο a 30 έχει την ίδια ιδιότητα, και

αντίστροφα).

Σύνολο, λοιπόν, μέχρι εδώ 22 16 352 αριθμοί.

Τέλος, μετράμε με το χέρι πόσα πολλαπλάσια του 2,3,5 περιέχονται στην

τελευταία ομάδα. Θα βρούμε ότι είναι 14 αριθμοί.

Ολικό σύνολο 352 14 366 .

Page 297: ΠΡΟΤΕΙΝΟΜΕΝΕΣ ΑΣΚΗΣΕΙΣ ΓΙΑ ΜΑΘΗΤΙΚΟΥΣ ΔΙΑΓΩΝΙΣΜΟΥΣ_ALL

http://www.mathematica.gr/forum/viewtopic.php?f=109&t=15584

Επιμέλεια: xr.tsif Σελίδα 15

ΘΕΜΑ 318 (Socrates)

Να βρείτε το μικρότερο θετικό ακέραιο n για τον οποίο ο αριθμός n 1

διαιρείται με το 19 και ο αριθμός n 1 διαιρείται με το 96 .

Λύση:

Ψάχνουμε το μικρότερο n της μορφής n 19k 1 96m 1 . Άρα

19k 96m 2 19·5m m 2 , οπότε 19 / m 2 . Το μικρότερο τέτοιο m είναι

m 17 . Δίνει 19k 96·17 2 , δηλαδή k 86 και αντίστοιχο n 1633 .

ΘΕΜΑ 319 (Socrates)

Οι θετικοί πραγματικοί αριθμοί a,b,c είναι τέτοιοι ώστε b c 3 1

a 2

και

a c3

b

.

α) Να προσδιορίσετε το λόγο a b

c

.

β) Να δείξετε ότι υπάρχει τρίγωνο με πλευρές a,b,c .

γ) Να προσδιορίσετε τις γωνίες του τριγώνου με πλευρές a,b,c .

Λύση:

α) Έχουμε:

b1

3 1ca 2

c

και

a1

c 3b

c

. Για ευκολία, θέτουμε

b ax , y

c c . Τότε έχουμε:

x 1 3 1

y 2

και

y 13

x

.

Λύνοντας το σύστημα βρίσκουμε ότι: 3 3

x1 3

, y 2 .

Page 298: ΠΡΟΤΕΙΝΟΜΕΝΕΣ ΑΣΚΗΣΕΙΣ ΓΙΑ ΜΑΘΗΤΙΚΟΥΣ ΔΙΑΓΩΝΙΣΜΟΥΣ_ALL

http://www.mathematica.gr/forum/viewtopic.php?f=109&t=15584

Επιμέλεια: xr.tsif Σελίδα 16

Άρα b 3 3

c 1 3

και

a2

c .

Και με πρόσθεση κατά μέλη έχουμε ότι b a 5 3 3

c 1 3

.

β) Από την υπόθεση έχουμε ότι:

b c 3 11 b c a

a 2

a c3 1 a c b

b

.

Επίσης από το (α) ερώτημα έχουμε: b a 5 3 3

1c 1 3

. Άρα b a c .

Από τα παραπάνω συμπεραίνουμε ότι τα a,b,c αποτελούν πλευρές τριγώνου.

γ) Από την υπόθεση βρίσκουμε ότι: a 2c , 3 3

b c1 3

.

Επειδή 1 3 1 3 2 3 2 1 3 2 2 3 3 3 2 2 3

3 3 3 31 c 2c b a

2(1 3) 1 3

.

Και επειδή είναι και c a βγαίνει το συμπέρασμα ότι η μεγαλύτερη πλευρά του

τριγώνου είναι η a .

Εύκολα παρατηρούμε (με απλές πράξεις) ότι 2 2 2 2 2b c 4c (2c) a , οπότε το

τρίγωνο είναι ορθογώνιο με υποτείνουσα a και επειδή a 2c θα πρέπει η γωνία

η απέναντι από την c να είναι o30 και φυσικά η άλλη οξεία γωνία θα είναι o

60 .

ΘΕΜΑ 320 (Socrates)

Βρείτε τις ακέραιες ρίζες της εξίσωσης 2 2x y x y 2 0 .

Page 299: ΠΡΟΤΕΙΝΟΜΕΝΕΣ ΑΣΚΗΣΕΙΣ ΓΙΑ ΜΑΘΗΤΙΚΟΥΣ ΔΙΑΓΩΝΙΣΜΟΥΣ_ALL

http://www.mathematica.gr/forum/viewtopic.php?f=109&t=15584

Επιμέλεια: xr.tsif Σελίδα 17

Λύση:

Αφού πολλαπλασιάσουμε επί 4 , η εξίσωση γράφεται 2 2(2x 1) (2y 1) 10 .

To 10 ως άθροισμα τετραγώνων γράφεται στους ακεραίους μόνο ως 2 2

10 ( 3) ( 1) . Συγκρίνοντας είναι 2x 1 3 , 2y 1 1 ή ανάποδα.

Τελικά έχουμε τις λύσεις (2,0) , (2,1) , ( 1,0) , ( 1, 1) ή ανάποδα.

Β τρόπος (ΣΧΗΜΑΤΙΚΑ)

Ζητάμε τις ακέραιες λύσεις της εξίσωσης 2 2x y x y 2 0 . Δηλαδή τα

σημεία τομής του κύκλου 2 2(2x 1) (2y 1) 10 με τις ευθείες

x 1 , x 0 , x 1 , x 2 .

ΘΕΜΑ 321 (Socrates)

Βρείτε όλους τους θετικούς ακεραίους που έχουν ακριβώς 9 (θετικούς)

διαιρέτες με άθροισμα 403 .

Page 300: ΠΡΟΤΕΙΝΟΜΕΝΕΣ ΑΣΚΗΣΕΙΣ ΓΙΑ ΜΑΘΗΤΙΚΟΥΣ ΔΙΑΓΩΝΙΣΜΟΥΣ_ALL

http://www.mathematica.gr/forum/viewtopic.php?f=109&t=15584

Επιμέλεια: xr.tsif Σελίδα 18

Λύση:

Έστω k ένας θετικός ακέραιος για τον οποίο ισχύει το ζητούμενο. Γράφουμε

τον k στη κανονική του μορφή 1 2 ma a a

1 2 mk p p ... p .

Θα πρέπει το πλήθος των διαιρετών του να είναι ίσο με 9 , δηλαδή

1 2 m(a 1)(a 1)...(a 1) 9 , άρα αναγκαστικά θα είναι m 2 και

1 2(a 1)(a 1) 3 3 δηλαδή

1 2a a 2 .

Επίσης θέλουμε το άθροισμα των διαιρετών του k να είναι ίσο με 403 δηλαδή

2 1 2 1

2 21 2

1 1 2 2

1 2

p 1 p 1403 (p p 1)(p p 1) 403

p 1 p 1

.

Όμως 403 13 31  και αφού 1 2

p p θα είναι

2

1 1p p 1 13 (1) και 2

2 2p p 1 31 (2).

Η (1) δίνει δεκτή λύση τη 1

p 3 και η (2) 2

p 5 .

Τελικά ο ζητούμενος αριθμός είναι ο 2 2 23 5 15 .

Να συμπληρώσω ότι αν ήταν m 1 , θα είχαμε 1

a 8 και 8k p με άθροισμα

διαιρετών 9

p 1

p 1

. Άρα θα έπρεπε :

9 3 3

2 3p 1 (p 1)(p 1)403 403 (p p 1)(p 1) 13 31

p 1 p 1

, πού δεν

ισχύει αφού κανένας από τους αριθμούς 12 , 30 δεν είναι τέλειος κύβος.

Page 301: ΠΡΟΤΕΙΝΟΜΕΝΕΣ ΑΣΚΗΣΕΙΣ ΓΙΑ ΜΑΘΗΤΙΚΟΥΣ ΔΙΑΓΩΝΙΣΜΟΥΣ_ALL

http://www.mathematica.gr/forum/viewtopic.php?f=109&t=15584

Επιμέλεια: xr.tsif Σελίδα 19

ΘΕΜΑ 322 (Socrates)

Να λυθεί το σύστημα:

1 1x 3

y z

1 1y 3

z x

1 1z 3

x y

.

Λύση:

Από τις δύο πρώτες εξισώσεις του συστήματος έχουμε:

1 1 1x y (x y)(1 ) 0

y x xy , και οι όμοιες σχέσεις απο το συνδυασμό

των άλλων εξισώσεων. Άρα θα είναι:

x y ή xy 1 , y z ή yz 1 , z x ή xz 1 .

Αν ίσχυαν και οι 3 από τη δεξιά μεριά τότε πολλαπλασιάζοντας τες θα είχαμε 2

(xyz) 1 (αδύνατο)

Αν ισχύουν και οι 3 από την αριστερή μεριά θα είναι x y z m και όπως

εύκολα βρίσκουμε m 1 ή m 2 .

Αν ισχύουν 2 από την αριστερή μεριά τότε θα ισχύει και η τρίτη. Μένει η

περίπτωση πού θα ισχύει μόνο μία από την αριστερή μεριά.

Ας είναι x y , yz 1 και xz 1 τότε θα είναι 1

x yz

και η τρίτη

εξίσωση γίνεται: z z z 3 z 3 και 1

x y3

.

Όμοια και οι άλλες περιπτώσεις.

Page 302: ΠΡΟΤΕΙΝΟΜΕΝΕΣ ΑΣΚΗΣΕΙΣ ΓΙΑ ΜΑΘΗΤΙΚΟΥΣ ΔΙΑΓΩΝΙΣΜΟΥΣ_ALL

http://www.mathematica.gr/forum/viewtopic.php?f=109&t=15584

Επιμέλεια: xr.tsif Σελίδα 20

ΘΕΜΑ 323 (Socrates)

Έστω a,b,c * διαφορετικοί ανά δύο και τέτοιοι ώστε a / b c bc ,

b / a c ac , c / a b ab . Να δειχθεί ότι ένας τουλάχιστον από τους a,b,c

δεν είναι πρώτος.

Λύση:

Έστω a b c πρώτοι. Δε μπορεί να ισχύει c 2 γιατί τότε δεν ισχύει η

διαιρετότητα 2 / a b ab .

Αφού a / b c bc , b / a c ac και οι a,b ειναι πρώτοι μεταξύ τους έχουμε

ab / a b c ac bc . Δε μπορεί να ισχύει γιατί

3ab a b c ac bc ab a(b c) b(a c) c a b (άτοπο), όποτε

ab a b c ac bc .

Ομοίως έχουμε ac / a b c ab bc 2ab ac ac / 2ab c / b (άτοπο).

Άρα δε μπορούν να ειναι και οι τρεις πρώτοι.

Και εδώ.

ΘΕΜΑ 324 (Socrates)

Να βρεθούν όλα τα ζεύγη θετικών ακέραιων (a,b) τέτοια ώστε ο αριθμός

4 3

2 2 2

a a 1

a b ab 1

να είναι ακέραιος.

Λύση:

Αν ήταν a b τότε 2 2 2 4 3a b ab 1 a a 1 , άτοπο

Αν a b , τότε λόγω της δοσμένης σχέσης παίρνουμε 2 2 2 4 3a b ab 1 / a a 1

δηλαδή 2 2 2 2 4 3 2 2 2 2 2 2a b ab 1 / b (a a 1) a (a b ab 1) b a .

Page 303: ΠΡΟΤΕΙΝΟΜΕΝΕΣ ΑΣΚΗΣΕΙΣ ΓΙΑ ΜΑΘΗΤΙΚΟΥΣ ΔΙΑΓΩΝΙΣΜΟΥΣ_ALL

http://www.mathematica.gr/forum/viewtopic.php?f=109&t=15584

Επιμέλεια: xr.tsif Σελίδα 21

Από την τελευταία παίρνουμε 2 2 2 2 2 2 2a b ab 1 | b a | a b απ' όπου

2 2 2 2(b 1)a b a b 1 0 και είναι φανερό ότι b 1 .

Αν όμως υποθέσουμε ότι υπάρχουν ζεύγη (a,b) ώστε να ικανοποιούν την

αρχική συνθήκη, τότε τα ίδια ζεύγη ικανοποιούν την παραπάνω δευτεροβάθμια

ως προς a ανίσωση η οποία έχει Διακρίνουσα 4Δ 3b 4 0 , άτοπο.

Άρα πρέπει a b και όλα τα ζεύγη της μορφής (a,a) με a N* ικανοποιούν

την αρχική.

ΘΕΜΑ 325 (Socrates)

Έστω n θετικός ακέραιος και μη – μηδενικοί ακέραιοι 1 2 n 1 2 n

x ,x ,...,x ,y ,y ,...,y

τέτοιοι ώστε 1 2 n 1 2 n

x x ... x y y ... y 0 και

2 2 2 2 2 2

1 1 2 2 n nx y x y ... x y . Να δείξετε ότι ο n είναι άρτιος.

Λύση:

Έστω m2 η μεγαλύτερη δύναμη του 2 που διαιρεί όλα τα

iix ,y . (Επιτρέπεται

m 0 .) Ορίζουμε m

i ia x / 2 και m

i ib y / 2 για 1 i n . Τότε έχουμε ότι τα

1 na , ,b είναι μη μηδενικοί ακέραιοι ώστε

1 n 1 na a b b 0 και

2 2 2 2

1 1 n na b a b . Επίσης, τουλάχιστον ένας από τους

1 na , ,b πρέπει να

είναι περιττός αφού αν ήταν όλοι άρτιοι τότε οι 1 n

x , ,y θα διαιρούνταν όλοι με

το m 12

, άτοπο.

Εξετάζουμε τώρα το 2 2a b . Αν a,b άρτιοι τότε το 2 2

a b είναι πολλαπλάσιο

του 4 , αν a,b άρτιοι τότε το 2 2a b είναι πολλαπλάσιο του 2 αλλά όχι του 4

και τέλος αν ακριβώς ένα από τα a,b είναι άρτιο και το άλλο περιττό τότε το 2 2

a b είναι περιττός. Άρα, αφού 2 2 2 2

1 1 n na b a b , τότε πρέπει να ισχύει

μια από τις πιο κάτω περιπτώσεις

α) Για κάθε 1 i n τα i i

a ,b είναι άρτιοι.

Page 304: ΠΡΟΤΕΙΝΟΜΕΝΕΣ ΑΣΚΗΣΕΙΣ ΓΙΑ ΜΑΘΗΤΙΚΟΥΣ ΔΙΑΓΩΝΙΣΜΟΥΣ_ALL

http://www.mathematica.gr/forum/viewtopic.php?f=109&t=15584

Επιμέλεια: xr.tsif Σελίδα 22

β) Για κάθε 1 i nτα i i

a ,b είναι περιττοί.

γ) Για κάθε 1 i nακριβώς ένα από τα i i

a ,b είναι άρτιος και τα άλλο είναι

περιττός.

Το α) απορρίπτεται αφού έχουμε δείξει πως δεν μπορεί να είναι όλοι άρτιοι. Αν

ισχύει το β), τότε αφού 1 n

a a 0 και κάθε i

a είναι περιττός, πρέπει και το

n να είναι περιττός. Αν ισχύει το γ) τότε για κάθε 1 i nτο i i

a b είναι

περιττός και αφού 1 1 n n

(a b ) (a b ) 0 τότε πρέπει και το n να είναι

περιττός.

ΘΕΜΑ 326 (Socrates)

Να δείξετε ότι υπάρχουν περιττοί (θετικοί) ακέραιοι 1 2 n

a ,a ,...,a τέτοιοι ώστε

1 2 n 1 2 na a ... a a ·a ...·a αν και μόνο αν 4 / n 1 .

Λύση:

Αν n 4k 1 παίρνουμε 1 2 3 n

a 2k 1,a 3,a a 1 και βλέπουμε ότι

1 n 1 na a (2k 1) 3 (4k 1) 6k 3 a a . Μένει να δείξουμε ότι αν

1 n 1 na a a a και

1 na a περιττοί τότε 4 / n 1 . Επειδή ο

1 na a είναι

περιττός τότε πρέπει και ο 1 n

a a να είναι περιττός και άρα πρέπει ο n να

είναι άρτιος. Ισχυρίζομαι ότι αν n 3mod4 και οι 1 n

a a είναι περιττοί, τότε

1 n 1 na a a a mod4 . Πράγματι αν k από τα

ia ισούνται με 3mod4 και

n k ισούνται με 1mod4 , τότε έχουμε

1 na a 3k (n k) n 2k 2k 3mod4 και άρα

1 na a 1mod4 αν k περιττός και 3mod4 αν k άρτιος. Αλλά

k

1 ka a ( 1) mod4 που ισούται με 1mod4 αν k άρτιος και 3mod4 αν k

περιττός. Άρα ο ισχυρισμός είναι πράγματι αληθής και άρα n 3mod4 και

αφού είναι περιττός πρέπει n 1mod4 .

Page 305: ΠΡΟΤΕΙΝΟΜΕΝΕΣ ΑΣΚΗΣΕΙΣ ΓΙΑ ΜΑΘΗΤΙΚΟΥΣ ΔΙΑΓΩΝΙΣΜΟΥΣ_ALL

http://www.mathematica.gr/forum/viewtopic.php?f=109&t=15584

Επιμέλεια: xr.tsif Σελίδα 23

ΘΕΜΑ 327 (Socrates)

Να βρεθούν όλοι οι φυσικοί αριθμοί n τέτοιοι ώστε οι αριθμοί 7n 2 και 9n 1

να είναι τέλεια τετράγωνα.

Λύση:

Έστω 29n 1 k (1) και 2

7n 2 m (2). Θα δείξουμε ότι δεν υπάρχει τέτοιος

n .

Έστω n άρτιος, τότε 2k 8r 1 (ως τετράγωνο περιττού) και 2

m 4s . Άρα

προσθέτοντας τις (1) και (2) έχουμε

16n 3 8r 1 4s , επομένως 8n 1 4r 2s , δηλ. άρτιος = περιττός, άτοπο.

Αν n περιττός, τότε ο k είναι άρτιος και ο mπεριττός και επαναλαμβάνουμε το

προηγούμενο επιχείρημα.

ΘΕΜΑ 328 (Socrates)

Είναι δυνατό να βρεθούν 100 ευθείες στο επίπεδο που να έχουν ακριβώς 1998

σημεία τομής; Εξηγείστε την απάντησή σας.

Λύση:

Θεωρούμε ορθοκανονικό σύστημα συντεταγμένων και φέρουμε τις ευθείες

x k , y λ με 1 k 40 και 1 λ 38 . Αυτές είναι 78 το πλήθος και

τέμνονται σε 38·40 1520 σημεία. Στη συνέχεια θεωρούμε τις ευθείες

x y 40 και x y 41 , οι οποίες προσφέρουν 2 2 4 , νέα σημεία τομής:

τα (1,39) , (40,0) , (2,39) και (1,40) .

Έπειτα, θεωρούμε τις παράλληλες ευθείες x y a για 17 a 37 , a 24 .

Αυτές είναι 20 το πλήθος.

Η x y 37 δίνει 1 3 4 νέα σημεία,

Page 306: ΠΡΟΤΕΙΝΟΜΕΝΕΣ ΑΣΚΗΣΕΙΣ ΓΙΑ ΜΑΘΗΤΙΚΟΥΣ ΔΙΑΓΩΝΙΣΜΟΥΣ_ALL

http://www.mathematica.gr/forum/viewtopic.php?f=109&t=15584

Επιμέλεια: xr.tsif Σελίδα 24

Η x y 36 δίνει 2 4 6 νέα σημεία,

....,

η x y 17 δίνει 21 23 44 νέα σημεία.

(έχουμε εξαιρέσει την x y 24 που θα μας έδινε 14 16 30 σημεία τομής).

Συνεπώς, οι τελευταίες 20 παράλληλες ευθείες τέμνουν τις 78 πρώτες σε 10

k 0

(4 2k) 30 474

νέα σημεία.

Συνολικά, λοιπόν, οι 78 2 20 100 ευθείες τέμνονται σε

1520 4 474 1998 σημεία.

ΘΕΜΑ 329 (Socrates)

Βρείτε όλους τους ακέραιους m,n για τους οποίους ο αριθμός m n18 9 1 είναι

τέλειο τετράγωνο ακεραίου.

Λύση:

Αν m 1 είναι φανερό ότι τότε θα πρέπει 2x 2mod4 που είναι αδύνατον.

Μένει να κοιτάξουμε για m 0 , m 1 .

Για m 0 έχουμε την εξίσωση n 29 2 x . Αν n άρτιος τότε θα πρέπει η

διαφορά δύο τετραγώνων να είναι ίση με 2 που δεν υπάρχουν λύσεις. Αν n

περιττός τότε θα υπάρχει y ώστε 2 29y 2 x , όμως αρκεί το mod2 για να

διαπιστώσουμε ότι λύσεις δεν υπάρχουν

Για m 1 η εξίσωση γίνετε n 29 19 x . Αν n άρτιος τότε με διαφορά

τετραγώνων βλέπουμε ότι μοναδική λύση είναι (m,n,x) (1,2,10) .

Για n 0 , n 1 η εξίσωση δεν έχει λύσεις.

Μένει να ελεγχθεί η περίπτωση όπου n περιττός οπού αρκεί επιλογή mod5 για

να δούμε ότι η εξίσωση δεν έχει λύσεις.

Page 307: ΠΡΟΤΕΙΝΟΜΕΝΕΣ ΑΣΚΗΣΕΙΣ ΓΙΑ ΜΑΘΗΤΙΚΟΥΣ ΔΙΑΓΩΝΙΣΜΟΥΣ_ALL

http://www.mathematica.gr/forum/viewtopic.php?f=109&t=15584

Επιμέλεια: xr.tsif Σελίδα 25

Μοναδική λύση η (m,n,x) (1,2,10) .

ΘΕΜΑ 330 (Socrates)

Βρείτε όλους τους ακέραιους x,y για τους οποίους x x x 22 3 4 y .

Λύση:

Διακρίνουμε τις εξής περιπτώσεις:

Αν ο x 3 , και περιττός τότε παίρνοντας mod4 έχουμε 2y 1mod4 .

Όμως το 1 δεν είναι τετραγωνικό υπόλοιπο mod4 .

Αν x 2 και άρτιος, τότε παίρνοντας mod3 έχουμε 2y 2mod3 .

Όμως το 2 δεν είναι τετραγωνικό υπόλοιπο mod3.

Αν x 0 τότε 2y 3 που δεν έχει ακέραιες λύσεις ως προς y ,

ενώ αν x 1 τότε 2y 9 δηλαδή y 3 .

Άρα οι μόνες λύσεις είναι οι (x,y) (1,3),(1, 3) .

ΘΕΜΑ 331 (Socrates)

Δείξτε ότι δεν υπάρχουν τέλεια τετράγωνα της μορφής a00...0b , όπου

a,b {1,2,3,...,9} .

Λύση:

Αν υποθέσουμε ότι ο δοσμένος αριθμός είναι τέλειο τετράγωνο, τότε με

δεδομένο ότι b 0 , θα πρέπει να λήγει σε 1 ή 4 ή 5 ή 6 ή 9 . Δηλαδή

b {1,4,5,6,9} .

Ας υποθέσουμε λοιπόν, ότι υπάρχει k , ώστε να είναι 2 n 2

α00...0b k α 10 b k (1) , όπου

Page 308: ΠΡΟΤΕΙΝΟΜΕΝΕΣ ΑΣΚΗΣΕΙΣ ΓΙΑ ΜΑΘΗΤΙΚΟΥΣ ΔΙΑΓΩΝΙΣΜΟΥΣ_ALL

http://www.mathematica.gr/forum/viewtopic.php?f=109&t=15584

Επιμέλεια: xr.tsif Σελίδα 26

από την υπόθεση προκύπτει ότι n 2 .

Διακρίνουμε τις εξής περιπτώσεις:

1η ΠΕΡΙΠΤΩΣΗ: b {1,4,9} .

Τότε 2b c , όπου c {1,2,3} . Οπότε η (1) γράφεται: n

α 10 (k c)(k c) .

Άρα θα υπάρχουν φυσικοί αριθμοί x,y,m,r , ώστε να

είναι mk c x 10 και r

k c y 10 , όπου x y α και m r n , με

x,y {1,2,...,9} . Από τις σχέσεις αυτές προκύπτει ότι r m2c y 10 x 10 (2) .

*** Αν m r , τότε m r m2c 10 (y 10 x)

, από την οποία για m 0 ,

συνεπάγεται ότι 10 / 2c , που είναι άτοπο, αφού c {1,2,3} .

(Επίσης αν m 0 , τότε έχουμε r n 2

2c y 10 x y 10 x 1 10 x 2c x 100 , που και πάλι είναι

άτοπο, αφού c {1,2,3} και x {1,2,...,9} )

*** Αν m r , με όμοιο τρόπο δείχνουμε το άτοπο.

*** Αν m r , τότε η σχέση (2) γράφεται: m2c 10 (y x) . Όμως

m r n 2 2m 2 m 1 , οπότε θα πρέπει 10 / 2c , που είναι άτοπο.

Αποκλείεται λοιπόν ο b {1,4,9} .

2η ΠΕΡΙΠΤΩΣΗ: b 5

Τότε η σχέση (1) γράφεται: n 2α 10 k 5 . Από εδώ προκύπτει ότι ο k , πρέπει

να λήγει σε 5 .

Δηλαδή k 10z 5 . Άρα: n 2 2α 10 (10z 5) 5 100z 100z 20 . Αφού δε

είναι n 2 , θα πρέπει 100 / 20 , που είναι άτοπο.

Page 309: ΠΡΟΤΕΙΝΟΜΕΝΕΣ ΑΣΚΗΣΕΙΣ ΓΙΑ ΜΑΘΗΤΙΚΟΥΣ ΔΙΑΓΩΝΙΣΜΟΥΣ_ALL

http://www.mathematica.gr/forum/viewtopic.php?f=109&t=15584

Επιμέλεια: xr.tsif Σελίδα 27

3η ΠΕΡΙΠΤΩΣΗ: b 6

Τότε η (1) γράφεται: n 2α 10 k 6 . Από εδώ προκύπτει ότι ο k , πρέπει να

λήγει σε 4 ή 6 .

Δηλαδή k 10z 4 ή k 10z 6 . Τότε : n 2α 10 100z 80z 10 ή

n 2α 10 100z 120z 30 . Και πάλι αφού n 2 , θα πρέπει : 100 / 10 ή

100 / 30 , που όμως είναι άτοπο.

Από τα παραπάνω, προκύπτει ότι δεν είναι δυνατόν, ο δοσμένος αριθμός να

είναι τέλειο τετράγωνο.

ΘΕΜΑ 332 (Socrates)

Βρείτε όλους τους θετικούς ακέραιους a,b για τους οποίους b aa b 2ab .

Λύση:

Αν ήταν και οι δύο μεγαλύτεροι του 2 τότε θα είχαμε b a 2 2a b a b 2ab ,

άτοπο. Άρα ένας τουλάχιστον είναι μικρότερος ή ίσος του 2 .

Αν ήταν b 2 τότε έχουμε την εξίσωση 2 aa 2 4a που έχει μοναδική λύση

a 2 (αν ήταν a 2 , καταλήγουμε εύκολα σε άτοπο όπως προηγουμένως).

Αν ήταν b 1 τότε έχουμε την εξίσωση a 1 2a που δίνει τη λύση a 1 .

Συνεπώς λύσεις είναι τα ζεύγη (a,b) (1,1),(2,2) .

ΘΕΜΑ 333 (Socrates)

Βρείτε όλους τους πρώτους p,q για τους οποίους ο αριθμός 2 2p q 5 είναι

επίσης πρώτος.

Εξισώσεις στους ακεραίους: viewtopic.php?f=109&t=20478

Λύση:

Αν είναι και οι 2 άρτιοι τότε 2 2A 2 2 5 21 όχι πρώτος.

Page 310: ΠΡΟΤΕΙΝΟΜΕΝΕΣ ΑΣΚΗΣΕΙΣ ΓΙΑ ΜΑΘΗΤΙΚΟΥΣ ΔΙΑΓΩΝΙΣΜΟΥΣ_ALL

http://www.mathematica.gr/forum/viewtopic.php?f=109&t=15584

Επιμέλεια: xr.tsif Σελίδα 28

Αν είναι και οι 2 περιττοί τότε A άρτιος , άρα πάλι μη πρώτος.

Αν είναι ένας άρτιος και ο άλλος περιττός, έστω ο p , τότε 2A 4p 5 .

Για p 3 o A ισούται με 41 , δεκτό. Για p 3 θα είναι 2p 1mod3 και

2A 4p 5 4 1 5 0(mod3) πάλι σύνθετος.

Άρα το ζητούμενο ικανοποιείται για (p,q) (3,2) ή (p,q) (2,3) .

Β τρόπος

Υποθέτουμε ότι οι 2 πρώτοι είναι μεγαλύτεροι του 3 . Άρα αν πάρουμε mod3

την παράσταση , διαιρείται ακριβώς γιατί 2 2p q 1(mod3) .Αυτό σημαίνει ότι

ένας από αυτούς θα είναι ίσος με το 3 .Έστω ο q 3 .Η παράσταση γίνεται

29p 5 .

Παίρνοντας τώρα mod6 τότε 29p 5 2mod6 που είναι άρτιος μεγαλύτερος

του 2 , άρα όχι πρώτος. Έτσι p 2 ή q 3 .Η δεύτερη απορρίπτεται και τελικά

έχουμε τις λύσεις (p,q) (3,2) ή (p,q) (2,3) .

ΘΕΜΑ 334 (ΔΗΜΗΤΡΗΣ ΙΩΑΝΝΟΥ )

Να υπολογίσετε το πλήθος των φυσικών αριθμών που δεν ξεπερνούν τον 10000

και διαιρούνται με κάθε φυσικό αριθμό που δεν ξεπερνάει τον 10 .

Λύση:

Αρκεί να υπολογίσουμε το πλήθος των πολλαπλασίων του Ε.Κ.Π. των 1,2,...,10

που είναι μικρότερα του 10000 . Είναι

3 2lcm(1,2, ,10) 2 3 5 7 2520 . Τα πολλαπλάσια του, που είναι μικρότερα

του 10000 είναι τα 2520,5040,7560 , στο πλήθος 3 .

Page 311: ΠΡΟΤΕΙΝΟΜΕΝΕΣ ΑΣΚΗΣΕΙΣ ΓΙΑ ΜΑΘΗΤΙΚΟΥΣ ΔΙΑΓΩΝΙΣΜΟΥΣ_ALL

http://www.mathematica.gr/forum/viewtopic.php?f=109&t=15584

Επιμέλεια: xr.tsif Σελίδα 29

ΘΕΜΑ 335 (ΔΗΜΗΤΡΗΣ ΙΩΑΝΝΟΥ )

Έστω n 1 *

nS 1 2 3 4 5 6 ... ( 1) n , (n N )

. Να βρείτε τον αριθμό

2011 2012S S .

Λύση:

2011 1

2011S 1 2 3 4 5 6 ..... ( 1) 2011

2010·11 2 3 4 5 6 ..... 2011 1 1006

2 .

2012 1

2012S 1 2 3 4 5 6 ..... ( 1) ·2012

2012·( 1)1 2 3 4 5 6 ..... 2012 1006

2

.

Επομένως 2011 2012

S S 0 .

ΘΕΜΑ 336 (Socrates)

Αν a b c d 0 με 2 2 2 2a b c d 1 να δείξετε ότι a b 1 c d .

Λύση:

Για την αριστερή.

Αρκεί ισοδύναμα να δείξουμε ότι 2 2 2 2 2 2 2(a b) a 2ab b 1 a b c d

το οποίο ανάγεται στην 2 22ab c d , από την πρώτη όμως υπόθεση του

προβλήματος έχουμε 2 2 2 22ab 2cc c c c d .

Ομοίως και για την δεξιά, αρκεί ισοδύναμα να είναι 2 2 2 2 2 2 2

(c d) c 2dc d 1 a b c d , δηλαδή 2 22dc a b που ισχύει

αφού ειναι 2 2 2 22dc 2bb b b a b .

ΘΕΜΑ 337 (sokratis lyras)

Να βρεθούν οι n Z ώστε: n 4mod6 και n 13mod21 .

Page 312: ΠΡΟΤΕΙΝΟΜΕΝΕΣ ΑΣΚΗΣΕΙΣ ΓΙΑ ΜΑΘΗΤΙΚΟΥΣ ΔΙΑΓΩΝΙΣΜΟΥΣ_ALL

http://www.mathematica.gr/forum/viewtopic.php?f=109&t=15584

Επιμέλεια: xr.tsif Σελίδα 30

Λύση:

n 4mod6 n 6k 4 21λ 96k 4 21λ 13 k

6n 13mod21 n 21λ 13

7λ 3k

2

.

Αν λ 2ρ με ρ Ζ , τότε 7λ 3 3

k 7ρ Ζ2 2

, άτοπο.

Άρα πρέπει λ 2ρ 1, με ρ Ζ , ώστε να έχουμε

7λ 3 7(2ρ 1) 3k 7ρ 5

2 2

.

Άρα n 6k 4 42ρ 34 , με ρ Ζ .

Δηλαδή οι αριθμοί που ζητάμε είναι οι 34,76,118,......

ΘΕΜΑ 337β

β

(Socrates)

Λύστε και το:

x 5mod6

x 4mod11

x 3mod17

.

Ψάξτε πρώτα για το κινεζικό θεώρημα υπολοίπων, πχ εδώ.

Λύση:

Όλα τα παρακάτω είναι σύμφωνα με τους συμβολισμούς στο κινέζικο θεώρημα

στην παραπομπή του socrates.

1 2 3M m m 187

2 1 3M m m 102

Page 313: ΠΡΟΤΕΙΝΟΜΕΝΕΣ ΑΣΚΗΣΕΙΣ ΓΙΑ ΜΑΘΗΤΙΚΟΥΣ ΔΙΑΓΩΝΙΣΜΟΥΣ_ALL

http://www.mathematica.gr/forum/viewtopic.php?f=109&t=15584

Επιμέλεια: xr.tsif Σελίδα 31

3 1 2M m m 66

Άρα υπάρχει i

n Z όπου i {1,2,3} ώστε: i i i

M n 1(modm ) . Άρα

απλοποιώντας τις 3 ισοτιμίες παίρνω:

1 1n 1(mod6) n 1

2 23n 1(mod11) n 4

3 315n 1(mod17) n 8 .

Άρα i i i 1 2 3x m n a (modm m m ) .

Κάνοντας τις πράξεις παίρνουμε ότι: x 1122y 785 .

Β τρόπος

Να δώσω την λύση διαφορετικά. Ας υπενθυμίσω πρώτα ότι ζητάμε την λύση

του συστήματος x 5mod6,x 4mod11,x 3mod17 .

Ας ξεκινήσουμε πρώτα από την λύση του συστήματος x 5mod6,x 4mod11 .

Η ουσία του κινεζικού θεωρήματος είναι ότι επειδή οι αριθμοί 6,11 είναι πρώτοι

μεταξύ τους, θα υπάρχει μοναδικό a {0,1,2, ,65} με x amod66 .

Μπορούμε βέβαια να χρησιμοποιήσουμε τον τύπο που δίνει το a όπως έκανε ο

Σωκράτης αλλά σε αυτές τις «μικρές» περιπτώσεις νομίζω είναι πιο εύκολο να

το κάνουμε στο χέρι. Αφού x 4mod11 , τότε απαραίτητα το a θα ισούται με

ένα από τα 4,15,26,37,48,59 . Κοιτάμε τώρα τους αριθμούς αυτούς και

παρατηρούμε ότι ο αριθμός που ισούται με 5mod6 είναι ο 59 . Φυσικά αυτά τα

κάνουμε στο πρόχειρο. Στην απάντηση αρκεί να γράψουμε:

Παρατηρούμε ότι x 59mod66 x 5mod6,x 4mod11 . Επειδή τα 6,11

είναι πρώτοι μεταξύ τους, από το κινέζικο θεώρημα έχουμε ισοδυναμία, δηλαδή

x 59mod66 x 5mod6,x 4mod11 .

Μένει τώρα να λύσουμε το σύστημα x 59mod66,x 3mod17 . Πάλι επειδή

τα 66,17 είναι πρώτοι μεταξύ τους, ψάχνουμε απάντηση της μορφής

Page 314: ΠΡΟΤΕΙΝΟΜΕΝΕΣ ΑΣΚΗΣΕΙΣ ΓΙΑ ΜΑΘΗΤΙΚΟΥΣ ΔΙΑΓΩΝΙΣΜΟΥΣ_ALL

http://www.mathematica.gr/forum/viewtopic.php?f=109&t=15584

Επιμέλεια: xr.tsif Σελίδα 32

x amod1122 . Εδώ αρχίζουμε να έχουμε κάπως πιο πολλές πράξεις. Ένας

τρόπος, όπως και προηγουμένως είναι να υπολογίσουμε τους 17 αριθμούς

59,59 66,59 2 66,...,59 16 66 και να βρούμε τον μοναδικό που ισούται

με 3mod17 . Πώς μπορούμε να το κάνουμε αυτό στα γρήγορα; Το 59 ισούται

με 8mod17 . Κάθε φορά που προσθέτουμε το 66 ουσιαστικά προσθέτουμε

15mod17 ή ισοδύναμα προσθέτουμε 2mod17 . Αν λοιπόν προσθέσουμε n

φορές το 66 θα φτάσουμε σε ένα αριθμό ισότιμο με (8 2n)mod17 . Θέλουμε

λοιπόν (8 2n) 3 1 od174m και παρατηρούμε ότι το n 11 δουλεύει.

Επομένως ο αριθμός που πρέπει να διαλέξουμε είναι ο 59 11 66 785 .

Πάλι το πιο πάνω είναι στο πρόχειρο. Στο χαρτί αρκεί να γράψουμε:

Παρατηρούμε ότι 785 59 11 66 59mod66 και

785 59 11 66 (8 22) 14 3mod17 . Επειδή οι 66,17 είναι πρώτοι

μεταξύ τους από το κινέζικο θεώρημα έχουμε

x 59mod66,x 3mod17 x 785mod1122 .

ΘΕΜΑ 338 (Socrates)

Για ποια 0

n ,ο αριθμός n 1 22 n

είναι δύναμη του 2 ; Για ποια 0

n είναι

πρώτος;

Λύση:

Για το πρώτο ερώτημα:

Ο n 0 είναι λύση.

Θεωρούμε ότι n 1 και ας υποθέσουμε ότι n 2 a12 n 2

για κάποιο φυσικό

αριθμό a , δηλαδή n 1 a 22 2 n

(1). Είναι φανερό από την τελευταία ότι

n 1 a οπότε ας είναι n 1 a k 0 , όπου k .

Τότε η (1) γίνεται a k 22 (2 1) n κι επειδή a k

(2 ,2 1) 1 άρα πρέπει

Page 315: ΠΡΟΤΕΙΝΟΜΕΝΕΣ ΑΣΚΗΣΕΙΣ ΓΙΑ ΜΑΘΗΤΙΚΟΥΣ ΔΙΑΓΩΝΙΣΜΟΥΣ_ALL

http://www.mathematica.gr/forum/viewtopic.php?f=109&t=15584

Επιμέλεια: xr.tsif Σελίδα 33

a 2

1

k 2

2

2 n

2 1

(2

3)n

)

(

με

1 2(n ,n ) 1 και

1 2n n n . Τότε

1a 2a και 1a

1n 2 .

Από την (3) αν ήταν k 2 τότε θα ήταν k2 1 1 mod4 , άτοπο διότι το 1

δεν είναι τετραγωνικό υπόλοιπο mod4 . Άρα k 1 απ' όπου 2

n 1 οπότε

1a

1n n 2 (4) . Από την άλλη λόγω της n 1 a k έχουμε

1n 1 2a 1

δηλαδή 1

n 2a (5).

Από τις (4) , (5) παίρνουμε τελικά 1a 1

12 a

που έχει τις λύσεις

1a 1 ή

1a 2

διότι για 1

a 3 ισχύει 1a 1

12 a

(εύκολο π.χ. με Bernoulli ή επαγωγή). Οπότε

n 2 ή n 4 . Άρα τελικά οι λύσεις είναι n 0 , n 2 , n 4 .

Για το δεύτερο ερώτημα:

Καταρχήν ο n 0 είναι φανερά ο μόνος άρτιος αριθμός ο οποίος αποτελεί λύση.

Έστω λοιπόν n 2k 1 με k 0 . Τότε αντικαθιστώντας , θέλουμε να ισχύει

2(k 1) 22 (2k 1) p

, όπου p 3 πρώτος δηλαδή

k 1 k 12 (2k 1) 2 (2k 1) p

οπότε αφού k 1 k 12 (2k 1) 2 (2k 1)

άρα θα πρέπει k 12 (2k 1) 1

η οποία έχει λύσεις τους αριθμούς k 0 ή

k 1 . Για k 2 είναι k 12 (2k 1) 3

(εύκολο π.χ. με επαγωγή).

Για k 0 παίρνουμε n 2 που είναι δεκτή λύση και για k 1 παίρνουμε n 4

που επίσης είναι δεκτή λύση.

ΘΕΜΑ 339 (Socrates)

Βρείτε το τελευταίο ψηφίο του abc , αν a,b,c πρώτοι αριθμοί διαφορετικοί

ανά δύο , και το τελευταίο ψηφίο του αριθμού 4 4 4α b c είναι 7 .

Page 316: ΠΡΟΤΕΙΝΟΜΕΝΕΣ ΑΣΚΗΣΕΙΣ ΓΙΑ ΜΑΘΗΤΙΚΟΥΣ ΔΙΑΓΩΝΙΣΜΟΥΣ_ALL

http://www.mathematica.gr/forum/viewtopic.php?f=109&t=15584

Επιμέλεια: xr.tsif Σελίδα 34

Λύση:

Κάθε πρώτος αριθμός είναι 1 ή 3 ή 7 ή 9(mod10) , εκτός αν ο πρώτος είναι ο

2 . Άρα για κάθε p παίρνω ότι 4p 1(mod10) . Αλλά 4 4 4

a b c 7mod10 ,

πράγμα το οποίο δεν γίνεται ακόμα και αν ένας εκ των πρώτων είναι ο 2 . Άρα

πρέπει να είναι ο 5 .

O ένας από τους πρώτους είναι ο 5 και στους άλλους οι τέταρτες δυνάμεις είναι

1(mod10) . Άρα το τελευταίο του abc είναι 5 αφού δεν γίνεται να λήγει σε 0 .

ΘΕΜΑ 340 (sokratis lyras)

Έστω a,b,c,x,y,z R με b c

ax 2

,

c ab

y 2

,

a bc

z 2

και

xy yz zx 67 ,x y z 2010 . Να βρείτε το xyz .

Λύση:

b c 2a a c 2b a b 2cx y z

a b c

2 2 2 2 2 2b c c b 2abc a c ac 2abc a b ab 2abc

abc

4abc (a b)(b c)(c a) (a b)(b c)(c a)4

abc abc

4 (x 2)(y 2)(z 2) 4 8 2(xy yz xz) 4(x y z) xyz

xyz 5892 .

Page 317: ΠΡΟΤΕΙΝΟΜΕΝΕΣ ΑΣΚΗΣΕΙΣ ΓΙΑ ΜΑΘΗΤΙΚΟΥΣ ΔΙΑΓΩΝΙΣΜΟΥΣ_ALL

http://www.mathematica.gr/forum/viewtopic.php?f=109&t=15584

Επιμέλεια: xr.tsif Σελίδα 35

ΘΕΜΑ 340b (sokratis lyras)

Aν2 2 2 2

2 2 2 2

x y x yk

x y x y

, τότε να βρεθεί η παρακάτω παράσταση

8 8 8 8

8 8 8 8

x y x y

x y x y

συναρτήσει τουk .

Λύση:

Έστω 8 8

8 8

1 x yE(x,y) E b , b

b x y

4 4 2 4 4 2 4 4 4 4 4 4

4 4 4 4 4 4 4 4 4 4

(x y ) (x y ) x y x y 1 x y2b 2b 2b a , a

(x y )(x y ) x y x y a x y

4 4 2 2 2 2 2 2 2 2 2 2

4 4 2 2 2 2 2 2 2 2

x y (x y ) (x y ) x y x ya 2a 2a k

x y (x y )(x y ) x y x y

4 4

4 4

k x y ka

2 x y 2

.

Άρα 2 2

k 2 k 4 k 42b 2b b

2 k 2k 4k

και επομένως

2 2 2

2 2

k 4 4k (k 4)E

4k k 4 4k(k 4)

.

Β τρόπος

Θέτοντας 2

2

xA

y η αρχική γίνεται

A 1 A 1k

A 1 A 1

, ισοδύναμα

2

2

2(A 1)k

A 1

, από όπου

2 k 2A

k 2

.

Τώρα, η ζητούμενη γράφεται 4 4 2 2

4 4 2

A 1 A 1 (k 4)...

A 1 A 1 4k(k 4)

.

Page 318: ΠΡΟΤΕΙΝΟΜΕΝΕΣ ΑΣΚΗΣΕΙΣ ΓΙΑ ΜΑΘΗΤΙΚΟΥΣ ΔΙΑΓΩΝΙΣΜΟΥΣ_ALL

http://www.mathematica.gr/forum/viewtopic.php?f=109&t=15584

Επιμέλεια: xr.tsif Σελίδα 36

ΘΕΜΑ 341 (Socrates)

Να βρεθεί η τιμή της παράστασης :

2 2x x 1 y y 1

(x y)(x z)(x y 1)(x z 1) (y x)(y z)(y x 1)(y z 1)

2z z 1

(z x)(z y)(z x 1)(z y 1)

.

Λύση:

Ο παρονομαστής του πρώτου κλάσματος γράφεται:

(x y) (x y) 1 (x z) (x z) 1

2 2 2 2 2 2 2 2(x y x y)(x z x z) x x (y y) x x (z z) .

Θέτουμε τώρα: 2 2 2x x 1 ky y 1 mz z 1 n .

Τότε το πρώτο κλάσμα της δοσμένης παράστασης γράφεται: k

(k m)(k n) .

Με τον ίδιο τρόπο βρίσκουμε και τα άλλα κλάσματα της παράστασης, οπότε

έχουμε (αν ονομάσουμε A την παράσταση:

k m nA

(k m)(k n) (m k)(m n) (n k)(n m)

k m n

(k m)(k n) (k m)(m n) (k n)(m n)

k(m n) m(k n) n(k m)0

(k m)(k n)(m n)

.

Page 319: ΠΡΟΤΕΙΝΟΜΕΝΕΣ ΑΣΚΗΣΕΙΣ ΓΙΑ ΜΑΘΗΤΙΚΟΥΣ ΔΙΑΓΩΝΙΣΜΟΥΣ_ALL

http://www.mathematica.gr/forum/viewtopic.php?f=109&t=15584

Επιμέλεια: xr.tsif Σελίδα 37

ΘΕΜΑ 342 (Socrates)

Να βρεθούν οι πρώτοι αριθμοί a,b,c τέτοιοι ώστε οι αριθμοί

b c 3 c a 2,

a b

και

a b 1

c

να είναι ακέραιοι.

Λύση:

Επειδή τα a,b,c είναι πρώτοι τότε a,b,c 2 . Διακρίνουμε τις ακόλουθες

περιπτώσεις :

α) Αν b c 3 a

οπότε τα άλλα δυο κλάσματα γράφονται στην μορφή c a 2 2c 5

1b b

,

a b 1 2b 41

c c

. Βλέπουμε ότι δεν υπάρχουν τιμές ώστε τα κλάσματα να

ειναι ακέραιοι.

β) Αν c a 2 b .

Όμοια δείχνουμε οτι δεν υπάρχουν τιμές.

γ) Αν a b 1 c

βρίσκουμε τις τιμές a 5,b 7,c 11 .

Προσοχή! Το παραπάνω συμπέρασμα δεν είναι σωστό εκτός εάν αποδειχθεί.

Γιατί να ισχύει π.χ. ότι b c 3 a και όχι ας πούμε b c 3 2a και γενικά

b c 3 ka , k N ;

Ας γράψω την εξήγηση:

Αν b c 3 2a και c a 2 2b και a b 1 2c τότε προσθέτοντας

έχουμε άτοπο.

Page 320: ΠΡΟΤΕΙΝΟΜΕΝΕΣ ΑΣΚΗΣΕΙΣ ΓΙΑ ΜΑΘΗΤΙΚΟΥΣ ΔΙΑΓΩΝΙΣΜΟΥΣ_ALL

http://www.mathematica.gr/forum/viewtopic.php?f=109&t=15584

Επιμέλεια: xr.tsif Σελίδα 38

ΘΕΜΑ 343 (Socrates)

Έστω οι ακολουθίες n

1 1a 1 ...

2 n ,

n 1 2 nb a a ... a και

1 2 n

n

b b bc ...

2 3 n 1

. Βρείτε τον αριθμό

2011 2010b c .

Λύση:

Παρατηρούμε ότι n

n 1 n 2 1b n

2 3 n

n 1 n 1

2 3 n n 1(n 1)a (1 ) (n 1)(a 1)

2 3 n n 1

.

Θέτω τώρα n n 1 n

d b c

. Τότε

n 1

n 1 n n 2 n 1 n 1 n n 2 n 2 n 2

bd d (b b ) (c c ) a a (a 1) 1

m 2

.

Άρα

2011 2010 2010 1 2 1 2010 2009b c d d (d d ) (d d ) 2 (1 1) 2011 .

[Για να βρούμε την λύση δουλέψαμε ανάποδα: Αφού ορίσαμε το n

d όπως πιο

πάνω, είδαμε ότι 1 2 3

d 2,d 3,d 4 , υποψιαστήκαμε ότι n 1

d n 1 , μετά

είδαμε ότι για να το αποδείξουμε πρέπει να ισχύει ότι n

n 1

ba 1

n 1

, το οποίο

είναι και το πρώτο πράγμα που δείξαμε στην πιο πάνω απόδειξη.]

ΘΕΜΑ 344 (Socrates)

Ο πρώτος όρος μιας ακολουθίας αριθμών είναι 2 και ο δεύτερος 6 . Κάθε

επόμενος όρος προκύπτει ως ο λόγος του τελευταίου προς τον προτελευταίο.

Να βρεθεί το άθροισμα των πρώτων 2011 όρων της ακολουθίας.

Page 321: ΠΡΟΤΕΙΝΟΜΕΝΕΣ ΑΣΚΗΣΕΙΣ ΓΙΑ ΜΑΘΗΤΙΚΟΥΣ ΔΙΑΓΩΝΙΣΜΟΥΣ_ALL

http://www.mathematica.gr/forum/viewtopic.php?f=109&t=15584

Επιμέλεια: xr.tsif Σελίδα 39

Λύση:

Λόγω της άσκησης παίρνουμε 1 2 3 4 5 6

1 1 1a 2,a 6,a 3,a ,a ,a

2 6 3 με

άθροισμα 1 6

a a 12 .

Επίσης ισχύει n 1

n

n 2

aa

a

και n 2

n 1

n 3

aa

a

. Πολλαπλασιάζοντας μεταξύ τους αυτές

τις σχέσεις παίρνουμε n

n 3

1a

a

και όμοια n 3

n 6

1a

a

. Από τις τελευταίες δύο

παίρνουμε n n 6

a a

.

Άρα επαγωγικά, αν n 6k υ , έχουμε n υ

a a .

Συνεπώς επειδή 2011 6·335 1 άρα το ζητούμενο άθροισμα είναι ίσο με:

1 2 2010 2011 1 2 3 4 5 6 1a a a a 335(a a a a a a ) a

335·12 2 4022 .

ΘΕΜΑ 345 (Socrates)

Σε ένα συνέδριο συμμετέχουν 2011 άτομα και υπάρχουν 4020 (αμοιβαίες)

γνωριμίες. Να δείξετε ότι κάποιος από τους συνέδρους γνωρίζει το πολύ 3

άλλους συνέδρους.

Λύση:

Αν υποθέσουμε ότι κάθε σύνεδρος γνωρίζει τουλάχιστον 4 άλλους συνέδρους

τότε ο αριθμός των (αμοιβαίων )γνωριμιών είναι 4 2011

40222

, άτοπο.

ΘΕΜΑ 346 (Socrates)

Βρείτε όλους τους θετικούς ακεραίους a,b για τους οποίους ο αριθμός 2

(ab) 4(a b) είναι τέλειο τετράγωνο ακεραίου.

Page 322: ΠΡΟΤΕΙΝΟΜΕΝΕΣ ΑΣΚΗΣΕΙΣ ΓΙΑ ΜΑΘΗΤΙΚΟΥΣ ΔΙΑΓΩΝΙΣΜΟΥΣ_ALL

http://www.mathematica.gr/forum/viewtopic.php?f=109&t=15584

Επιμέλεια: xr.tsif Σελίδα 40

Λύση:

Υπόδειξη – λύση:

Παρατηρούμε ότι 2 2(ab) 4(a b) (ab) οπότε 2 2

(ab) 4(a b) (ab 1) .

Όμως, η εξίσωση 2 2(ab) 4(a b) (ab 1) είναι αδύνατη γιατί τα δύο μέλη

έχουν διαφορετική αρτιότητα.

Άρα, 2 2(ab) 4(a b) (ab 2) δηλαδή (a 1)(b 1) 2 .

Αν a,b 3 έχουμε άτοπο.

Άρα κάποιος, π.χ. ο a είναι 1 a 2 που σημαίνει a 1 , a 2 .

Προκύπτουν δυο εξισώσεις (λύνονται π.χ. με συμπλήρωση τετραγώνου.)

ΘΕΜΑ 347 (Socrates)

Μας δίνονται 13 ακέραια βάρη. Γνωρίζουμε ότι αφαιρώντας οποιοδήποτε βάρος,

μπορούμε να χωρίζουμε τα υπόλοιπα 12 σε δύο ομάδες των έξι με το ίδιο

βάρος. Να δείξετε ότι όλα τα βάρη είναι ίσα.

Λύση:

Υπόδειξη – λύση:

Έστω 1 132

x ,x ...,x τα βάρη. Θεωρούμε την ακολουθία i i

y S 13x όπου S το

άθροισμά τους.

Τότε i

T y 0 και i

2 / T y δηλαδή i

2 / y για κάθε i .

Διαιρούμε όλα τα i

y με το 2 και αναγόμαστε στο ίδιο πρόβλημα.

Δηλαδή διαιρούμε με το 2 άπειρες φορές, που σημαίνει i

y 0 για κάθε i .

Page 323: ΠΡΟΤΕΙΝΟΜΕΝΕΣ ΑΣΚΗΣΕΙΣ ΓΙΑ ΜΑΘΗΤΙΚΟΥΣ ΔΙΑΓΩΝΙΣΜΟΥΣ_ALL

http://www.mathematica.gr/forum/viewtopic.php?f=109&t=15584

Επιμέλεια: xr.tsif Σελίδα 41

ΘΕΜΑ 348 (Socrates)

Έχουμε δυο σακούλια με κόκκινους και μπλε βόλους, έναν τουλάχιστον από

κάθε χρώμα σε κάθε σακούλι. Υποθέτουμε ότι αν διαλέξουμε στην τύχη ένα από

τα σακούλια και έπειτα έναν βόλο από αυτό, η πιθανότητα να είναι κόκκινος

είναι ίση με την αντίστοιχη πιθανότητα αν βάλουμε όλους τους βόλους σε ένα

σακούλι και διαλέξουμε ένα βόλο. Αν το πρώτο σακούλι περιέχει 7 βόλους και

το δεύτερο 5 κόκκινους βόλους, να βρείτε πόσους βόλους περιέχει συνολικά το

δεύτερο σακούλι.

Λύση:

Έστω mο αριθμός των μπλε βόλων του δεύτερου σακουλιού και k ο αριθμός

των κόκκινων βόλων του πρώτου σακουλιού. Τότε οι πιθανότητες επιλογής

κόκκινου βόλου από το πρώτο σακούλι είναι k

7 και από το δεύτερο

5

5 m. Οι

πιθανότητες επιλογής κόκκινου βόλου από ένα σακούλι στο οποίο έχουμε ρίξει

όλους τους βόλους είναι 5 k

12 m

. Άρα από υπόθεση έχουμε ότι

5 k12k km 35 7k k(5 m 3

k) 5

7 12 m

, οπότε με περιπτώσεις

καταλήγουμε στα δεκτά (k,m) (1,30),(5,2),(3,10) . Το πρώτο ζευγάρι δίνει

1 6

7 42 που ισχύει. Το δεύτερο δίνει

5 10

7 14 που ισχύει. Το τρίτο δίνει

3 8

7 22

που δεν ισχύει.

Άρα το δεύτερο σακούλι έχει είτε 5 30 35 βόλους είτε 5 2 7 βόλους.

Β τρόπος

Ας συμβολίσουμε με k τον αριθμό των κόκκινων βόλων μέσα στο πρώτο

σακούλι και με n το σύνολο των βόλων στο 2ο σακούλι. Τότε η πιθανότητα να

τραβήξουμε κόκκινο βόλο διαλέγοντας βόλους από το πρώτο σακούλι είναι k

7

ενώ η πιθανότητα να τραβήξουμε κόκκινο βόλο διαλέγοντας βόλους από το

Page 324: ΠΡΟΤΕΙΝΟΜΕΝΕΣ ΑΣΚΗΣΕΙΣ ΓΙΑ ΜΑΘΗΤΙΚΟΥΣ ΔΙΑΓΩΝΙΣΜΟΥΣ_ALL

http://www.mathematica.gr/forum/viewtopic.php?f=109&t=15584

Επιμέλεια: xr.tsif Σελίδα 42

δεύτερο σακούλι είναι 5

n. Συνεπώς η πιθανότητα να τραβήξουμε κόκκινο βόλο

(διαλέγοντας αρχικά στην τύχη το σακούλι) είναι 1 k 5

2 7 n

(1).

Αν βάλουμε όλους τους βόλους σε ένα σακούλι τότε θα έχουμε k 5 κόκκινους

και σύνολο n 7 , συνεπώς η πιθανότητα να διαλέξουμε τότε κόκκινο βόλο είναι

5 k

7 n

(2).

Από τις (1) , (2) και λόγω των δεδομένων του προβλήματος παίρνουμε

1 k 5 k 5

2 7 n n 7

απ' όπου τελικά (απαλοιφή παρονομαστών) παίρνουμε

2kn 7kn 35n 245 0 δηλαδή (n 7)(kn 35) 0 απ' όπου παίρνουμε

n 7 ή (k,n) (1,35), (5,7) (διότι ο αριθμός των κόκκινων σφαιρών δεν

υπερβαίνει τις 6 στο πρώτο σακούλι).

Άρα τελικά n 7 ή n 35 και οι δύο δεκτές λύσεις.

ΘΕΜΑ 349 (ΔΗΜΗΤΡΗΣ ΙΩΑΝΝΟΥ )

Να δειχθεί ότι η ακολουθία των αριθμών 6,10,14,18,...,(4k 2),... δεν περιέχει

κανένα τέλειο τετράγωνο φυσικού αριθμού.

Λύση:

Παίρνουμε τετραγωνικά υπόλοιπα mod4 και παρατηρούμε πως είναι τα 1 και

0 . Άρα δεν υπάρχει αριθμός της μορφής 4k 2 o οποίος να είναι τέλειο

τετράγωνο ακεραίου.

Β τρόπος

Θα εργαστούμε με την μέθοδο της εις άτοπον απαγωγής:

Ας υποθέσουμε ότι υπάρχει n φυσικός αριθμός έτσι ώστε να είναι

Page 325: ΠΡΟΤΕΙΝΟΜΕΝΕΣ ΑΣΚΗΣΕΙΣ ΓΙΑ ΜΑΘΗΤΙΚΟΥΣ ΔΙΑΓΩΝΙΣΜΟΥΣ_ALL

http://www.mathematica.gr/forum/viewtopic.php?f=109&t=15584

Επιμέλεια: xr.tsif Σελίδα 43

2 2 24k 2 n 2(2k 1) n n άρτιος. Άρα θα είναι και ο n επίσης άρτιος

(εύκολα αποδεικνύεται ότι αν kn άρτιος ή περιττός, τότε και ο n θα είναι άρτιος

ή περιττός αντιστοίχως)

Αφού λοιπόν ο n είναι άρτιος άρα n 2p,p N . Τότε από την σχέση

2 2 22(2k 1) n 2(2k 1) (2p) 2k 1 2p . Τούτο όμως είναι άτοπο,

αφού το πρώτο μέλος είναι περιττός και το δεύτερο άρτιος.

ΘΕΜΑ 350 (Socrates)

α) Πόσες ακέραιες ρίζες έχει η εξίσωση 2 2 2011x y 4 3 ; Πόσες θετικές

ακέραιες;

β) Πόσες θετικές ακέραιες ρίζες έχει η εξίσωση n(4α b)(4b α) 2010 όπου

n δεδομένος θετικός ακέραιος;

Λύση:

Για το (α) πρώτα:

Έχουμε:

2 2 2011 2011x y 4 3 (x y)(x y) 4 3 . Άρα υπάρχουν οι εξής περιπτώσεις:

1η Περίπτωση

mx y 4 3

nx y 3

όπου m n 2011 και m,n 0,1,2,3,...,2011 .

Προσθέτοντας όμως κατά μέλη τις δύο παραπάνω σχέσεις, βρίσκουμε m n n

2x 4 3 3 2 / 3 , που όμως είναι άτοπο.

Άρα η περίπτωση αυτή απορρίπτεται.

Page 326: ΠΡΟΤΕΙΝΟΜΕΝΕΣ ΑΣΚΗΣΕΙΣ ΓΙΑ ΜΑΘΗΤΙΚΟΥΣ ΔΙΑΓΩΝΙΣΜΟΥΣ_ALL

http://www.mathematica.gr/forum/viewtopic.php?f=109&t=15584

Επιμέλεια: xr.tsif Σελίδα 44

2η Περίπτωση

mx y 3

nx y 4 3 όπου m n 2011 και m,n 0,1,2,3,...,2011 .

Τότε και πάλι με όμοιο όπως και πριν τρόπο καταλήγουμε σε άτοπο.

3η Περίπτωση

mx y 2 3

nx y 2 3 όπου m n 2011 και m,n 0,1,2,3,...,2011 .

Τότε με πρόσθεση κατά μέλη βρίσκουμε m nx 3 3 και ύστερα n m

y 3 3

όπου βλέπουμε ότι οι αριθμοί x,y είναι ακέραιοι. Το πλήθος των λύσεων εύκολα

διαπιστώνουμε ότι είναι 2012 .

Αν τώρα ζητάμε το πλήθος των θετικών ακεραίων, τότε απαιτούμε να είναι

n m3 3 0 n m m 0,1,2,...,1005 και συνεπώς το πλήθος των

θετικών ακεραίων λύσεων είναι 1006 .

Για το (β) νομίζω ότι θα εργασθούμε παρόμοια.

ΘΕΜΑ 351 (ΔΗΜΗΤΡΗΣ ΙΩΑΝΝΟΥ )

Να βρεθεί το άθροισμα: A 2 99 3 98 4 97 ... 50 51 .

Λύση:

Είναι

50 50 502

n 1 n 1 n 1

50·51 50·51·101A 100 n(101 n) 101 n n 101 85850

2 6

.

Άρα A 85750 .

Page 327: ΠΡΟΤΕΙΝΟΜΕΝΕΣ ΑΣΚΗΣΕΙΣ ΓΙΑ ΜΑΘΗΤΙΚΟΥΣ ΔΙΑΓΩΝΙΣΜΟΥΣ_ALL

http://www.mathematica.gr/forum/viewtopic.php?f=109&t=15584

Επιμέλεια: xr.tsif Σελίδα 45

ΘΕΜΑ 352 (ΔΗΜΗΤΡΗΣ ΙΩΑΝΝΟΥ )

Να βρεθεί ο αριθμός των ζευγών (m,n) θετικών ακεραίων που είναι μικρότεροι

ή ίσοι του 1000 και τέτοιοι ώστε: m m 1

2n 1 n

.

Λύση:

H ανίσωση είναι ισοδύναμη με την 2n 1 m 2(n 1) (*).

Συμβολίζουμε με 1

K το σύνολο των ακεραίων έως το 1000 που περιέχονται στο

διάστημα ( 2 1,2 2) , με 2

K το σύνολο των ακεραίων έως το 1000 που

περιέχονται στο διάστημα (2 2 1,3 2) , με 3

K το σύνολο των ακεραίων έως

το 1000 που περιέχονται στο διάστημα (3 2 1,4 2) .

... ... ... ... ... ...

με 707

K το σύνολο των ακεραίων έως το 1000 που περιέχονται στο διάστημα

(707 2 1,708 2) .

(Ο δείκτης 707 εντοπίζεται με δοκιμές αν κάνουμε χρήση της πληροφορίας

2 1,414... )

Σταματάμε στο συγκεκριμένο δείκτη γιατί στο σύνολο 707

K ανήκει το 1000 και

τα σύνολα 708 709

K K, ,... είναι κενά. Συμβολίζουμε με n

K το πλήθος των

στοιχείων του συνόλου n

K . Ο ζητούμενος αριθμός είναι το άθροισμα

1 2 707S | K | | K | ... | K | .

Τα σύνολα 1 2 707

K ,K ,...,K έχουν συνολικά 1000 ακριβώς στοιχεία,

συγκεκριμένα τους αριθμούς 1,2,3,...,1000 . Το ζητούμενο πλήθος S είναι

μεγαλύτερο του 1000 αφού τα σύνολα 1 2 707

K ,K ,...,K έχουν κοινά στοιχεία.

Για παράδειγμα ο αριθμός 2 ανήκει και στο 1

K και στο 2

K .

Page 328: ΠΡΟΤΕΙΝΟΜΕΝΕΣ ΑΣΚΗΣΕΙΣ ΓΙΑ ΜΑΘΗΤΙΚΟΥΣ ΔΙΑΓΩΝΙΣΜΟΥΣ_ALL

http://www.mathematica.gr/forum/viewtopic.php?f=109&t=15584

Επιμέλεια: xr.tsif Σελίδα 46

Θα πρέπει το 2 να το μετρήσουμε δύο φορές.

Τα σύνολα 1 2 707

K ,K ,...,K μπορούν να έχουν κοινά στοιχεία μόνο αν οι δείκτες

τους είναι διαδοχικοί αριθμοί.

Τα σύνολα n n 1

K ,K

για n 1,2,...,706 ''συναντώνται'' στο διάστημα

(n 1) 2 1,(n 1) 2 . Αυτό το διάστημα έχει πλάτος ακριβώς 1 και έχει

άκρα άρρητους αριθμούς, άρα περιέχει ακριβώς έναν ακέραιο.

Επομένως έχουμε ακριβώς 706 ακεραίους που θα πρέπει να μετρηθούν από δύο

φορές. Άρα το ζητούμενο πλήθος ζευγών είναι 1000 706 1706 .

ΘΕΜΑ 353 (Socrates)

Να δείξετε ότι αν a b c 0 , τότε

a b c b c c a a b( )( ) 9b c c a a b a b c

.

Λύση:

Θέτουμε a b c

x, y, zb c c a a b

. Τότε έχουμε:

2 2 2x z (a c) (a c b) (c a) ( b b) 2(c a)

...y b(b c)(a b) b(b c)(a b) (b c)(a b)

.

Με όμοιο τρόπο δείχνουμε ότι:

2 2x y 2(a b) y z 2(b c)

,z (b c)(c a) x (c a)(a b)

.

Άρα: 3 3 3

x z x y y z (c a) (a b) (b c)2

y z x (c a)(a b)(b c)

(1).

Όμως (c a) (a b) (b c) 0 και άρα το άθροισμα των κύβων τους θα

ισούται με το τριπλάσιο γινόμενό τους.

Page 329: ΠΡΟΤΕΙΝΟΜΕΝΕΣ ΑΣΚΗΣΕΙΣ ΓΙΑ ΜΑΘΗΤΙΚΟΥΣ ΔΙΑΓΩΝΙΣΜΟΥΣ_ALL

http://www.mathematica.gr/forum/viewtopic.php?f=109&t=15584

Επιμέλεια: xr.tsif Σελίδα 47

Άρα η (1) γράφεται:

x z x y y z 3(c a)(a b)(b c)2 6

y z x (c a)(a b)(b c)

x z x y y z1 1 1 9

y y z z x x

x x x y y y z z z9

y z x z x y x y z

1 1 1 1 1 1 1 1 1x( ) y( ) z( ) 9

x y z x y z x y z

1 1 1(x y z)( ) 9

x y z , και άρα το ζητούμενο.

ΘΕΜΑ 354 (Socrates)

Να λυθεί το σύστημα 2 2 2 2

x y z t 6

1 x 4 y 9 z 16 t 8

.

Λύση:

Θα δώσω μια λύση, αλλά μάλλον θα υπάρχει και πιο απλή.

Αρχικά παρατηρούμε ότι

2 2 2 21 x 0,4 y 0,9 z 0,16 t 0 x 1, y 2, z 3, t 4 .

Για τις τετράδες των αριθμών:

1 x , 2 y , 3 z , 4 t και

1 x , 2 y , 3 z , 4 t εφαρμόζουμε την ταυτότητα του Lagrange και

έχουμε: (και με δεδομένο ότι :

Page 330: ΠΡΟΤΕΙΝΟΜΕΝΕΣ ΑΣΚΗΣΕΙΣ ΓΙΑ ΜΑΘΗΤΙΚΟΥΣ ΔΙΑΓΩΝΙΣΜΟΥΣ_ALL

http://www.mathematica.gr/forum/viewtopic.php?f=109&t=15584

Επιμέλεια: xr.tsif Σελίδα 48

1 x 0,1 x 0,2 y 0,2 y 0,3 z 0,3 z 0,4 t 0,4 t 0 ),

(1 x 2 y 3 z 4 t)(1 x 2 y 3 z 4 t)

2

2 2 2 21 x 4 y 9 z 16 t

2 2

(1 x)(2 y) (1 x)(2 y) 1 x)(3 z) (1 x)(3 z)

2

... (3 z)(4 t) (3 z)(4 t)

22

16 4 8 (1 x)(2 y) (1 x)(2 y) ...

2

(3 z)(4 t) (3 z)(4 t)

2 2

0 (1 x)(2 y) (1 x)(2 y) ... (3 z)(4 t) (3 z)(4 t)

Από την ισότητα αυτή, έχουμε το εξής σύστημα ανεξάρτητων εξισώσεων:

(1 x)(2 y) (1 x)(2 y)

(1 x)(3 z) (1 x)(3 z)

(1 x)(4 t) (1 x)(4 t) . Άρα: y 2x , z 3x , t 4x .

Και επειδή x y z t 6 , θα έχουμε x 2x 3x 4x 6 και άρα 6

x10

.

Οπότε τελικά είναι: 3 6 9 12

x ,y ,z ,t5 5 5 5

.

ΘΕΜΑ 355 (Socrates)

Έστω 1 2 3 2011

(a ,a ,a ,...,a ) μια αναδιάταξη των αριθμών 1,2,3,...,2011 . Δείξτε ότι

υπάρχουν j,k έτσι ώστε 1 j k 2011 και j k

| a j | | a k | .

Λύση:

Ωραίο πρόβλημα αλλά μάλλον δύσκολο.

Page 331: ΠΡΟΤΕΙΝΟΜΕΝΕΣ ΑΣΚΗΣΕΙΣ ΓΙΑ ΜΑΘΗΤΙΚΟΥΣ ΔΙΑΓΩΝΙΣΜΟΥΣ_ALL

http://www.mathematica.gr/forum/viewtopic.php?f=109&t=15584

Επιμέλεια: xr.tsif Σελίδα 49

Έστω ότι έχουμε 2011 κουτιά τα οποία ονομάζουμε 0,1,2,...,2010 . Για κάθε

1 i 2011 , βάζουμε το i στο κουτί με την ονομασία i

| a i | . Αν υπάρχουν

j k που μπήκαν στο ίδιο κουτί τότε τελειώσαμε. Αν αυτό δεν ισχύει, τότε

επειδή έχουμε 2011κουτιά και 2011αριθμούς, κάθε κουτί πρέπει να περιέχει

ακριβώς ένα αριθμό.

Έστω ότι τον αριθμό 1 i 2011 τον βάλαμε στο κουτί f (i) . Δηλαδή τα

f (1), ,f (2011) είναι οι αριθμοί 0,1,2,...,2010 με κάποια σειρά. Επειδή όμως

1 2 2011(a 1) (a 2) (a 2011) 0 , πρέπει να μπορούμε να βρούμε

κατάλληλα πρόσημα ώστε να ισχύει η ισότητα 1 2 2010 0 .

Ισχυρίζομαι όμως πως όποια πρόσημα και να βάλουμε ο αριθμός είναι περιττός.

(Οπότε αυτό είναι αδύνατο και η απόδειξη θα έχει ολοκληρωθεί.) Πράγματι αν

όλα τα πρόσημα είναι , τότε το άθροισμα ισούται με

20092010 1005 2009

2 είναι περιττός. Αν τώρα αλλάξουμε το πρόσημο των

αριθμών 1 r

a , ,a από σε το άθροισμα θα μειωθεί κατά 1 r

(a a )2

οπότε το αποτέλεσμα θα είναι πάλι περιττός.

ΘΕΜΑ 356 (Socrates)

Ένας φυσικός αριθμός n βρίσκεται αυστηρά μεταξύ δυο διαδοχικών τελείων

τετραγώνων. Το μικρότερο από αυτά τα τετράγωνα προκύπτει με αφαίρεση k

από τον n ενώ το μεγαλύτερο προσθέτοντας στον n . Δείξτε ότι ο αριθμός

n k είναι τέλειο τετράγωνο ακεραίου.

Λύση:

Αν 2a  είναι το μικρότερο από τα παραπάνω τετράγωνα τότε εξ' υποθέσεως

έχουμε:

2n k a (1)

2n (a 1)   (2)

Page 332: ΠΡΟΤΕΙΝΟΜΕΝΕΣ ΑΣΚΗΣΕΙΣ ΓΙΑ ΜΑΘΗΤΙΚΟΥΣ ΔΙΑΓΩΝΙΣΜΟΥΣ_ALL

http://www.mathematica.gr/forum/viewtopic.php?f=109&t=15584

Επιμέλεια: xr.tsif Σελίδα 50

2(1) n a k (3)

2(2) n (a 1) , άρα 2 2

 a k (a 1) 2a 1 k  (4).

Από (3) , (4) είναι

2 2 2 2n k a k k(2a 1 k) a k 2ak k k (a k) .

ΘΕΜΑ 357 (Socrates)

Να εξετάσετε αν υπάρχει ακέραιοςn τέτοιος ώστε η εξίσωση 4 2

x 2011x n 0 να έχει 4 ακέραιες ρίζες.

Λύση:

Ας υποθέσουμε ότι mείναι μια ακέραια ρίζα της δοσμένης εξίσωσης

Τότε αφού m είναι ρίζα, θα πρέπει 4 2 4 2m 2011m n 0 n m 2011m

Τότε η εξίσωση γράφεται:

4 2 4 2 2 2 2 2 2 2x 2011x m 2011m 0 (x m )(x m ) 2011(x m ) 0

2 2 2 2(x m )(x m 2011) 0 2 2

x m 2011 0 ή 2 2x m .

Άρα x m ή 2 2x m 2011 .

Θα αποδείξουμε τώρα, ότι η εξίσωση 2 2x m 2011 δεν έχει λύση στο σύνολο

των ακεραίων

Διακρίνουμε τις περιπτώσεις:

1η ΠΕΡΙΠΤΩΣΗ: m 2k , k Z .

Τότε 2 2 2x 4k 2011 x είναι περιττός άρα και ο x είναι περιττός και άρα

x 2t 1 όπου t Z . Άρα 2 24t 4t 4k 2010 που όμως είναι άτοπο αφού ο

4 δεν διαιρεί τον 2010 .

2η ΠΕΡΙΠΤΩΣΗ: m 2k 1 , k Z .

Page 333: ΠΡΟΤΕΙΝΟΜΕΝΕΣ ΑΣΚΗΣΕΙΣ ΓΙΑ ΜΑΘΗΤΙΚΟΥΣ ΔΙΑΓΩΝΙΣΜΟΥΣ_ALL

http://www.mathematica.gr/forum/viewtopic.php?f=109&t=15584

Επιμέλεια: xr.tsif Σελίδα 51

Τότε 2 2 2 2x (2k 1) 2011 x 2011 (2k 1) και επειδή η διαφορά

περιττών είναι άρτιος άρα ο 2x είναι άρτιος και άρα και ο x είναι άρτιος. Άρα

x 2t , t Z . Τότε θα έχουμε:

2 2 2 2 2 2(2t) 2011 (2k 1) 4t 4k 4k 1 2011 4(t k k) 2010 ,

που όμως είναι άτοπο, αφού ο 2010 δεν είναι πολλαπλάσιο του 4 .

Άρα η δοσμένη εξίσωση δεν μπορεί να έχει 4 ακέραιες ρίζες.

ΘΕΜΑ 358 (ΔΗΜΗΤΡΗΣ ΙΩΑΝΝΟΥ )

Θεωρούμε το σύνολο A 1,2,3,...,100 καθώς και τα υποσύνολα αυτού :

1 2 50B b ,b ,...,b , όπου

1 2 50b b ... b και 1 2 50

C c ,c ,...,c , όπου

1 2 50c c ... c . Τα σύνολα B,C είναι ξένα μεταξύ τους.

Να αποδειχθεί ότι : 1 1 2 2 50 50

b c b c ... b c 2500 .

Λύση:

Σε κάθε ποσότητα της μορφής i i

b c με i 1,2,...,50 πρέπει ο ένας όρος να

είναι μεγαλύτερος του 50 και ο άλλος μικρότερος ή ίσος του 50 ,γιατί αν ήταν

και οι 2 μεγαλύτεροι του 50 για παράδειγμα θα είχαμε i (50 i 1) 51

ακέραιους αριθμούς μεγαλύτερους του 50 και μικρότερους του 100 ,άτοπο.

Όμοια αν και οι δυο αριθμοί είναι μικρότεροι του 50 .

Η παράσταση τελικά κάνει:

51 52 ... 100 (1 2 ... 50) 50 50 (1 2 ... 50) (1 2 ... 50)

2500 .

Page 334: ΠΡΟΤΕΙΝΟΜΕΝΕΣ ΑΣΚΗΣΕΙΣ ΓΙΑ ΜΑΘΗΤΙΚΟΥΣ ΔΙΑΓΩΝΙΣΜΟΥΣ_ALL

http://www.mathematica.gr/forum/viewtopic.php?f=109&t=15584

Επιμέλεια: xr.tsif Σελίδα 52

ΘΕΜΑ 359 (vzf)

Για κάθε m,x,y,N,k και k 0 , να δείξετε ότι:

2 22

2 2 mx y m N my NxNx k y N

k k k

.

Λύση:

Θα αποδείξουμε ότι:

2 2 2mx y my Nx N m

Nk k k

.

Ξεκινάμε από το πρώτο μέλος:

2 2 2 2 2 2 2 2 2

2

mx y my Nx N(m x 2mxy y ) m y N x 2mnxyN

k k k

2 2 2 2 2 2 2 2 2 2

2 2 2

m (Nx y ) N(Nx y ) (m N)(Nx y ) (m N)( k) N m

k k k k

.

ΘΕΜΑ 360 (vzf)

Να λύσετε την ανισότητα για θετικά x :

x(8 1 x 1 x) 11 1 x 16 1 x .

Λύση:

Έχουμε από την υπόθεση x 0 , ενώ επί πλέον πρέπει 1 x 0 x 1 .

Άρα τελικά πρέπει 0 x 1 .

Τώρα η δοσμένη ανίσωση γράφεται:

8x 1 x x 1 x 11 1 x 16 1 x 8(x 2) 1 x (11 x) 1 x

Και αφού είναι 0 x 1 τότε θα έχουμε ισοδύναμα:

Page 335: ΠΡΟΤΕΙΝΟΜΕΝΕΣ ΑΣΚΗΣΕΙΣ ΓΙΑ ΜΑΘΗΤΙΚΟΥΣ ΔΙΑΓΩΝΙΣΜΟΥΣ_ALL

http://www.mathematica.gr/forum/viewtopic.php?f=109&t=15584

Επιμέλεια: xr.tsif Σελίδα 53

2 2 3 264(x 2) (1 x) (11 x) (1 x) ... 65x 171x 99x 135 0 .

Με το σχήμα του HORNER βρίσκουμε:

23 3(x )(65x 210x 225) 0 x

5 5 και αφού είναι και

30 x 1 x 1

5 .

ΘΕΜΑ 361 (vzf)

Πόσα ψηφία έχει ο αριθμός 100125 ;

Λύση:

100 3 100 300125 (5 ) 5 . Ψάχνουμε φυσικό k ώστε k 300 k 1

10 5 10 .

Ξεκινάμε από την παρατήρηση 3 75 2 . Υψώνουμε και τα δύο μέλη στην 30 και

μετά πολλαπλασιάζουμε κατά μέλη με 2105 . Προκύπτει 300 210

5 10 .

Ας εξετάσουμε αν 209 30010 5 . Είναι αρκετό να δείξουμε ότι 209 91

2 5 .

Είναι 22 5 οπότε θα ήταν αρκετό 207 90

2 5 Οι 207,90 διαιρούνται με το 9 . Θα

αρκούσε λοιπόν να δείξουμε ότι 23 102 5 . Όμως αυτό ισχύει αφού

23 22 1024 ·8 1.100.000·8 9.000.000 ενώ 10 2

5 3125 9.000.000 .

Άρα 209 300 21010 5 10 , οπότε το πλήθος των ψηφίων του 300

5 είναι 210 .

ΘΕΜΑ 362 (ΔΗΜΗΤΡΗΣ ΙΩΑΝΝΟΥ )

Αν 5 3a a a 3 να αποδείξετε ότι:

α) a 0 .

β) 6a 5 .

Page 336: ΠΡΟΤΕΙΝΟΜΕΝΕΣ ΑΣΚΗΣΕΙΣ ΓΙΑ ΜΑΘΗΤΙΚΟΥΣ ΔΙΑΓΩΝΙΣΜΟΥΣ_ALL

http://www.mathematica.gr/forum/viewtopic.php?f=109&t=15584

Επιμέλεια: xr.tsif Σελίδα 54

Λύση:

6

5 3 4 2

2

a 10 3 a a a a(a a 1) a

a 1

. Αφού ο παράγοντας

6

2

a 1

a 1

είναι θετικός, έπεται και ο άλλος, ο a θα είναι θετικός.

Επίσης, αφού 2a 1 2a , ισοδύναμα

2

a 1

a 1 2

, έχουμε

6

6

2

a 1 10 3 a· (a 1)

a 1 2

, οπότε 6

a 5 .

Β τρόπος

Διαφορετικά :

α) 5 3 4 2 2 21 30 a a a a(a a 1) a[(a ) ]

2 4 , άρα a 0 .

β) Πολλαπλασιάζουμε τη δοσμένη σχέση με 2a 1 και παίρνουμε

6 3 3a 3a 1 2 3a· 1 5

a a .

β τρόπος για το β)

πιο απλά ...

6 6 6 5 3 2 4 2a 5 a 2 3 1 a 2(a a a) 1 (a 1) (a a 1) 0 .

ΘΕΜΑ 363 (ΔΗΜΗΤΡΗΣ ΙΩΑΝΝΟΥ )

Τα γράμματα της λέξης ΘΑΛΗΣ μπορούν να μετατεθούν με όλους τους

δυνατούς τρόπους για να κάνουν 120 διαφορετικές λέξεις. Αν οι λέξεις αυτές

γραφούν με αλφαβητική σειρά (όπως στα λεξικά), ποιο θα είναι το τελευταίο

γράμμα της 111ης λέξης;

Page 337: ΠΡΟΤΕΙΝΟΜΕΝΕΣ ΑΣΚΗΣΕΙΣ ΓΙΑ ΜΑΘΗΤΙΚΟΥΣ ΔΙΑΓΩΝΙΣΜΟΥΣ_ALL

http://www.mathematica.gr/forum/viewtopic.php?f=109&t=15584

Επιμέλεια: xr.tsif Σελίδα 55

Λύση:

αλφαβητικά έχουμε : Α,Η,Θ,Λ,Σ

με πρώτο γράμμα :

το Α, υπάρχουν 4! 24 λέξεις

το H, υπάρχουν 4! 24 λέξεις

το Θ, υπάρχουν 4! 24 λέξεις

το Λ, υπάρχουν 4! 24 λέξεις

---------------------------------------------------μέχρι εδώ έχουμε 96 .

με πρώτο το Σ και δεύτερο το Α, έχουμε 3! 6 λέξεις ... 96 6 102 .

με πρώτο το Σ και δεύτερο το Η, έχουμε 3! 6 λέξεις... 102 6 108 .

με πρώτο το Σ και δεύτερο το Θ, έχουμε 3! 6 λέξεις... 108 6 114 .

------------------------------------------------------

άρα η 109η λέξη είναι η : ΣΘΑΗΛ

η 110η είναι η : ΣΘΑΛΗ

η 111η είναι η : ΣΘΗΑΛ.

ΘΕΜΑ 364 (ΔΗΜΗΤΡΗΣ ΙΩΑΝΝΟΥ )

Να εξετάσετε αν υπάρχουν φυσικοί αριθμοί k,nώστε το τρίγωνο με πλευρές

k (n!),2 (n 1)!,(n 2)! να είναι ορθογώνιο.

Λύση:

Δεν υπάρχουν.

Page 338: ΠΡΟΤΕΙΝΟΜΕΝΕΣ ΑΣΚΗΣΕΙΣ ΓΙΑ ΜΑΘΗΤΙΚΟΥΣ ΔΙΑΓΩΝΙΣΜΟΥΣ_ALL

http://www.mathematica.gr/forum/viewtopic.php?f=109&t=15584

Επιμέλεια: xr.tsif Σελίδα 56

Επειδή είναι 2(n 1)! 2(n 1)n! και (n 2)! (n 2)(n 1)n! , αρκεί να

αποδείξουμε ότι τα k,2(n 1),(n 1)(n 2) δεν μπορεί να είναι πλευρές

ορθογωνίου τριγώνου.

Αν k max{k,2(n 1),(n 1)(n 2)} , αρκεί να αποδείξουμε ότι η εξίσωση

2 2 2k [(n 1)(n 2)] 4(n 1) (1) είναι αδύνατη στους φυσικούς.

Πράγματι, είναι 2 2 2 2[(n 1)(n 2)] 4(n 1) (n 1) (n 4n 8)

και επειδή 2 2 2(n 2) n 4n 8 (n 3) , η (1) είναι αδύνατη.

Αν τώρα, n 2 max{k,2(n 1),(n 1)(n 2)} , αρκεί να αποδείξουμε ότι είναι

αδύνατη η εξίσωση 2 2 2 2(n 2) (n 1) k 4(n 1) δηλαδή είναι

2 2 2k (n 4n)(n 1) (2).

Και πάλι, το 2n 4n βρίσκεται ανάμεσα σε δύο διαδοχικά τετράγωνα,

2 2 2(n 1) n 4n (n 2) , οπότε η (2) είναι αδύνατη.

ΘΕΜΑ 365 (ΜΠΑΜΠΗΣ ΣΤΕΡΓΙΟΥ )

Αν a,b,c 0 , με a b c abc , να αποδειχθεί ότι: 2 2 2(1 a )(1 b )(1 c ) 64 .

Λύση:

Κάνοντας πράξεις αρκεί ν.δ.ο. 2 2 2 2 2 2 2 2 2 2 2 2a b c a b c a b c a b c 63 .

Επίσης έχω: 2 2 2 2 2 2 2 2 2a b c a b c abc(a b c) a b c (1), και

2 2 23a b c 3 abc a b c 27 (2).

Από (1) και (2) αρκεί ν.δ.ο. 2 2 2a b c 9 .

Αλλά 2 2 2 2 2 23a b c 3 a b c 9 , από AM – GM.

Page 339: ΠΡΟΤΕΙΝΟΜΕΝΕΣ ΑΣΚΗΣΕΙΣ ΓΙΑ ΜΑΘΗΤΙΚΟΥΣ ΔΙΑΓΩΝΙΣΜΟΥΣ_ALL

http://www.mathematica.gr/forum/viewtopic.php?f=109&t=15584

Επιμέλεια: xr.tsif Σελίδα 57

Β τρόπος

Καταρχάς, από την ανισότητα ΑΜ – ΓΜ είναι

2 23abc a b c 3 (abc) (abc) 27 (1).

Από την ανισότητα Huygens (η οποία είναι άμεση συνέπεια της ανισότητας

Hölder) έχουμε

2 2 2 2 3 333

(1)

(1 a )(1 b )(1 c ) 1 (abc) (1 27) 64( ) .

Γ τρόπος (και κατασκευαστικά...)

Ισχύει 2 a b c (a b)(a c)1 a 1 1 a

bc bc

.

Οπότε, αρκεί να δείξουμε ότι

2 2 2(a b)(a c)(b c)(b a)(c a)(c b) 64a b c ή

(a b)(b c)(c a) 8abc , όπου ισχύει απο AM – GM.

ΘΕΜΑ 366 (sokratis lyras)

Έστω a,b θετικοί πραγματικοί αριθμοί. Να αποδειχθεί ότι:

2(a b) a b

a b a2 4

.

Λύση:

Είναι a b

ab2

(1). Επίσης, είναι 21 1

a a a2 4

( ) (2) ,

και ομοίως 1

b b4

(3).

Προσθέτουμε κατά μέλη τις (2) , (3) και την ανισότητα που προκύπτει την

πολλαπλασιάζουμε κατά μέλη με την (1), οπότε προκύπτει η ζητούμενη.

Page 340: ΠΡΟΤΕΙΝΟΜΕΝΕΣ ΑΣΚΗΣΕΙΣ ΓΙΑ ΜΑΘΗΤΙΚΟΥΣ ΔΙΑΓΩΝΙΣΜΟΥΣ_ALL

http://www.mathematica.gr/forum/viewtopic.php?f=109&t=15584

Επιμέλεια: xr.tsif Σελίδα 58

Β τρόπος

Από ΑΜ – ΓΜ έχω: 24a 4ab a b 8a b . Ομοίως για το b ,διαιρώ τις

σχέσεις με 8 και τις προσθέτω και προκύπτει το ζητούμενο.

ΘΕΜΑ 367 (sokratis lyras)

Να βρεθούν οι ακέραιες λύσεις της εξίσωσης: x x 4 3 29 3 y 2y y 2y .

Λύση:

Έχουμε : x x 4 3 2 x x 29 3 y 2y y 2y 3 (3 1) y(y 2)(y 1) .

Για x 0 έχουμε y 0 ή y 2 . Oι αριθμοί y , y 2 , 2y 1 είναι ανά 2

πρώτοι και δεδομένου ότι ο 2y 1 δεν είναι ποτέ πολλαπλάσιο του 3 θα πρέπει

x3 y ή x

3 y 2 και βλέπουμε ότι δεν έχει άλλες λύσεις.

Μοναδικές λύσεις οι (x,y) (0,0) , (0, 2) , (1,1) .

Β τρόπος

Η δοσμένη εξίσωση γράφεται:

x x 23 (3 1) y(y 2)(y 1) (1).

Αρχικά, βλέπουμε ότι για x 0 είναι y 0 ή y 2 ενώ για x 1 είναι y 1 ,

που είναι και η μόνη ακέραια ρίζα της εξίσωσης 26 y(y 2)(y 1) .

Άρα έχουμε βρει τις λύσεις (x,y) (0,0),(x,y) (0, 2),(x,y) (1,1) .

Θα αποδείξουμε τώρα ότι για κάθε x 0 , η (1) δεν έχει ακέραιες λύσεις,

Πράγματι, έχουμε:

x 0 x x x x xx 0 3 3 0 3 1 1 3 1 0 3 3 (3 1) 0 .

Άρα δεν είναι ακέραιος ο αριθμός 2y(y 2)(y 1) , πράγμα που είναι άτοπο.

Page 341: ΠΡΟΤΕΙΝΟΜΕΝΕΣ ΑΣΚΗΣΕΙΣ ΓΙΑ ΜΑΘΗΤΙΚΟΥΣ ΔΙΑΓΩΝΙΣΜΟΥΣ_ALL

http://www.mathematica.gr/forum/viewtopic.php?f=109&t=15584

Επιμέλεια: xr.tsif Σελίδα 59

Συνεπώς αν υπάρχουν άλλες ακέραιες λύσεις, θα είναι για x 1 με x φυσικό

αριθμό .

Θα διακρίνουμε τώρα δύο περιπτώσεις:

1η ΠΕΡΙΠΤΩΣΗ: x 2 , y 1

Παρατηρούμε ότι ο αριθμός 2y(y 2)(y 1) είναι πολλαπλάσιο του x

3 όπως

προκύπτει από την σχέση (1). Επίσης εύκολα διαπιστώνουμε ότι αν κάποιος από

τους 2y,(y 2),(y 1) είναι πολλαπλάσιος του 3 τότε οι δύο άλλοι δεν μπορούν

να είναι πολλαπλάσιοι του 3 . Αυτό σημαίνει ότι ένας μόνο από αυτούς είναι

πολλαπλάσιο του x3 . Οπότε διακρίνουμε τις παρακάτω υποπεριπτώσεις

(1Α) xy k 3

Τότε η σχέση (1) γράφεται:

x x x x 2 2x x x 2 2x3 (3 1) k 3 (k 3 2)(k 3 1) 3 1 k(k 3 2)(k 3 1)

η οποία όμως είναι αδύνατη, αφού προφανώς το δεύτερο μέλος είναι μεγαλύτερο

από το πρώτο.

(1Β) xy 2 k 3

Τότε με παρόμοιο τρόπο καταλήγουμε σε άτοπο.

(1Γ) Το ίδιο και όταν 2 xy 1 k 3

2η ΠΕΡΙΠΤΩΣΗ: x 2 , y 2

(ΣΗΜΕΙΩΣΗ: Η περίπτωση y 2 έχει εξεταστεί χωριστά, ενώ αν y 1 η

δοσμένη εξίσωση εύκολα βλέπουμε ότι είναι αδύνατη)

Πάλι διακρίνουμε τις εξής περιπτώσεις:

(2Α) xy k 3 , k αρνητικός ακέραιος

Page 342: ΠΡΟΤΕΙΝΟΜΕΝΕΣ ΑΣΚΗΣΕΙΣ ΓΙΑ ΜΑΘΗΤΙΚΟΥΣ ΔΙΑΓΩΝΙΣΜΟΥΣ_ALL

http://www.mathematica.gr/forum/viewtopic.php?f=109&t=15584

Επιμέλεια: xr.tsif Σελίδα 60

Τότε η σχέση (1) γράφεται: x x 2 2x3 1 k(k 3 2)(k 3 1) . Βλέπουμε ότι ο

αριθμός xk(k 3 2) είναι θετικός ακέραιος ενώ ο 2 2x x

(k 3 1) 3 1 για κάθε

x 2 και άρα η Σχέση (1) είναι αδύνατη

(2Β) xy 2 k 3 , k αρνητικός ακέραιος

Τότε η σχέση (1) γράφεται x x 2 2x x3 1 k(k 3 2)(k 3 4k 3 5)

Πάλι παρατηρούμε ότι ο αριθμός xk(k 3 2) είναι θετικός ακέραιος για κάθε

k αρνητικό ακέραιο και x 2 ενώ ο αριθμός 2 2x xk 3 4k 3 5 είναι

μεγαλύτερος από τον x3 1 για κάθε τέτοιο k και x 2 . Συνεπώς και πάλι η

σχέση (1) είναι αδύνατη.

(2Γ) 2 x

y 1 k 3 όπου αφού πρέπει ο k θετικός ακέραιος. Τότε πάλι όπως και

στην 1η ΠΕΡΙΠΤΩΣΗ καταλήγουμε σε άτοπο.

Τελικά οι μόνες λύσεις που βρήκαμε είναι οι

(x,y) (0,0),(x,y) (0, 2),(x,y) (1,1) .

Γ τρόπος

Αρχικά βρίσκουμε εύκολα τις προφανείς λύσεις της εξίσωσης που βρήκε ο

κύριος Δημήτρης παραπάνω. Στη συνέχεια θεωρούμε την εξίσωση τριώνυμο ως

προς x3 με Διακρίνουσα 4 3 2

4y 8y 8yΔ 4y 1 . Η Διακρίνουσα πρέπει

να είναι τέλειο τετράγωνο ακεραίου αλλά: 2 2 2 2(2y 2y) Δ (2y 2y 1) άρα

η Διακρίνουσα δεν είναι ποτέ τέλειο τετράγωνο και συνεπώς οι παραπάνω

λύσεις είναι μοναδικές.

Προκύπτουν δύο ερωτήματα :

(α) Πως βρίσκουμε τις ''προφανείς'' λύσεις , αν δεν ακολουθήσουμε τον τρόπο

του Δημήτρη ; Διότι θα μπορούσε πχ κάποιος να είχε βρει μόνο τις δύο από τις

3 (νομίζω) λύσεις και με τον τρόπο σου να δικαιολογήσει ότι είναι και οι

μοναδικές (που θα ήταν τότε το λάθος;)

Page 343: ΠΡΟΤΕΙΝΟΜΕΝΕΣ ΑΣΚΗΣΕΙΣ ΓΙΑ ΜΑΘΗΤΙΚΟΥΣ ΔΙΑΓΩΝΙΣΜΟΥΣ_ALL

http://www.mathematica.gr/forum/viewtopic.php?f=109&t=15584

Επιμέλεια: xr.tsif Σελίδα 61

(β) Γιατί να ισχύει 2 2Δ (2y 2y) ; Αυτό δεν ισχύει όταν 8y 1 0 ; Αλλά που

το ξέρουμε αυτό ,αφού y Z

Μπορούμε να πούμε γι' αυτήν την άσκηση ότι αν πολλαπλασιάσουμε με 4 και

προσθέσουμε τον άσσο, τότε στο αριστερό μέλος προκύπτει ένα τέλειο

τετράγωνο, άρα το δεξί πρέπει να είναι και αυτό τέλειο τετράγωνο. Βρίσκουμε

με τις ανισότητες ποιο πρέπει να είναι και έτσι βρίσκουμε το y . Μετά όλα είναι

απλά.

Για το (α) Σε τέτοιου είδους ασκήσεις βλέπουμε πρώτα τί γίνεται στις

τετριμμένες περιπτώσεις 2, 1,0,1,2,3 (ανάλογα με την άσκηση) επειδή

πολλές φορές η λύση που ακολουθεί ισχύει π.χ. για x 3 ή κάτι ανάλογο.

Δηλαδή οι 'προφανείς' λύσεις θα είναι μοναδικές ή όχι ανάλογα με την συνέχεια

της επίλυσης αφού η συνέχεια της λύσης θα αφορά όλους τους ακέραιους εκτός

των περιπτώσεων που έχουμε πάρει στην αρχή. Συγκεκριμένα τώρα στη λύση

μου υπάρχει ένα κενό, το οποίο θα κοιτάξω και θα επανέλθω. Ελπίζω να είμαι

κατανοητός.

Δεν εξετάζουμε εκ των προτέρων τις τετριμμένες περιπτώσεις διότι απλά δεν τις

γνωρίζουμε. Αν σε κάποιες ασκήσεις δεις εκ των προτέρων να εξετάζονται

κάποιες τετριμμένες περιπτώσεις τότε αυτό σημαίνει ότι ο λύτης απλά έχει

επιλέξει να τις παραθέσει στην αρχή (όμως αυτό έχει γίνει ΑΦΟΥ έχει λυθεί η

άσκηση στο χαρτί οπότε στην πορεία της άσκησης παρακάτω πρέπει να φαίνεται

γιατί δε χρειάζεται να εξετάσει κάποιες άλλες περιπτώσεις με το χέρι). Ο

αριθμός των περιπτώσεων που πρέπει να εξεταστούν με το χέρι μπορεί να

αλλάζει από πρόβλημα σε πρόβλημα ή ακόμα και από λύση σε λύση αν

ακολουθηθεί άλλη μέθοδος. Θα προσπαθήσω λοιπόν να είμαι όσο πιο

αναλυτικός γίνεται παρακάτω.

Καταρχήν η παραπάνω σχέση που έχω σημειώσει ως (1) στη λύση του Σωκράτη

ισχύει για όλα τα y 2 (το αριστερό μέλος της ανισότητας καταλήγει στην

8y 1 0 που ισχύει για y 1 και το δεξί καταλήγει στην σχέση 2y y 0 που

ισχύει για y 2 ). Άρα για y 2 είμαστε βέβαιοι ότι η ποσότητα Δ δεν είναι

τέλειο τετράγωνο ακεραίου. Μόλις έφυγε ένα μεγάλο βάρος από πάνω μας!

Όλοι οι θετικοί οι μεγαλύτεροι ή ίσοι του 2 .

Page 344: ΠΡΟΤΕΙΝΟΜΕΝΕΣ ΑΣΚΗΣΕΙΣ ΓΙΑ ΜΑΘΗΤΙΚΟΥΣ ΔΙΑΓΩΝΙΣΜΟΥΣ_ALL

http://www.mathematica.gr/forum/viewtopic.php?f=109&t=15584

Επιμέλεια: xr.tsif Σελίδα 62

Για y 4 ισχύει ότι 2 2 2 2(2y 2y 1) Δ (2y 2y) (το αριστερό μέλος

καταλήγει στην y(y 3) 0 που ισχύει για y 4 και το δεξί καταλήγει στην

8y 1 0 που ισχύει για y 1 ). Άρα για y 4 είμαστε βέβαιοι ότι η

ποσότητα Δ δεν είναι τέλειο τετράγωνο ακεραίου. Μόλις έφυγε άλλο ένα

μεγάλο βάρος από πάνω μας! Όλοι οι αρνητικοί οι μικρότεροι ή ίσοι του 4 .

Έχουμε "διώξει" λοιπόν από τη μέση τις "πολλές" περιπτώσεις για το y .

Τι έμεινε; Να εξετάσουμε ξεχωριστά τις "λίγες" περιπτώσεις y 3, 2, 1,0,1 .

Για y 3, 1 δεν έχουμε λύσεις.

Για y 2 παίρνουμε x 0 άρα έχουμε τη λύση (x,y) (0, 2) .

Για y 0 παίρνουμε x 0 άρα έχουμε τη λύση (x,y) (0,0)

Για y 1 παίρνουμε x 1 άρα έχουμε τη λύση (x,y) (1,1)

Συμπέρασμα: Πρώτα απορρίπτουμε με κάποιο τρόπο (όπως παραπάνω) τις

"πολλές" τιμές του y και μετά εξετάζουμε με το χέρι εκείνες τις "λίγες" που

έμειναν.

ΘΕΜΑ 368 (sokratis lyras)

Έστω a,b,c,d,e,f ,g,h,k διαφορετικά ανά δύο μη μηδενικά ψηφία. Βρείτε το

ελάχιστο του E abc def ghk .

Λύση:

Είναι από ΑΜ – GΜ

33E abc def ghk 3 abcdefghk 3 1 2 3 4 5 6 7 8 9 213

οπότε, αφού E είναι E 214 .

Η τιμή αυτή πιάνεται π.χ.

γιαa 1,b 8,c 9,d 3,e 4,f 6,g 2,h 5,k 7 .

Page 345: ΠΡΟΤΕΙΝΟΜΕΝΕΣ ΑΣΚΗΣΕΙΣ ΓΙΑ ΜΑΘΗΤΙΚΟΥΣ ΔΙΑΓΩΝΙΣΜΟΥΣ_ALL

http://www.mathematica.gr/forum/viewtopic.php?f=109&t=15584

Επιμέλεια: xr.tsif Σελίδα 63

ΘΕΜΑ 369 (sokratis lyras)

Δίνονται a,b,c,x,y,z θετικοί αριθμοί. Να αποδειχθεί ότι:

a(y z) b(x z) c(y x)3(xy yz zx)

b c a c b a

.

Λύση:

Πρόκειται για κλασική, αλλά δυσκολούτσικη ανισότητα.

Ας θέσουμε για ευκολία a b c

A ,B ,Cb c c a a b

.

Τότε είναι 3

AB BC CA4

(1) (απλές πράξεις).

Τώρα, είναι από την ανισότητα Cauchy – Schwarz

a(y z) b(x z) c(y x)A(y z) B(z x) C(x y)

b c a c b a

(A B C)(x y z) (Ax By Cz)

2 2 2 2 2 2A B C 2(AB BC CA) x y z 2(xy yz zx)

(Ax By Cz) 2 (AB BC CA)(xy yz zx)

2 2 2 2 2 2(A B C )(x y z ) (Ax By Cz)

2 (AB BC CA)(xy yz zx) , οπότε, λόγω της (1), προκύπτει η

ζητούμενη.

Β τρόπος

Ισχύει 2[a(x y) b(y z) c(z x)] 4(ab bc ca)(xy yz zx) .

Για c a b

(a,b,c) , ,a b b c c a

η ανισότητα γίνεται

Page 346: ΠΡΟΤΕΙΝΟΜΕΝΕΣ ΑΣΚΗΣΕΙΣ ΓΙΑ ΜΑΘΗΤΙΚΟΥΣ ΔΙΑΓΩΝΙΣΜΟΥΣ_ALL

http://www.mathematica.gr/forum/viewtopic.php?f=109&t=15584

Επιμέλεια: xr.tsif Σελίδα 64

2

a(y z) b(x z) c(y x)

b c a c b a

ab bc ca4(xy yz zx)

(b c)(a c) (a c)(a b) (a b)(b c)

Όμως, ab bc ca 3

(b c)(a c) (a c)(a b) (a b)(b c) 4

γιατί ισοδύναμα

έχουμε ότι

3ab(a b) (a b)(b c)(c a) ab(a b) 6abc

4 , η οποία ισχύει

λόγω της AM – GM.

Άρα, ισχύει 2

a(y z) b(x z) c(y x) 34(xy yz zx)· 3(xy yz zx)

b c a c b a 4

.

ΘΕΜΑ 370 (dimitris.ligonis)

Για a,b,c θετικούς πραγματικούς, με abc 1 και 3a 36 , να δειχθεί ότι:

2

2 2ab c ab bc ac

3 .

Λύση:

Είναι abc 1 . Άρα 1

bca

. Αρκεί λοιπόν να αποδείξουμε ότι:

3 3

2 2 2a 1 a 1b c ab ac (b c) 2bc a(b c)

3 a 3 a

3 2

2 2a 2 1 a 3(b c) a(b c) (b c) a(b c) 0

3 a a 3 a (1).

Page 347: ΠΡΟΤΕΙΝΟΜΕΝΕΣ ΑΣΚΗΣΕΙΣ ΓΙΑ ΜΑΘΗΤΙΚΟΥΣ ΔΙΑΓΩΝΙΣΜΟΥΣ_ALL

http://www.mathematica.gr/forum/viewtopic.php?f=109&t=15584

Επιμέλεια: xr.tsif Σελίδα 65

Η Διακρίνουσα του τριωνύμου είναι: 3 4 3

2 a 3 4a 3a 36Δ a 4( )

3 a 3a

Στόχος είναι να αποδείξουμε ότι η Διακρίνουσα είναι αρνητικός αριθμός ή

μηδέν. Αρκεί λοιπόν να αποδείξουμε ότι

4 3 34a 3a 36 0 a (4a 3) 36 .

Όμως 3a 36 , από την υπόθεση και 4a 3 1 προφανώς αφού a 1 . Άρα

3a (4a 3) 36 και έτσι η (1) είναι αληθής άρα έχουμε και το ζητούμενο.

ΘΕΜΑ 371 (Socrates)

Αν a,b,c θετικοί πραγματικοί αριθμοί τέτοιοι ώστε abc 1 , να δείξετε ότι

2 2 21 1 12a 2b 2c

a b c 31 1 1

b 1 c 1 a 1a b c

.

Λύση:

2 2 1a a 3a

a και

1 ab a 1b 1

a a

.

Άρα αρκεί ν.δ.ο.: 2

aLHS 1

ab a 1

.

Αλλά από το λήμμα Cauchy (ή Andreescu) έχω ότι: 2

(a b c)LHS

ab bc ca a b c 3

.Αλλά:

2

2 2 2(a b c)1 a b c ab bc ca a b c 3

ab bc ca a b c 3

.

Αφού όμως: ab bc ca 3 αρκεί ν.δ.ο.: 2

2 2 2 (a b c)a b c a b c a b c a b c 3

3

που ισχύει.

Page 348: ΠΡΟΤΕΙΝΟΜΕΝΕΣ ΑΣΚΗΣΕΙΣ ΓΙΑ ΜΑΘΗΤΙΚΟΥΣ ΔΙΑΓΩΝΙΣΜΟΥΣ_ALL

http://www.mathematica.gr/forum/viewtopic.php?f=109&t=15584

Επιμέλεια: xr.tsif Σελίδα 66

ΘΕΜΑ 372 (Socrates)

Αν 1 2 n

x ,x ,...,x πραγματικοί αριθμοί τέτοιοι ώστε n 1

k k 1 1 nk 1

min(x ,x ) min(x ,x )

,

να δείξετε ότι n 1

kk 2

x 0

.

Λύση:

Έχουμε: n 1

k k 1 1 n 1 2 2 3 3 4k 1

min(x ,x ) min(x ,x ) min(x ,x ) min(x ,x ) min(x ,x ) ...

n 1 n 1 nmin(x ,x ) min(x ,x )

2 3 3 4 n 1 n 1 n 1 2min(x ,x ) min(x ,x ) ... min(x ,x ) min(x ,x ) min(x ,x )

n 1

k k 1 1 n 1 2k 2

min(x ,x ) min(x ,x ) min(x ,x )

.

Διακρίνουμε τώρα τις τρεις παρακάτω περιπτώσεις:

1η περίπτωση Έστω

1 2 3 nx x x ... x .

θα ισχύει:n 1

1 n 1 2 1 1 n 2 k k 1k 2

min(x ,x ) min(x ,x ) x x x x 0 min(x ,x ) 0

.

2η περίπτωση Έστω

1 2 3 nx x x ... x .

Τώρα ισχύει: n 1

1 n 1 2 1 1 k k 1k 2

min(x ,x ) min(x ,x ) x x 0 min(x ,x ) 0

.

3η περίπτωση Έστω

1 2 3 nx x x ... x .

Άρα: n 1

1 n 1 2 n 2 k k 1k 2

min(x ,x ) min(x ,x ) x x 0 min(x ,x ) 0

.

Page 349: ΠΡΟΤΕΙΝΟΜΕΝΕΣ ΑΣΚΗΣΕΙΣ ΓΙΑ ΜΑΘΗΤΙΚΟΥΣ ΔΙΑΓΩΝΙΣΜΟΥΣ_ALL

http://www.mathematica.gr/forum/viewtopic.php?f=109&t=15584

Επιμέλεια: xr.tsif Σελίδα 67

Συνδυάζοντας τώρα τα παραπάνω αποτελέσματα έχουμε: n 1

k k 1k 2

min(x ,x ) 0

.

Για τις περιπτώσεις 1. και 2. εύκολα προκύπτει ότι:

n 1

kk 2

x 0

(από την ώρα που k k 1 k

min(x ,x ) x

).Για την τρίτη περίπτωση δεν

καταλαβαίνω γιατί βρίσκω διαφορετικό αποτέλεσμα από αυτό της

άσκησης...Συγκεκριμένα, βρίσκω:

n 1

k k 1 n 2 2 3 3 4 n 1 nk 2

min(x ,x ) x x min(x ,x ) min(x ,x ) ... min(x ,x )

n 2 3 4 5 n n 2 3 4 5 n 1 2x x x x x ... x x x x x x ... x x

n 2

k 1 2k 2

x x

.

Δηλαδή: n 1 n 1 n 1 n 2 n 1

k k 1 k 1 k 1 n k nk 2 k 1 k 2 k 2 k 2

x x x x x x x 0

. Αποδείχτηκε με

λίγα λόγια οτι η σειρά που ψάχνουμε είναι μικρότερη του 0 , (άτοπο)!

ΘΕΜΑ 373 (Socrates)

Αν a,b,c θετικοί πραγματικοί αριθμοί τέτοιοι ώστε abc 1 , να δείξετε ότι

1 1 1ab bc bc ca ca ab (1 2a)(1 2b)(1 2c)

ca ab bc

.

Λύση:

Αρχικά είναι

1LHS (ab bc ) (ab bc b) b(a c 1) (a c 1)

ca .

Άρα αρκεί να δείξουμε ότι:

2(a c 1) (1 2a) ab(a b) a 2a 3ab 2abc 1

24ab 2a 8abc 1 ab(a b) a ab 6 .

Page 350: ΠΡΟΤΕΙΝΟΜΕΝΕΣ ΑΣΚΗΣΕΙΣ ΓΙΑ ΜΑΘΗΤΙΚΟΥΣ ΔΙΑΓΩΝΙΣΜΟΥΣ_ALL

http://www.mathematica.gr/forum/viewtopic.php?f=109&t=15584

Επιμέλεια: xr.tsif Σελίδα 68

Όμως ισχύει 2a ab και 2 2

ab(a b) a b b a 6 (από ΑΜ – ΓΜ)

και το ζητούμενο δείχθηκε.

Η ισότητα ισχύει αν και μόνον αν a b c 1 .

Β τρόπος

Κάνοντας τον μετασχηματισμό 1 1 1

(a,b,c) , ,x y z

με xyz 1 η ανισότητα

γίνεται (x xy y)(y yz z)(z zx x) (x 2)(y 2)(z 2) .

Αυτή όμως ισχύει, αφού από την CS έχουμε ότι 2 2

(x xy y)(yz z y) (1 1 y) (y 2) . Δουλεύοντας κυκλικά έχουμε

το ζητούμενο.

ΘΕΜΑ 374 (Socrates)

Αν x,y,z θετικοί πραγματικοί αριθμοί να δείξετε ότι

2

1 1 1 x y z(x y z) 4

x y z xy 1 yz 1 zx 1

.

Λύση:

Ισχύει απο τις ανισότητες CS και AM – GM ότι

2

2

x x x 1 1x x x

xy 1 (xy 1) 4xy 4 x

.

ΘΕΜΑ 375 (Socrates)

Να λυθεί η εξίσωση 2 22(a 1)(b 1) (a 1)(b 1)(ab 1) .

Page 351: ΠΡΟΤΕΙΝΟΜΕΝΕΣ ΑΣΚΗΣΕΙΣ ΓΙΑ ΜΑΘΗΤΙΚΟΥΣ ΔΙΑΓΩΝΙΣΜΟΥΣ_ALL

http://www.mathematica.gr/forum/viewtopic.php?f=109&t=15584

Επιμέλεια: xr.tsif Σελίδα 69

Λύση:

Ισχύει (π.χ. Cauchy – Schwarz ή πράξεις)

2 22(a 1) (a 1) και η ισότητα ισχύει αν και μόνο αν a 1 .

2 22(b 1) (b 1) και η ισότητα ισχύει αν και μόνο αν b 1 .

2 2 2(a 1)(b 1) (ab 1) και η ισότητα ισχύει αν και μόνο αν a b .

Με πολλαπλασιασμό αυτών βρίσκουμε

2 22(a 1)(b 1) (a 1)(b 1)(ab 1) .

Φανερά τώρα, το μοναδικό ζεύγος – λύση της εξίσωσης είναι το (1,1) .

ΘΕΜΑ 376 (Socrates)

Οι θετικοί πραγματικοί αριθμοί x,y,z είναι τέτοιοι ώστε 1 1 1

x y zx y z

.

Να δείξετε ότι 3 2

x y zx y z xyz

.

Λύση:

Απο CS έχουμε ότι 3 1 1

x y z 3 x

.

Μένει να δείξουμε ότι 2 2 1

xy yz zx 3xyz 3 x

.

Αυτή όμως ισχύει γιατί

2(xy yz zx) 3xyz(x y z) (xy yz zx) xy yz zx 3 , λόγω της

υπόθεσης xyz(x y z) xy yz zx .

Page 352: ΠΡΟΤΕΙΝΟΜΕΝΕΣ ΑΣΚΗΣΕΙΣ ΓΙΑ ΜΑΘΗΤΙΚΟΥΣ ΔΙΑΓΩΝΙΣΜΟΥΣ_ALL

http://www.mathematica.gr/forum/viewtopic.php?f=109&t=15584

Επιμέλεια: xr.tsif Σελίδα 70

ΘΕΜΑ 377 (Αρχιμήδης 6)

Να εξεταστεί αν η εξίσωση έχει λύση στους ακέραιους για x,y,z 3

x! y! z! 21 3 5 7 w .

Λύση:

Θα δείξουμε ότι η παραπάνω εξίσωση δεν έχει λύσεις στους ακεραίους. Ας

υποθέσουμε λοιπόν, αντίθετα, ότι έχει.

Λόγω του μικρού θεωρήματος του Fermat αφού (3,7) 1 και (5,7) 1 άρα

63 1 mod7 και 6

5 1 mod7 (*).

Συνεπώς αφού για x 3 έχουμε x! 0(mod6) , άρα 1

x! 6x οπότε

1

1x

xx! 63 3 1 1 mod7 .

Όμοια y!5 1 mod7 και τελικά παίρνοντας την αρχική εξίσωση mod7

παίρνουμε τελικά:

2w 1 1 1 0 3 mod7 . Όμως το 3 δεν είναι τετραγωνικό υπόλοιπο

mod7 κι έτσι καταλήγουμε σε άτοπο.

(*) Φυσικά για τη διαπίστωση αυτών δε χρειάζεται η γνώση του μικρού

θεωρήματος του Fermat αφού απλές πράξεις μπορούν να επαληθεύσουν τον

ισχυρισμό. Βοηθάει όμως η εκ των προτέρων γνώση του.

Β τρόπος

Θα δώσω και την δική μου σκέψη με την οποία κατασκεύασα την εξίσωση.

x! y! z! 21 3 5 7 w .

Αφού x,y,z 3 τότε x! , y! , z! 0(mod4)

Έτσι το τελευταίο ψηφίο του x!3 θα είναι το 1 .

Page 353: ΠΡΟΤΕΙΝΟΜΕΝΕΣ ΑΣΚΗΣΕΙΣ ΓΙΑ ΜΑΘΗΤΙΚΟΥΣ ΔΙΑΓΩΝΙΣΜΟΥΣ_ALL

http://www.mathematica.gr/forum/viewtopic.php?f=109&t=15584

Επιμέλεια: xr.tsif Σελίδα 71

Το τελευταίο ψηφίο του y!5 θα είναι το 5 .

Το τελευταίο ψηφίο του z!7 θα είναι το 1 .

Άρα το τελευταίο ψηφίο του x! y! z!1 3 5 7 θα είναι το 8 .

Όμως το 8 δεν μπορεί να είναι το τελευταίο ψηφίο ενός τέλειου τετραγώνου

αφού το τελευταίο των τετραγώνων είναι μόνο τα 1,4,5,6,9 .

Άρα δεν υπάρχουν λύσεις!

ΘΕΜΑ 378 (ΔΗΜΗΤΡΗΣ ΙΩΑΝΝΟΥ )

Δίνεται η συνάρτηση f με f (x) 0 για κάθε x πραγματικό αριθμό διάφορο του

μηδενός και επί πλέον ισχύει ότι 1 r

fx f (x)

για κάθε x R* όπου r είναι μη

τετράγωνος ρητός αριθμός . Να αποδείξετε ότι ο αριθμός

1 1 1P f(1) f (2) f (3) ... f (2012) f ( ) f ( ) ... f ( )

2 3 2012 είναι άρρητος.

Λύση:

1 1 1P f(1) f (2) f (3) ... f (2012) f ( ) f ( ) ... f ( )

2 3 2012

1 1 1f(1) f (2) f ( ) f (3) f ( ) ... f (2012) f ( )

2 3 2012

2011f (1) r r ... r f(1) r .

Αφού 1 r

fx f (x)

, για x 1 έχουμε 21 r

f f (1) r f (1) rf (1)1

.

Άρα 2011 2011P f(1) r r r και αφού ο r είναι μη τετράγωνος ρητός αριθμός

συμπεραίνουμε ότι ο αριθμός P είναι άρρητος.

Page 354: ΠΡΟΤΕΙΝΟΜΕΝΕΣ ΑΣΚΗΣΕΙΣ ΓΙΑ ΜΑΘΗΤΙΚΟΥΣ ΔΙΑΓΩΝΙΣΜΟΥΣ_ALL

http://www.mathematica.gr/forum/viewtopic.php?f=109&t=15584

Επιμέλεια: xr.tsif Σελίδα 72

ΘΕΜΑ 379 (Socrates)

Να βρεθούν οι μη αρνητικές ακέραιες λύσεις της εξίσωσης x y z

1005 2011 1006 .

Λύση:

Έχουμε x y z1005 2011 1006 . Προφανώς z 1 , x 0 .

Αν εξετάσουμε το τελευταίο διψήφιο τμήμα κάθε δύναμης ξεχωριστά θα δούμε

ότι...

x1005 25mod100 οπότε το τελευταίο διψήφιο τμήμα είναι το 25 .

10k 12011 11mod100

10k 22011 21mod100

.

.

.

10k 92011 91mod100

10k 102011 01mod100

.

z1006 36,16,96,76,56mod100 ( z 1 ) εύκολα βλέπουμε ότι y περιττός.

Αφού y περιττός τότε

y2011 1mod503 και z

1006 0mod503 . Οπότε x άρτιος.

Τώρα δουλεύω mod8 που βολεύει πολύ αφού ο z1006 διαιρείται με το 8 για

z 2 . Οπότε αφού x άρτιος θα έχουμε..

21005 1mod8 Οπότε x

1005 1mod8 ,

22011 1mod8 Οπότε y

2011 3mod8 .

Page 355: ΠΡΟΤΕΙΝΟΜΕΝΕΣ ΑΣΚΗΣΕΙΣ ΓΙΑ ΜΑΘΗΤΙΚΟΥΣ ΔΙΑΓΩΝΙΣΜΟΥΣ_ALL

http://www.mathematica.gr/forum/viewtopic.php?f=109&t=15584

Επιμέλεια: xr.tsif Σελίδα 73

Άρα x y1005 2011 4mod8 . Αυτό θα ισχύει μόνο αν z 3 .

Mένει να ελεγχθεί η περίπτωση που z 2 ή z 1 .

Πράγματι μοναδικές λύσεις (x,y,z) (2,1,2) και (x,y,z) (1,0,1) . Ξέχασα την

περίπτωση z 1 που μόλις την πρόσθεσα.

2 1 2 21005 2011 1006 1012036 1006 .

ΘΕΜΑ 380 (Socrates)

Έστω T ένα σύνολο θετικών ακεραίων τέτοιο ώστε 2 2 2 2a ab b / a b για κάθε

a,b T . Να δείξετε ότι το T είναι πεπερασμένο.

Λύση:

Έστω λοιπόν a T . Αρκεί να δείξουμε ότι για κάθε d / a υπάρχουν

πεπερασμένα b T με (a,b) d , όπου με (a,b) δηλώνουμε τον μέγιστο κοινό

διαιρέτη των a,b .

Αν λοιπόν b T και (a,b) d τότε μπορούμε να γράψουμε a Ad και b Bd

όπου (A,B) 1 . Επειδή a,b T έχουμε 2 2 2 4 2 2d (A AB B ) / d A B και άρα

2 2 2 2 2/A AB B d A B .

Επειδή (A,B) 1 , έχουμε 2 2 2(A AB B ,A) (B ,A) 1 . Ομοίως έχουμε

2 2(A AB B ,B) 1 και άρα 2 2 2 2

(A AB B ,A B ) 1 .

Επομένως 2 2 2A AB B d/ και άρα 2 2 4

a ab b d/ . Αλλά 2 2 2

a ab b (a b) ab ab .

Πρέπει λοιπόν 4b d / a αφού αλλιώς θα είχαμε 2 2 4

a ab b d , άτοπο. Ο

αρχικός ισχυρισμός λοιπόν και άρα και το ζητούμενο αποδείχθηκε.

Page 356: ΠΡΟΤΕΙΝΟΜΕΝΕΣ ΑΣΚΗΣΕΙΣ ΓΙΑ ΜΑΘΗΤΙΚΟΥΣ ΔΙΑΓΩΝΙΣΜΟΥΣ_ALL

http://www.mathematica.gr/forum/viewtopic.php?f=109&t=15584

Επιμέλεια: xr.tsif Σελίδα 74

ΘΕΜΑ 381 (Socrates)

Να βρείτε όλους τους ακέραιους n ώστε ο αριθμός n36 6 να γράφεται ως

γινόμενο (δύο ή περισσότερων) διαδοχικών ακεραίων.

Λύση:

Καταρχήν το γινόμενο τεσσάρων ή περισσοτέρων διαδοχικών ακεραίων είναι

πολλαπλάσιο του 4 ενώ n36 6 0 2 2 mod4 .

Άρα το n36 6 είναι γινόμενο το πολύ τριών διαδοχικών ακεραίων.

Αν το n36 6 γράφεται ως γινόμενο δύο διαδοχικών ακεραίων

τότε n36 6 m(m 1) δηλαδή 2 n

m m 36 6 0 του οποίου η

Διακρίνουσα είναι ίση με nΔ 4·36 23 και πρέπει να είναι τέλειο τετράγωνο

ακεραίου. Άρα πρέπει να υπάρχει ακέραιος k ώστε

n 24·36 23 k ή n n

2·6 k 2·6 k 23 δηλαδή

n2·6 k 1 και n

2·6 k 23 από όπου n 1 που είναι δεκτή λύση.

Αν το n36 6 γράφεται ως γινόμενο τριών διαδοχικών ακεραίων

τότε n36 6 (m 1)m(m 1) .

Όμως n36 6 1 6 2 mod7 ενώ 3

m m 2 mod7 για οποιοδήποτε

ακέραιο m (Συγκεκριμένα αν 1

m 7m υ τότε για υ 0,1,2,3,4,5,6 , η

ποσότητα 3m m είναι αντίστοιχα ισότιμη με 0,0,6,3,4,1,0(mod7) ).

Άρα το n36 6 δε μπορεί να γραφτεί ως γινόμενο τριών διαδοχικών ακεραίων.

Β τρόπος

Προφανώς, ο αριθμός n θα πρέπει να είναι θετικός ακέραιος.

Page 357: ΠΡΟΤΕΙΝΟΜΕΝΕΣ ΑΣΚΗΣΕΙΣ ΓΙΑ ΜΑΘΗΤΙΚΟΥΣ ΔΙΑΓΩΝΙΣΜΟΥΣ_ALL

http://www.mathematica.gr/forum/viewtopic.php?f=109&t=15584

Επιμέλεια: xr.tsif Σελίδα 75

Ακόμη το γινόμενο 4 διαδοχικών ακεραίων διαιρείται με το 4 , αφού οι αριθμοί

αυτοί αφήνουν διαφορετικά υπόλοιπα με το 4 κι επομένως κάποιος από αυτούς

θα διαιρείται με το 4 . Όμως, ο αριθμός n36 6 δε διαιρείται με το 4 κι

επομένως δεν μπορεί να γραφεί ως γινόμενο 4 ή περισσότερων διαδοχικών

αριθμών. Οπότε, μένει να εξετάσουμε την περίπτωση που γράφεται ως γινόμενο

2 ή 3 διαδοχικών.

Για την πρώτη περίπτωση θα υπάρχει αριθμός m N , ώστε

n n 2 236 6 m(m 1) (6 ) m m 6 .

Για m 6 ,είναι 2 n 2 2(m 1) (6 ) m , άτοπο.

Οπότε m 5 και με έναν απλό έλεγχο, βρίσκω ότι (m,n) (5,1) .

Για τη δεύτερη περίπτωση θα υπάρχει αριθμός m N , ώστε

n n 236 6 (m 1)(m 2)(m 3) 36 m[(m 3) 2] .

Για να μη διαιρείται ο αριθμός (m 1)(m 2)(m 3) με το 4 , θα πρέπει

m 4k κι έτσι n 2 n 1 n 236 4k[(4k 3) 2] 4 9 k[(4k 3) 2]

.

Επειδή 2(4k 3) 2 0(mod2) , θα είναι n 1

4 / k (1).

Αν 3 / k , τότε 2 n(4k 3) 2 0(mod3) 9 / k , άτοπο (γιατί;).

Οπότε n 29 / (4k 3) 2 (2).

Λόγω των σχέσεων (1) και (2) συμπεραίνουμε ότι:

n 1 2 n 2k 4 ,(4k 3) 2 (3 )

,άτοπο αφού 2 n 2 2(4k 3) (3 ) (4k 2) .

Δείτε και εδώ: http://www.artofproblemsolving.com/Foru ... 6&t=418785

ΘΕΜΑ 382 (Socrates)

Αν a,b,x,y πραγματικοί, τέτοιοι ώστε να ισχύει a b x y και

4 4 4 4x y a b , να αποδείξετε ότι n n n n

x y a b για κάθε θετικό ακέραιο n .

Page 358: ΠΡΟΤΕΙΝΟΜΕΝΕΣ ΑΣΚΗΣΕΙΣ ΓΙΑ ΜΑΘΗΤΙΚΟΥΣ ΔΙΑΓΩΝΙΣΜΟΥΣ_ALL

http://www.mathematica.gr/forum/viewtopic.php?f=109&t=15584

Επιμέλεια: xr.tsif Σελίδα 76

Λύση:

Έχουμε: 4 4 4 4 2 2 2 2 2 2 2 2x a b y (x a )(x a ) (b y )(b y )

2 2 2 2(x a)(x a ) (b y)(b y ) με την προϋπόθεση ότι είναι x a .

Άρα έχουμε: 3 2 2 3 3 2 2 3x a x ax a b by b y y (1).

Με όμοιο τρόπο έχουμε:

4 4 4 4 3 2 2 3 2 2 2 3x b a y x b x bx b a ay a y y (2).

με την προϋπόθεση ότι x b .

Αφαιρώντας κατά μέλη τις σχέσεις (1) και (2) βρίσκουμε ότι: (με την

προϋπόθεση ότι a b ,

2 2 2 2(a b)(a b)(x y) (a b)(x y ) 2(a b)(a ab b ) 0

2 2 2 2(a b)(x y) x y 2a 2ab 2b 0

2 2 2 2 2 2(x y)(x y) x y a a 2ab b b 0

2 2 2 2 2 2(x y) x y a (a b) b 0 x y a b 0 , που είναι

άτοπο αφού υποθέσαμε ότι a b .

Άρα υποχρεωτικά θα είναι a b , οπότε τότε έχουμε από τις υποθέσεις:

x y 2a και 4 4 4x y 2a . Άρα

2 2 2 2 2(x y) 4a x y 2xy 4a ,

2 2 2 2 2 4 2 2 2 2 4(x y ) 2x y 2a (4a 2xy) 2x y 2a

2 2 2 4 2x y 8a xy 7a 0 xy a ή 2

xy 7a .

1η ΠΕΡΙΠΤΩΣΗ : 2xy a

Page 359: ΠΡΟΤΕΙΝΟΜΕΝΕΣ ΑΣΚΗΣΕΙΣ ΓΙΑ ΜΑΘΗΤΙΚΟΥΣ ΔΙΑΓΩΝΙΣΜΟΥΣ_ALL

http://www.mathematica.gr/forum/viewtopic.php?f=109&t=15584

Επιμέλεια: xr.tsif Σελίδα 77

Τότε από το σύστημα 2xy a ,x y 2a βρίσκουμε ότι τα x,y είναι οι ρίζες

της εξίσωσης 2 2t 2at a 0 t a x y a ,

που είναι άτοπο αφού έχουμε ήδη υποθέσει ότι x a .

2η ΠΕΡΙΠΤΩΣΗ: 2xy 7a .

Τότε ομοίως έχουμε ότι τα x,y είναι οι ρίζες της εξίσωσης

2 2t 2at 7a 0 , η οποία για να έχει ρίζα πραγματική , θα πρέπει να είναι

a 0 και άρα a b 0 , οπότε από την σχέση 4 4 4 4

x y a b x y 0 x y 0 , που και πάλι είναι άτοπο, αφού

έχουμε υποθέσει ότι x a .

Από τον παραπάνω συλλογισμό συμπεραίνουμε ότι υποχρεωτικά θα είναι x a

από όπου προκύπτει ότι και y b (ή x b ,y a και επομένως

n n n nx y a b .

Β τρόπος

Είναι 4 4 4 4 4 4 2 2 2(a b) a b (x y) x y 4ab(a b) 2a b

2 2 2 24xy(x y) 2x y 2(x y) (xy ab) (xy ab)(xy ab) . Υποθέτω ότι

xy ab και τότε 2 2 22(x y) xy ab (x y) (a b) xy ab .

Όμως είναι 2 2(x y) (a b) 4xy 4ab κι επειδή xy ab 0 θα πρέπει

xy ab 0 και τελικά εύκολα βλέπουμε ότι θα πρέπει x y a b 0 και άρα

xy ab , άτοπο. Άρα xy ab . Θα δείξω τον ισχυρισμό του προβλήματος με

μαθηματική επαγωγή. Για n 1 προφανώς ισχύει. Υποθέτω πως αν ισχύει για

κάθε k n θα ισχύει και για n 1 . Αυτό προκύπτει εύκολα από το διωνυμικό

θεώρημα.

ΘΕΜΑ 383 (Socrates)

Βρείτε όλους τους πρώτους p τέτοιους ώστε ο αριθμός 2p 1007 να έχει

λιγότερους από 7 (θετικούς) διαιρέτες.

Page 360: ΠΡΟΤΕΙΝΟΜΕΝΕΣ ΑΣΚΗΣΕΙΣ ΓΙΑ ΜΑΘΗΤΙΚΟΥΣ ΔΙΑΓΩΝΙΣΜΟΥΣ_ALL

http://www.mathematica.gr/forum/viewtopic.php?f=109&t=15584

Επιμέλεια: xr.tsif Σελίδα 78

Λύση:

Χρησιμοποιούμε το παρακάτω βασικό λήμμα (του οποίου η απόδειξη αφήνεται

ως απλή άσκηση – κάπου το έχουμε ξαναδεί στο forum):

**************************************************

ΛΗΜΜΑ: Αν p 5 πρώτος τότε ισχύει κάθε μία από τις παρακάτω προτάσεις:

Α) p 1 mod6

Β) p 1,5,7,11 mod12

Γ) 2p 1 mod24 .

**************************************************

Ας είναι 2A p 1007 .

Αν p 2 τότε A 1011 3·337 που έχει 4 θετικούς διαιρέτες άρα είναι δεκτή

λύση.

Αν p 3 τότε 3A 2 ·127 που έχει 8 θετικούς διαιρέτες άρα απορρίπτεται.

Έστω λοιπόν παρακάτω ότι p 5 τότε από το Λήμμα Γ παραπάνω έχουμε

2p 1 mod24 , άρα A 0 mod24 .

• Αν ο A έχει 6 διαιρέτες τότε είναι της μορφής 5A q όπου q πρώτος ή

2A q t όπου q,t πρώτοι διαφορετικοί μεταξύ τους.

Όμως δε μπορεί να είναι της μορφής 5A q αφού το πρώτο μέλος είναι άρτιος

άρα θα έπρεπε q 2 , που δε δίνει λύση.

Αν ήταν q 2 τότε 2A 2 t άρα από τη μία A 0 mod24 ενώ απ' την άλλη

22t 2 mod24 , άτοπο

Page 361: ΠΡΟΤΕΙΝΟΜΕΝΕΣ ΑΣΚΗΣΕΙΣ ΓΙΑ ΜΑΘΗΤΙΚΟΥΣ ΔΙΑΓΩΝΙΣΜΟΥΣ_ALL

http://www.mathematica.gr/forum/viewtopic.php?f=109&t=15584

Επιμέλεια: xr.tsif Σελίδα 79

Αν ήταν t 2 τότε A 4q άρα από τη μία A 0 mod24 ενώ απ' την άλλη

4q 4 mod24 ή 4q 4 mod24 ανάλογα αν ήταν q 1 mod6 ή

q 1 mod6 (Λήμμα Α παραπάνω), άτοπο.

• Αν ο A έχει 5 διαιρέτες τότε είναι της μορφής 4A q όπου q πρώτος.

Όμως δε μπορεί να είναι της μορφής 4A q αφού το πρώτο μέλος είναι άρτιος

άρα θα έπρεπε q 2 , που δε δίνει λύση.

• Αν ο A έχει 4 διαιρέτες , τότε είναι της μορφής 3A q όπου q πρώτος ή

A q t όπου q,t πρώτοι διαφορετικοί μεταξύ τους.

Όμως δε μπορεί να είναι της μορφής 3A q αφού το πρώτο μέλος είναι άρτιος

άρα θα έπρεπε q 2 , που δε δίνει λύση.

Αν ήταν q 2 τότε A 2t άρα από τη μία A 0 mod24 ενώ απ' την άλλη

2t 2,10,14,20 mod24 , ανάλογα αν ήταν q 1,5,7,11 mod12 (Λήμμα Β

παραπάνω), άτοπο.

Όμοια αν ήταν t 2 .

• Αν ο A έχει 3 διαιρέτες , τότε είναι της μορφής 2A q όπου q πρώτος.

Όμως δε μπορεί να είναι της μορφής 2A q αφού το πρώτο μέλος είναι άρτιος

άρα θα έπρεπε q 2 , που δε δίνει λύση.

• Αν ο A έχει 2 διαιρέτες, τότε είναι της μορφής A q όπου q πρώτος.

Όμως δε μπορεί να είναι της μορφής A q αφού το πρώτο μέλος είναι άρτιος

άρα θα έπρεπε q 2 , που δε δίνει λύση.

Μοναδική λύση λοιπόν p 2 .

Τώρα που το σκέφτομαι καλύτερα αν p 5 τότε αφού A 0 mod24 άρα ο A

περιέχει στην ανάλυσή του σε πρώτους παράγοντες το 324 2 ·3 δηλαδή

Page 362: ΠΡΟΤΕΙΝΟΜΕΝΕΣ ΑΣΚΗΣΕΙΣ ΓΙΑ ΜΑΘΗΤΙΚΟΥΣ ΔΙΑΓΩΝΙΣΜΟΥΣ_ALL

http://www.mathematica.gr/forum/viewtopic.php?f=109&t=15584

Επιμέλεια: xr.tsif Σελίδα 80

τουλάχιστον 8 διαιρέτες, άτοπο και η άσκηση ολοκληρώνεται. Οπότε αυτά που

έγραψα στην προηγούμενη δημοσίευση δε χρειάζονται. Ας τα αφήσω μόνο και

μόνο για να φαίνεται η μορφή που πρέπει να έχει ένας αριθμός όταν έχει

6,5,4,3,2 διαιρέτες.

ΘΕΜΑ 384 (Socrates)

Βρείτε όλους τους περιττούς θετικούς ακεραίους n για τους οποίους υπάρχουν

περιττοί ακέραιοι 1 2 n

x ,x ,...,x τέτοιοι ώστε 2 2 2 4

1 2 nx x ... x n .

Λύση:

Εργαζόμενοι mod8 έχουμε 2 2 2

1 2 nx x ... x 1 1 ... 1 n και 4

n 1

οπότε n 1 mod8 .

Αντίστροφα, αν n 8k 1 τότε

4 4 2 2 2 2n (8k 1) (64k 16k 1) (64k 16k 1 2)

2 2 2(64k 16k 1) 4(64k 16k 1) 4

2 2 2 2 2 2 2(64k 16k 1) 16 k 64k (64k 16k 1) (16k 1) 32k 1

2 2 2 2 2 2 2 2 2 2 2 2

k 1

(64k 16k 1) (16k 1) k(5 1 1 1 1 1 1 ) 1 ... 1

.

ΘΕΜΑ 385 (ΔΗΜΗΤΡΗΣ ΙΩΑΝΝΟΥ )

Οι αριθμοί a,b είναι ανάλογοι με τους αριθμούς 5 και 6 . Οι αριθμοί b,c είναι

αντιστρόφως ανάλογοι με τους αριθμούς 3 και 4 . Να εξετάσετε αν οι αριθμοί 2 2

a b και 2 2b c είναι τέλεια τετράγωνα.

(ΕΥΚΛΕΙΔΗΣ Β Γυμνασίου, 1995 – 96)

Page 363: ΠΡΟΤΕΙΝΟΜΕΝΕΣ ΑΣΚΗΣΕΙΣ ΓΙΑ ΜΑΘΗΤΙΚΟΥΣ ΔΙΑΓΩΝΙΣΜΟΥΣ_ALL

http://www.mathematica.gr/forum/viewtopic.php?f=109&t=15584

Επιμέλεια: xr.tsif Σελίδα 81

Λύση:

Αφού οι a,b ανάλογοι με τους 5 και 6 πρέπει a b

k5 6 a 5k , b 6k και

αφού οι b,c αντιστρόφως ανάλογοι με τους 3 και 4 , πρέπει

b 4 9k3b 4c 4c 18k c

c 3 2 . Επομένως : 2 2 2

a b 61k και δεν

είναι τέλειο τετράγωνο ρητού. Ενώ

22

2 2 225k 15kb c

4 2

, που είναι τέλειο

τετράγωνο.

ΘΕΜΑ 386 (ΔΗΜΗΤΡΗΣ ΙΩΑΝΝΟΥ )

Αν p 3 πρώτος, να βρεθεί το πλήθος των λύσεων της εξίσωσης

3 3 2 2 2011x y x y xy p , με x,y .

Λύση:

Κανένα ζεύγος (a,a) δεν είναι λύση, ενώ αν (x,y) είναι λύση τότε και (y,x)

είναι επίσης λύση, διαφορετική της πρώτης.

Αρκεί, λοιπόν , να βρούμε τις λύσεις με x y .

Η εξίσωση γράφεται 2 2011(x y) (x y) p οπότε a

x y p και bx y p .

Εύκολα βλέπουμε ότι τότε x,y .

Παρατηρούμε ότι για δύο λύσεις 1 1

(x ,y ) και 2 2

(x ,y ) με αντίστοιχα 1 1

(a ,b ) και

2 2(a ,b ) ισχύει

1 1 2 2 1 1 2 2(x ,y ) (x ,y ) a ,b ) (a ,b ) – ( .

Οπότε το πλήθος των ριζών της δοθείσας είναι το πλήθος των ριζών της

2a b 2011 .

O b είναι περιττός, b 2 1, 0 και a 0 . Έτσι έχουμε την εξίσωση

a 1005, a, 0 . Αυτή έχει 1006 λύσεις.

Page 364: ΠΡΟΤΕΙΝΟΜΕΝΕΣ ΑΣΚΗΣΕΙΣ ΓΙΑ ΜΑΘΗΤΙΚΟΥΣ ΔΙΑΓΩΝΙΣΜΟΥΣ_ALL

http://www.mathematica.gr/forum/viewtopic.php?f=109&t=15584

Επιμέλεια: xr.tsif Σελίδα 82

Τελικά, η αρχική έχει 2 1006 2012 λύσεις.

ΘΕΜΑ 387 (ΔΗΜΗΤΡΗΣ ΙΩΑΝΝΟΥ )

Ο Γιάννης και η Μαρία τρέχουν 10 χιλιόμετρα. Αρχίζουν από το ίδιο σημείο,

τρέχουν 5 χιλιόμετρα τον ανήφορο και επιστρέφουν στην αφετηρία από την ίδια

διαδρομή. Ο Γιάννης άρχισε 10 λεπτά γρηγορότερα από την Μαρία και έχει

ταχύτητα 15 χιλιόμετρα την ώρα στον ανήφορο και 20 χιλιόμετρα την ώρα

στον κατήφορο. Η Μαρία τρέχει 16 χιλιόμετρα την ώρα στον ανήφορο και 22

χιλιόμετρα την ώρα στον κατήφορο. Σε ποια απόσταση από την κορυφή θα

συναντηθούν οι δύο δρομείς τρέχοντας σε αντίθετες κατευθύνσεις;

Λύση:

Αν A η αφετηρία και K η κορυφή, ο Γιάννης έκανε την διαδρομή AK σε χρόνο

t και άρα AK 15t . Άρα 5 15t , άρα 1

t3

ώρες.

Επειδή η Μαρία ξεκίνησε να τρέχει 10 λεπτά αργότερα, δηλαδή 1

6 της ώρας

αργότερα, θα ισχύει ότι: 1

AB 16(t )6

με B το σημείο που θα βρίσκεται η

Μαρία, όταν ο Γιάννης είναι στην κορυφή K . Δηλαδή

1 1 16AB 16( ) Km

3 6 6 . Τώρα ο Γιάννης αρχίζει να κατεβαίνει και η Μαρία

εξακολουθεί να ανεβαίνει και ας υποθέσουμε ότι θα συναντηθούν στο σημείο

T . Για τον Γιάννη, έχουμε KT 20x και για την Μαρία έχουμε BT 16x ,

όπου x είναι ο χρόνος κινήσεώς τους.

Αλλά 16

BT TK AK AB 56

, άρα 7

16x 20x3

, άρα 7

x108

και

άρα 7

KT 20108

Km.

Page 365: ΠΡΟΤΕΙΝΟΜΕΝΕΣ ΑΣΚΗΣΕΙΣ ΓΙΑ ΜΑΘΗΤΙΚΟΥΣ ΔΙΑΓΩΝΙΣΜΟΥΣ_ALL

http://www.mathematica.gr/forum/viewtopic.php?f=109&t=15584

Επιμέλεια: xr.tsif Σελίδα 83

ΘΕΜΑ 388 (ΔΗΜΗΤΡΗΣ ΙΩΑΝΝΟΥ )

Δύο άνισοι κύκλοι εφάπτονται εξωτερικά. Οι κοινές εξωτερικές εφαπτόμενες

των κύκλων, τέμνονται στο σημείο P . Έστω ότι η μια από τις κοινές αυτές

εφαπτόμενες τέμνει τον μικρό κύκλο στο σημείο A και τον μεγάλο στο B . Αν

PA AB 4 , να υπολογίσετε το εμβαδόν του μικρού κύκλου.

Λύση:

Ας θέσουμε ΚΒ R και ΛΑ ρ . Τα ορθογώνια τρίγωνα ΡΑΛ και ΡΚΒ είναι

όμοια, άρα ρ ΡΑ 1

R 2ρR ΡΒ 2 και ΚΛ ΛΡ R ρ από το Π.Θ έχουμε

2 2 2 2 2 2 2ΛΡ ρ 16 (R ρ) ρ 16 9ρ ρ 16 8ρ 16 ρ 2 .

Άρα το εμβαδόν του μικρού κύκλου είναι 2Ε π ρ 2π τ.μ.

Page 366: ΠΡΟΤΕΙΝΟΜΕΝΕΣ ΑΣΚΗΣΕΙΣ ΓΙΑ ΜΑΘΗΤΙΚΟΥΣ ΔΙΑΓΩΝΙΣΜΟΥΣ_ALL

http://www.mathematica.gr/forum/viewtopic.php?f=109&t=15584

Επιμέλεια: xr.tsif Σελίδα 84

ΘΕΜΑ 389 (ΔΗΜΗΤΡΗΣ ΙΩΑΝΝΟΥ )

Μια ευθεία (ε) τέμνει τις πλευρές AB,AC τριγώνου ABC στα σημεία M,K

αντίστοιχα έτσι ώστε το εμβαδόν του τριγώνου BMK να είναι ίσο με το

εμβαδόν του τετραπλεύρου MBCK . Να αποδείξετε ότι BM BK 1

AM KC CAC 3

.

Λύση:

Έστω BM e και BK d . Οπότε αρκεί να αποδείξουμε ότι :

4(d e) a b c . Από την σχέση 2de ac , παίρνουμε ότι το ελάχιστο της

παράστασης d e συμβαίνει όταν d e . Όμως αν a 2c , το ελάχιστο

συμβαίνει όταν e c και a

d2

. Από τη δεύτερη περίπτωση, είναι προφανές ότι

b a c . Εάν τώρα υποθέσουμε ότι a 2c 4a και d e , αρκεί να

αποδείξουμε ότι a c

3c a το οποίο προφανώς ισχύει.

Β τρόπος

Προφανώς, το εμβαδόν του τριγώνου BMK είναι μεγαλύτερο από το εμβαδόν

του τριγώνου AMK , έτσι έχουμε BM MA . Ομοίως, έχουμε BK KC .

Άρα έχουμε: 3(BM BK) (BM BK) 2(AM KC)

3(BM BK) AM KC (BM MA) (BK KC)

3(BM BK) AM KC BA BC AM KC AC .

ΘΕΜΑ 390 (ΔΗΜΗΤΡΗΣ ΙΩΑΝΝΟΥ )

Σε ένα χορό πήραν μέρος 8 αγόρια και κορίτσια. Κάθε αγόρι χόρεψε με μερικά

κορίτσια και κάθε κορίτσι με μερικά αγόρια. Μετά το τέλος του χορού, κάθε

άτομο έγραψε τον αριθμό των χορών που χόρεψε, Έτσι πήραμε τους αριθμούς:

{3,3,3,3,3,3,4,6,6,6,6,6,6,6,6,6} . Να αποδείξετε ότι οι αριθμοί αυτοί δεν είναι

σωστοί, γιατί κάπου υπάρχει λάθος. (ΕΥΚΛΕΙΔΗΣ Β ΓΥΜΝΑΣΙΟΥ, 1995)

Page 367: ΠΡΟΤΕΙΝΟΜΕΝΕΣ ΑΣΚΗΣΕΙΣ ΓΙΑ ΜΑΘΗΤΙΚΟΥΣ ΔΙΑΓΩΝΙΣΜΟΥΣ_ALL

http://www.mathematica.gr/forum/viewtopic.php?f=109&t=15584

Επιμέλεια: xr.tsif Σελίδα 85

Λύση:

Το άθροισμα των δοσμένων αριθμών είναι 76 .

Ας συμβολίσουμε με 1 2 8

k ,k ,..,k τους αριθμούς που σημείωσαν τα κορίτσια και

1 2 8α ,α ,..,α τους αριθμούς που σημείωσαν τα αγόρια. Αν οι αριθμοί που

σημείωσαν τα 16 παιδιά ήταν ακριβείς, θα έπρεπε να είχαμε

1 2 8 1 2 8k k ... k α α ... α 38 αφού από την υπόθεση σε κάθε χορό

εμπλέκονται άτομα διαφορετικού φύλου. Προσθέτοντας τριάρια και εξάρια

βρίσκουμε πάντα πολλαπλάσιο του 3 , αν προσθέσουμε και ένα τεσσάρι θα

πάρουμε αριθμό της μορφής 3k 1 . Άρα δεν μπορεί να προκύψει η παραπάνω

ισότητα γιατί το 38 είναι της μορφής 3k 2 .

Με αφορμή την άσκηση 390, ας θυμίσω μερικές ασκήσεις θεωρίας γραφημάτων

από το Δημήτρη:

viewtopic.php?f=109&t=18854

viewtopic.php?f=109&t=18855

viewtopic.php?f=109&t=18856

viewtopic.php?f=109&t=18869

viewtopic.php?f=111&t=18870

viewtopic.php?f=111&t=18910

viewtopic.php?f=111&t=18911

ΘΕΜΑ 391 (ΔΗΜΗΤΡΗΣ ΙΩΑΝΝΟΥ )

Έστω τετράγωνο KLMN πλευράς 21 και A,B,C,D σημεία των

KL,LM,MN,NK ώστε KA 9 , LB 5 , MC 12 , ND 13 . Να δειχθεί ότι:

DA DB 1 AB AC . (ΕΥΚΛΕΙΔΗΣ Β ΓΥΜΝΑΣΙΟΥ, 1996)

Λύση:

Από πυθαγόρειο στο τρίγωνο ABLέχουμε: 2 2(AB) 12 5 13 . Επίσης η

ACείναι παράλληλη της LM οπότε AC 21 . Θέλουμε λοιπόν να δείξουμε

Page 368: ΠΡΟΤΕΙΝΟΜΕΝΕΣ ΑΣΚΗΣΕΙΣ ΓΙΑ ΜΑΘΗΤΙΚΟΥΣ ΔΙΑΓΩΝΙΣΜΟΥΣ_ALL

http://www.mathematica.gr/forum/viewtopic.php?f=109&t=15584

Επιμέλεια: xr.tsif Σελίδα 86

ότι AD DB 33 . Πάλι από

πυθαγόρειο στο τρίγωνο AKD έχουμε

2 2(AD) 9 8 145 12 . Επίσης

DB (KL) 21 . Άρα τελικά

AD DB 33 και το ζητούμενο

αποδείχθηκε.

ΘΕΜΑ 392 (ΔΗΜΗΤΡΗΣ ΙΩΑΝΝΟΥ )

Έστω a,b,c φυσικοί αριθμοί με a b c 20 , 3a 2b 3c 57 . Να βρεθεί η

τιμή της παράστασης: A (2a b 2c)(4a 3b 4c) . (Β ΓΥΜΝΑΣΙΟΥ, 1998)

Λύση:

3a 2b 3c 57 2(a b c) a c 57 40 a c 57 a c 17 .

A (a b c a c)(3a 2b 3c a b c) (20 17)(57 20)

37 77 2849 .

(Δεν είναι απαραίτητο να είναι φυσικοί οι αριθμοί a,b,c ).

ΘΕΜΑ 393 (ΔΗΜΗΤΡΗΣ ΙΩΑΝΝΟΥ )

Το σημείο 1

M είναι το μέσον του AB , το 2

M το μέσον του 1

AM , το 3

M το

μέσον του 2

AM κλπ ... και 10

M είναι το μέσον του 9

AM . Αν 11AB 2 3 να

βρεθεί το 10

AM . (ΕΥΚΛΕΙΔΗΣ Β ΓΥΜΝΑΣΙΟΥ, 1998).

Λύση:

11AB 2 3

Page 369: ΠΡΟΤΕΙΝΟΜΕΝΕΣ ΑΣΚΗΣΕΙΣ ΓΙΑ ΜΑΘΗΤΙΚΟΥΣ ΔΙΑΓΩΝΙΣΜΟΥΣ_ALL

http://www.mathematica.gr/forum/viewtopic.php?f=109&t=15584

Επιμέλεια: xr.tsif Σελίδα 87

11 1

1

ABAM 2 3

2

11 21

2

AMAM 2 3

2

......

......

......

11 10

10AM 2 3 2 3 6

.

ΘΕΜΑ 394 (ΔΗΜΗΤΡΗΣ ΙΩΑΝΝΟΥ )

Να δειχθεί ότι η ακολουθία των αριθμών 6,10,14,18,...,(2k 2),... δεν περιέχει

τέλειο τετράγωνο φυσικού αριθμού. (Γ ΓΥΜΝΑΣΙΟΥ)

Λύση:

Όλοι οι όροι ειναι ισουπόλοιποι με 2(mod4) το οποίο δεν είναι τετραγωνικό

υπόλοιπο mod4 .

ΘΕΜΑ 395 (ΔΗΜΗΤΡΗΣ ΙΩΑΝΝΟΥ )

α) Να βρεθούν οι γωνίες του τριγώνου ABC αν n 1AB 3 3 2

, n 2 n 1 n

BC 2 2 2 , n 1 n n 1

AC 2 2 2 , όπου n N,n 0 .

β) Να προσδιοριστεί το n αν η περίμετρος του ABC είναι 3(3 3) .

(Γ ΓΥΜΝΑΣΙΟΥ)

Λύση:

Κατ' αρχάς είναι n 2 nBC 2 (2 2 1) 3 2 και

n 1 n3AC 2 (2 1 2 ) 2

2

.

Page 370: ΠΡΟΤΕΙΝΟΜΕΝΕΣ ΑΣΚΗΣΕΙΣ ΓΙΑ ΜΑΘΗΤΙΚΟΥΣ ΔΙΑΓΩΝΙΣΜΟΥΣ_ALL

http://www.mathematica.gr/forum/viewtopic.php?f=109&t=15584

Επιμέλεια: xr.tsif Σελίδα 88

Τώρα παρατηρούμε ότι 2 2 2n 2n 2n n 2 227 9AB AC 2 2 9 2 (3 2 ) BC

4 4 ,

οπότε το ABC είναι ορθογώνιο με oA 90

.

Ακόμα παρατηρούμε ότι η υποτείνουσα BC είναι διπλάσια της πλευράς AC

άρα θα πρέπει να είναι oB 30

και συνεπώς oC 60

.

Για το n :

n 1 n n n 133 3 2 3 2 2 3(3 3) 2 3( 3 2 1) 3(3 3)

2

n 12 3(3 3) 3(3 3) n 1

.

ΘΕΜΑ 396 (ΔΗΜΗΤΡΗΣ ΙΩΑΝΝΟΥ )

Έστω τρίγωνο ABC , D σημείο της BC και I το μέσον της AD . Η BI τέμνει

την ACστο E και η CI την AB στο Z . Από το D φέρνουμε DH / /AC

( H σημείο της BI ) και DG / /AB (G σημείο της CI ). Να αποδείξετε ότι το

EZHG είναι παραλληλόγραμμο. (Γ ΓΥΜΝΑΣΙΟΥ)

Λύση:

Αφού DG / /AB έχουμε ότι τα

τρίγωνα AZI και DGI είναι ίσα

(γιατί;). Άρα ZI IG (1).

Όμοια DH / /AC και τότε τα

τρίγωνα AEI και DHI είναι ίσα

(γιατί;). Άρα HI IE (2).

Από (1) και (2) έχουμε ότι οι

διαγώνιες του τετραπλεύρου

EZHG διχοτομούνται, άρα είναι

παραλληλόγραμμο.

Page 371: ΠΡΟΤΕΙΝΟΜΕΝΕΣ ΑΣΚΗΣΕΙΣ ΓΙΑ ΜΑΘΗΤΙΚΟΥΣ ΔΙΑΓΩΝΙΣΜΟΥΣ_ALL

http://www.mathematica.gr/forum/viewtopic.php?f=109&t=15584

Επιμέλεια: xr.tsif Σελίδα 89

ΘΕΜΑ 397 (ΔΗΜΗΤΡΗΣ ΙΩΑΝΝΟΥ )

Δίνεται τρίγωνο ABΓ ορθογώνιο και ισοσκελές με υποτείνουσα BΓ και

AB AΓ α . Φέρνουμε ευθεία xAy έτσι ώστε η γωνία xAΓ

να είναι ίση με

o30 . Από τα Γ και B φέρνουμε κάθετες προς την xAy που την τέμνουν στα Δ

και E , αντίστοιχα. Να υπολογίσετε το εμβαδόν του τραπεζίου BΓΔE

συναρτήσει του α . (Γ ΓΥΜΝΑΣΙΟΥ)

Λύση:

(BΓΔE) (ΑΒΕ) (ΑΒΓ) (ΑΓΔ)

21 1 1ΑΕ ΒΕ α ΑΔ ΓΔ

2 2 2

2 2 21 α α 3 α α 3 α

22 2 2 2 4 2

22 3α

4

.

ΘΕΜΑ 398 (ΔΗΜΗΤΡΗΣ ΙΩΑΝΝΟΥ )

Σε κύκλο κέντρου O θεωρούμε δύο χορδές AB και CD που είναι κάθετες

μεταξύ τους και δεν περνάνε από το κέντρο του κύκλου. Οι δύο χορδές

τέμνονται στο σημείο K , έτσι ώστε να είναι AK KB . Έστω M το

συμμετρικό του B ως προς κέντρο συμμετρίας το K . Να αποδείξετε ότι το

σημείο M είναι το σημείο τομής των υψών του τριγώνου ACD .

(Γ ΓΥΜΝΑΣΙΟΥ)

Page 372: ΠΡΟΤΕΙΝΟΜΕΝΕΣ ΑΣΚΗΣΕΙΣ ΓΙΑ ΜΑΘΗΤΙΚΟΥΣ ΔΙΑΓΩΝΙΣΜΟΥΣ_ALL

http://www.mathematica.gr/forum/viewtopic.php?f=109&t=15584

Επιμέλεια: xr.tsif Σελίδα 90

Λύση:

Φέρουμε CB , BD . Το τετράπλευρο

ACBD είναι εγγεγραμμένο (1).

(1) ACK ABD (2). Φέρουμε

DM και έστω L DM AC .

Οπότε αρκεί CKMLνα είναι εγγράψιμο.

Το τρίγωνο BDM είναι ισοσκελές

MBD DMB (3). Το ζητούμενο

έπεται απο τις (2) , (3).

ΘΕΜΑ 399 (ΔΗΜΗΤΡΗΣ ΙΩΑΝΝΟΥ )

Θεωρούμε τα πολυώνυμα 4 3P(x) x 3x x 3 , 2

g(x) x 2x 3 ,

2R(x) x 5x a .

(α) Να ορίσετε το a έτσι ώστε το πολυώνυμο R(x) να διαιρείται από το x 2 .

(β) Να αναλύσετε σε γινόμενα παραγόντων τα πολυώνυμα P(x) ,g(x) ,R(x) .

(γ) Να δείξετε ότι η παράσταση 2x x P(x) :g(x) 15 είναι τέλειο

τετράγωνο. (Σημείωση: έχει ξανατεθεί)

Λύση:

(α) Αν κάνουμε την διαίρεση του πολυωνύμου R(x) με το πολυώνυμο

Q(x) x 2 θα πάρουμε πηλίκο W(x) και υπόλοιπο μηδέν όπως αναφέρεται

στην άσκηση. Άρα θα ισχύει ότι R(x) (x 2)W(x) και για x 2 βρίσκουμε

ότι R(2) 0 από όπου προκύπτει ότι a 14 .

(β) 3 2

P(x) x (x 3) (x 3) (x 3)(x 1)(x x 1)

Page 373: ΠΡΟΤΕΙΝΟΜΕΝΕΣ ΑΣΚΗΣΕΙΣ ΓΙΑ ΜΑΘΗΤΙΚΟΥΣ ΔΙΑΓΩΝΙΣΜΟΥΣ_ALL

http://www.mathematica.gr/forum/viewtopic.php?f=109&t=15584

Επιμέλεια: xr.tsif Σελίδα 91

2g(x) x 3x x 3 x(x 3) (x 3) (x 3)(x 1)

2 2 2R(x) x 5x 14 x 5x 10 4 (x 4) 5(x 2)

(x 2)(x 2) 5(x 2) (x 2)(x 7) .

(γ) 2 2 2 2x x P(x) :g(x) 15 x x x x 1 15 16 4 .

ΘΕΜΑ 400 (ΔΗΜΗΤΡΗΣ ΙΩΑΝΝΟΥ )

Έστω ότι a 0,b 0,c 0 και 1987 a 1987 b 2 1987 c . Να

αποδείξετε ότι 1

(a b) c2

.

Λύση:

Θέτουμε για ευκολία 1987 k .

Υψώνουμε την (1) στο τετράγωνο και παίρνουμε:

2k a b 2 (a k)(b k) 4k 4c a b 2 (a k)(b k) 2k 4c .

Όμως από AM – GM:

2k 4c a b 2 (a k)(b k) a b a k b k 2k 2(a b)

a b 2c , όπως θέλαμε.

Β τρόπος

Από την προφανή ανισότητα (π.χ. Cauchy – Schwarz ή πράξεις)

x y 2(x y) έχουμε

2 1987 c 1987 a 1987 b 2(1987 a 1987 b) ,

και με ύψωση στο τετράγωνο προκύπτει αμέσως η ζητούμενη.

Page 374: ΠΡΟΤΕΙΝΟΜΕΝΕΣ ΑΣΚΗΣΕΙΣ ΓΙΑ ΜΑΘΗΤΙΚΟΥΣ ΔΙΑΓΩΝΙΣΜΟΥΣ_ALL

http://www.mathematica.gr/forum/viewtopic.php?f=109&t=15584

Επιμέλεια: xr.tsif Σελίδα 1

Page 375: ΠΡΟΤΕΙΝΟΜΕΝΕΣ ΑΣΚΗΣΕΙΣ ΓΙΑ ΜΑΘΗΤΙΚΟΥΣ ΔΙΑΓΩΝΙΣΜΟΥΣ_ALL

http://www.mathematica.gr/forum/viewtopic.php?f=109&t=15584

Επιμέλεια: xr.tsif Σελίδα 2

ΠΡΟΤΕΙΝΟΜΕΝΕΣ

ΑΣΚΗΣΕΙΣ ΓΙΑ ΜΑΘΗΤΙΚΟΥΣ

ΔΙΑΓΩΝΙΣΜΟΥΣ

ΕΚΦΩΝΗΣΕΙΣ

ΤΕΥΧΟΣ 5ο

ΑΣΚΗΣΕΙΣ 401 - 500

Αφιερωμένο σε κάθε μαθητή που ασχολείται ή πρόκειται να ασχοληθεί με

Μαθηματικούς διαγωνισμούς

Τσιφάκης Χρήστος : xr.tsif

Page 376: ΠΡΟΤΕΙΝΟΜΕΝΕΣ ΑΣΚΗΣΕΙΣ ΓΙΑ ΜΑΘΗΤΙΚΟΥΣ ΔΙΑΓΩΝΙΣΜΟΥΣ_ALL

http://www.mathematica.gr/forum/viewtopic.php?f=109&t=15584

Επιμέλεια: xr.tsif Σελίδα 3

ΘΕΜΑ 401 (ΔΗΜΗΤΡΗΣ ΙΩΑΝΝΟΥ)

Δίνεται τρίγωνο ABC με AB AC . Πάνω στην βάση BC παίρνουμε σημείο

D και πάνω στην πλευρά AC παίρνουμε το σημείο E έτσι ώστε η γωνία DAB

να είναι ίση με το διπλάσιο της γωνίας CDE

. Να αποδειχθεί ότι AD AE .

ΘΕΜΑ 402 (ΔΗΜΗΤΡΗΣ ΙΩΑΝΝΟΥ)

Δύο κύκλοι 1 1 2 2

(O ,R ),(O ,R ) βρίσκονται ο ένας εκτός του άλλου. Να βρεθεί

α) Το μικρότερο ευθύγραμμο τμήμα που συνδέει σημεία των κύκλων.

β) Το μεγαλύτερο ευθύγραμμο τμήμα που συνδέει σημεία των κύκλων.

(Να γίνουν οι αποδείξεις).

ΘΕΜΑ 403 (ΔΗΜΗΤΡΗΣ ΙΩΑΝΝΟΥ)

Να δείξετε ότι δεν υπάρχουν φυσικοί αριθμοί m,n τέτοιοι ώστε οι αριθμοί 2

m 2n και 2n 2m να είναι τετράγωνα φυσικών αριθμών.

ΘΕΜΑ 404 (ΔΗΜΗΤΡΗΣ ΙΩΑΝΝΟΥ)

Δίνονται οι πραγματικοί αριθμοί x,y για τους οποίους ισχύει ότι:

x y 2a 4 και 2x y a 3a 5 , όπου και ο a είναι πραγματικός αριθμός.

Να βρεθεί η ελάχιστη τιμή του 2 2x y .

ΘΕΜΑ 405 (Karanus)

Εάν οι x,ψ,z πραγματικοί θετικοί αριθμοί τέτοιοι ώστε 2 2 2ψ 3x z , να

αποδείξετε ότι 2 2 2

3 1 ψ 1 z 1 x3

2 x 2 ψ 2 z 2

. (Αρχιμήδης 2008)

Page 377: ΠΡΟΤΕΙΝΟΜΕΝΕΣ ΑΣΚΗΣΕΙΣ ΓΙΑ ΜΑΘΗΤΙΚΟΥΣ ΔΙΑΓΩΝΙΣΜΟΥΣ_ALL

http://www.mathematica.gr/forum/viewtopic.php?f=109&t=15584

Επιμέλεια: xr.tsif Σελίδα 4

ΘΕΜΑ 406 (Karanus)

Να βρεθεί ο μέγιστος θετικός ακέραιος x για τον οποίο ο αριθμός 18 x2 700

A 2 4 8 είναι τετράγωνο ακεραίου αριθμού.

ΘΕΜΑ 407 (Karanus)

Αν x,ψ,z είναι θετικοί πραγματικοί αριθμοί τέτοιοι ώστε 2 2 2ψ 25x z να

προσδιορίσετε την ελάχιστη δυνατή τιμή της παράστασης yz zy

z yA

x

x

x .

ΘΕΜΑ 408 (Socrates)

Να βρείτε όλους τους ακεραίους n , για τους οποίους ο αριθμός n n n2 12 2011

είναι τετράγωνο ακεραίου.

viewtopic.php?f=111&t=21878

2011 USAJMO:

http://www.artofproblemsolving.com/Foru ... &t=404350&

ΘΕΜΑ 409 (Socrates)

Να βρείτε όλους τους ακέραιους n για τους οποίους ο αριθμός n n n11 12 13

είναι τέλειο τετράγωνο ακεραίου.

ΘΕΜΑ 410 (Socrates)

Αν a,b,c θετικοί πραγματικοί αριθμοί για τους οποίους c a b να δείξετε ότι 3 3 3 2

a b c 3abc 2(a b) c .

ΘΕΜΑ 411 (Socrates)

Αν a,b είναι πραγματικοί αριθμοί τέτοιοι ώστε 3a b και 3b a , να δείξετε ότι

a b·( 3a b 3b a) 4 ab .

Page 378: ΠΡΟΤΕΙΝΟΜΕΝΕΣ ΑΣΚΗΣΕΙΣ ΓΙΑ ΜΑΘΗΤΙΚΟΥΣ ΔΙΑΓΩΝΙΣΜΟΥΣ_ALL

http://www.mathematica.gr/forum/viewtopic.php?f=109&t=15584

Επιμέλεια: xr.tsif Σελίδα 5

ΘΕΜΑ 412 (Socrates)

Να βρείτε τον μικρότερο τετραψήφιο αριθμό n για τον οποίο το σύστημα

3 3 2 2

2 2

x y x y y x n

x y x y n 1

έχει ακέραια ρίζα.

ΘΕΜΑ 413 (Socrates)

Να βρείτε όλα τα ζεύγη φυσικών αριθμών (m,n)τέτοια ώστε

2 2 2 2τ (m) τ (n 15) 1 3τ (n 3n), όπου με τ(n) συμβολίζουμε το πλήθος

των (θετικών) διαιρετών του φυσικού αριθμού n .

ΘΕΜΑ 414 (Socrates)

Τρείς μαθητές, οι A , B , συμμετέχουν σε ένα μαθηματικό διαγωνισμό. Στο

διαγωνισμό δόθηκαν για λύση 5 προβλήματα. Η μέγιστη βαθμολογία για κάθε

πρόβλημα είναι θετικός ακέραιος αριθμός, διαφορετικός για κάθε πρόβλημα.

Ο μαθητής A έλυσε πλήρως 4 από τα προβλήματα και συγκέντρωσε 21

βαθμούς, ενώ ο μαθητής B έλυσε πλήρως 3 προβλήματα και η συνολική του

βαθμολογία ήταν 22 βαθμοί.

Αν ο μαθητής έλυσε πλήρως όλα τα προβλήματα, να βρείτε τη μέγιστη και

την ελάχιστη τιμή της βαθμολογίας του.

ΘΕΜΑ 415 (Socrates)

Δίνεται ισόπλευρο τρίγωνο πλευράς n , όπου n ένας θετικός ακέραιος. Σε κάθε

πλευρά θεωρούμε σημεία που τη διαιρούν σε n ίσα τμήματα. Χρησιμοποιώντας

αυτά τα σημεία φέρουμε παράλληλες στις πλευρές του τριγώνου. Έστω n

c το

πλήθος των ρόμβων πλευράς 1 , που σχηματίζονται. Να δείξετε ότι η

μεγαλύτερη λύση της ανίσωσης n

c 2009 είναι πρώτος αριθμός.

Page 379: ΠΡΟΤΕΙΝΟΜΕΝΕΣ ΑΣΚΗΣΕΙΣ ΓΙΑ ΜΑΘΗΤΙΚΟΥΣ ΔΙΑΓΩΝΙΣΜΟΥΣ_ALL

http://www.mathematica.gr/forum/viewtopic.php?f=109&t=15584

Επιμέλεια: xr.tsif Σελίδα 6

ΘΕΜΑ 416 (Socrates)

Μπορούμε να χωρίσουμε ένα ισόπλευρο τρίγωνο σε 2011 μικρότερα τρίγωνα

χρησιμοποιώντας 122 ευθείες;

ΘΕΜΑ 417 (Socrates)

Να δείξετε ότι 2 2 2

2 2 2 2 2 2

x y z1

y yz z x xz z x xy y

, για όλους τους

πραγματικούς αριθμούς x,ψ,z για τους οποίους ορίζονται τα κλάσματα.

ΘΕΜΑ 418 (Socrates)

Έστω *m,n . Αν 2

m / 3n και ο αριθμός 2n m είναι τέλειο τετράγωνο

ακεραίου, να δείξετε ότι 2m 3n .

ΘΕΜΑ 419 (Socrates)

Ένα σύνολο ακεραίων λέγεται καλό αν κανένα στοιχείο του δε διαιρεί το

άθροισμα των υπολοίπων.

Πόσα το πολύ στοιχεία έχει ένα καλό υποσύνολο του συνόλου {1,2,3,...,63} ;

ΘΕΜΑ 420 (Socrates)

Να εξετάσετε αν υπάρχουν ρητοί a,b,c,d τέτοιοι ώστε

2 21 3 (a b 3) (c d 3) .

ΘΕΜΑ 421 (Socrates)

Να προσδιορίσετε τους ακεραίους x,ψ έτσι ώστε

x y 2

2 3 4 15 3 5

.

Page 380: ΠΡΟΤΕΙΝΟΜΕΝΕΣ ΑΣΚΗΣΕΙΣ ΓΙΑ ΜΑΘΗΤΙΚΟΥΣ ΔΙΑΓΩΝΙΣΜΟΥΣ_ALL

http://www.mathematica.gr/forum/viewtopic.php?f=109&t=15584

Επιμέλεια: xr.tsif Σελίδα 7

ΘΕΜΑ 422 (Socrates)

Μπορούμε να τοποθετήσουμε 2011 σημεία στο επίπεδο έτσι ώστε:

η απόσταση μεταξύ δύο οποιονδήποτε σημείων να είναι διαφορετική από

1 και

κάθε μοναδιαίος κύκλος με κέντρο ένα από αυτά τα σημεία να αφήνει

ακριβώς 1005 από αυτά τα σημεία στο εξωτερικό του;

ΘΕΜΑ 423 (Socrates)

Πόσα στοιχεία έχει το σύνολο 2

2

n 7{x | x , n 1,2,...,2012}

n n 6

;

ΘΕΜΑ 424 (Socrates)

Να βρείτε τις θετικές ακέραιες ρίζες της εξίσωσης 3 2(x y) (x y 6) .

ΘΕΜΑ 425 (Karanus)

Να αποδειχθεί ότι υπάρχουν θετικοί πραγματικοί αριθμοί α και β για τους

οποίους, αν ισχύει 2000 2000 1998 1998α β α β ,τότε θα ισχύει 2 2

α β 2 .

(ΑΡΧΙΜΗΔΗΣ 1999)

ΘΕΜΑ 426 (Karanus)

Να βρεθούν όλοι οι φυσικοί αριθμοί n ώστε η 4 3 2

A n 4n 5n 6n να είναι

τετράγωνο φυσικού αριθμού. (ΑΡΧΙΜΗΔΗΣ 1997)

ΘΕΜΑ 427 (Karanus)

Να εξετάσετε εάν μπορούμε να ξαναγράψουμε τους αριθμούς

1,2,3,4,5,6,7,8,9,10 σε μία σειρά ώστε:

Page 381: ΠΡΟΤΕΙΝΟΜΕΝΕΣ ΑΣΚΗΣΕΙΣ ΓΙΑ ΜΑΘΗΤΙΚΟΥΣ ΔΙΑΓΩΝΙΣΜΟΥΣ_ALL

http://www.mathematica.gr/forum/viewtopic.php?f=109&t=15584

Επιμέλεια: xr.tsif Σελίδα 8

α) το άθροισμα οποιωνδήποτε τριών διαδοχικών αριθμών στη νέα σειρά ,να μην

υπερβαίνει το 16 .

β) το άθροισμα οποιωνδήποτε τριών διαδοχικών αριθμών στη νέα σειρά ,να μην

υπερβαίνει το 15 .

ΘΕΜΑ 428 (ΔΗΜΗΤΡΗΣ ΙΩΑΝΝΟΥ)

Να λυθεί στο Ν το σύστημα: x 7y 42z 210w 1209

z y x 7

.

ΘΕΜΑ 429 (Socrates)

Έστω M ένα σύνολο θετικών ακεραίων τέτοιο ώστε:

i) 1 M

ii) Αν x M τότε 2x 3 M

iii) Αν 4x 5 M τότε x M

Να δείξετε ότι το M περιέχει 8 τουλάχιστον πρώτους και 8 τουλάχιστον

σύνθετους αριθμούς.

ΘΕΜΑ 430 (Socrates)

Να βρείτε το n αν ισχύει

5 6 n 4 15 5...

21 5 6 1 6 7 1 n 4 n 5

.

ΘΕΜΑ 431 (freyia)

Υποθέτουμε ότι υπάρχουν τρεις πραγματικές συναρτήσεις f ,g,h ορισμένες στο

A για τις οποίες ισχύουν 2 2 2f (x)g(x) h (x),g(x)h(x) f (x),h(x)f (x) g (x) ,

για κάθε x A . Να αποδείξετε ότι f (x) g(x) h(x) .

Page 382: ΠΡΟΤΕΙΝΟΜΕΝΕΣ ΑΣΚΗΣΕΙΣ ΓΙΑ ΜΑΘΗΤΙΚΟΥΣ ΔΙΑΓΩΝΙΣΜΟΥΣ_ALL

http://www.mathematica.gr/forum/viewtopic.php?f=109&t=15584

Επιμέλεια: xr.tsif Σελίδα 9

ΘΕΜΑ 432 (gauss1988)

Να εξετάσετε αν ο αριθμός 800 5003 2 6 είναι πρώτος.

ΘΕΜΑ 432β (Παύλος Μαραγκουδάκης)

Έστω 5 5 5A 1821 1822 ... 2012 . Να βρεθεί το υπόλοιπο της διαίρεσης του

A με το 7 .

Ενισχύοντας την προσπάθεια του Θάνου να γίνουν κατανοητές οι λύσεις τέτοιου

είδους ασκήσεων από μαθητές Γυμνασίου, να δώσω ένα είδος μεθοδολογίας για

αυτές.

Θα γράψω όλα όσα είναι απαραίτητα να γνωρίζουμε για τις ισοτιμίες:

(α) Γράφουμε a b(modn) αν και μόνο αν ο φυσικός αριθμός n διαιρεί την

διαφορά a b των ακεραίων a,b .

(β) Αν a b(modn) και το υπόλοιπο της διαίρεσης του b με τον n είναι u ,

τότε a u(modn) .

(γ) m ma b(modn) a b (modn) , m N .

(δ) a b(modn) aq bq(modn),q N .

(ε) a b(modn),c d(modn)

a c b d(modn)

a c b d(modn)

ac bd(modn)

(στ) a b(modn) a k b k(modn) για κάθε k Z .

Page 383: ΠΡΟΤΕΙΝΟΜΕΝΕΣ ΑΣΚΗΣΕΙΣ ΓΙΑ ΜΑΘΗΤΙΚΟΥΣ ΔΙΑΓΩΝΙΣΜΟΥΣ_ALL

http://www.mathematica.gr/forum/viewtopic.php?f=109&t=15584

Επιμέλεια: xr.tsif Σελίδα 10

(ζ) Σε ασκήσεις τέτοιου είδους, όπου θέλουμε να βρούμε το υπόλοιπο της

διαίρεσης του ma με το n , προσπαθούμε (αν αυτό είναι εφικτό), να

καταλήξουμε σε ισοτιμία της μορφής:

ka 0(modn) ή k

a 1(modn) ή ka 1(modn) .

ΘΕΜΑ 433 (Παύλος Μαραγκουδάκης)

α) Να βρεθεί η μεγαλύτερη δύναμη k7 που διαιρεί το 100!

(100! 1·2·3·...·99·100 ).

β) Αν k

100!P

7 , να βρεθεί το υπόλοιπο της διαίρεσης του P με το 7 .

ΘΕΜΑ 434 (Socrates)

Να προσδιορίσετε όλες τις τριάδες (p,m,n) όπου p πρώτος και m,n μη

αρνητικοί ακέραιοι, που είναι λύσεις της εξίσωσης m 3p n 8 .

Ένα συγγενές θέμα (άσκηση 16) είδαμε εδώ: viewtopic.php?p=69084#p69084

ΘΕΜΑ 435 (Socrates)

Να βρεθούν οι ακέραιες ρίζες της εξίσωσης 4 4 4 4x 2y 4z 8t 16xyzt .

ΘΕΜΑ 436 (Socrates)

Οι ρητοί αριθμοί x και y και ο περιττός θετικός ακέραιος n είναι τέτοιοι ώστε

n nx 2x y 2y . Να δείξετε ότι x y .

Page 384: ΠΡΟΤΕΙΝΟΜΕΝΕΣ ΑΣΚΗΣΕΙΣ ΓΙΑ ΜΑΘΗΤΙΚΟΥΣ ΔΙΑΓΩΝΙΣΜΟΥΣ_ALL

http://www.mathematica.gr/forum/viewtopic.php?f=109&t=15584

Επιμέλεια: xr.tsif Σελίδα 11

ΘΕΜΑ 437 (Socrates)

Οι θετικοί πραγματικοί αριθμοί p,q είναι τέτοιοι ώστε 1 1

1p q . Να δείξετε

ότι 1 1 1 1

3 p(p 1) q(q 1) 2

. Ποια η ελάχιστη τιμή της παράστασης

1 1

p(p 1) q(q 1)

;

ΘΕΜΑ 438 (Socrates)

Διαθέτουμε 100 βαρίδια βάρους 1,2,3,...,100 κιλών αντίστοιχα. Τοποθετούμε

όλα τα παραπάνω βαρίδια σε ζυγαριά ώστε αυτή να ισορροπεί. Να δείξετε ότι

μπορούμε να αφαιρέσουμε δύο βαρίδια από κάθε πλευρά της ζυγαριάς και αυτή

να εξακολουθεί να ισορροπεί!

ΘΕΜΑ 439 (Socrates)

Να προσδιορίσετε όλα τα ζεύγη θετικών ακεραίων (m,n) τέτοια ώστε οι

αριθμοί 2

2

m 2n

n 2m

και

2

2

n 2m

m 2n

να είναι ακέραιοι.

ΘΕΜΑ 440 (ΔΗΜΗΤΡΗΣ ΙΩΑΝΝΟΥ)

Δίνεται ισοσκελές τρίγωνο ABC με βάση BC . Έστω D σημείο της πλευράς

AB , M το μέσον της BC και E σημείο της πλευράς AC έτσι ώστε να είναι 2

MB DB EC . Να αποδείξετε ότι τα τρίγωνα BDM , DEM , CEM είναι

όμοια.

ΘΕΜΑ 441 (Socrates)

Θεωρούμε τραπέζιο ABCD με AD / /BC και AD BC . Έστω E το μέσο της

διαγωνίου BD και F το ίχνος της καθέτου από το B στην AD . Να δείξετε ότι το

τραπέζιο είναι ισοσκελές αν και μόνο αν το συμμετρικό του A ως προς το F και

το συμμετρικό του C ως προς το E συμπίπτουν.

Page 385: ΠΡΟΤΕΙΝΟΜΕΝΕΣ ΑΣΚΗΣΕΙΣ ΓΙΑ ΜΑΘΗΤΙΚΟΥΣ ΔΙΑΓΩΝΙΣΜΟΥΣ_ALL

http://www.mathematica.gr/forum/viewtopic.php?f=109&t=15584

Επιμέλεια: xr.tsif Σελίδα 12

ΘΕΜΑ 442 (Socrates)

Στις κάθετες πλευρές AB και AC ορθογωνίου τριγώνου ABC

κατασκευάζουμε εξωτερικά τα τετράγωνα ABDE και ACFG . Αν

DC AB {U}, BF AC {V}, UV BD {P}, UV CF {Q} , να δείξετε

ότι DF PQ UV .

ΘΕΜΑ 443 (Socrates)

Το σημείο D είναι το μέσο της πλευράς ACτριγώνου ABC και οι DE , DF

διχοτόμοι των γωνιών ADB, CDB

αντίστοιχα. Αν EF DB {M} να δείξετε

ότι EF 2DM .

ΘΕΜΑ 444 (Παύλος Μαραγκουδάκης)

α) Να αποδείξετε ότι ο αριθμός 2007

5 13A

32

είναι ακέραιος.

β) Να βρείτε τα τελευταία 2 ψηφία του αριθμού A .

γ) Να βρείτε τα 7 τελευταία ψηφία του αριθμού 20125 .

ΘΕΜΑ 445 (ΔΗΜΗΤΡΗΣ ΙΩΑΝΝΟΥ)

Να βρεθούν οι τιμές των ακεραίων αριθμών x,y , που επαληθεύουν την

εξίσωση: 2 2x y x y .

ΘΕΜΑ 446 (vzf)

Αν για τους πραγματικούς αριθμούς a,b ισχύει ότι 2 2

a b 2a 2b ab 4 να

βρεθούν οι λύσεις της εξίσωσης 3 3 3(2x a) (x b) x 0 .

Page 386: ΠΡΟΤΕΙΝΟΜΕΝΕΣ ΑΣΚΗΣΕΙΣ ΓΙΑ ΜΑΘΗΤΙΚΟΥΣ ΔΙΑΓΩΝΙΣΜΟΥΣ_ALL

http://www.mathematica.gr/forum/viewtopic.php?f=109&t=15584

Επιμέλεια: xr.tsif Σελίδα 13

ΘΕΜΑ 447 (vzf)

Αν οι αριθμοί x,y,z είναι τέτοιοι ώστε x 0,y 1 0,z 2 0 και

x y z 3 , να αποδείξετε ότι x(y 1) (y 1)(z 2) x(z 2)

3x y 1 y z 3 x z 2

.

Για ποιές τιμές των x,y,z ισχύει η ισότητα;

ΘΕΜΑ 448 (Socrates)

Να λυθεί στους ακεραίους η εξίσωση 3 3 2x 2y 4x 5y z 2012 .

είναι η 405 viewtopic.php?f=109&t=15584&start=860

ΘΕΜΑ 449 (Socrates)

Αν a,b,c 0 με a b c 3 να δείξετε ότι 3 3 3

a 2 b 2 c 23

b 2 c 2 a 2

.

Πριν δώσω την αναλυτική απόδειξη, θα γράψω μια πολύ χρήσιμη ανισότητα

(που αναφέρεται ως "Γενικευμένη ανισότητα του Andreescu" )

Αν 1 2 n 1 2 n

a ,a ,...,a ,b ,b ,...,b 0,m 1,0 r m 1 , τότε:

m m m m

1 2 n 1 2 n

r r r m r 1 r

1 2 n 1 2 n

a a a (a a ... a )...

b b b n (b b ... b )

.

H ισότητα ισχύει, όταν 1 n

1 n

a a...

b b .

ΘΕΜΑ 450 (Socrates)

Αν a,b,c 0 με 1

ab bc ca3

να δείξετε ότι

Page 387: ΠΡΟΤΕΙΝΟΜΕΝΕΣ ΑΣΚΗΣΕΙΣ ΓΙΑ ΜΑΘΗΤΙΚΟΥΣ ΔΙΑΓΩΝΙΣΜΟΥΣ_ALL

http://www.mathematica.gr/forum/viewtopic.php?f=109&t=15584

Επιμέλεια: xr.tsif Σελίδα 14

2 2 2

a b c 1

a bc 1 b ca 1 c ab 1 a b c

.

Θα χρησιμοποιήσουμε την ανισότητα Andreescu και την ταυτότητα του Euler

H ανισότητα Andreescu έχει αναφερθεί προηγουμένως.

Η ταυτότητα του Euler είναι η εξής:

3 3 3 2 2 2a b c 3abc (a b c)(a b c ab ac bc)

2 2 21(a b c)[(a b) (b c) (c a) ]

2 .

ΘΕΜΑ 451 (Socrates)

Στον πίνακα είναι γραμμένος ο αριθμός 200613 . Σβήνουμε το τελευταίο του

ψηφίο και στον αριθμό που προκύπτει προσθέτουμε το τετραπλάσιο αυτού του

ψηφίου.

Μπορούμε επαναλαμβάνοντας τη διαδικασία να πάρουμε τον αριθμό 132006 ;

ΘΕΜΑ 452 (Socrates)

Ένα μη κενό σύνολο *

A είναι τέτοιο ώστε για κάθε x A να υπάρχουν

y,z A με y z έτσι ώστε x y z .

α) Βρείτε ένα τέτοιο σύνολο με 2012 στοιχεία.

β) Ποιος είναι ο ελάχιστος αριθμός στοιχείων ενός τέτοιου συνόλου;

ΘΕΜΑ 453 (Socrates)

Αν n 1 ακέραιος και pπρώτος, τέτοιοι ώστε n / p 1 και 3p / n 1 να δείξετε

ότι ο αριθμός 4p 3 είναι τέλειο τετράγωνο ακεραίου.

Page 388: ΠΡΟΤΕΙΝΟΜΕΝΕΣ ΑΣΚΗΣΕΙΣ ΓΙΑ ΜΑΘΗΤΙΚΟΥΣ ΔΙΑΓΩΝΙΣΜΟΥΣ_ALL

http://www.mathematica.gr/forum/viewtopic.php?f=109&t=15584

Επιμέλεια: xr.tsif Σελίδα 15

ΘΕΜΑ 454 (Socrates)

Αν a,b,c 0 με 1 1 1 a b c

a b c b c a , να δείξετε ότι

a b c 3

bc(b c) ca(c a) ab(a b) 2

.

ΘΕΜΑ 455 (Socrates)

Βρείτε το ελάχιστο της παράστασης 1 1

1 a 1 b

όπου a,b 0 με

a b 1 .

ΘΕΜΑ 456 (Socrates)

Να βρείτε τις ακέραιες ρίζες της εξίσωσης 2010 2009 2008x 2006 4y 4y 2007y .

ΘΕΜΑ 457 (vzf)

Βρείτε τους x,y,u,v που ικανοποιούν το σύστημα των εξισώσεων:

x 7y 3v 5u 16

8x 4y 6v 2u 16

2x 6y 4v 8u 16

5x 3y 7v u 16

.

ΘΕΜΑ 458 (vzf)

Να βρείτε τις ακέραιες ρίζες της εξίσωσης 2 25x 5xy 5y 7x 14y .

ΘΕΜΑ 459 (vzf)

Να αποδείξετε ότι αν n

p είναι ο n – οστός πρώτος αριθμός, τότε ισχύει n 12

np 2

(η ισότητα ισχύει μόνο για n 1 ).

Page 389: ΠΡΟΤΕΙΝΟΜΕΝΕΣ ΑΣΚΗΣΕΙΣ ΓΙΑ ΜΑΘΗΤΙΚΟΥΣ ΔΙΑΓΩΝΙΣΜΟΥΣ_ALL

http://www.mathematica.gr/forum/viewtopic.php?f=109&t=15584

Επιμέλεια: xr.tsif Σελίδα 16

ΘΕΜΑ 460 (vzf)

Να βρεθούν οι πραγματικές λύσεις (x,y,z) του συστήματος:

2

2

2

ax by (x y)

by cz (y z)

cz ax (z x)

με a,b,c δεδομένους θετικούς πραγματικούς αριθμούς.

ΘΕΜΑ 461 (vzf)

Έστω S ένα σύνολο με n στοιχεία (n 2 ) και έστω 1 2 m

A ,A ,...,A υποσύνολα

του S (m 2 ). Αν για κάθε δύο διαφορετικά στοιχεία x,y S υπάρχει ένα

υποσύνολο i

A τέτοιο ώστε i

x A και i

y A ή i

x A και i

y A , αποδείξτε ότι m

2 n .

Είναι από βαλκανιάδες:

461. http://www.artofproblemsolving.com/Foru ... f2#p495412

460. http://www.artofproblemsolving.com/Foru ... f2#p822805.

ΘΕΜΑ 462 (vzf)

Έστω n θετικός ακέραιος. Δείξτε ότι ο n[(2 3) ] είναι περιττός αριθμός.

(Όπου με [x] συμβολίζουμε το ακέραιο μέρος του αριθμού x ).

ΘΕΜΑ 463 (freyia)

Να αποδείξετε ότι ο αριθμός 13

n n , διαιρείται με τον αριθμό 2730 , με n

ακέραιο .

Mε αφορμή την ΑΣΚΗΣΗ 463 και την λύση που έδωσε ο Θάνος, θα γράψω το

θεώρημα του Fermat, ώστε να μπορούμε πλέον να το χρησιμοποιούμε.

***********************************************************

Page 390: ΠΡΟΤΕΙΝΟΜΕΝΕΣ ΑΣΚΗΣΕΙΣ ΓΙΑ ΜΑΘΗΤΙΚΟΥΣ ΔΙΑΓΩΝΙΣΜΟΥΣ_ALL

http://www.mathematica.gr/forum/viewtopic.php?f=109&t=15584

Επιμέλεια: xr.tsif Σελίδα 17

ΜΙΚΡΟ ΘΕΩΡΗΜΑ ΤΟΥ FERMAT

Αν pπρώτος, a Z και (a,p) 1 τότε: p 1a 1(modp)

.

***********************************************************

Άμεση συνέπεια αυτού του θεωρήματος είναι το εξής:

Αν pπρώτος και a Z τότε pa a(modp) .

***********************************************************

Για παράδειγμα: 13 13a a(mod13) a a πολ13 .

Για εξοικείωση με το θεώρημα του Fermat, δίνονται οι παρακάτω ασκήσεις:

ΘΕΜΑ 464 (ΔΗΜΗΤΡΗΣ ΙΩΑΝΝΟΥ)

Δείξτε ότι: 17 17 17 171 2 3 ... 17 διαιρείται με το 17 .

Λύση:

Νομίζω ότι η συγκεκριμένη άσκηση λύνεται απλούστερα χωρίς το θεώρημα

Fermat, παρά μόνο με χρήση της ιδιότητας n n(a b) / (a b ) όταν n είναι

περιττός φυσικός.

Πράγματι, το 17 διαιρεί όλα τα αθροίσματα

17 17 17 17 17 17 17 17 17 17 17 17 17 17 17 171 16 , 2 15 , 3 14 , 4 13 , 5 12 , 6 11 , 7 10 , 8 9

και φυσικά διαιρεί και το 17

17 .

Β τρόπος

Με Fermat, γράφω την λύση για εξοικείωση με το θεώρημα αυτό:

Page 391: ΠΡΟΤΕΙΝΟΜΕΝΕΣ ΑΣΚΗΣΕΙΣ ΓΙΑ ΜΑΘΗΤΙΚΟΥΣ ΔΙΑΓΩΝΙΣΜΟΥΣ_ALL

http://www.mathematica.gr/forum/viewtopic.php?f=109&t=15584

Επιμέλεια: xr.tsif Σελίδα 18

171 1(mod17)

172 2(mod17)

.............

………..

1717 17(mod17)

Mε πρόσθεση, έχουμε:

17 17 171 2 ... 17 1 2 ... 17(mod17) 9 17(mod17) πολ17 .

ΘΕΜΑ 465 (ΔΗΜΗΤΡΗΣ ΙΩΑΝΝΟΥ)

Δείξτε ότι το υπόλοιπο της διαίρεσης του 7221 με το 73 είναι 1 .

Λύση:

Από το μικρό θεώρημα του Fermat :

73 1 7221 1(mod73) 21 1(mod73)

. Επομένως το υπόλοιπο είναι 1 .

ΘΕΜΑ 466 (ΔΗΜΗΤΡΗΣ ΙΩΑΝΝΟΥ)

Δείξτε ότι ο αριθμός 5 3

9a 10a 4a διαιρείται με το 15 για κάθε *

a N .

Λύση:

Θα αποδείξουμε ότι η παράσταση διαιρείται από τους 3 και 5 , οπότε, επειδή

είναι πρώτοι μεταξύ τους, θα διαιρείται και από το γινόμενό τους, που είναι το

15 .

Από το θεώρημα του Fermat είναι 5a a(mod5) και 3

a a(mod3) .

Page 392: ΠΡΟΤΕΙΝΟΜΕΝΕΣ ΑΣΚΗΣΕΙΣ ΓΙΑ ΜΑΘΗΤΙΚΟΥΣ ΔΙΑΓΩΝΙΣΜΟΥΣ_ALL

http://www.mathematica.gr/forum/viewtopic.php?f=109&t=15584

Επιμέλεια: xr.tsif Σελίδα 19

Δουλεύοντας mod3 έχουμε :

5 3 5 5 59a 10a 4a 9a 10a 4a 9a 6a 3(3a 2a) 0(mod3) .

Δουλεύοντας mod5 έχουμε

5 3 3 3 39a 10a 4a 9a 10a 4a 5a 10a 5(a 2a ) 0(mod5) .

Τελειώσαμε.

Β τρόπος

5 3 5 3 39a 10a 4a 3(3a 3a a) a a

5 33(3a 3a a) (a 1)a(a 1) πoλ3 .

(*) το γινόμενο 3 διαδοχικών ακεραίων (a 1)a(a 1) πoλ3 .

5 3 5 3 5 3 39a 10a 4a 10a 10a 5a [(a 5a 4a) (5a 5a)]

5 3 35(2a 2a a) (a 2)(a 1)a(a 1)(a 2) 5(a a) πoλ5 .

(**) το γινόμενο 5 διαδοχικών ακεραίων (a 2)(a 1)a(a 1)(a 2) πoλ5 .

και 3 , 5 πρώτοι μεταξύ τους οπότε προκύπτει το ζητούμενο.

ΘΕΜΑ 467 (ΔΗΜΗΤΡΗΣ ΙΩΑΝΝΟΥ)

Να βρεθεί το υπόλοιπο της διαίρεσης του αριθμού 24n 5

10

με το 13 .

Λύση:

Επειδή οι αριθμοί 10 ,13 είναι πρώτοι μεταξύ τους, από το θεώρημα Fermat

ισχύει 12 24n 24n 5 510 1(mod13) 10 1(mod13) 10 10 mod13)(

.

Εκτελώντας τη διαίρεση 10000:13 βρίσκουμε υπόλοιπο 3 , το οποίο είναι και

το ζητούμενο.

Page 393: ΠΡΟΤΕΙΝΟΜΕΝΕΣ ΑΣΚΗΣΕΙΣ ΓΙΑ ΜΑΘΗΤΙΚΟΥΣ ΔΙΑΓΩΝΙΣΜΟΥΣ_ALL

http://www.mathematica.gr/forum/viewtopic.php?f=109&t=15584

Επιμέλεια: xr.tsif Σελίδα 20

ΘΕΜΑ 468 (ΔΗΜΗΤΡΗΣ ΙΩΑΝΝΟΥ)

Αν p,q είναι δύο διαφορετικοί πρώτοι αριθμοί, να αποδείξετε ότι:

q 1 p 1p q 1(modpq)

.

Λύση:

Επειδή οι p,q είναι διαφορετικοί πρώτοι, είναι πρώτοι μεταξύ τους. Άρα, από

το θεώρημα Fermat έχουμε

q 1p 1(modq)

και ισχύει προφανώς p 1q 0(modq)

,

άρα q 1 p 1p q 1(modq)

(1) .

Με τον ίδιο ακριβώς τρόπο προκύπτει και q 1 p 1p q 1(modp)

(2) .

Επειδή οι p,q είναι διαφορετικοί πρώτοι, είναι πρώτοι μεταξύ τους, οπότε

q 1 p 1p q 1(modpq)

.

ΘΕΜΑ 469 (ΔΗΜΗΤΡΗΣ ΙΩΑΝΝΟΥ)

Να εξετάσετε αν υπάρχει πρώτος αριθμός p τέτοιος ώστε να είναι: pp | 2 3 .

Λύση:

Ο μοναδικός πρώτος με αυτή την ιδιότητα είναι ο 5 .

Πράγματι, από το θεώρημα Fermat είναι p2 2(modp) οπότε ισχύει

p2 3(modp) 2 3(modp) p | 5 p 5 .

ΘΕΜΑ 470 (ΔΗΜΗΤΡΗΣ ΙΩΑΝΝΟΥ)

Να αποδείξετε ότι το κλάσμα 3

4 2

n 2n

n 3n 1

είναι ανάγωγο, για κάθε ακέραιο n .

Page 394: ΠΡΟΤΕΙΝΟΜΕΝΕΣ ΑΣΚΗΣΕΙΣ ΓΙΑ ΜΑΘΗΤΙΚΟΥΣ ΔΙΑΓΩΝΙΣΜΟΥΣ_ALL

http://www.mathematica.gr/forum/viewtopic.php?f=109&t=15584

Επιμέλεια: xr.tsif Σελίδα 21

ΘΕΜΑ 471 (ΔΗΜΗΤΡΗΣ ΙΩΑΝΝΟΥ)

Να βρεθούν όλοι οι ακέραιοι n , για τους οποίους ο αριθμός

3A n (n 4) 2n(7n 10) 10 , να είναι τέλειο τετράγωνο ακεραίου.

ΘΕΜΑ 472 (Socrates)

Βρείτε τις μη αρνητικές ακέραιες λύσεις της εξίσωσης 3 2 3x 7x 35x 27 y .

ΘΕΜΑ 473 (Socrates)

Αν x,y,z διαδοχικοί ακέραιοι τέτοιοι ώστε 1 1 1 1

x y z 45 να βρείτε το βρείτε

το μέγιστο του x y z .

ΘΕΜΑ 474 (Socrates)

Αν x,y θετικοί αριθμοί τέτοιοι ώστε (1 x)(1 y) 2 να δείξετε ότι

1xy 6

xy .

ΘΕΜΑ 475 (Socrates)

Έστω x,y,z μη αρνητικοί αριθμοί τέτοιοι ώστε xy yz zx 0 . Να δείξετε ότι

2 2 2x y y z y z z x z x x y x y z

5y z x y z x y z x y z x xy yz zx

.

Πότε ισχύει η ισότητα;

ΘΕΜΑ 476 (Socrates)

Αν a,b,c θετικοί αριθμοί , να δείξετε ότι

2 2 2 2 2 2

92

cyc

5a 5c 8b 8(a b) (b c) (c a)3·

4ac (abc)

.

Page 395: ΠΡΟΤΕΙΝΟΜΕΝΕΣ ΑΣΚΗΣΕΙΣ ΓΙΑ ΜΑΘΗΤΙΚΟΥΣ ΔΙΑΓΩΝΙΣΜΟΥΣ_ALL

http://www.mathematica.gr/forum/viewtopic.php?f=109&t=15584

Επιμέλεια: xr.tsif Σελίδα 22

Μια λύση:

http://www.artofproblemsolving.com/Foru ... b#p2250521.

ΘΕΜΑ 477 (Socrates)

Αν οι ακέραιοι x,y,z είναι τέτοιοι ώστε 3 3 3x y z 0 να δείξετε ότι 3 / xyz .

ΘΕΜΑ 478 (Socrates)

Να λυθεί το σύστημα

2 3

2 3

x y y

y x x

.

ΘΕΜΑ 479 (Socrates)

Βρείτε όλους τους θετικούς ακεραίους a,b για τους οποίους

(a b) 1 (a b)(b ) 1/ 1a .

ΘΕΜΑ 480 (Socrates)

(α) Αν οι αριθμοί x y και 2x y όπου x,y είναι ρητοί είναι υποχρεωτικά

οι x,y ρητοί;

(β) Αν οι αριθμοί x y , 2x y και 3

x y όπου x,y είναι ρητοί είναι

υποχρεωτικά οι x,y ρητοί;

ΘΕΜΑ 481 (Socrates)

Αν

n

1x 1

n

και

n 1

1y 1

n

να συγκρίνετε τους αριθμούς y xx ,y .

ΘΕΜΑ 482 (Socrates)

Να δείξετε ότι 4 2 2 4 3 32(a a b b ) 3(a b ab ), a,b .

Page 396: ΠΡΟΤΕΙΝΟΜΕΝΕΣ ΑΣΚΗΣΕΙΣ ΓΙΑ ΜΑΘΗΤΙΚΟΥΣ ΔΙΑΓΩΝΙΣΜΟΥΣ_ALL

http://www.mathematica.gr/forum/viewtopic.php?f=109&t=15584

Επιμέλεια: xr.tsif Σελίδα 23

ΘΕΜΑ 483 (Socrates)

Έστω S υποσύνολο του A {1,2,3,...,9} τέτοιο ώστε να μην υπάρχουν δύο ίσα

αθροίσματα a b, a,b S, a b . Πόσα το πολύ στοιχεία έχει το S ;

http://www.artofproblemsolving.com/Foru ... 6b#p479091.

ΘΕΜΑ 484 (Socrates)

Έστω a b θετικοί ακέραιοι ίδιας αρτιότητας. Δείξτε ότι η εξίσωση 2 2 2 2 2

x (a a 1)(x b 1) (b 1) 0 έχει θετικές ακέραιες ρίζες καμία από

τις οποίες δεν είναι τέλειο τετράγωνο ακεραίου.

ΘΕΜΑ 485 (Socrates)

Οι θετικοί αριθμοί x,y,z είναι τέτοιοι ώστε

33 3x 2yz y 2zx z 2xy 2(x y z) . Να δείξετε ότι x y z 6 .

ΘΕΜΑ 486 (Socrates)

Θεωρούμε τραπέζιο ABCD με AD / /BC τέτοιο ώστε AB AD BC . Να

δείξετε ότι η διχοτόμος της A

διχοτομεί την πλευρά CD .

ΘΕΜΑ 487 (Socrates)

Να δείξετε ότι a a b b c c

3( a b c)bc ca ab

, για όλους τους

θετικούς αριθμούς a,b,c με ab bc ca 1 .

ΘΕΜΑ 488 (Socrates)

Να δείξετε ότι 3 3 3

2 2 2

a b c 3

b c c a a b 1 3

, για όλους τους θετικούς

αριθμούς a,b,c με 2 2 2

a b c 1 .

Page 397: ΠΡΟΤΕΙΝΟΜΕΝΕΣ ΑΣΚΗΣΕΙΣ ΓΙΑ ΜΑΘΗΤΙΚΟΥΣ ΔΙΑΓΩΝΙΣΜΟΥΣ_ALL

http://www.mathematica.gr/forum/viewtopic.php?f=109&t=15584

Επιμέλεια: xr.tsif Σελίδα 24

ΘΕΜΑ 489 (Socrates)

Να δείξετε ότι 2

2cyc

a 16

a a 1

για όλους τους θετικούς αριθμούς a,b,c με

cyc

13

a .

Λύση:

Είναι από την x y 2 xy .

2 2

2

2 2 2

a 1 a a 1 a aa a 1 2 a

a a 1 a a 1 a a 1( )

.

Αρκεί τώρα, να αποδειχθεί ότι a b c 3 .

Όμως, από την ανισότητα ΑΜ – ΓΜ είναι

6a b c 3 abc 3 , αφού abc 1 .

Αυτό συμβαίνει επειδή πάλι από την ΑΜ – ΓΜ έχουμε

3

1 1 1 13 3 abc 1

a b c abc .

ΘΕΜΑ 490 (Socrates)

Να δείξετε ότι x y z

xy yz zxa b c για όλους τους θετικούς αριθμούς

a,b,c,x,y,z με 2 2 2

a b c 3 .

Λύση:

Πάλι Cauchy-Schwarz:

Είναι

Page 398: ΠΡΟΤΕΙΝΟΜΕΝΕΣ ΑΣΚΗΣΕΙΣ ΓΙΑ ΜΑΘΗΤΙΚΟΥΣ ΔΙΑΓΩΝΙΣΜΟΥΣ_ALL

http://www.mathematica.gr/forum/viewtopic.php?f=109&t=15584

Επιμέλεια: xr.tsif Σελίδα 25

2 2( x y z ) ( x y z )x y z

xy yz zxa b c a b c 3

,

όπου έγινε χρήση της 2 2 2

a b c 3(a b c )

και της 2(a b c) 3(ab bc ca) για a x κ.τ.λ.

ΘΕΜΑ 491 (Socrates)

Να δείξετε ότι για όλους τους θετικούς αριθμούς a,b,c

2 2 2 2 2 22ab 2bc 2ac(a b ) c (b c ) a (c a ) b 0

a c a b b c

.

Λύση:

Καταρχήν λόγω της 2 2 22(a b ) (a b) παίρνουμε ότι

2 22ab(a b )

ab(a b)a b

(1) .

Οι τριάδες 2 2 2 2 2 2(a,b,c), (a b ,a c ,b c ), (ab,ac,bc) έχουν την ίδια διάταξη

συνεπώς το ίδιο συμβαίνει και με τις τριάδες 2 2 2 2 2 2

(a,b,c), (2ab(a b ), 2ac(a c ),2bc(b c )) ,

άρα τελικά οι τριάδες

2 2 2 2 2 2(2ab(a b ),2ac(a c ),2bc(b c )) και

1 1 1, ,

a b a c b c

έχουν

αντίθετη διάταξη.

Συνεπώς από την ανισότητα της αναδιάταξης παίρνουμε:

2 2 2 2 2 2 2 22ab(a b ) 2ac(a c ) 2bc(b c ) 2ab(a b )

a c b c a b b

a

Page 399: ΠΡΟΤΕΙΝΟΜΕΝΕΣ ΑΣΚΗΣΕΙΣ ΓΙΑ ΜΑΘΗΤΙΚΟΥΣ ΔΙΑΓΩΝΙΣΜΟΥΣ_ALL

http://www.mathematica.gr/forum/viewtopic.php?f=109&t=15584

Επιμέλεια: xr.tsif Σελίδα 26

2 2 2 2 (1)

2ac(a c ) 2bc(b c )

ab(a b) ac(a c) bc(b c)a c b c

2 2 2 2 2 2c(a b ) b(a c ) a(b c ) , που είναι η ζητούμενη..

ΘΕΜΑ 492 (Socrates)

Έστω *x,y

με x y 1 . Βρείτε το ελάχιστο του

1 1x y

x y .

ΘΕΜΑ 493 (Socrates)

(a) Βρείτε μια λύση της εξίσωσης 2 2x y 1010 στο σύνολο των ακεραίων.

(b) Δείξτε ότι η εξίσωση 2 2 2x y 1000 10z έχει άπειρες ακέραιες λύσεις.

Είναι από εδώ: http://forum.gil.ro/viewtopic.php?f=38&t=571&start=0

ΣΗΜΕΙΩΣΕΙΣ

Θα προσπαθήσω να εξηγήσω εδώ ποιοί θετικοί ακέραιοι γράφονται σαν

άθροισμα δύο ακέραιων τετραγώνων και με πόσους τρόπους μπορεί να γίνει

αυτό.

Το πρώτο λήμμα είναι αρκετά απλό και η απόδειξή του αφήνεται στον

αναγνώστη.

Λήμμα 1: Αν n 3(mod4) τότε ο n δεν μπορεί να γραφτεί σαν άθροισμα δύο

τετραγώνων.

Το επόμενο λήμμα μάλλον είναι γνωστό σε αρκετούς.

Μια απόδειξή του βασίζεται σε τετραγωνικά ισοϋπόλοιπα για τα οποία ίσως να

μιλήσουμε άλλη φορά.

Λήμμα 2: Αν p 3(mod4) πρώτος και 2 2

p / m n , τότε p / m και p / n .

Page 400: ΠΡΟΤΕΙΝΟΜΕΝΕΣ ΑΣΚΗΣΕΙΣ ΓΙΑ ΜΑΘΗΤΙΚΟΥΣ ΔΙΑΓΩΝΙΣΜΟΥΣ_ALL

http://www.mathematica.gr/forum/viewtopic.php?f=109&t=15584

Επιμέλεια: xr.tsif Σελίδα 27

(Άρα 2 2 2p / m n .)

Πόρισμα 3: Αν ο αριθμός n γράφεται σαν άθροισμα δύο τετραγώνων, και

p 3(mod4) πρώτος που διαιρεί τον n τότε υπάρχει k ώστε ο 2kp διαιρεί τον

n αλλά ο 2k 1p

δεν τον διαιρεί.

Η απόδειξη είναι σχετικά εύκολη και αφήνεται στον αναγνώστη.

Θα δούμε σε λίγο ότι ισχύει και το αντίστροφο. Για αρχή

Θεώρημα 4 (Fermat): Κάθε πρώτος της μορφής p 1(mod4) μπορεί να

γραφτεί σαν άθροισμα δύο τετραγώνων

Μπορείτε να βρείτε αρκετές αποδείξεις εδώ .

Παρατήρηση 5: Αν ο n και ο m μπορούν να γραφούν σαν αθροίσματα δύο

τετραγώνων (επιτρέπουμε την χρησιμοποίηση του 0), τότε το ίδιο ισχύει και για

τον n m .

Η απόδειξη υπάρχει και στο παραπάνω link, αλλά την βάζω διότι είναι σύντομη,

εξαιρετικά όμορφη και θα χρησιμεύσει αργότερα.

Απόδειξη: Αν 2 2

n b και 2 2

m c d τότε 2 2nm ( c bd) (ad bc) .

Επειδή ο 2 γράφεται σαν άθροισμα δύο τετραγώνων, και κάθε τετράγωνο

γράφεται σαν άθροισμα δύο τετραγώνων έχουμε :

Θεώρημα 6: Ο αριθμός n γράφεται σαν άθροισμα δύο τετραγώνων αν και μόνο

αν όλοι οι διαιρέτες του της μορφής p 3(mod4) βρίσκονται σε άρτια δύναμη

στην παραγοντοποίηση του n .

Τώρα ξέρουμε ποιοί αριθμοί γράφονται σαν άθροισμα δύο τετραγώνων αλλά με

Page 401: ΠΡΟΤΕΙΝΟΜΕΝΕΣ ΑΣΚΗΣΕΙΣ ΓΙΑ ΜΑΘΗΤΙΚΟΥΣ ΔΙΑΓΩΝΙΣΜΟΥΣ_ALL

http://www.mathematica.gr/forum/viewtopic.php?f=109&t=15584

Επιμέλεια: xr.tsif Σελίδα 28

πόσους τρόπους μπορεί να γίνει αυτό;

Θεώρημα 4β: Κάθε πρώτος της μορφής p 1(mod4) μπορεί να γραφτεί με

μοναδικό τρόπο σαν άθροισμα δύο τετραγώνων.

Για παράδειγμα, ο μόνος τρόπος να γράψουμε το 1997 ως άθροισμα δύο

τετραγώνων είναι 2 21997 34 29 .

Το 22 1997 που μας απασχόλησε σε άλλη άσκηση μπορεί να γραφτεί σαν

2 2 2 2 2 2 22 1997 (1 1 ) (29 34 ) (29 34 ) και χρησιμοποιώντας τον τύπο στην

απόδειξη της παρατήρησης 5 βρίσκουμε 2 2 2 2 2 2 2 2 2 2 2

2 1997 (1 1 ) (29 34 ) (29 34 ) (1 1 ) (1997 0 ) 2 2

1997 1997 2 2 2 2 2 2 2 2 2 2 2

2 1997 (1 1 ) (29 34 ) (29 34 ) (1 1 ) (1972 315 ) 2 2

2287 1657 .

Το θεώρημα λέει πως αν δουλέψουμε όπως πιο πάνω τότε μπορούμε να βρούμε

όλους τους τρόπους που γράφεται ένας αριθμός σαν άθροισμα δύο τετραγώνων.

(Δεν είμαι σίγουρος αν αυτό το θεώρημα είναι του Gauss ή όχι. Μπορεί όμως να

αποδειχθεί χρησιμοποιώντας Gaussian integers.)

Μπορεί κάποιος να μετρήσει με πόσους τρόπους μπορεί να γίνει αυτό αλλά είναι

λίγο πολύπλοκο και θέλει αρκετή προσοχή. (Για παράδειγμα, στην άσκηση που

μας ενδιέφερε είχα πει πως γίνεται με τρεις τρόπους επειδή μετρούσα το 2 2

1997 1997 μία φορά και το 2 2

2287 1657 δύο φορές.)

ΘΕΜΑ 494 (Socrates)

Έστω a,b,c,d με 2 2

a b 1 . Δείξτε ότι 2 2 2 2 2

(1 bd) (a b 1)(c d 1) .

Page 402: ΠΡΟΤΕΙΝΟΜΕΝΕΣ ΑΣΚΗΣΕΙΣ ΓΙΑ ΜΑΘΗΤΙΚΟΥΣ ΔΙΑΓΩΝΙΣΜΟΥΣ_ALL

http://www.mathematica.gr/forum/viewtopic.php?f=109&t=15584

Επιμέλεια: xr.tsif Σελίδα 29

ΘΕΜΑ 495 (sokratis lyras)

Έστω η ακολουθία n

a με 2

n

1 2 n 1

n 1

a ca a,a b,c 0,a

a

και ab 0 . Nα

δείξετε ότι 2 2

i

a b ca Z a,b, Z

ab

για κάθε i N .

Πρόκειται για το Πρόβλημα 3 της Βαλκανικής Μαθηματικής Ολυμπιάδας του

1986.

ΘΕΜΑ 496 (Socrates)

Λύστε στους πρώτους την εξίσωση 2 2

p q r s t .

ΘΕΜΑ 497 (Socrates)

Να βρείτε όλους τους θετικούς ακεραίους που είναι ίσοι με 300 φορές το

άθροισμα των ψηφίων τους.

ΘΕΜΑ 498 (Socrates)

Υπάρχουν ακέραιοι 0 a b c , τέτοιοι ώστε 1 1 1 1

b a cb 2 a c

;

ΘΕΜΑ 499 (Socrates)

Να λυθεί η εξίσωση 1 1 1 1 1

a b [a,b] (a,b) 2 .

ΘΕΜΑ 500 (Socrates)

Αν οι διαφορετικοί ανά δύο και μη μηδενικοί αριθμοί a,b,c,d είναι τέτοιοι ώστε

ac bd και a b c d

4b c d a , να βρείτε το μέγιστο του

a c b d

c a d b .

http://forum.gil.ro/viewtopic.php?f=19&t=3394

Page 403: ΠΡΟΤΕΙΝΟΜΕΝΕΣ ΑΣΚΗΣΕΙΣ ΓΙΑ ΜΑΘΗΤΙΚΟΥΣ ΔΙΑΓΩΝΙΣΜΟΥΣ_ALL

http://www.mathematica.gr/forum/viewtopic.php?f=109&t=15584

Επιμέλεια: xr.tsif Σελίδα 1

Page 404: ΠΡΟΤΕΙΝΟΜΕΝΕΣ ΑΣΚΗΣΕΙΣ ΓΙΑ ΜΑΘΗΤΙΚΟΥΣ ΔΙΑΓΩΝΙΣΜΟΥΣ_ALL

http://www.mathematica.gr/forum/viewtopic.php?f=109&t=15584

Επιμέλεια: xr.tsif Σελίδα 2

ΠΡΟΤΕΙΝΟΜΕΝΕΣ

ΑΣΚΗΣΕΙΣ ΓΙΑ ΜΑΘΗΤΙΚΟΥΣ

ΔΙΑΓΩΝΙΣΜΟΥΣ

ΤΕΥΧΟΣ 6ο

ΑΣΚΗΣΕΙΣ 501 - 600

Αφιερωμένο σε κάθε μαθητή που ασχολείται ή πρόκειται να ασχοληθεί με

Μαθηματικούς διαγωνισμούς

Τσιφάκης Χρήστος : xr.tsif

Page 405: ΠΡΟΤΕΙΝΟΜΕΝΕΣ ΑΣΚΗΣΕΙΣ ΓΙΑ ΜΑΘΗΤΙΚΟΥΣ ΔΙΑΓΩΝΙΣΜΟΥΣ_ALL

http://www.mathematica.gr/forum/viewtopic.php?f=109&t=15584

Επιμέλεια: xr.tsif Σελίδα 3

ΘΕΜΑ 501 (sokratis lyras)

Να βρείτε όλα τα διαδοχικά ζεύγη n n 1

a ,a

της ακολουθίας n

na 2 49,n N

έτσι ώστε n n 1

a pq , a rs

με p,q,r,s P,p q,r s και q p s r όπου

P το σύνολο των πρώτων αριθμών.

ΘΕΜΑ 502 (ΔΗΜΗΤΡΗΣ ΙΩΑΝΝΟΥ)

Να αποδείξετε ότι η εξίσωση 2x z 1(2x) 1 y

, δεν έχει λύση στο σύνολο των

φυσικών αριθμών, όταν ο y είναι πρώτος.

ΘΕΜΑ 503 (vzf)

Να λύσετε την εξίσωση 2 2 2(x 5x 3) 3(x 5x 3) 3(x 1) .

ΘΕΜΑ 504 (vzf)

Να λύσετε την εξίσωση x x

5 24 5 24 10 .

ΘΕΜΑ 505 (Socrates)

Δείξτε ότι 3 3 3 3

2 2 2

a b c (a b c),

1 9b ac 1 9c ab 1 9a bc 18

για όλους τους

θετικούς πραγματικούς αριθμούς a,b,c για τους οποίους ab bc ca 1 .

ΘΕΜΑ 506 (Socrates)

Δείξτε ότι σε κάθε τρίγωνο ισχύει

2 2 2

a b c

2 2 2

a b c

r r ra b c2· .

r r r 3ra b c

ΘΕΜΑ 507 (Socrates)

Οι θετικοί ακέραιοι m,n είναι τέτοιοι ώστε 3 3 3 3n (n 1) (n 2) m . Δείξτε

ότι 4 / n 1 .

Page 406: ΠΡΟΤΕΙΝΟΜΕΝΕΣ ΑΣΚΗΣΕΙΣ ΓΙΑ ΜΑΘΗΤΙΚΟΥΣ ΔΙΑΓΩΝΙΣΜΟΥΣ_ALL

http://www.mathematica.gr/forum/viewtopic.php?f=109&t=15584

Επιμέλεια: xr.tsif Σελίδα 4

ΘΕΜΑ 508 (Socrates)

Να βρεθούν οι ακέραιες ρίζες της εξίσωσης 4n 4n n n 2n 2na b a b 3a b 0 , όπου

n N* .

ΘΕΜΑ 509 (Socrates)

Βρείτε όλους τους θετικούς ακεραίους m,n,p όπου p πρώτος, τέτοιους ώστε ο

αριθμός m n

m n

5 2 p

5 2 p

να είναι τέλειο τετράγωνο ακεραίου.

ΘΕΜΑ 510 (Socrates)

Αν οι πραγματικοί αριθμοί x,y,α είναι τέτοιοι ώστε

2 2 22x 2y 2αx 2αy α 4 , να δείξετε ότι | x y | 2 .

ΘΕΜΑ 511 (Socrates)

Αν σε τρίγωνο ABC οι αριθμοί R,S,r αποτελούν με αυτή τη σειρά διαδοχικούς

όρους αριθμητικής προόδου , όπου S το εμβαδόν του τριγώνου, R και r είναι η

ακτίνα του περιγεγραμμένου και του εγγεγραμμένου κύκλου αντίστοιχα, να

δείξετε ότι 2 2 2

a b c

9m m m

4 .

ΘΕΜΑ 512 (Socrates)

Να δείξετε ότι 1 1 1

(abc 1)( ) 3 a b ca b c

για κάθε a,b,c 1 .

ΘΕΜΑ 513 (Socrates)

Αν x,y ακέραιοι, x,y 1 τέτοιοι ώστε 4 4

x 1 y 1Z

y 1 x 1

, να δείξετε ότι

4 44x 1| x y 1 .

Page 407: ΠΡΟΤΕΙΝΟΜΕΝΕΣ ΑΣΚΗΣΕΙΣ ΓΙΑ ΜΑΘΗΤΙΚΟΥΣ ΔΙΑΓΩΝΙΣΜΟΥΣ_ALL

http://www.mathematica.gr/forum/viewtopic.php?f=109&t=15584

Επιμέλεια: xr.tsif Σελίδα 5

ΘΕΜΑ 514 (Socrates)

α) Να βρείτε τη μέγιστη τιμή της σταθερής C έτσι ώστε 2 2x y 1 C(x y) ,

για κάθε x,y R .

β) Να βρείτε τη μέγιστη τιμή της σταθερής C έτσι ώστε 2 2

x y xy 1 C(x y) , για κάθε x,y R .

ΘΕΜΑ 515 (Socrates)

Βρείτε όλους τους πρώτους pγια τους οποίους οι αριθμοί

2 2 2p 2,2p 1,3p 4 είναι επίσης πρώτοι.

ΘΕΜΑ 516 (Socrates)

Έστω n 0 ένας ακέραιος. Να προσδιορίσετε όλα τα πολυώνυμα βαθμού 2nμε

πραγματικούς συντελεστές και μη αρνητικές ρίζες, της μορφής

2n 2n 1 2n 2 2

2 2n 2P(X) X (2n 10)X a X ... a X (2n 10)X 1

.

ΘΕΜΑ 517 (Socrates)

Να λυθεί η εξίσωση 2x 6x [x] 7 0 .

ΘΕΜΑ 518 (Socrates)

Να λυθεί το σύστημα

3 2

2 2

x 3xy 49

x 8xy y 8y 17x

.

ΘΕΜΑ 519 (Socrates)

Έστω m,n θετικοί ακέραιοι. Αν ο αριθμός 2 2

2 2

n mp

n m

είναι πρώτος, να

δείξετε ότι p 1(mod8) .

Page 408: ΠΡΟΤΕΙΝΟΜΕΝΕΣ ΑΣΚΗΣΕΙΣ ΓΙΑ ΜΑΘΗΤΙΚΟΥΣ ΔΙΑΓΩΝΙΣΜΟΥΣ_ALL

http://www.mathematica.gr/forum/viewtopic.php?f=109&t=15584

Επιμέλεια: xr.tsif Σελίδα 6

ΘΕΜΑ 520 (Socrates)

Να δείξετε ότι σε κάθε τρίγωνο ισχύει

2 2 2

A B C B A C C A B

2 r 2 r 2 ra b c 4(R 3r)

r r r r r r r r r

.

ΘΕΜΑ 521 (Socrates)

Να βρεθούν όλοι οι άρτιοι θετικοί ακέραιοι n τέτοιοι ώστε

1 2 k

1 1 1 1620

d d d 1003 όπου

1 2 3 kd d d ... d οι διαιρέτες του.

Λύση:

Γράφω λίγα περισσότερα από ότι χρειάζονται για την λύση για να δούμε και την

σχετική θεωρία.

Αν ο d είναι διαιρέτης του n τότε και ο n

d πρέπει να είναι διαιρέτης του n . Άρα

οι 1

n

d,…,

k

n

d είναι επίσης διαιρέτες του n . Επίσης είναι διαφορετικοί μεταξύ

τους και άρα πρέπει να είναι όλοι οι διαιρέτες του n . Επομένως η συνθήκη

μετασχηματίζεται σε 1 kd d 1620

n n 1003 ή ισοδύναμα σε 1620n 1003σ(n) ,

όπου με (n) συμβολίζουμε το άθροισμα των διαιρετών του n .

Ας μελετήσουμε λοιπόν λίγο περισσότερο αυτήν την συνάρτηση.

Έστω ότι 1 2 kr r r

1 2 kn p p p η παράσταση του n σε γινόμενο διαφορετικών πρώτων

παραγόντων. Τότε κάθε διαιρέτης του n πρέπει να είναι της μορφής t1 2 ss s

1 2 tp p p

όπου i i

0 s r για κάθε i .

Page 409: ΠΡΟΤΕΙΝΟΜΕΝΕΣ ΑΣΚΗΣΕΙΣ ΓΙΑ ΜΑΘΗΤΙΚΟΥΣ ΔΙΑΓΩΝΙΣΜΟΥΣ_ALL

http://www.mathematica.gr/forum/viewtopic.php?f=109&t=15584

Επιμέλεια: xr.tsif Σελίδα 7

Άρα για να βρούμε το (n) πρέπει να προσθέσουμε όλους αυτούς τους

αριθμούς. Γνωρίζοντας ήδη το αποτέλεσμα ισχυρίζομαι πως

t1 1 rr r2 2 2

1 1 1 2 2 2 t t t(n) (1 p p p )(1 p p p ) (1 p p p ) .

Για την απόδειξη του ισχυρισμού παρατηρούμε ότι αν κάνουμε όλους τους

πολλαπλασιασμούς στο πιο πάνω γινόμενο θα πάρουμε άθροισμα αριθμών της

μορφής t1 2 ss s

1 2 tp p p με

i i0 s r για κάθε i . Επιπλέον κάθε τέτοιος αριθμός

εμφανίζεται στο άθροισμα ακριβώς μία φορά οπότε το άθροισμα πράγματι

ισούται με (n) .

Επομένως ισχύει ότι

t1 rr2 2

1 1 1 t t t

(n) 1 1 1 1 1 11 1

n p p p p p p

.

Από εδώ παρατηρούμε άμεσα ότι αν ο m είναι πολλαπλάσιο του n με m n

τότε σ(m) σ(n)

m n .

Αυτό γιατί στο αντίστοιχο γινόμενο για το σ(m)

m κάθε ένας από τους

παράγοντες είτε θα είναι ο ίδιος είτε θα έχει επιπλέον όρους στο άθροισμά του.

Επιπλέον το αντίστοιχο γινόμενο για το σ(m)

m μπορεί να έχει και άλλους

παράγοντες που αντιστοιχούν σε πρώτους που δεν διαιρούν m αλλά όχι τον n .

Τέλος επειδή m n δεν μπορεί να είναι όλοι οι παράγοντες των δύο γινομένων

ακριβώς οι ίδιοι.

Πάμε τώρα πίσω στο πρόβλημα. Ξέρουμε ότι 1620n 1003σ(n) επομένως ο n

είναι πολλαπλάσιο του 2003 . (Αφού οι 1620 και 2003 δεν έχουν κοινούς

παράγοντες.)

Επειδή ο n είναι άρτιος, πρέπει να είναι και πολλαπλάσιο του 2006 2·17·59 .

Άρα 1620 σ(n) σ(2006) 3 18 60 1620

· ·1003 n 2006 2 17 59 1003

.

Page 410: ΠΡΟΤΕΙΝΟΜΕΝΕΣ ΑΣΚΗΣΕΙΣ ΓΙΑ ΜΑΘΗΤΙΚΟΥΣ ΔΙΑΓΩΝΙΣΜΟΥΣ_ALL

http://www.mathematica.gr/forum/viewtopic.php?f=109&t=15584

Επιμέλεια: xr.tsif Σελίδα 8

Όπως έχουμε δείξει πιο πάνω η ισότητα ισχύει αν και μόνο αν n 2006 .

ΘΕΜΑ 522 (ΔΗΜΗΤΡΗΣ ΙΩΑΝΝΟΥ)

Nα λυθεί η εξίσωση: 2 2 2 2

2 2 2 2

x 4x 3 x 4x 3 x 6x 8 x 6x 8

x 4x 3 x 4x 3 x 6x 8 x 6x 8

.

ΘΕΜΑ 523 (ΔΗΜΗΤΡΗΣ ΙΩΑΝΝΟΥ)

Αν ο αριθμός 9 9 4n 1

A 9n2 2

, όπου n N , είναι θετικός ρητός, να

αποδείξετε ότι και ο αριθμός A , είναι επίσης θετικός ρητός.

ΘΕΜΑ 524 (ΔΗΜΗΤΡΗΣ ΙΩΑΝΝΟΥ)

Αποδείξτε ότι ο αριθμός: A 111...11122...2225 ,όπου οι άσσοι έχουν πλήθος

1997 και τα δυάρια έχουν πλήθος 1998 , είναι τέλειο τετράγωνο.

(Βαλκανική Μαθηματική Ολυμπιάδα για "Μικρούς")

ΘΕΜΑ 525 (ΔΗΜΗΤΡΗΣ ΙΩΑΝΝΟΥ)

Αν a,b,c 0 , a b c 1 , να δειχθεί ότι: 2 2 2112abc a b c 1 2abc

27 .

(Από το περιοδικό "ΕΥΚΛΕΙΔΗΣ" της Ε.Μ.Ε)

ΘΕΜΑ 526 (Socrates)

Να βρεθούν οι ακέραιες λύσεις της εξίσωσης n 2

3 7 k .

ΘΕΜΑ 527 (Socrates)

Να αποδείξετε ότι 2(abc) (a b c)(b c a)(c a b) για κάθε x,y,z 0

με a b c 3 .

Page 411: ΠΡΟΤΕΙΝΟΜΕΝΕΣ ΑΣΚΗΣΕΙΣ ΓΙΑ ΜΑΘΗΤΙΚΟΥΣ ΔΙΑΓΩΝΙΣΜΟΥΣ_ALL

http://www.mathematica.gr/forum/viewtopic.php?f=109&t=15584

Επιμέλεια: xr.tsif Σελίδα 9

ΘΕΜΑ 528 (Socrates)

Αν a,b,c 0 να δείξετε την ισοδυναμία

a b 1 a c 1 a d 1· · · 3 a bcd

b 1 cd c 1 bd d 1 cb

.

ΘΕΜΑ 529 (Socrates)

Θεωρούμε την ακολουθία 2 2

1 2 n 2 n 1 na 2,a 5, a (2 n )a (2 n )a , n 1

.

Υπάρχουν p,q,r ώστε p q r

a a a ;

ΘΕΜΑ 530 (Socrates)

Αν a,b,c θετικοί πραγματικοί αριθμοί τέτοιοι ώστε a b c 1 , να δείξετε ότι

3 2 2 3 2 2 3 2 2

a b c 81

a b c c b b c a a c c a b b a 27abc

.

ΘΕΜΑ 531 (Socrates)

Υπάρχουν ακέραιοι x,y,z,t ώστε 5 5 5 5x y z t 93 ;

ΘΕΜΑ 532 (Socrates)

Να λυθεί η εξίσωση [x]·{x} 2005x .

ΘΕΜΑ 533 (Socrates)

Στο επίπεδο, θεωρούμε σημεία 1 2 2010

A ,A , ,A και αυθαίρετο κύκλο (C)

ακτίνας 1 .Να δείξετε ότι υπάρχει σημείο S του κύκλου τέτοιο ώστε

1 2 2010SA SA ... SA 2010 .

Page 412: ΠΡΟΤΕΙΝΟΜΕΝΕΣ ΑΣΚΗΣΕΙΣ ΓΙΑ ΜΑΘΗΤΙΚΟΥΣ ΔΙΑΓΩΝΙΣΜΟΥΣ_ALL

http://www.mathematica.gr/forum/viewtopic.php?f=109&t=15584

Επιμέλεια: xr.tsif Σελίδα 10

ΘΕΜΑ 534 (Freyia)

Να λυθεί στους φυσικούς αριθμούς ( *N ), η εξίσωση:

(7x 3y 16)(17y 15) 32 .

(Να αποφύγετε να πάρετε περιπτώσεις).

ΘΕΜΑ 535 (Socrates)

Βρείτε τους πρώτους p,q αν 2 3p q q p .

ΘΕΜΑ 536 (Socrates)

Βρείτε τους πρώτους p,q αν 2 2p q q 145p (1) .

ΘΕΜΑ 537 (Socrates)

Βρείτε τους θετικούς ακεραίους x,y για τους οποίους ο αριθμός 2

xy

x y είναι

πρώτος.

ΘΕΜΑ 538 (Socrates)

Να βρείτε τους φυσικούς a,b,c αν ισχύουν 8 9 11[ab,c] 2 , [bc,a] 2 , [ca,b] 2 .

ΘΕΜΑ 539 (Socrates)

Να βρείτε όλους τους φυσικούς αριθμούς n , για τους οποίους ο αριθμός n n

(2 1)(3 2) διαιρείται από τον n

5 .

ΣΗΜΕΙΩΣΗ: Υπενθυμίζουμε ότι:

α) Αν ένας αριθμός λήγει σε 0,1,5,6 τότε οποιαδήποτε δύναμη με βάση αυτόν

τον αριθμό, θα λήγει επίσης σε 0,1,5,6 .

β) Αν ένας αριθμός λήγει σε 4,9 τότε για να βρούμε που θα λήγει μια δύναμη με

βάση αυτόν τον αριθμό, παίρνουμε δύο περιπτώσεις για τον εκθέτη n :

Page 413: ΠΡΟΤΕΙΝΟΜΕΝΕΣ ΑΣΚΗΣΕΙΣ ΓΙΑ ΜΑΘΗΤΙΚΟΥΣ ΔΙΑΓΩΝΙΣΜΟΥΣ_ALL

http://www.mathematica.gr/forum/viewtopic.php?f=109&t=15584

Επιμέλεια: xr.tsif Σελίδα 11

1η ΠΕΡΙΠΤΩΣΗ : n 2k

2η ΠΕΡΙΠΤΩΣΗ : n 2k 1

γ) Αν ένας αριθμός λήγει σε 2,3,7,8 τότε για να βρούμε που θα λήγει μια

δύναμη με βάση αυτόν τον αριθμό, παίρνουμε τέσσερις περιπτώσεις για τον

εκθέτη n :

1η ΠΕΡΙΠΤΩΣΗ : n 4k

2η ΠΕΡΙΠΤΩΣΗ : n 4k 1

3η ΠΕΡΙΠΤΩΣΗ : n 4k 2

4η ΠΕΡΙΠΤΩΣΗ : n 4k 3

Επανερχόμενοι τώρα στο πρόβλημά μας, έχουμε:

1η ΠΕΡΙΠΤΩΣΗ : n 4k .

Τότε n 4k k

2 1 2 1 16 1 και άρα λήγει σε 7 , που δεν μας κάνει.

2η ΠΕΡΙΠΤΩΣΗ : n 4k 1 .

Τότε n 4k k

2 1 2 2 1 16 2 1 και άρα λήγει σε 3 , που επίσης δεν μας

κάνει

3η ΠΕΡΙΠΤΩΣΗ : n 4k 2 .

Τότε n 4k 2 k

2 1 2 2 1 16 4 1 , και άρα λήγει 5 . Αυτό ίσως μας κάνει,

αλλά όμως: n 4k 2 k

3 2 3 2 81 9 2 , που λήγει σε 1 και άρα ούτε η

περίπτωση αυτή μας ενδιαφέρει.

4η ΠΕΡΙΠΤΩΣΗ : n 4k 3 .

Τότε n k

2 1 16 8 1 , που λήγει σε 9 .

Συνεπώς , μόνο για n 1 , έχουμε το ζητούμενο.

Page 414: ΠΡΟΤΕΙΝΟΜΕΝΕΣ ΑΣΚΗΣΕΙΣ ΓΙΑ ΜΑΘΗΤΙΚΟΥΣ ΔΙΑΓΩΝΙΣΜΟΥΣ_ALL

http://www.mathematica.gr/forum/viewtopic.php?f=109&t=15584

Επιμέλεια: xr.tsif Σελίδα 12

ΘΕΜΑ 540 (Socrates)

Να συγκριθούν οι αριθμοί p q q p και p p q q .

ΘΕΜΑ 541 (Socrates)

Να υπολογίσετε το άθροισμα 3 3 3 3 3 3

3 3 3 3 3

3 1 5 2 4013 2006S ....

2 1 3 2 2007 2006

.

ΘΕΜΑ 542 (Socrates)

Για ποιους ακέραιους n είναι ο αριθμός 4 2

n 3n 9 πρώτος;

ΘΕΜΑ 543 (Socrates)

Οι πραγματικοί αριθμοί a,b,c,d είναι τέτοιοι ώστε 2 2 2 2 2 2

a b b c c d 1 . Να δείξετε ότι ab ac ad bc bd cd 3 .

ΘΕΜΑ 544 (ΔΗΜΗΤΡΗΣ ΙΩΑΝΝΟΥ)

Αν x,y,z θετικοί αριθμοί με 2 2 2x y z 1 , τότε να βρεθεί η ελάχιστη τιμή της

συνάρτησης: 2 2 2 2 2 2

x x y y z zf (x,y,z)

y z x z x y .

(Από το περιοδικό της ΕΜΕ ¨ΕΥΚΛΕΙΔΗΣ Β¨)

ΘΕΜΑ 545 (Socrates)

Να δείξετε ότι

1 1 1 2 2 2... 1 ...

852 853 2554 2* 3* 4 5* 6* 7 2552* 2553* 2554 .

Page 415: ΠΡΟΤΕΙΝΟΜΕΝΕΣ ΑΣΚΗΣΕΙΣ ΓΙΑ ΜΑΘΗΤΙΚΟΥΣ ΔΙΑΓΩΝΙΣΜΟΥΣ_ALL

http://www.mathematica.gr/forum/viewtopic.php?f=109&t=15584

Επιμέλεια: xr.tsif Σελίδα 13

ΘΕΜΑ 546 (Socrates)

Να λυθεί το σύστημα

2 2

2 2

2 2

x y z 1

y z x 1

z x y 1

.

ΘΕΜΑ 547 (Socrates)

Βρείτε το υπόλοιπο της διαίρεσης του αριθμού

N 1 3 5 ... 2549 2 4 6 ... 2550 με το 2551 .

ΘΕΜΑ 548 (Socrates)

Να λυθεί το σύστημα

2 2

2 2

2 2

x y z

y z x

z x y

.

ΘΕΜΑ 549 (Socrates)

Να βρεθούν οι ακέραιες ρίζες της a 2 2

4 4a 4 b .

ΘΕΜΑ 550 (Socrates)

Αν x,y,z θετικοί αριθμοί να δείξετε ότι ένας, τουλάχιστον από τους αριθμούς

x y z xyz και xy yz zx 3 είναι θετικός.

Βρείτε παράδειγμα τέτοιων αριθμών για τους οποίους οι αριθμοί x y z xyz

και xy yz zx 3 είναι ετερόσημοι (μη μηδενικοί).

ΘΕΜΑ 551 (Socrates)

Αν η εξίσωση (x m)(x n) x m n έχει ακέραια ρίζα να δείξετε ότι

1 m2

2 n .

Page 416: ΠΡΟΤΕΙΝΟΜΕΝΕΣ ΑΣΚΗΣΕΙΣ ΓΙΑ ΜΑΘΗΤΙΚΟΥΣ ΔΙΑΓΩΝΙΣΜΟΥΣ_ALL

http://www.mathematica.gr/forum/viewtopic.php?f=109&t=15584

Επιμέλεια: xr.tsif Σελίδα 14

ΘΕΜΑ 552 (Socrates)

Αν a,b,c [0,1] τότε 1 a b c 2(ab bc ca) 3(1 a)(1 b)(1 c) 9 .

Πότε έχουμε ισότητα;

ΘΕΜΑ 553 (Socrates)

Να βρεθούν οι ρητοί x,y αν x 5 y 5 6 5 10 .

ΘΕΜΑ 554 (Socrates)

Να λυθεί το σύστημα

2

2

2

x y z 1

y z x 1

z x y 1

.

ΘΕΜΑ 555 (Socrates)

Να λυθεί το σύστημα

2

2

2

x y z 1

y z x 1

z x y 1

.

ΘΕΜΑ 556 (Socrates)

Ένα ημικυκλικό κομμάτι χαρτί, ακτίνας 10 , διπλώνεται ώστε να σχηματιστεί

κώνος. Να βρεθεί το ύψος του.

ΘΕΜΑ 557 (Socrates)

Οι πρώτοι αριθμοί p,q είναι τέτοιοι ώστε 2q / p 1 και 2

p / q 1 .

Δείξτε ότι ο αριθμός p q 1 είναι σύνθετος.

Page 417: ΠΡΟΤΕΙΝΟΜΕΝΕΣ ΑΣΚΗΣΕΙΣ ΓΙΑ ΜΑΘΗΤΙΚΟΥΣ ΔΙΑΓΩΝΙΣΜΟΥΣ_ALL

http://www.mathematica.gr/forum/viewtopic.php?f=109&t=15584

Επιμέλεια: xr.tsif Σελίδα 15

ΘΕΜΑ 558 (ΔΗΜΗΤΡΗΣ ΙΩΑΝΝΟΥ)

Να αποδείξετε ότι:

Αν : 2 2 2 21 2 3 ... 50 a , τότε: 1 2 2 3 ... 100 101 8a .

Και μάλιστα ισχύει 2n n

2 2

k 1 k 1

(k k) 8 k

, για κάθε θετικό ακέραιο n .

ΘΕΜΑ 559 (ΔΗΜΗΤΡΗΣ ΙΩΑΝΝΟΥ)

Δίνεται η εξίσωση 2

ax bx c 0 , όπου a,b,c R και a 0 . Αν είναι μια

πραγματική ρίζα της εξίσωσης, δείξτε ότι: 2

2 | ac | b| |

| ab |

.

Δίνεται μια ιδιότητα των απολύτων τιμών: | a b | | a | | b | , | a b | | a | | b | .

ΘΕΜΑ 560 (ΔΗΜΗΤΡΗΣ ΙΩΑΝΝΟΥ)

Θεωρούμε δύο ομόκεντρους κύκλους (O,R) και (O,2R) .

Φέρνουμε την χορδή AB του μεγαλύτερου κύκλου, ώστε να είναι εφαπτόμενη

στον μικρότερο κύκλο, και ονομάζουμε M , το σημείο επαφής.

Από το σημείο A φέρνουμε την εφαπτομένη AN στον μικρό κύκλο.

Να αποδείξετε ότι το εμβαδόν της περιοχής που περικλείεται από τα "ελάσσονα"

τόξα AB , και NM , και από τα ευθύγραμμα τμήματα AN και MB , είναι ίσο με

το εμβαδόν του μικρότερου κύκλου.

ΘΕΜΑ 561 (Socrates)

Η πράξη *: R R R είναι τέτοια ώστε x 1

* x 12

και (x * y)z (xz)* (yz)

για κάθε x,y,z R Βρείτε τον αριθμό 3* 4 και παράδειγμα τέτοιας πράξης.

Page 418: ΠΡΟΤΕΙΝΟΜΕΝΕΣ ΑΣΚΗΣΕΙΣ ΓΙΑ ΜΑΘΗΤΙΚΟΥΣ ΔΙΑΓΩΝΙΣΜΟΥΣ_ALL

http://www.mathematica.gr/forum/viewtopic.php?f=109&t=15584

Επιμέλεια: xr.tsif Σελίδα 16

ΘΕΜΑ 562 (ΔΗΜΗΤΡΗΣ ΙΩΑΝΝΟΥ)

Αν 1 2

x ,x είναι ρίζες των εξισώσεων 2x ax b 0 και 2n n n n

x a x b 0 ,

όπου *n N και b 0 , τότε: n n n

1 2 1 2(x x ) x x .

ΘΕΜΑ 563 (ΘΑΝΑΣΗΣ KARKAR)

Η πάνω και η κάτω πλευρές είναι παράλληλες .

Βρείτε το E .

ΘΕΜΑ 564 (Socrates)

Ένα αυτοκίνητο που κινείται σε ευθύ δρόμο με μία λωρίδα κυκλοφορίας ανά

κατεύθυνση, θέλει να προσπεράσει ένα φορτηγό που βρίσκεται 5mμπροστά

του. Μπαίνει στο αντίθετο ρεύμα κυκλοφορίας και μόλις το προσπεράσει κατά

5m επανέρχεται στο ρεύμα κυκλοφορίας του.

Αν το αυτοκίνητο κινείται με Km

100h

και το φορτηγό με Km

90h

βρείτε το

διάστημα που θα διανύσει το αυτοκίνητο στο αντίθετο ρεύμα κυκλοφορίας.

Δίνεται ότι το μήκος του φορτηγού είναι 15m .

ΘΕΜΑ 565 (Socrates)

Στον πίνακα είναι γραμμένος ο αριθμός 1000 . Δύο μαθητές παίζουν το

ακόλουθο παιχνίδι:

Εναλλάξ, επιλέγουν ένα γνήσιο διαιρέτη (διάφορο του ίδιου του αριθμού) του

αριθμού που υπάρχει στον πίνακα και αντικαθιστούν τον αριθμό που είναι

γραμμένος στον πίνακα με τη (θετική) διαφορά του αριθμού αυτού από τον

διαιρέτη που επέλεξαν.

Page 419: ΠΡΟΤΕΙΝΟΜΕΝΕΣ ΑΣΚΗΣΕΙΣ ΓΙΑ ΜΑΘΗΤΙΚΟΥΣ ΔΙΑΓΩΝΙΣΜΟΥΣ_ALL

http://www.mathematica.gr/forum/viewtopic.php?f=109&t=15584

Επιμέλεια: xr.tsif Σελίδα 17

Χάνει όποιος δεν μπορεί να παίξει.

Ποιος έχει στρατηγική νίκης;

ΘΕΜΑ 566 (ΔΗΜΗΤΡΗΣ ΙΩΑΝΝΟΥ)

Να αποδείξετε ότι ο αριθμός: 3 3 3 3A 6 13 20 ... (7n 1) 22n , είναι

πολλαπλάσιο του 7 , για κάθε *

n N .

ΘΕΜΑ 567 (ΔΗΜΗΤΡΗΣ ΙΩΑΝΝΟΥ)

Να βρεθεί η ελάχιστη τιμή της παράστασης:

2 2A 5x 4xy y 6x 6y 33 με x,y R .

ΘΕΜΑ 568 (ΘΑΝΟΣ ΜΑΓΚΟΣ)

Έστω a Z . Να εξετάσετε αν υπάρχουν ακέραιοι x,y για τους οποίους

2 2 245x 78y a 1 .

ΘΕΜΑ 569 (ΔΗΜΗΤΡΗΣ ΙΩΑΝΝΟΥ)

Να αποδειχθεί ότι:

20 20 20 20 30 30 30 30 11 .

ΘΕΜΑ 570 (ΔΗΜΗΤΡΗΣ ΙΩΑΝΝΟΥ)

Αν 1 2 3 n

a ,a ,a ,...,a , είναι φυσικοί αριθμοί διάφοροι ανά δύο και όλοι τους

μεγαλύτεροι ή ίσοι του 2 , να αποδείξετε ότι:

2

1 2 2 3 n 1 n n 1a a a a ... a a a a n 4n

, για κάθε n N , με n 2 .

ΘΕΜΑ 571 (Socrates)

Να βρεθούν οι θετικές ακέραιες ρίζες της εξίσωσης n

2 a! b! c! .

Page 420: ΠΡΟΤΕΙΝΟΜΕΝΕΣ ΑΣΚΗΣΕΙΣ ΓΙΑ ΜΑΘΗΤΙΚΟΥΣ ΔΙΑΓΩΝΙΣΜΟΥΣ_ALL

http://www.mathematica.gr/forum/viewtopic.php?f=109&t=15584

Επιμέλεια: xr.tsif Σελίδα 18

ΘΕΜΑ 572 (Socrates)

Αν ο πρώτος αριθμός p γράφεται στη μορφή 5 5x y όπου x,y ακέραιοι, να

δείξετε ότι 2

4p 1 v 1

5 2

για κάποιον περιττό v .

ΘΕΜΑ 573 (Socrates)

Να βρεθούν οι ακέραιες ρίζες της εξίσωσης 2 4 3 2

y 2y x 20x 104x 40x 2003 .

ΘΕΜΑ 574 (Socrates)

Βρείτε όλες τις τριάδες θετικών ακεραίων (p,q,n) όπου p,q πρώτοι ώστε

p(p 3) q(q 3) n(n 3) .

ΘΕΜΑ 575 (Socrates)

Να λυθεί η εξίσωση (x 1)(x 2)(x 3)(x 4)(x 5)(x 6) 720 .

ΘΕΜΑ 576 (Socrates)

Προσδιορίστε τα ζεύγη πρώτων αριθμών (p,q)με 2 p,q 100 ώστε οι αριθμοί

p 6,p 10,q 4,q 10 και p q 1 να είναι επίσης πρώτοι.

ΘΕΜΑ 577 (Socrates)

Δείξτε ότι ο αριθμός 2005

2005 γράφεται ως άθροισμα δύο τετραγώνων ακεραίων

αλλά όχι ως άθροισμα δύο κύβων ακεραίων.

ΘΕΜΑ 578 (Socrates)

Να λυθεί το σύστημα

2 2

2 2

y (x 8)(x 2)

y (8 4x)y 5x 16x 16

.

Page 421: ΠΡΟΤΕΙΝΟΜΕΝΕΣ ΑΣΚΗΣΕΙΣ ΓΙΑ ΜΑΘΗΤΙΚΟΥΣ ΔΙΑΓΩΝΙΣΜΟΥΣ_ALL

http://www.mathematica.gr/forum/viewtopic.php?f=109&t=15584

Επιμέλεια: xr.tsif Σελίδα 19

ΘΕΜΑ 579 (Socrates)

Υπάρχουν ακέραιοι x,y και z ώστε 2 2 2z (x 1)(y 1) n αν

(α) n 2006

(β) n 2007 ;

ΘΕΜΑ 580 (Socrates)

Βρείτε τους πρώτους p και q ώστε ο p να διαιρεί τον q 6 και ο q τον p 7 .

ΘΕΜΑ 581 (Socrates)

Σε πόσα μηδενικά τελειώνει ο αριθμός 2007!; Ποιο το τελευταίο μη μηδενικό

ψηφίο του;

ΘΕΜΑ 582 (Socrates)

Έστω 1 2 3

p ,p ,p και 4

p πρώτοι, διαφορετικοί ανά δύο, τέτοιοι ώστε

1 2 3 42p 3p 5p 7p 162

1 2 3 411p 7p 5p 4p 162

Βρείτε όλες τις δυνατές τιμές του γινομένου 1 2 3 4

p p p p .

ΘΕΜΑ 583 (Socrates)

Βρείτε τον ελάχιστο k για τον οποίο ο αριθμός 2010 μπορεί να εκφραστεί ως

άθροισμα k τετραγώνων ακεραίων.

ΘΕΜΑ 584 (Socrates)

Να βρεθεί η μέγιστη τιμή της σταθερής k ώστε 4 2 2 4 4 x x y y k(x y) για

κάθε x,y .

Page 422: ΠΡΟΤΕΙΝΟΜΕΝΕΣ ΑΣΚΗΣΕΙΣ ΓΙΑ ΜΑΘΗΤΙΚΟΥΣ ΔΙΑΓΩΝΙΣΜΟΥΣ_ALL

http://www.mathematica.gr/forum/viewtopic.php?f=109&t=15584

Επιμέλεια: xr.tsif Σελίδα 20

ΘΕΜΑ 585 (Socrates)

Για ποιους θετικούς ακεραίους n είναι ο αριθμός 3 2

n 8n 1

3n

πρώτος ;

ΘΕΜΑ 586 (Socrates)

Να βρεθούν οι ακέραιες λύσεις της εξίσωσης 2 21 x y x 2xy 2x y .

ΘΕΜΑ 587 (Socrates)

α) Δείξτε ότι υπάρχουν πραγματικοί x,y,z τέτοιοι ώστε

x y z 0 και xy yz zx 3 .

β) Αν οι πραγματικοί x,y,z είναι τέτοιοι ώστε x y z 0 και

xy yz zx 3 να βρείτε την τιμή της παράστασης 3 3 3x y y z z x .

ΘΕΜΑ 588 (ΔΗΜΗΤΡΗΣ)

Δείξτε ότι: 5 6 7 104 1 1 1 1

... ...1 3 2 4 3 5 100 102 2 3 4 340

.

ΘΕΜΑ 589 (Socrates)

Να δείξετε ότι η παράσταση

2 2

2 2

n 0 n 1 n 2 ... n n 1 n n

n 0 n 1 n 2 ... n n 1 n n

είναι σταθερή, δηλαδή ανεξάρτητη του θετικού ακεραίου n .

ΘΕΜΑ 590 (Socrates)

Βρείτε όλες τις τριάδες (x,y,p) όπου x,y ακέραιοι και pπρώτος τέτοιες ώστε

Page 423: ΠΡΟΤΕΙΝΟΜΕΝΕΣ ΑΣΚΗΣΕΙΣ ΓΙΑ ΜΑΘΗΤΙΚΟΥΣ ΔΙΑΓΩΝΙΣΜΟΥΣ_ALL

http://www.mathematica.gr/forum/viewtopic.php?f=109&t=15584

Επιμέλεια: xr.tsif Σελίδα 21

2 2 2x 3xy p y 12p .

ΘΕΜΑ 591 (Socrates)

Να λυθεί η εξίσωση 6 3x 2[x] 1 0 .

ΘΕΜΑ 592 (Socrates)

Να βρεθούν οι ακέραιες λύσεις του συστήματος

2

1 1 2

2

2 2 3

2

n n 1

a a 2a 1

a a 2a 1

...

a a 2a 1

.

ΘΕΜΑ 593 (Socrates)

Βρείτε όλους τους πρώτους p ώστε ο αριθμός p(p 17) να γράφεται ως

γινόμενο δύο διαδοχικών θετικών ακεραίων.

ΘΕΜΑ 594 (ΔΗΜΗΤΡΗΣ)

Έστω 2 2 2 2

1 1 1 1x ...

2 3 4 100 . Δείξτε ότι:

x0,2 0,3

11 .

ΘΕΜΑ 595 (ΔΗΜΗΤΡΗΣ)

Αν a,b,c είναι πρώτοι αριθμοί και 2 a 7 b c 3 , να βρεθούν οι a,b,c .

ΘΕΜΑ 596 (ΔΗΜΗΤΡΗΣ)

Δείξτε ότι ο αριθμός 2009 2009 2009 2009

...1 3 3 5 5 7 2007 2009

είναι φυσικός.

ΘΕΜΑ 597 (ΔΗΜΗΤΡΗΣ)

Page 424: ΠΡΟΤΕΙΝΟΜΕΝΕΣ ΑΣΚΗΣΕΙΣ ΓΙΑ ΜΑΘΗΤΙΚΟΥΣ ΔΙΑΓΩΝΙΣΜΟΥΣ_ALL

http://www.mathematica.gr/forum/viewtopic.php?f=109&t=15584

Επιμέλεια: xr.tsif Σελίδα 22

Αν *n N ,a N , με a 2 , τότε ο αριθμός

n 1

n

1 aA

1 a

δεν μπορεί να είναι

φυσικός.

ΘΕΜΑ 598 (ΔΗΜΗΤΡΗΣ)

Δείξτε ότι: 2012 2013 2012 2013 2012 2013 2013 .

ΘΕΜΑ 599 (ΔΗΜΗΤΡΗΣ)

Αν a,b,c Q και ab bc ca 2013 , δείξτε ότι ο αριθμός:

2 2 2A (2013 a )(2013 b )(2013 c ) είναι ρητός.

ΘΕΜΑ 600 (ΔΗΜΗΤΡΗΣ)

Αν a,b N,a,b 2 και αν επί πλέον είναι a b , δείξτε ότι: ab a b 1 .

Page 425: ΠΡΟΤΕΙΝΟΜΕΝΕΣ ΑΣΚΗΣΕΙΣ ΓΙΑ ΜΑΘΗΤΙΚΟΥΣ ΔΙΑΓΩΝΙΣΜΟΥΣ_ALL

http://www.mathematica.gr/forum/viewtopic.php?f=109&t=15584

Επιμέλεια: xr.tsif Σελίδα 1

Page 426: ΠΡΟΤΕΙΝΟΜΕΝΕΣ ΑΣΚΗΣΕΙΣ ΓΙΑ ΜΑΘΗΤΙΚΟΥΣ ΔΙΑΓΩΝΙΣΜΟΥΣ_ALL

http://www.mathematica.gr/forum/viewtopic.php?f=109&t=15584

Επιμέλεια: xr.tsif Σελίδα 2

ΠΡΟΤΕΙΝΟΜΕΝΕΣ

ΑΣΚΗΣΕΙΣ ΓΙΑ ΜΑΘΗΤΙΚΟΥΣ

ΔΙΑΓΩΝΙΣΜΟΥΣ

ΤΕΥΧΟΣ 7ο

ΑΣΚΗΣΕΙΣ 601 - 700

Αφιερωμένο σε κάθε μαθητή που ασχολείται ή πρόκειται να ασχοληθεί με

Μαθηματικούς διαγωνισμούς

Τσιφάκης Χρήστος : xr.tsif

Page 427: ΠΡΟΤΕΙΝΟΜΕΝΕΣ ΑΣΚΗΣΕΙΣ ΓΙΑ ΜΑΘΗΤΙΚΟΥΣ ΔΙΑΓΩΝΙΣΜΟΥΣ_ALL

http://www.mathematica.gr/forum/viewtopic.php?f=109&t=15584

Επιμέλεια: xr.tsif Σελίδα 3

ΘΕΜΑ 601 (ΔΗΜΗΤΡΗΣ ΙΩΑΝΝΟΥ)

Να γραφτεί ο αριθμός: 2n 2n 1 2n 2A 10 10 10

, ως άθροισμα τετραγώνων

τριών φυσικών αριθμών (n N* ).

ΘΕΜΑ 602 (ΔΗΜΗΤΡΗΣ ΙΩΑΝΝΟΥ)

(α) Να αποδείξετε ότι: Αν x,y 0 με x y , τότε: x y

xy2

.

(β) Να αποδείξετε ότι: 2010 2013

1 2 3 ... 20104

.

ΘΕΜΑ 603 (ΔΗΜΗΤΡΗΣ ΙΩΑΝΝΟΥ)

Δίνεται ένας εξαψήφιος αριθμός, ο οποίος έχει ψηφίο των μονάδων του το 6 . Αν

πάρουμε το ψηφίο των μονάδων και το τοποθετήσουμε μπροστά από τον αρχικό

αριθμό χωρίς να μεταβάλλουμε τα άλλα ψηφία του, τότε προκύπτει νέος

εξαψήφιος αριθμός, ο οποίος είναι τετραπλάσιος από τον προηγούμενο αριθμό.

Ποιος είναι ο αρχικός αριθμός;

ΘΕΜΑ 604 (ΔΗΜΗΤΡΗΣ ΙΩΑΝΝΟΥ)

Να κατατάξετε κατά σειρά μεγέθους από τον μικρότερο προς τον μεγαλύτερο,

τους αριθμούς:

1001 1001 1000 1000 1001x 1001 ,y 1000 1000 ,z 1001 1000 .

ΘΕΜΑ 605 (ΔΗΜΗΤΡΗΣ ΙΩΑΝΝΟΥ)

Αν x,y N , και αν 2 y

x 2069 2 x , να βρεθούν οι αριθμοί x,y .

Page 428: ΠΡΟΤΕΙΝΟΜΕΝΕΣ ΑΣΚΗΣΕΙΣ ΓΙΑ ΜΑΘΗΤΙΚΟΥΣ ΔΙΑΓΩΝΙΣΜΟΥΣ_ALL

http://www.mathematica.gr/forum/viewtopic.php?f=109&t=15584

Επιμέλεια: xr.tsif Σελίδα 4

ΘΕΜΑ 606 (ΔΗΜΗΤΡΗΣ ΙΩΑΝΝΟΥ)

Δίνονται οι αριθμοί: 2 2m m m m 8 2

x 5 9 ,y n 2(15p 1) , όπου *

m,n,p N .

Να βρείτε ποιο μπορεί να είναι το ψηφίο των μονάδων του αριθμού xy .

ΘΕΜΑ 607 (ΔΗΜΗΤΡΗΣ ΙΩΑΝΝΟΥ)

Δείξτε ότι δεν υπάρχει φυσικός αριθμός n , τέτοιος ώστε ο αριθμός:

n n 1 n 2 n 3A 2 3 5 7

, να είναι τετράγωνος.

ΘΕΜΑ 608 (ΔΗΜΗΤΡΗΣ ΙΩΑΝΝΟΥ)

Τους φυσικούς αριθμούς 1,2,3,... τους χωρίζουμε σε ομάδες ως εξής:

1 2 3A (1,2),A (3,4,5,6),A (7,8,9,10,11,12),...

Να βρείτε ποια είναι η ομάδα 11

A .

ΘΕΜΑ 609 (ΔΗΜΗΤΡΗΣ ΙΩΑΝΝΟΥ)

Να συγκριθούν οι αριθμοί: 2011 2010 4020 4023 4024x 5 5 ,y 2 2 2 .

ΣΗΜΕΙΩΣΗ: Ένας άλλος τρόπος να το δείξουμε (αλλά δεν είναι επιπέδου

Γυμνασίου) είναι με την ανισότητα του Bernoulli,

που λέει ότι αν a 1 και *

n N , τότε: n(1 a) 1 na .

ΘΕΜΑ 610 (ΔΗΜΗΤΡΗΣ ΙΩΑΝΝΟΥ)

Αν για τους φυσικούς αριθμούς x,y,z ισχύει ότι:

2x y 2

2x

2 11 2y 1 z 1

4 9 3y 1 3z 1

, να βρεθούν οι αριθμοί αυτοί.

Page 429: ΠΡΟΤΕΙΝΟΜΕΝΕΣ ΑΣΚΗΣΕΙΣ ΓΙΑ ΜΑΘΗΤΙΚΟΥΣ ΔΙΑΓΩΝΙΣΜΟΥΣ_ALL

http://www.mathematica.gr/forum/viewtopic.php?f=109&t=15584

Επιμέλεια: xr.tsif Σελίδα 5

ΘΕΜΑ 611 (ΔΗΜΗΤΡΗΣ ΙΩΑΝΝΟΥ)

Να βρεθεί ο διψήφιος αριθμός ab , αν γνωρίζουμε ότι:

7 ab ba abba :11 1 .

ΘΕΜΑ 612 (ΜΠΑΜΠΗΣ ΣΤΕΡΓΙΟΥ)

Να βρεθούν όλοι οι τριψήφιοι αριθμοί που αν διαιρεθούν με τους αριθμούς

7,8,9 δίνουν αντίστοιχα υπόλοιπα 1,4,7 .

ΘΕΜΑ 613 (Socrates)

Δείξτε ότι: αν a,b,c 0 τότε 2 2 2

(a bc)(b ca)(c ab) abc(a b)(b c)(c a) .

Λύση:

Προτού δώσουμε την λύση στο θέμα αυτό, είναι χρήσιμο να αναφερθούμε σε

ένα θεώρημα, που ισχύει στις ανισότητες (το οποίο βέβαια είναι για πολύ

προχωρημένους στα διαγωνιστικά μαθηματικά μαθητές Γυμνασίου).

Το γράφω, όπως είναι δημοσιευμένο στο βιβλίο ΑΛΓΕΒΡΙΚΕΣ ΑΝΙΣΟΤΗΤΕΣ

των αγαπητών συναδέλφων Μπάμπη Στεργίου και Νίκου Σκομπρή :

Έστω 1 2 n 1 2 n

x (x ,x ,...,x ),y (y ,y ,...,y ) δύο n-άδες θετικών αριθμών και

1 2 nw (w ,w ,...,w ) μια n-άδα στην οποία οι αριθμοί

1 2 nw ,w ,...,w είναι οι

αριθμοί 1 2 n

y ,y ,...,y , με διαφορετική ίσως σειρά, τότε:

Αν οι n-άδες x και y είναι όμοια διατεταγμένες, δηλαδή 1 2 n

x x ... x και

1 2 ny y ... y (ή αντιστρόφως οι φορές των ανισοτήτων), τότε:

1 1 2 2 n n 1 w1 2 2 n n(x y )(x y )...(x y ) (x )(x w )...(x w ) .

ΕΝΩ, αν οι n-άδες x και y , έχουν αντίθετη διάταξη, τότε ισχύει και πάλι η

παραπάνω ανισότητα, αλλά με αντίθετη την φορά της.

Page 430: ΠΡΟΤΕΙΝΟΜΕΝΕΣ ΑΣΚΗΣΕΙΣ ΓΙΑ ΜΑΘΗΤΙΚΟΥΣ ΔΙΑΓΩΝΙΣΜΟΥΣ_ALL

http://www.mathematica.gr/forum/viewtopic.php?f=109&t=15584

Επιμέλεια: xr.tsif Σελίδα 6

ΘΕΜΑ 614 (Socrates)

Βρείτε τους μη αρνητικούς ακέραιους x,y,z για τους οποίους zx y 2

(2 1)(2 1) 2 1 .

ΘΕΜΑ 615 (Socrates)

Οι ακέραιοι m και n ικανοποιούν τη σχέση 3m 4n 100 . Να βρεθεί η

ελάχιστη τιμή του |m n | .

http://forum.gil.ro/viewtopic.php?f=38&t=5617&start=0.

ΘΕΜΑ 616 (Socrates)

Έστω a,b,c θετικοί ακέραιοι τέτοιοι ώστε 5 5a |b ,b | c και 5

c | a . Δείξτε οτι

31abc | (a b c) .

ΘΕΜΑ 617 (Socrates)

Να λυθεί το σύστημα

(x 1)yz 12

(y 1)zx 4

(z 1)xy 4

.

ΘΕΜΑ 618 (Socrates)

Έστω a,b,c τα μήκη των πλευρών ενός τριγώνου. Αν ισχύει ab bc ca 1 να

δείξετε ότι (a 1)(b 1)(c 1) 4 .

ΘΕΜΑ 619 (Socrates)

Οι μη μηδενικοί πραγματικοί αριθμοί a,b,c είναι τέτοιοι ώστε 2 2 2 3 3 3

a b c a b c a b c . Βρείτε παράδειγμα τέτοιων αριθμών

(διαφορετικών ανά δύο) και υπολογίστε την τιμή της παράστασης

1 1 1

a b c 2a b c

.

Page 431: ΠΡΟΤΕΙΝΟΜΕΝΕΣ ΑΣΚΗΣΕΙΣ ΓΙΑ ΜΑΘΗΤΙΚΟΥΣ ΔΙΑΓΩΝΙΣΜΟΥΣ_ALL

http://www.mathematica.gr/forum/viewtopic.php?f=109&t=15584

Επιμέλεια: xr.tsif Σελίδα 7

ΘΕΜΑ 620 (Socrates)

Χρωματίζουμε το επίπεδο με δύο χρώματα. Δείξτε ότι υπάρχει ορθογώνιο και

ισοσκελές τρίγωνο με κορυφές ίδιου χρώματος.

ΘΕΜΑ 621 (ΔΗΜΗΤΡΗΣ ΙΩΑΝΝΟΥ)

Αν x,y 0 , x y και αν 5 5x y x y , να αποδείξετε ότι: 4 4

x y 1 .

ΘΕΜΑ 622 (ΔΗΜΗΤΡΗΣ ΙΩΑΝΝΟΥ)

Να βρεθούν τα δύο τελευταία ψηφία του αριθμού 20137 .

ΘΕΜΑ 623 (Socrates)

Βρείτε όλους τους πραγματικούς αριθμούς a για τους οποίους η εξίσωση

| x | x | a | | x | x | a | 2 έχει ακριβώς τρεις ρίζες.

ΘΕΜΑ 624 (Socrates)

Οι θετικοί αριθμοί x,y είναι τέτοιοι ώστε 3 3x y 2xy . Να δείξετε ότι

2

3x,y 2 .

ΘΕΜΑ 625 (Socrates)

Οι ακέραιοι a,b είναι τέτοιοι ώστε οι αριθμοί a 2b και b 2a να είναι τέλεια

τετράγωνα. Δείξτε ότι 3 / a,b .

ΘΕΜΑ 626 (Socrates)

Βρείτε όλους τους πραγματικούς αριθμούς a για τους οποίους η εξίσωση 2

x (a 2)x 1 3 | x | έχει ακριβώς τρεις ρίζες.

ΘΕΜΑ 627 (Socrates)

Έστω n ένας θετικός ακέραιος τέτοιος ώστε οι αριθμοί 2n 1 και 3n 1 να

είναι τέλεια τετράγωνα. Δείξτε ότι ο αριθμός 5n 3 είναι σύνθετος.

Page 432: ΠΡΟΤΕΙΝΟΜΕΝΕΣ ΑΣΚΗΣΕΙΣ ΓΙΑ ΜΑΘΗΤΙΚΟΥΣ ΔΙΑΓΩΝΙΣΜΟΥΣ_ALL

http://www.mathematica.gr/forum/viewtopic.php?f=109&t=15584

Επιμέλεια: xr.tsif Σελίδα 8

ΘΕΜΑ 628 (Socrates)

Χρωματίζουμε το επίπεδο με δύο χρώματα. Δείξτε ότι υπάρχει ισόπλευρο

τρίγωνο με κορυφές ίδιου χρώματος.

ΘΕΜΑ 629 (Socrates)

Ποιος ο μέγιστος αριθμός σημείων που μπορούμε να τοποθετήσουμε στο

εσωτερικό και την περιφέρεια μοναδιαίου τετραγώνου ώστε να μην υπάρχουν

σημεία που απέχουν λιγότερο από 1

2;

ΘΕΜΑ 630 (Socrates)

Ο θετικός ακέραιος n έχει δύο διαφορετικούς διαιρέτες a και b τέτοιους ώστε

(a 1)(b 2) n 2 .

Δείξτε ότι ο αριθμός 2n είναι τέλειο τετράγωνο ακεραίου.

ΘΕΜΑ 631 (Socrates)

Στον πίνακα είναι γραμμένοι οι αριθμοί 1,2,...,9 . Σβήνουμε δύο από αυτούς,

έστω a και b , και γράφουμε τον αριθμό 2 2

a 7ab 3b .

Έτσι έχουμε τώρα 8 αριθμούς. Συνεχίζουμε με τον ίδιο τρόπο μέχρι να μείνει

μόνο ένας αριθμός στον πίνακα.

Μπορεί αυτός ο αριθμός να είναι ο 2010 ;

ΘΕΜΑ 632 (Socrates)

Δείξτε ότι ένας πρώτος αριθμός δε μπορεί να εκφρασθεί ως άθροισμα 2 ή

περισσοτέρων διαδοχικών θετικών περιττών αριθμών.

ΘΕΜΑ 633 (Socrates)

Να δείξετε ότι 2 2 2 21(a b c) a b c 2(a b 1)

3 .

Page 433: ΠΡΟΤΕΙΝΟΜΕΝΕΣ ΑΣΚΗΣΕΙΣ ΓΙΑ ΜΑΘΗΤΙΚΟΥΣ ΔΙΑΓΩΝΙΣΜΟΥΣ_ALL

http://www.mathematica.gr/forum/viewtopic.php?f=109&t=15584

Επιμέλεια: xr.tsif Σελίδα 9

ΘΕΜΑ 634 (Socrates)

Έστω C {1,2,3,4,5,6,7,8,9,10,11,12,13,14,15,16,17,18,19,20} και

S {4,5,9,14,23,37} .

Βρείτε δύο σύνολα Aκαι B τέτοια ώστε

(α) A B

(β) A B C

(γ) το άθροισμα δύο διαφορετικών στοιχείων του A δεν ανήκει στο S .

(δ) το άθροισμα δύο διαφορετικών στοιχείων του B δεν ανήκει στο S .

ΘΕΜΑ 635 (Socrates)

Έστω S(n) το άθροισμα των ψηφίων (στο δεκαδικό σύστημα) του αριθμού n .

Για παράδειγμα, S(2012) 2 0 1 2 5 .

Δείξτε ότι δεν υπάρχει ακέραιος n 0 για τον οποίο n S(n) 9990 .

ΘΕΜΑ 636 (ΔΗΜΗΤΡΗΣ ΙΩΑΝΝΟΥ)

Έστω A(n) 1 3 5 ... (2n 1) και B(n) 2 4 6 ... (2n) ,

όπου *

n N . Να αποδείξετε ότι: A(2)A(3)...A(n) 1

B(1)B(2)...B(n) n 1

.

ΘΕΜΑ 637 (Socrates)

Να δείξετε ότι 1

| a 2 b |2(a b)

για όλους τους θετικούς ακεραίους a,b .

ΘΕΜΑ 638 (Socrates)

Βρείτε όλους τους πρώτους pγια τους οποίους ο αριθμός p 45 4p είναι τέλειο

τετράγωνο ακεραίου.

Page 434: ΠΡΟΤΕΙΝΟΜΕΝΕΣ ΑΣΚΗΣΕΙΣ ΓΙΑ ΜΑΘΗΤΙΚΟΥΣ ΔΙΑΓΩΝΙΣΜΟΥΣ_ALL

http://www.mathematica.gr/forum/viewtopic.php?f=109&t=15584

Επιμέλεια: xr.tsif Σελίδα 10

ΘΕΜΑ 639 (Socrates)

Στον πίνακα είναι γραμμένοι 6 διαφορετικοί θετικοί ακέραιοι. Δύο μαθητές, ο

A και ο B , υπολογίζουν όλα τα αθροίσματα μεταξύ δύο από αυτούς τους

αριθμούς.

Ο Aπαρατηρεί ότι υπάρχουν ακριβώς 10 πρώτοι ανάμεσα σ' αυτά τα

αθροίσματα ενώ ο B , 9 πρώτοι αριθμοί.

Ποιος έχει δίκιο; Βρείτε παράδειγμα τέτοιων αριθμών.

ΘΕΜΑ 640 (Socrates)

Να δείξετε ότι

2

2 2 2

1 1 1 1 1 1 1

2a bc 2b ca 2c ab 9 a b c

,

για όλους τους θετικούς πραγματικούς αριθμούς a,b,c .

ΘΕΜΑ 640β (G.Bas)

Η παρακάτω αλλά πιο ισχυρή Ανισότητα ισχύει επίσης:

Αν a,b,c 0 τότε να δείξετε ότι 2

2 2 2

1 1 1 a b c

2a bc 2b ca 2c ab ab bc ca

.

Υπάρχει μια όμορφη λύση με Cauchy – Schwarz.

ΘΕΜΑ 641 (Socrates)

Να δείξετε ότι 2

3 3 3

1 1 1 (ab bc ca)

a bc b ca c ab 6

,

για όλους τους θετικούς πραγματικούς αριθμούς a,b,c με abc 1 .

Page 435: ΠΡΟΤΕΙΝΟΜΕΝΕΣ ΑΣΚΗΣΕΙΣ ΓΙΑ ΜΑΘΗΤΙΚΟΥΣ ΔΙΑΓΩΝΙΣΜΟΥΣ_ALL

http://www.mathematica.gr/forum/viewtopic.php?f=109&t=15584

Επιμέλεια: xr.tsif Σελίδα 11

ΘΕΜΑ 642 (Socrates)

Να δείξετε ότι a b c a b c

a b b c c a 2

, για όλους τους θετικούς

πραγματικούς αριθμούς a,b,c με 2 2 2

a b c a b c .

ΘΕΜΑ 643 (Socrates)

Να δείξετε ότι 4 5 6x y (x y) 2 για όλους τους πραγματικούς αριθμούς

x,y [0,1] . Πότε ισχύει η ισότητα;

ΘΕΜΑ 644 (Socrates)

Να δείξετε ότι 2 2 2

a b c1

a 2 b 2 c 2

, για όλους τους θετικούς

πραγματικούς αριθμούς a,b,c με abc 1 .

ΘΕΜΑ 645 (Socrates)

Να δείξετε ότι 2 2 2 2 2 2

x y z

2(x y ) 2(y z ) 2(z x )

2 2 2 2 2 2

2 2 2 2 2 2

4x y 4y z 4z x9

x 4y y 4z z 4x

, για όλους τους θετικούς πραγματικούς

αριθμούς x,y,z .

Είναι το 9 η καλύτερη σταθερά;

ΘΕΜΑ 646 (Socrates)

Να λυθεί το σύστημα 2 2

2(x y 2) y(x y 2)

x (y 1) y (x 1) xy 1

.

Page 436: ΠΡΟΤΕΙΝΟΜΕΝΕΣ ΑΣΚΗΣΕΙΣ ΓΙΑ ΜΑΘΗΤΙΚΟΥΣ ΔΙΑΓΩΝΙΣΜΟΥΣ_ALL

http://www.mathematica.gr/forum/viewtopic.php?f=109&t=15584

Επιμέλεια: xr.tsif Σελίδα 12

ΘΕΜΑ 647 (Socrates)

Να δείξετε ότι 2 2 2

2x 2y 2z3

1 x 1 y 1 z

για όλους τους θετικούς

πραγματικούς αριθμούς x,y,z με 1 1 1 1

x y z xyz .

ΘΕΜΑ 648 (Socrates)

Το άθροισμα 11, διαφορετικών ανά δύο, θετικών ακεραίων είναι μικρότερο του

96 . Δείξτε ότι υπάρχουν δύο με άθροισμα 13 .

ΘΕΜΑ 649 (Socrates)

Να προσδιορίσετε τους τετραψήφιους αριθμούς abcd για τους οποίους

abcd 1 (ac 1)(bd 1) (1) .

ΘΕΜΑ 650 (Socrates)

Να προσδιορίσετε τους πραγματικούς a,b,c αν ισχύουν

2 2 2a b c 26, a b 5, b c 7 .

ΘΕΜΑ 651 (Socrates)

α) Χρησιμοποιώντας τους αριθμούς 1,2,3,...,10 , μια φορά τον καθένα,

μπορούμε να σχηματίσουμε 5 ζεύγη τέτοια ώστε το άθροισμα κάθε ζεύγους να

είναι διαφορετικό και πρώτος αριθμός;

β) Χρησιμοποιώντας τους αριθμούς 1,2,3,...,20 μια φορά τον καθένα,

μπορούμε να σχηματίσουμε 10 ζεύγη τέτοια ώστε το άθροισμα κάθε ζεύγους

να είναι διαφορετικό και πρώτος αριθμός;

Page 437: ΠΡΟΤΕΙΝΟΜΕΝΕΣ ΑΣΚΗΣΕΙΣ ΓΙΑ ΜΑΘΗΤΙΚΟΥΣ ΔΙΑΓΩΝΙΣΜΟΥΣ_ALL

http://www.mathematica.gr/forum/viewtopic.php?f=109&t=15584

Επιμέλεια: xr.tsif Σελίδα 13

ΘΕΜΑ 652 (Socrates)

Να λυθεί η εξίσωση 8 9 10

{x} x [x] (1) .

ΘΕΜΑ 653 (Socrates)

Δείξτε ότι ανάμεσα σε 10 , διαφορετικούς ανά δύο, θετικούς αριθμούς,

υπάρχουν δύο, έστω x,y τέτοιοι ώστε (1 x)(1 y)

0 x y9

.

ΘΕΜΑ 654 (Socrates)

Στην περιφέρεια κύκλου, μήκους s , βρίσκονται 7 διαφορετικά σημεία. Τα τρία

από αυτά σχηματίζουν ισόπλευρο τρίγωνο ενώ τα υπόλοιπα τετράγωνο.

Δείξτε ότι υπάρχουν δύο σημεία από αυτά τέτοια ώστε το κυρτογώνιο τόξο που

ορίζουν να έχει μήκος όχι μεγαλύτερο από s

24.

ΘΕΜΑ 655 (Socrates)

Μια ακολουθία θετικών ακεραίων 0 1 2 n

a ,a ,a , ,a , κατασκευάζεται ως εξής:

[*] αν το τελευταίο ψηφίο του n

a είναι μικρότερο ή ίσο του 5 τότε το ψηφίο

αυτό σβήνεται και ο n 1

a

είναι ο αριθμός που μένει

(αν ο n 1

a

δεν περιέχει κανένα ψηφίο τότε η διαδικασία τερματίζεται.)

[*] αλλιώςn 1 n

a a9 .

Μπορούμε να επιλέξουμε τον 0

a ώστε να προκύψει άπειρη ακολουθία ;

ΘΕΜΑ 656 (Socrates)

Υπάρχουν αριθμοί 1 2 2009

x ,x , ,x { 1,1} , έτσι ώστε

1 2 2 3 3 4 2008 2009 2009 1x x x x x x x x x x 999 ;

Page 438: ΠΡΟΤΕΙΝΟΜΕΝΕΣ ΑΣΚΗΣΕΙΣ ΓΙΑ ΜΑΘΗΤΙΚΟΥΣ ΔΙΑΓΩΝΙΣΜΟΥΣ_ALL

http://www.mathematica.gr/forum/viewtopic.php?f=109&t=15584

Επιμέλεια: xr.tsif Σελίδα 14

ΘΕΜΑ 657 (Socrates)

Βρείτε όλους τους ακεραίους n τέτοιους ώστε 3

2

n 3Z

n 7

.

ΘΕΜΑ 658 (ΔΗΜΗΤΡΗΣ ΙΩΑΝΝΟΥ)

Να λυθεί η εξίσωση: 2[x] 4 3x , όπου x R .

ΘΕΜΑ 659 (Socrates)

Βρείτε όλους τους ακεραίους n για τους οποίους ο αριθμός n7 147 είναι

τέλειο τετράγωνο ρητού.

ΘΕΜΑ 660 (Socrates)

Έστω m,n θετικοί ακέραιοι τέτοιοι ώστε ο αριθμός m n 1 να είναι πρώτος

και διαιρέτης του 2 22(m n ) 1 . Δείξτε ότι m n .

ΘΕΜΑ 661 (Socrates)

Βρείτε όλους τους θετικούς ακεραίους μικρότερους του 1000 τέτοιους ώστε ο

κύβος του αθροίσματος των ψηφίων τους να ισούται με το τετράγωνο του

αριθμού.

ΘΕΜΑ 662 (Socrates)

Οι πραγματικοί αριθμοί 1 2 3 4 5

x ,x ,x ,x ,x είναι τέτοιοι ώστε

2 1 3 2 4 3 5 4 1 5| x x | 2 | x x | 3 | x x | 4 | x x | 5 | x x | .

Δείξτε ότι 1 2 3 4 5

x x x x x .

ΘΕΜΑ 663 (Socrates)

Σε ένα τραπέζι υπάρχουν 24 φλιτζάνια. Τρία από αυτά είναι τοποθετημένα

ανάποδα. Μπορούμε να επιλέξουμε τέσσερα φλιτζάνια και να τα γυρίσουμε

ανάποδα.

Page 439: ΠΡΟΤΕΙΝΟΜΕΝΕΣ ΑΣΚΗΣΕΙΣ ΓΙΑ ΜΑΘΗΤΙΚΟΥΣ ΔΙΑΓΩΝΙΣΜΟΥΣ_ALL

http://www.mathematica.gr/forum/viewtopic.php?f=109&t=15584

Επιμέλεια: xr.tsif Σελίδα 15

Είναι δυνατόν κάποια στιγμή όλα τα φλιτζάνια να είναι στην όρθια θέση τους με

100 το πολύ εφαρμογές της διαδικασίας;

ΘΕΜΑ 664 (Socrates)

Στο επίπεδο μοναδιαίου τετραγώνου θεωρούμε 100 σημεία 1 2 100

M ,M ,...,M .

Δείξτε ότι υπάρχουν δύο διαδοχικές κορυφές του τετραγώνου, έστω A και B ,

τέτοιες ώστε το άθροισμα των περιμέτρων των τριγώνων

1 2 100AM B,AM B,...,AM B να είναι μεγαλύτερο από 241 .

ΘΕΜΑ 665 (Socrates)

Στην περιφέρεια ενός κύκλου είναι γραμμένοι 268 αριθμοί. Παρατηρήθηκε ότι

το άθροισμα οποιονδήποτε 20 διαδοχικών αριθμών είναι ίσο με 75 . Αν στις

θέσεις 17,83 και 144 είναι γραμμένοι οι αριθμοί 3,4 και 9 αντίστοιχα, να

βρείτε τον αριθμό στη θέση 210 .

ΘΕΜΑ 666 (Socrates)

Να λυθεί το σύστημα: 3 3 3

| x | | y | 1340

x y 2010xy 670

.

ΘΕΜΑ 667 (ΔΗΜΗΤΡΗΣ ΙΩΑΝΝΟΥ)

Να υπολογίσετε το άθροισμα: 20141A (1 11 111 1111 ... 11...1) 10

81 ,

όπου τα ψηφία στον αριθμό 11...1 είναι 2013 .

ΘΕΜΑ 668 (Socrates)

Έστω a,b θετικοί αριθμοί με άθροισμα 1 . Αν οι αριθμοί 3

a και 3

b είναι ρητοί,

τότε και οι a,b είναι ρητοί.

ΘΕΜΑ 669 (Socrates)

α) Να συγκρίνετε τους αριθμούς 2 148 149

a 1 3 3 ... 3 3 και 99

b 1 5 ... 5 .

Page 440: ΠΡΟΤΕΙΝΟΜΕΝΕΣ ΑΣΚΗΣΕΙΣ ΓΙΑ ΜΑΘΗΤΙΚΟΥΣ ΔΙΑΓΩΝΙΣΜΟΥΣ_ALL

http://www.mathematica.gr/forum/viewtopic.php?f=109&t=15584

Επιμέλεια: xr.tsif Σελίδα 16

β) Να συγκρίνετε τους αριθμούς 2013 2013

200 243 και 2013 2013

216 225 .

ΘΕΜΑ 670 (Socrates)

Να προσδιορίσετε τους τετραψήφιους αριθμούς abcd για τους οποίους ισχύει 2 2 2 2 6 6 6 6

a(a b c d)(a b c d )(a 2b 3c 4d ) abcd .

ΘΕΜΑ 671 (Socrates)

Από τους ακεραίους 1,2,3,4,...,2011,2012 ποιος είναι ο μεγαλύτερος δυνατός

αριθμός ακεραίων που μπορούμε να επιλέξουμε έτσι ώστε αν προσθέτουμε

δύο οποιουσδήποτε από αυτούς που επιλέξαμε να προκύπτει ακέραιος που να

μην είναι πολλαπλάσιο του 11.

ΘΕΜΑ 672 (Socrates)

Να δείξετε ότι 2 2 2

1 1 11

x yz 3 y zx 3 z xy 3

για όλους τους

θετικούς πραγματικούς αριθμούς x,y,z με 2 2 2x y z 9 .

ΘΕΜΑ 673 (Socrates)

Για τους ακέραιους a,b,c ισχύει 2 2

a 4b c . Δείξτε ότι ο αριθμός 2

a 2b

γράφεται ως άθροισμα δύο τετραγώνων ακεραίων.

ΘΕΜΑ 674 (Socrates)

Δείξτε ότι ανάμεσα σε 20132 2 πρώτους, υπάρχουν δύο που η διαφορά τους

διαιρείται με τον 20142 .

ΘΕΜΑ 675 (Socrates)

Θεωρούμε ευθεία d και δύο διαφορετικά σημεία A,B επί αυτής. Έστω ακόμη

2013 σημεία, διαφορετικά μεταξύ τους και με τα A,B που δεν ανήκουν στο

τμήμα AB .

Page 441: ΠΡΟΤΕΙΝΟΜΕΝΕΣ ΑΣΚΗΣΕΙΣ ΓΙΑ ΜΑΘΗΤΙΚΟΥΣ ΔΙΑΓΩΝΙΣΜΟΥΣ_ALL

http://www.mathematica.gr/forum/viewtopic.php?f=109&t=15584

Επιμέλεια: xr.tsif Σελίδα 17

Είναι δυνατόν το άθροισμα των αποστάσεων των σημείων αυτών από το A να

ισούται με το άθροισμα των αποστάσεων τους από το B ;

ΘΕΜΑ 676 (Socrates)

Υπάρχουν ακέραιοι a,b,c ώστε 2 2 2

a b c 20122012 ;

ΘΕΜΑ 677 (Socrates)

Βρείτε τις ακέραιες λύσεις της εξίσωσης 2 2

a 2b b 2a a | 2a 1 2b | .

ΘΕΜΑ 678 (ΣΩΤΗΡΗΣ ΛΟΥΡΙΔΑΣ)

Έστω ότι έχουμε: 0 a x a 1 (1) , με τον a ρητό και τον x ΜΗ τέλειο

τετράγωνο ρητού. Αποδείξτε ότι: 2 2

x a x a 1a x a

2a 1 2a 1 8a 4

.

(*) To a στην σχέση (1) , καλείται «ελλείπουσα τιμή» της x εντός της

μονάδας.

ΘΕΜΑ 679 (ΔΗΜΗΤΡΗΣ ΙΩΑΝΝΟΥ)

Δείξτε ότι για κάθε n N , ο αριθμός 5n n διαιρείται με το 30 .

ΘΕΜΑ 680 (ΔΗΜΗΤΡΗΣ ΙΩΑΝΝΟΥ)

Έστω N (6x y z)(x 6y z)(x y 6z) , με x,y,z N .

(α) Να αποδείξετε ότι ο N είναι άρτιος.

(β) Αν 5 | N , δείξτε ότι 250 | N .

ΘΕΜΑ 681 (Socrates)

Να προσδιορίσετε τους θετικούς ακεραίους 1 2 n n 1

x x ... x x

αν

n n 1 1 2 nx x 2(x x ... x )

.

Page 442: ΠΡΟΤΕΙΝΟΜΕΝΕΣ ΑΣΚΗΣΕΙΣ ΓΙΑ ΜΑΘΗΤΙΚΟΥΣ ΔΙΑΓΩΝΙΣΜΟΥΣ_ALL

http://www.mathematica.gr/forum/viewtopic.php?f=109&t=15584

Επιμέλεια: xr.tsif Σελίδα 18

ΘΕΜΑ 682 (Socrates)

Να λυθεί το σύστημα:

x(x 1) yz

y(y 2) zx

z(z 3) xy

.

ΘΕΜΑ 683 (Socrates)

Να αποδείξετε ότι 2 2 2

2 2 2

a b c 3

a b b c c a 2

, για όλους τους θετικούς

πραγματικούς αριθμούς a,b,c με a b c 3 .

ΘΕΜΑ 684 (Socrates)

Να αποδείξετε ότι 2 2 2x xy xyz 4xyz 4 , για όλους τους θετικούς

πραγματικούς αριθμούς x,y,z .

ΘΕΜΑ 685 (Socrates)

Θεωρούμε τους μη αρνητικούς ακέραιους a,b,c καθώς και τους αριθμούς 2 2 2

x (a 1) 4b, y (b 1) 4c, z (c 1) 4a .

Αν x y z 2013 , να προσδιορίσετε τους a,b,c .

ΘΕΜΑ 686 (Socrates)

Για την ακολουθία n

(a ),n 1 ισχύει 1

1a

6 και

n 1 n

n 1 1a a

n 3 2

.

Βρείτε τον αριθμό 2010

a .

ΘΕΜΑ 687 (Socrates)

Το ορθογώνιο X (δείτε το σχήμα στο σύνδεσμο) διαιρείται σε 9 μικρότερα

ορθογώνια με ευθείες παράλληλες στις πλευρές του.

Page 443: ΠΡΟΤΕΙΝΟΜΕΝΕΣ ΑΣΚΗΣΕΙΣ ΓΙΑ ΜΑΘΗΤΙΚΟΥΣ ΔΙΑΓΩΝΙΣΜΟΥΣ_ALL

http://www.mathematica.gr/forum/viewtopic.php?f=109&t=15584

Επιμέλεια: xr.tsif Σελίδα 19

Αν τα ορθογώνια A,B,C,D,E έχουν εμβαδόν 2 2 2 2 23cm ,9cm ,1cm ,2cm ,8cm

αντίστοιχα, να βρείτε το εμβαδόν του X .

http://forum.gil.ro/viewtopic.php?f=38& ... bad5725d55

ΘΕΜΑ 688 (Socrates)

Προσδιορίστε τους φυσικούς a και n 0 καθώς και το ψηφίο b ώστε a

6 1 bb b , όπου το b εμφανίζεται n φορές.

http://forum.gil.ro/viewtopic.php?f=36&t=5809&start=0

ΘΕΜΑ 689 (Socrates)

Βρείτε τους πρώτους p q r τέτοιους ώστε 25pq r 2004 και ο αριθμός

pqr 1 είναι τέλειο τετράγωνο.

Τι γίνεται αν αφαιρέσουμε τον περιορισμό p q r ;

http://erdos.fciencias.unam.mx/omm/primer18omm.pdf.

ΘΕΜΑ 690 (Socrates)

Βρείτε όλους τους m,n N και τον πρώτο p 5 ώστε

2 nm(4m m 12) 3(p 1) .

ΘΕΜΑ 691 (Socrates)

Να βρείτε τον ελάχιστο θετικό ακέραιο αριθμό που μπορεί να γραφεί σαν

άθροισμα 9,10 και 11 διαδοχικών θετικών ακεραίων αριθμών.

ΘΕΜΑ 692 (Socrates)

Βρείτε τους πραγματικούς αριθμούς x,y,z για τους οποίους

2 2 2 3x y z xy yz zx 2(x y z)

2 .

Page 444: ΠΡΟΤΕΙΝΟΜΕΝΕΣ ΑΣΚΗΣΕΙΣ ΓΙΑ ΜΑΘΗΤΙΚΟΥΣ ΔΙΑΓΩΝΙΣΜΟΥΣ_ALL

http://www.mathematica.gr/forum/viewtopic.php?f=109&t=15584

Επιμέλεια: xr.tsif Σελίδα 20

ΘΕΜΑ 693 (Socrates)

Στην ακολουθία n

(a ) με γενικό όρο n

a 12n p 12 , ο p είναι πρώτος

αριθμός. Να βρείτε το μέγιστο πλήθος διαδοχικών όρων της ακολουθίας που

είναι πρώτοι αριθμοί.

ΘΕΜΑ 694 (Socrates)

Να βρείτε τις ακέραιες θετικές λύσεις (x,y) της εξίσωσης

2 23 1(x y ) xy 83

8 4 .

ΘΕΜΑ 695 (Socrates)

Να εξετάσετε αν το σύνολο A {1,2,3,...,2012} μπορεί να χωριστεί σε

υποσύνολα, έτσι ώστε το μέγιστο στοιχείο κάθε τέτοιου υποσυνόλου να ισούται

με το άθροισμα όλων των υπολοίπων στοιχείων του συνόλου A .

ΘΕΜΑ 696 (Socrates)

Αν οι θετικοί πραγματικοί αριθμοί a,b,c είναι τέτοιοι ώστε

2 2 2 2 2 2 3 3 3a b a c b c 4 a b c , να δείξετε ότι ab ac bc 2 abc .

Πότε ισχύει η ισότητα;

ΘΕΜΑ 697 (Socrates)

Ένας καλαθοσφαιριστής έχει ποσοστό ευστοχίας στις ελεύθερες βολές

μικρότερο του 80%.

Λίγο καιρό αργότερα το ποσοστό του στις ελεύθερες βολές βελτιώθηκε και έγινε

μεγαλύτερο από 80%.

Δείξτε ότι κάποια χρονική στιγμή το ποσοστό ευστοχίας του στις ελεύθερες

βολές ήταν ακριβώς 80%.

Page 445: ΠΡΟΤΕΙΝΟΜΕΝΕΣ ΑΣΚΗΣΕΙΣ ΓΙΑ ΜΑΘΗΤΙΚΟΥΣ ΔΙΑΓΩΝΙΣΜΟΥΣ_ALL

http://www.mathematica.gr/forum/viewtopic.php?f=109&t=15584

Επιμέλεια: xr.tsif Σελίδα 21

ΘΕΜΑ 698 (Socrates)

Να λυθεί η εξίσωση [x] [2x] ... [10x] 3 .

ΘΕΜΑ 699 (Socrates)

Δείξτε ότι για κάθε a,b,c,x,y,z 0 ισχύει

ax by cz 2 ab 2 bc 2 ca a b c

y z z x x y 2

.

ΘΕΜΑ 700 (Socrates)

Βρείτε όλα τα ζεύγη μη αρνητικών ακεραίων (a,b) τέτοια ώστε ο αριθμός a b

3 7 να είναι τέλειο τετράγωνο.